Location via proxy:   [ UP ]  
[Report a bug]   [Manage cookies]                

Presented by 15 Batch 1 Fhcs - Eusl

Download as pdf or txt
Download as pdf or txt
You are on page 1of 453

1301

Presented by 15th Batch 1 FHCS | EUSL


HB 01

Presented by 15th Batch 2 FHCS | EUSL


MOLECULAR ORGANIZTION OF HUMAN BODY
EME QUESTIONS

10TH BATCH EME - MCQs

01) Regarding Carbohydrates,


a) Lactose is the major dietary component in milk
b) Heparin binds with a central linear core protein to create proteoglycans
c) D-galactose is a C2 epimer of D-glucose
d) Starch contain 80-85% of amylose & 15-20% Of amylopectin
e) β (1,4) glycosidic bond of cellulose cannot be hydrolyzed by mammalian constitutive protein.

02) Write regards to protein


a) Heparin binds with a central linear core protein to create proteoglycans
b) Only 20 different type of proteins found in animals
c) Denaturation of protein always lead to irreversible loss in tertiary structure
d) Polypeptide chain of quaternary structure are associated through covalent interaction
e) Protein with a single peptide chain generally process a quaternary structure

03) Regarding fatty acid


a) Most naturally occurring unsaturated long-chain fatty acids are in cis-configuration
b) Polyunsaturated fatty acids have more than one double bond
c) Short chain volatile fatty acids are normally found in triglycerides
d) Unsaturated fatty acids generally predominate over the saturated one in higher plants
e) Unsaturated fatty acids impart a higher melting point to cell membranes of animals

04) With regard to DNA


a) The bases carry genetic information
b) Heat stability of CΞG base pair is greater than A=T base pair
c) β form DNA twists in right-handed double helix manner
d) It is a polyribonucleotide polymer
e) In ovum DNA is present with a single strand

05) Enzymes,
a) Becomes more resistant to heat denaturation in presence of substrate
b) Active site has functional group of amino acid residues
c) Activity is measured during initial velocity of the reactions
d) Increase the rate of reaction by increasing kinetic energy of reactants
e) Bind with substrate by multiple weak attractions

SBR

06) Which one of the following statements is correct?


a) Glutamine contain a hydroxyl group in it side chain

Presented by 15th Batch 3 FHCS | EUSL


b) Serine can form disulfide bonds
c) Cysteine consist of the smallest side chain
d) Isoleucine is nearly always found buried in center of protein
e) Glycine contains an amide group in its side chain

07) The Michaelis constant Km is


a) Numerically is equals to Vmax
b) Depend on the enzyme concentration
c) Independent in pH
d) Numerically equal to the substrate concentration that gives half maximum velocity
e) Increase in the presence of non-competitive inhibitor

08) A fatty acid is not synthesized in human body & only get from diet
a) Palmitic acid
b) Oleic acid
c) Linoleic acid
d) Stearic acid
e) Valeric acid

09) DNA replication is called as semi discontinuous because,


a) The polymerase remains attached to the DNA molecules at the replication fork
b) Leading strand is synthesized continuously but Lagging strand is synthesized discontinuously
c) β DNA is a right-handed helix
d) DNA is being replicated in both direction
e) None of above

10) Which of the following is a Keto triose?


a. Xylulose
b. Dihydroxyacetone
c. Glyceraldehyde
d. Erythrose
e. Fructo

SEQ
1. Briefly describe on
1.1. Storage of polysaccharide [25 marks]
1.2. Secondary structure of protein [25 marks]
1.3. Protein synthesis [25 marks]
1.4. Functions of prostaglandins [25 marks]

Answers
MCq
1. A T
BT
CF
D
ET
Presented by 15th Batch 4 FHCS | EUSL
2.A.T
BF
CF
DF
EF

3.A T
BT
C
DT
ET

4.A T
BT
CT
DF
EF

5.A
BT
CT
DT
EF
SBR

6. d
7. D
8.C
9.B
10.B

1.Storage polysaccharide

Starch

• It is one of the most Important dietary sources For human beings and is Also one of the main Storage
polysaccharide of

Plants.

• We can find high content Of starch in cereals, roots And also in some other

Vegetable

Presented by 15th Batch 5 FHCS | EUSL


Glycogen

2. Secondary structure of protein

Secondary

• Structure that arises as a result of interactions between backbone groups that are close to one

another in protein

• Helix and alpha chain are not same

Alpha helix

Intra chain H bonds (CO of one AA to NH of AA 4

residues ahead

H bonds parallel to imaginary axis

Presented by 15th Batch 6 FHCS | EUSL


Disrupt α helix

➢ Pro, OH-Pro (by secondary NH2 group)

➢ Gly (its R group has high flexibility)

• Destabilize α helix

➢ Glu, Asp, His, Lys, Arg (by charged / bulky

R groups)

➢ Trp, Val, Ile ( branched β carbon )

Found in -> α keratin, Myoglobin, Haemoglobin

B pleated

Inter Chain or different segments of same

Chain – H bonds

Perpendicular to the axis

Favored by AAs having small R groups

In both fibrous & globular proteins

3 .Protein synthesis

Presented by 15th Batch 7 FHCS | EUSL


Translation

Converting the coded message in the mRNA into the amino acid sequence of the protein.
Presented by 15th Batch 8 FHCS | EUSL
Translation requires,

• mRNA

• tRNA

• Ribosomes

➢ free ribosomes – synthesize proteins required in cytosol, nucleus, mitochondria,

Peroxisomes.

➢ RER bound ribosomes – synthesize secretory proteins, SER, Golgi, plasma & lysosomes

• Translation factors – Initiation, Elongation & Termination factors

• AAs

• Aminoacyl tRNA synthetase

• Energy

Presented by 15th Batch 9 FHCS | EUSL


Refer Lippincott page 461 for protein synthesis summary

4. Prostaglandins

First recognized in prostate gland

♦ they are synthesized at the site of action

Presented by 15th Batch 10 FHCS | EUSL


♦ made in almost all tissues

Functions of Prostaglandins

– alter the cellular activities by

Cellular signaling

11TH BATCH EME - MCQs

01) Regarding the Carbohydrate,


a) Malrotation gives α, β anomers.
b) Sucrose is non-reducing sugar because it has no anomeric OH.
c) Carbohydrates are found in structurally in co-enzyme.

Presented by 15th Batch 11 FHCS | EUSL


d) Fructose is oxidized faster than glucose.
e) Galactose is component of blood group antigen.

02) Amino acid that disrupt α helix of protein,


a) Aspartate
b) Glycine
c) Glutamic
d) Proline
e) Valine

03) Protein denaturation,


a) No irreversible.
b) Increase water solubility.
c) Do not occur in extreme PH
d) Occur with temperature.
e) Will break down primary structure.

04) Glycoprotein act as,


a) Blood group determination.
b) Cell recognition site.
c) Structural molecule.
d) Steroid hormones.
e) Structural components.

05) Regarding enzyme inhibitors,


a) Irreversible inhibitors bind to allosteric site of enzyme.
b) Suicide inhibitors bind to allosteric site of enzyme.
c) Reversible inhibitors bind by non-covalent bond.
d) Competitive inhibition can be revers by increase concentration of [S].
e) Non-competitive inhibitors modify the shape of allosteric site.

06) Cholesterol is,


a) Structural component of the cell.
b) Is synthesis by liver.
c) Major steroid of plant.
d) Hydrophilic in nature.
e) A precursor of progesterone.

07) Regarding omega-3 fatty acids,


a) Mainly found in vegetable oil.
b) Increase heart attack risk.
c) Eicsopentenoic acid is important in brain development.
d) Act as anti-inflammatory.
e) Derived from α Linolenic acid.

08) Nucleic acid,


a) r-RNA acts as a messenger for protein synthesis.
b) t-RNA shows dual specificity.
c) Eukaryotes have multiple replication bubbles.
d) Introns are not translated.

Presented by 15th Batch 12 FHCS | EUSL


e) The two strands of DNA double helix are held together by hydrophobic interaction.

09) Enzymes are regulated by,


a) Allosteric activation.
b) Covalent modification.
c) Feedback inhibition.
d) Induction of enzyme synthesis.
e) Proteolysis cleavage.

10) Regarding to Electron transport chain,


a) It is embedded inner membrane space.
b) High doses of synthetic uncouple cause death in human.
c) Ubiquinone is a protein derivative.
d) FMN is prosthetic group of NADH dehydrogenase.
e) Thermogenic is rich in brown adipose tissue.
SBR

11) Which is not a phospholipid


a) Cephalic
b) Diacyl glycerol
c) Lecithin
d) Phosphatidyl inositol
e) Sphingomyelin

12) Basic side chain amino acid,


a) Arginine
b) Glutamate
c) Leucine
d) Methionine
e) Proline

13) Small piece of DNA can be amplified by,


a) Cloning
b) Gel electrophoresis
c) Polymerase chain reaction
d) Southern blotting
e) Western blotting

14) Enzyme having slightly different molecular structure but perform identical activates are,
a) Apo enzyme
b) Co enzyme
c) Holo-enzyme
d) Iso enzyme
e) Pro enzyme

15) Pyruvate cannot be converted directly into,


a) Acetyl co enzyme
b) Alanine
c) Citrate
d) Glucose

Presented by 15th Batch 13 FHCS | EUSL


e) Lactic acid

11TH BATCH EME – SEQ

01.
1.1. Briefly describe the biochemical functions of carbohydrates. (25 marks)
1.2. Outline the stages of cellular respiration. (25 marks)
1.3. List the different level of structural organization of a protein. (05 marks)
1.4. Describe the two of the above mentioned levels. (45 marks)
02.
2.1. A patient with myocardial infraction was admitted to hospital. Routine clinical manifestations including ECG
were done and serum biochemical parameters were tested on the second day from the onset of infraction. Briefly
explain, what are the biochemical parameters would be elevated on the above condition? (20 marks)
2.2. Explain the biochemical basis of the followings;
a. Ethanol is used as antidote in methanol poisoning (25 marks)
b. Dietary lipids on atherosclerosis (25 marks)
2.3. Describe the double helix structure of DNA. (30 marks)

11TH BATCH EME – MCQ ANSWERS

1.
A
B True
C True Pentose sugar group in nucleotides .(Ribose in NADH)
D True Glycolytic pathway of fructose bypass the major rate limiting step of the glycolysis PFK1
E True Galactose is a component of ABO antigen

2 A True
Amino acids with charge R groups not compatible with alpha helix- glutamate, aspartate , histidine , lysine , arginine
B False
C True
D True Proline is a helix breaker
E True Amino acids with bulky R groups not compatible with alpha helix.
tryptophan , valine or isoleucine
3 A False Careful denaturation can lead to renaturation
B False Decrease the water solubility , increase the viscosity , surface tension reduced and
can be easily digested
C False Occur due to physical and chemical factors
D True
E False Denaturation loss the secondary , tertiary & quaternary structure. But primary
structure cannot damaged . To cleave the peptide bonds of primary structure
11th Batch – HB 1 – EME
prolonged exposure to a high temperature with a strong acid to a long time
period needed.
04 A True Cell recognition (galactose found in ABO antigens on cell membrane )
B True
C True Cell membrane
Presented by 15th Batch 14 FHCS | EUSL
D False
E True
05 A False Bind to active site of the enzyme
B False
C True
D True
E False Change the shape of the active site.
06 A True
B True De – novo synthesis of cholesterol mainly occur in liver & other extra hepatic
tissue cells also can cholesterol synthesis.
C False Major sterol in animal
D False Amphipathic in nature
E True Precursor of bile acids, bile salts , Vitamin D & steroid hormones.
07
A False Mainly found in fish oil
11th Batch – HB 1 – EME
B False Decrease the heart attack risk
C
D True
E
08
A False m-RNA
B True Dual specificity refers to the fact that each t RNA has its particular anticodon,
which corresponds to the specific amino acid to which it can be bonded.
C True Multiple Ori regions in eukaryotes, A,T rich regions. Prokaryotes have a single
ori region
D True Introns and exons are nucleotide sequences within a gene. Introns are removed
by RNA splicing as RNA matures, meaning that they are not expressed in the
final messenger RNA (mRNA) product, while exons go on to be covalently
bonded to one another in order to create mature m-RNA. (Intrones are bad exons
are good )
E False Held together by hydrogen bonds which are between complementary bases.
09
A True
B True
C True
D Feedback regulation.
E True
11th Batch – HB 1 – EME
10 A True
B True Reduce ATP production and denaturation of body enzymes lead to death
C False Ubiquinone = Coenzyme Q --- all the electron carriers in ETC are proteins
except Coenzyme Q (coenzyme q is a liphophilic substance)
D True
E True
11 B glyceride
12 A Basic side chain amino acids--- lysine, arginine & histidine
13 C Polymerase chain reaction –test tube method for amplifying specific DNA
sequence.
14 D

Presented by 15th Batch 15 FHCS | EUSL


15 D

SEQ

1 biochemical functions of carbohydrate

Carbohydrates are chief energy source, in many animals,

They are instant source of energy.

Eg: Glucose is broken down by glycolysis/ kreb’s cycle to yield ATP.

♠ Source of storage of energy.

Eg: It is stored as glycogen in animals and starch in plants.

♠ Protein sparing effect of carbohydrates

Stored carbohydrates acts as energy source instead of proteins.

♠ Carbohydrates are intermediates in biosynthesis of fats and

Proteins.

♠ Carbohydrates gets associated with lipids and proteins

To form surface antigens, receptor molecules, etc

They form structural and protective components,

Eg: in cell wall of plants and microorganisms.

♠ In animals they are important constituent of

Connective tissues.

♠ Component of nucleic acid (Ribose & deoxyribose)

♠ They participate in biological transport, cell-cell

Communication and activation of growth factors.

♠ Carbohydrates that are rich in fibre content help to

Prevent constipation.

♠ Also they help in modulation of immune system

Presented by 15th Batch 16 FHCS | EUSL


Eg: During sports / exercise CHO reduces stress

2. Stages of cellular respiration

Cellular respiration is the process by which the chemical

Energy of “food” molecules is released and partially

Captured in the form of ATP.

• Carbohydrates, fats, and proteins can all be used as

Fuels in cellular respiration, but glucose is most

Commonly used as an example to examine the reactions

And pathways involved

An Oxidation-Reduction Process or REDOX Reaction

•Oxidation of GLUCOSE
Cellular Respiration (4-stages)

• Glycolysis

• Formation of acetyl Coenzyme A

• Krebs Cycle (Citric Acid Cycle)

• Electron Transport Chain (ETC)

Glycolysis

Oxidation of one glucose (6C) to two pyruvate(3C) molecules

♠ Occurs virtually in all tissues

♠ RBC & nervous tissues derive its energy mainly from glycolysis

♠ Unique pathway

- Can utilize O2

If available (aerobic)

- Can function in absence of O2

(anaerobic)
Presented by 15th Batch 17 FHCS | EUSL
Krebs cycle
Each turn of the Krebs Cycle also produces 3NADH,

1FADH2

, and 2CO2

• Therefore, For each Glucose molecule, the Krebs

Cycle produces 6NADH, 2FADH2

, 4CO2

, and 2ATP
Anaplerosis is the act of replenishing TCA cycle intermediates that

Have been extracted for biosynthesis (in what are called

Cataplerotic reactions).

• The TCA cycle is a hub of metabolism, with central importance in both

Energy production and biosynthesis.

Four reactions are classed as anaplerotic, although the production of oxaloacetate from

Pyruvate is probably the most important physiologically. The anaplerotic reactions are:

1. Carboxylation of pyruvate to oxaloacetate (gluconeogenesis)

2. Transamination of aspartate to oxaloacetate


By aspartate aminotransferase.

3. Hydration of glutamate to α-ketoglutarate

4. Β-oxidation of fatty acids to succinyl-C

Pyruvate oxidation

ETC
Energy rich molecules (Glucose), are metabolized by a series of

Oxidation reactions -------Yielding CO2 + H2O

Metabolic intermediates , donate electrons to specific coenzymes

Presented by 15th Batch 18 FHCS | EUSL


(NAD+ & FAD) ------- energy rich reduced forms (NADH & FADH2)

Each reduced coenzyme donate a pair of electrons to a specialized

Electron carriers
5 separate protein cplx (I—V)

– IV are part of ETC--- accept or

Donate electrons to the

Relatively mobile electron

Carriers ( CoQ & Cytochrome c)

Except CoQ.
Cplx I – NADH dehydrogenase

– Succinate dehydrogenase

– Cyt c reductase

– Cyt c oxidase

- ATP synthase

3,4 structural organisation of protein and explanation


Primary

• Nature & sequence of AAs in polypeptide chains that a protein is comprised of

• Each protein has a unique primary structure determined by gene code

• Proteins have a common backbone, different R group sequence

• 3D arrangement depends in this

2) Secondary

• Structure that arises as a result of interactions between backbone groups that are close to one

Another in protein

• Helix and alpha chain are not same


Secondary

Presented by 15th Batch 19 FHCS | EUSL


• Structure that arises as a result of interactions between backbone groups that are close to one

Another in protein

• Helix and alpha chain are not same

Alpha helix

Intra chain H bonds (CO of one AA to NH of AA 4

Residues ahead

H bonds parallel to imaginary axis

Disrupt α helix

➢ Pro, OH-Pro (by secondary NH2 group)

➢ Gly (its R group has high flexibility)

• Destabilize α helix

➢ Glu, Asp, His, Lys, Arg (by charged / bulky

R groups)

➢ Trp, Val, Ile ( branched β carbon )

Found in -> α keratin, Myoglobin, Haemoglobin

B pleated

Inter Chain or different segments of same

Presented by 15th Batch 20 FHCS | EUSL


Chain – H bonds

Perpendicular to the axis

Favored by AAs having small R groups

In both fibrous & globular proteins

Tertiary

• structure due to interactions of R group in protein with

➢ aqueous environment

➢ other R groups far apart

Such as

✓ H bonds

✓ Hydrophobic interactions

✓ Ionic interactions

✓ Disulphide linkag
eMyoglobin

➢ Haem protein – heart & skeletal muscles

➢ O2 storing & O2 transfer within muscle cells

➢ Single polypeptide chain & single haem unit

➢ Eight α helical segments (80% of polypeptide

Chain)

➢ Haem unit stabilized by ‘His’ & hydrophobic

Interactions.

➢ Globular functional protein

Lysozyme

➢ some globular proteins have both α helical &

Β pleated sheet structures in the same


Presented by 15th Batch 21 FHCS | EUSL
Molecule
Quaternary

• In functional molecule

• 2 types

a) Monomeric

b) Oligomeric

• Individual polypeptide chain – protomers / monomers / sub units

• Quaternary structure; characteristic manner in which individual polypeptide chains held

Together by weak non covalent

➢ H bonds

➢ Ionic linkage

➢ Hydrophobic interactions

• One type of subunit; - homo multimer

• Several different subunits; - hetero multimer

Eg: - Hb

✓ α2 β2 –tetramer

✓ 2 chains within each dimer (α β dimer) held tightly by hydrophobic interactions.

Primarily dimers are held by polar bond

• Sub units may function independently of each other or may work cooperatively as in Hb

Proteins Simple

Conjugated = Apo protein + non protein

❖ Chromo protein = protein +coloured compound (Eg; haemoglobin =

Globin + haem)

❖ Glycoprotein & proteoglycan = protein + carbohydrates (Eg; mucin of

Presented by 15th Batch 22 FHCS | EUSL


Saliva)

❖ Lipoprotein – protein + lipid (Eg; cell membrane, plasma lipoprotein)

❖ Nucleoprotein – protein + nucleic acid (Eg; chromosome, ribosome)

❖ Phosphoprotein – protein + phosphate (Eg; casein linked via Ser, Thr)

2.1 MI parameters

2.2 a Ethanol antidote

Methanol Poisoning is treated by Ethanol. By competitive nature of substrates

2.2 b lipids in atherosclerosis


Steroid are mainly formation of atherosclerotic plaque

Presented by 15th Batch 23 FHCS | EUSL


2. 3 double helix of DNA

Structure of DNA (Watson & Crick Model)


• a polymer of deoxyribonucleoside monophosphates
• 2 polynucleotide chains coiled around a common axis (a few viruses that contain singlestranded (ss) DNA)
• Double helical (for replication, transcription)
• Antiparallel
5’ 3’
3’ 5’
• Complementary (not identical)
• Right-handed coiling
• 10 bp/turn,20' A diameter
• Sugar phosphate backbone outside
• Hydrophilic
• Polar sugar molecules
• Negatively charged phosphate groups - Polyanionic
backbone

Presented by 15th Batch 24 FHCS | EUSL


• Make railing
• Bases inside
• Hydrophobic
• stacked inside the helix
• perpendicular to sugar-phosphate backbone
• Make steps with H bonds
• When DNA double helix is viewed from outside 2 groves can be observed
1. Major groove
2. Minor groove
Both for drug attachment
Additional DNA strand can form a triplet stranded DNA at the groove.

• Base pairing in the double helix;(Chargaff’s rule)

O A=T & G= C

O A+G= T+C (number of purines=number of pyrimidines)

O A+T/G+C=k

O Complementary base pairing sequence of bases in a single chain will automatically

Determine the sequence of the other chain.

O bases are always written in sequence from the 5’-end to the 3’-end.

12TH BATCH EME – MCQ

1) Followings are composed exclusively of glucose,


a. Amylose
b. Cellulose
c. Glycogen
d. Lactose
e. Maltose

2) Regarding active site of the enzymes,


a. It is a three dimensional cleft
b. It is responsible for catalysis
c. Substrate are bound to enzyme by weak attaches
d. Interaction with substrate increase free energy of activation
e. Specificity of enzyme binding depend on the arrangement of Carbon atoms in active site

3) Amino acid Glutamine,


a. It has amide group
b. Single letter is Q
c. It has titratable structure
Presented by 15th Batch 25 FHCS | EUSL
d. It is charged neutral amino acid
e. Migration to cathode at physiological PH 7. 2

4) Regarding proteins,
a. Motif is a secondary structure of protein
b. Proline is always in β bend
c. Primary structure is stabilized by H bonds
d. Tertiary structure has one or more polypeptide chain
e. β sheet exist only in anti-parallel form

5) Function of essential fatty acid,


a. Atherogenic effect
b. Provide cell membrane integrity
c. Brain develop
d. As a energy source
e. Synthesis of eicosanoids
6) Regarding enzyme,
a. Catalyze only one type of chemical reaction of substrate
b. High Km of the enzyme needs low [S] level to meet Vmax
c. When temperature increases in 10 0C rate of reaction doubles
d. Km is an index of affinity towards substrate concentration
e. Maltase is a hydrolase enzyme

7) Cholesterol precursor of,


a. Bile
b. Ketone bodies
c. Steroid hormones
d. Vitamin A
e. Vitamin D

8) Regarding nucleic acids,


a. Certain protein can bind with in grooves of DNA double helix
b. Complementary base pairing is found
c. Purine bases common in both nucleic acid
d. N-base order is important to determine the genetic information of a molecule
e. Most predominant structure found in living organism is B-DNA

9) The conversion of pyruvate into acetyl CoA & CO2,


a. Regulated by coenzyme – biotin
b. Contain thymine pyrophosphate
c. Occur in mitochondria
d. Reversible reaction
e. Pyruvate decarboxylase catalyze it

10) Regarding plasma enzyme level,


a. Alkaline phosphate increase in bone cancers
b. Amylase falls in acute pancreatis
c. LDH rise in leukemia
d. Lipase rises in acute pancreatis
e. Transaminase fall in myocardial infraction

Presented by 15th Batch 26 FHCS | EUSL


SBR

11) Which is the correct one,


a. β bend often contain proline
b. Motif are a type of secondary structure
c. The ά-helix can be composed of more than one poly peptide chain
d. The ά-helix stabilized by ionic interaction between the side chains of amino acids
e. Β sheet exist only in anti-parallel form

12) Which of the following is not in GAGs


a. Amylopectin
b. Dermatan sulfate
c. Heparin
d. Hyaluronic acid
e. Keratin sulfate

13) Recently diagnosed hypertensive patient has been prescribed an ACE (Angiotensin converting enzyme inhibitor)
which is known to act by low Vmax. What is the possible mechanism to inhibit drug?
a. Cofactor
b. Competitive inhibitors
c. Non-competitive inhibitor
d. Prosthetic group
e. Suicide inhibitor

14) DNA highly negative charged. Its stability,


a. Depend on H bond
b. Does not hydrophobic interaction
c. Stacking interaction base planarity
d. Increase in increasing ionic interactions
e. Depend on G-C content

15)The enzyme that does not creates an electron carrier that produce 3 ATP with electron transport chain,
a. Gleceraldehyde-3 phosphate
b. Glycerol 3 phosphate dehydrogenase
c. Isocitrate dehydrogenase
d. Malate dehydrogenase
e. Pyruvate dehydrogenase

12TH BATCH EME – SEQ

1.
1.1.
1.1.1. Briefly describe the salient features of an 𝛼 – helix. (25 marks)
1.1.2. Name two modified amino acids found in proteins and state their biological function.
(20 marks)
1.2.
1.2.1. Explain how the types of RNA molecules are structurally modified to do the specific functions. (30
marks)
Presented by 15th Batch 27 FHCS | EUSL
1.2.2. List five uses of molecular diagnostic techniques in Medicine with examples. (25 marks)

2.
2.1. Explain the biochemical basis of the followings;
2.1.1. Statin drugs are used to treat hyperlipidemic conditions. (25 marks)
2.1.2. Cyanide poisoning causes death. (25 marks)
2.2. Outline the functions of
2.2.1. Glycoproteins (25 marks)
2.2.2. Eicosanoids (25 marks)

Answers

1A T
BT
CT
DF
ET

2A T
BT
CT
DF
EF

3A T
BT
CT
DF
E

4A T
BF
CF
D
EF

5A F(ANTI ATHEROGENIC)
BT
CT
D
ET

6A T
BF
CT
DT
ET

7AT
Presented by 15th Batch 28 FHCS | EUSL
BT
CT
D
ET

8A T
BT
CT
DT
E

9A F PANTATHONIC ACID
BT
CT
D
EF

10A T
BF
CT
DT
EF

SBR
11. b
12.A
13.B
14.C
15. A ALL OTHERS USE NAD+

13TH BATCH EME – MCQ

1) Define sources of complex carbohydrates includes


A. Chocolate
B. Fruits
C. Refined sugar
D. Vegetable
E. Whole grain

2) Glycogen
A. Can generate energy even in the absence of oxygen
B. In the muscle involves in the maintenance of blood glucose level
C. Is a heteropolysaccharide
D. Is mobilized slowly
E. Is stored in skeletal muscle and liver

Presented by 15th Batch 29 FHCS | EUSL


3) Non-standard amino acids
A. Histidine
B. Beta alanine
C. Arginine
D. Hydroxylysine
E. ornithine

4) Peptide bond
A. Charged polar
B. Has partial double bond character
C. Has trans configuration
D. Broken by heat
E. Is stable

5) Hyaluronic acid
A. Contributes tensile strength to tendons
B. Gives consistency to vitreous humour of the eye
C. Has sulphate group
D. Is a lubricant of the synovial fluid of joints
E. Is a potent anticoagulant

6) Natural antioxidants include/s


A. Cycopene
B. Selenium
C. Vitamin C
D. Vitamin E
E. Zinc

7) Regarding competitive inhibitors


A. Angiotensin converting enzyme inhibitors are example for competitive inhibitor
B. Competitive inhibitors decrease apparent Vmax
C. Do not interfere with substrate to bind active site
D. substrate concentration should be raised to achieve Vmax/2
E.

8) Regarding lipids
A. Provide 7Kcal/g calories
B. Insoluble in polar solutions
C. Naturally amphipathic
D. Precursor of water soluble vitamins
E.

9) Regarding glycolysis
A. ATP consumed in the step catalysed by the pyruvate kinase
B. High level of citrate inhibit glycolysis
C. It occurs only in the cytosol of the cell
D. Lactate is an intermediate

Presented by 15th Batch 30 FHCS | EUSL


E. The committed step is catalysed by the hexokinase

10) Regarding RNA A. Deoxyribose is sugar moiety


B. RNA has shorter half life than DNA
C. rRNA is synthesized in nucleolus
D. tRNA consist of genetic information for protein synthesis
E. tRNA is double stranded 11) What is the composition of glutathione
A. Cysteine, Glycine, Glutamic acid
B. Glutamine, Glycine, Cysteine
C. Glutamine, Lysine, Cysteine
D. Glycine, Cysteine, Proline
E. Glycine, Glutamic acid, Proline

12) Pyruvate dehydrogenase requires oxidized (NAD+ ) for enzymatic activity, In the reaction Pyruvate oxidized to
acetyl CoA as NAD+ reduced to NADH2 is functioning as,
A. Holoenzyme
B. Cofactor
C. Coenzyme
D. Apoenzyme
E. Isoenzyme

13) Which from the followings is not a derivative of cholesterol?


A. Bile
B. Lecithin
C. Oestrogen
D. Progesterone
E. Vitamin D

14) which of the following is not diagnostic tool for acute MI?
A. alanine aminotransferase
B. Aspartate aminotransferase
C. Creatine phosphokinase
D. Lactate dehydrogenase
E. Troponin I

15) Which is the following incorrect regarding citric acid cycle?


A. Generate NADH
B. Is the common pathway for all metabolism
C. Isocitrate dehydrogenase is one of the regulating enzyme
D. Occur in mitochondrial matrix
E. Vitamin A act as a coenzyme for regulatory enzymes.

13TH BATCH EME – SEQ

1. 1.1. Classify the amino acids based on


1.1.1 Side chain structure

Presented by 15th Batch 31 FHCS | EUSL


1.1.2 Side chain character
1.1.3 Metabolic fate (30 marks)
.2. Briefly write on molecular technique cloning (30 marks)
1.3. Describe the process of eukaryotic DNA replication (40 marks)
2. 2.1. Briefly describe the role of lipoproteins in the human body (25 marks)
2.2. Outline the regulatory mechanisms of enzyme activity (25 marks)
2.3. Explain the mechanism of oxidative phosphorylation (30 marks)
2.4. Briefly explain the role of uncoupling protein in the electron transport chain (20 marks)

Answers

1
AF
BT
CF
DT
ET
2
AT
BF
CF
DF
ET
3
AF
BT
CF
DT
ET

4
AF
BT
CT
DF
ET
5
AT
BT
CF
DT
EF
6
A F LYCOPENE
BT
CT
DT
ET
7
Presented by 15th Batch 32 FHCS | EUSL
AT
B F UNCHANGED
CF
DT
E
8
AT
BT
CT
DF
E
9
AF
BT
CT
DT
EF
10
AF
B T
CT
DF
E
SBR
11 A
12 C LIPPINCOT PG 110
13 B
14 E
15 A ALT USED FIR LIVER DISEASE DAIGNOSIS

Classify the amino acids based on


1.1.1 Side chain structure
Based on side chain structure amino acids can be classified as follows,
01.Aliphatic a.a
• Mono amino mono carboxylic acids
• Simple a.a
o Glycine
o Alanine
• Branched chain a.a
o Valine
o Leucine
o Isoleucine
• OH group containing a.a
o Serine
o Threonine
• Sulphur containing a.a
o Cysteine
o Methionine
• Amide group containing a.a

Presented by 15th Batch 33 FHCS | EUSL


o Asparagine
o Glutamine
• Mono amino dicarboxylic acid
o Aspartic
o Glutamic
• Di amino mono carboxylic acids •
Lysine
o Arginine
02.Aromatic a.a
Phenylalanine
Tyrosine
03.Heterocyclic a.a
Tryptophan
Histidine
04.Imino acids
Proline
05.Derived a.a
Hydroxy proline
Hydroxy lysine
Ornithine
Citruline
Homocysteine
Arginosuccinic
Beta alanine
GABA
Delta amino levulinic

1.1.2 Side chain character


1. Nonpolar side chains
Glycine
Alanine
Valine
Leucine
Isoleuine
Phenylalanine
Tryptophan
Methionine
Proline
2. Uncharged polar side chains
Serine
Threonine
Tyrosine
Aspargine
Glutamine
Cysteine
3. Acidic side chains
Aspartic acids
4. Basic side chain
Histidine
Lysine

Presented by 15th Batch 34 FHCS | EUSL


Arginine

1.1.3 Metabolic Fate


1. Both glucogenic a.a
Isoleuine
Tyrosine
Phenylalanine
Tryptophan
2. Purely ketogenic
Leucine
Lysine
3. Purely glucogenic
Remaining 14 a.a.

2.see at lipin pgno 484, figure 34.6

1.3.

Importance of DNA replication,


• Provide a (accurate) copy of the genome for the cells in cell division.
• Occurs in the ‘S’ phase of the cell cycle.

Replication Process
• Replication occurs in 3 stages,
1. Initiation
2. Elongation
3. Termination

Initiation
• strands of the parental double helical DNA must be separated (“melt”) because the polymerases use only ssDNA
as a template
• begins at a single, unique nucleotide sequence—a site called the origin of replication (oriC).
• It is a highly conserved A:T rich repeat sequence that facilitate melting
• Two strands open forming Replication Forks (Y-shaped region)
• In eukaryotes, multiple replication sites – to rapidly replicate the great length of the eukaryotic DNA
1. DnaA (Initiation factor) - binds to ‘oriC’ and induces melting of DNA strands.
2. DnaB (Principle Helicase in E-coli) - binds to ssDNA and begins further unwinding of DNA strands – requires ATP.
3. Single-stranded DNA-binding (SSB) proteins (not enzymes) –
• binds and stabilize ssDNA.
• shift the equilibrium between dsDNA and ssDNA in the direction of ssDNA
• protect the DNA from nucleases that degrade ssDNA
4. Primase - binds and synthesizes RNA primer.
5. Topoisomerases - relieve topological strain.
6. DNA pol III - binds and begins to add dNTPs – deoxynucleotide triphosphates, to the primer.
• As the 2 DNA strands open at the origin, Replication Bubbles form
1. Prokaryotes (bacteria) have a single bubble
2. Eukaryotes have MANY bubbles

Elongation
Presented by 15th Batch 35 FHCS | EUSL
• Mainly done by DNA polymerase - III,
❖ Reads the template from 3´ to 5´ end.
❖ Synthesize new DNA strand in 5´ to 3´ end. (Polymerase activity)
❖ Has proofreading activity. (3´ to 5´ exonuclease activity)
❖ Synthesizes DNA from both strands simultaneously
❖ High processivity / high fidelity – remains bound to the template strand as it moves along, and does not diffuse
away and then rebind
❖ Processivity is the result of - β subunit forming a ring that encircles and moves along the template strand of the
DNA, serving as a sliding DNA clamp
❖ High rate of synthesis
❖ Two DNA strands grow in opposite directions.
• Helicase continues to unwind the DNA.
• SSB proteins stabilize the SS-DNA.
• Topoisomerase relieves the strain.
• Replication of one strand is discontinuous, because DNA polymerase activity is in 5’ to 3’ direction only
Leading strand
• Continuous synthesis
• Chain elongation in the same direction as replication fork movement.
Lagging strand
• Synthesized in the opposite direction to the fork movement.
• Limits of DNA polymerase III - can only build onto 3 end of an existing DNA strand
• Discontinuous synthesis.
• Synthesized short fragments – Okazakifragments.

• Each Okazaki fragment has its own RNA

Primer. Energy of Replication

• The nucleotide substrates are 5’-deoxyribo nucleoside triphosphates

• The nucleotides arrive as nucleosides – DNA bases with P–P–P

• DNA bases arrive with their own energy source for bonding

• Pyrophosphate (PPi) is released when each new deoxynucleoside monophosphate is added to the growing chain

• Hydrolysis of PPi to 2Pi break down 2 high-energy bonds release the energy

DNA polymerase – I,

• 5´ to 3´ exonuclease activity – hydrolytically remove the RNA primer in the 5’→3’ direction

• 5´ to 3´ polymerase activity – extends strand up to adjacent Okazaki fragment and replaces RNA primer with DNA,
because new lagging strand consists with RNA-DNA unjoined pieces of RNA-DNA combinations.

• 3´ to 5´ exonuclease activity – proofreading DNA Ligase – “spot welder” enzyme

• Joins adjacent Okazaki fragments.

• Forms the final phosphodiester bond between the 5’-phosphate group on the DNA chain synthesized by DNA
polymerase III and the 3’- OH group on the chain made by DNA polymerase1.

• ATP dependent.

Termination

Presented by 15th Batch 36 FHCS | EUSL


• Replication forks from bi-directional replication run into each other and terminates Or One fork

STOPS and WAITS for the other.

• Events that occur at the terminus that result in separation of daughter stands.

2.2.1

–chylomicrons-

Transport exogenous TG synthesized in intestine, to peripheral tissues (adipose,

Skeletal & Cardiac). Formation fluctuates with load of TG absorbed. Give turbidity after a fatty meal, rapidly
catabolized and cleaved from circulation (1-2 hrs)

VLDL-

Transport endogenous TG synthesized in liver to extra hepatic tissue.

Less affected by dietary TG. Precursor for LDL.

LDL-Forward transport of cholesterol produced in plasma during intra vascular metabolism of VLDL.

HDL-Reverse transport of cholesterol produced in liver and intestine.

2.2.

Enzyme regulation operates at 02 main levels

• Changing the concentration of the enzyme – long term

- Via regulation of gene expression


• Altering the activity of the preexisting enzyme – short term

- via effector molecules (allosteric regulation)

- via covelent modification

Achieved by regulation of gene expression

• Transcription

• Processing of the mRNA (Post-transcription)

• Translation

• Co/Post translational modifications

• Targeted proteolysis

Regulation via gene expression is a long term adaptation.

Regulation by altering the activity of the enzyme

1. Via effector molecules/Allosteric regulation allosteric enzymes – The enzymes that are regulated by effector
molecules

-These are mainly multi- subunit enzyme


Presented by 15th Batch 37 FHCS | EUSL
• regulate the enzyme activity by binding at a site other than the active site of enzyme

• binding site is known as regulatory site

• Binding of an effector molecule to the enzyme creates conformational changes in the enzyme resulting in altered
activity

• It is the principle for fine tuning and control of metabolic activity

• Cause positive or negative influence on the enzyme Positive effectors negative effectors

• When activators bind – the substrate binding to the enzyme is increased

• When inhibitors bind – decrease the binding affinity of the enzyme to substrate

❖ Mediate feedback inhibition and feed forward activation reaction pathways

Regulation by altering the activity of the enzyme

1. Via effector molecules/Allosteric regulation


Allosteric enzymes – The enzymes that are regulated by effector molecules

-These are mainly multi- subunit enzyme

• regulate the enzyme activity by binding at a site other than the active site of the Enzyme

• binding site is known as regulatory site

• Binding of an effector molecule to the enzyme creates conformational changes in the enzyme resulting in altered
activity

• It is the principle for fine tuning and control of metabolic activity

• Cause positive or negative influence on the enzyme

Positive effectors negative effectors

• When activators bind – the substrate binding to the enzyme is increased

• When inhibitors bind – decreasethe binding affinity of the enzyme to substrate

❖ Mediate feedback inhibition and feed forward activation reaction pathways

2. Via phosphorylation
• Phosphorylation of enzymes by specific protein kinases is a widespread mechanism for regulation of enzyme
activity

• It is a reversible means of regulation

• Plays a central role in signal transduction in eukaryotes

• The phosphorylation occurs mainly on Ser/Thr and on Tyr residues

3. Via proteolytic cleavage


• Some enzymes are synthesized in an inactive form called a zymogen or a proenzyme, which is only activated
when the enzyme is needed for use

• Activation by proteolytic cleavage is usually a one-time event – and NOT reversible

• Activation requires one or more key peptide bond(s) to be cut,

• It allows the enzyme’s tertiary structure to rearrange so that the final active state can form.
Presented by 15th Batch 38 FHCS | EUSL
2.3.

As the electrons are transferred, some electron energy is lost with each transfer.

This energy is used to pump protons (H+ ) across the membrane from the Matrix to the inner membrane space. A
proton gradient is established.

The higher negative charge in the matrix attracts the protons (H+ ) back from the

Intermembrane space to the matrix. The accumulation of protons in the intermembrane space drives protons into
the matrix via diffusion.

Most protons move back to the matrix through ATP synthase. ATP synthase is a membrane-bound enzyme that uses
the energy of the proton gradient to synthesize ATP from ADP + Pi. This is called as oxidative phosphorylation of
etc.

2.4.

Coupling with oxidative phosphorylation is a key step for ATP production.

However, in specific cases, uncoupling the two processes may be biologically useful.

The uncoupling protein, thermogenin—present in the inner mitochondrial

Membrane of brown adipose tissue—provides for an alternative flow of protons back to the inner mitochondrial
matrix.

This alternative flow results in thermogenesis rather than ATP production.

Synthetic uncouplers (e.g., 2,4-dinitrophenol) also exist, and, at high doses, are lethal.

This rapid respiration produces heat, and is particularly important as a way of

Maintaining body temperature for hibernating animals, although these proteins may also

Have a more general function in cells’ responses to stress.

14TH BATCH EME – MCQ

01. Glycogen,
A) Can generate energy even in the absence of oxygen.
B) Is a linear homopolysaccharides.
C) Is stored in anhydrous form.
D) Is stored in the endoplasmic reticulum of the cell.
E) Has α (1-4) glycosidic bonds only.

Presented by 15th Batch 39 FHCS | EUSL


02. Hyaluronic acid,
A) Act as a lubricant in the synovial fluid of joints.
B) Contributes to tensile strength of tendons.
C) Gives consistency to vitreous humor.
D) Is sulphated glycosaminoglycan.
E) Is an anticoagulant.

03. Regarding amino acids,


A) Glutamic acids are proton donors.
B) Proline interrupts the alpha helical found in globular proteins.
C) Serine is a basic amino acid.
D) Tryptophan has zero net charge at neutral pH.
E) Valine is a surface protein in soluble proteins.

04. Nonstandard amino acids are,


A) Arginine
B) Beta alanine
C) Hydroxyproline
D) Ornithine
E) Proline.

05. Enzymes accelerate the rate of reaction by,


A) Increasing the collision of molecules.
B) Lowering the number of molecules in higher transition state.
C) Lowering the activation energy with lower transition state
D) Providing energy to substrate
E) Providing more chance for substrate to react together by reducing energy

06. Regarding allosteric inhibition of an enzyme,


A) Binds to the active sites
B) Causes the enzyme to work faster
C) Denaturates the enzyme
D) Participates in feedback regulation
E) Reduces the substrate binding affinity

07. Cholesterol,
A) Mainly exists as cholesterol esters
B) Maintain the fluidity of plasma membrane
C) Is a precursor of peptide hormone
D) Rich in coconut
E) Is transported free in plasma

08. The elongation of leading strand during DNA synthesis,


A) Complementary to an existing template
B) Depends on the action of DNA polymerase
C) Occurs at 3'-5' direction
D) Produces okazaki fragments
E) Progresses towards the replication fork

Presented by 15th Batch 40 FHCS | EUSL


09. Pyruvate dehydrogenase complex,
A) Involves in oxidative decarboxylation
B) Is activated by Acetyl CoA.
C) Is stimulated by insulin.
D) Needs vitamin B12 complex for its action.
E) Produces one NADH molecule.

10. What are the high energy compounds?


A) Acetyl CoA
B) Adenosine mono phosphate
C) Creatinine phosphate
D) Glucose 6 phosphate
E) Phospho Enol Pyruvate

SBR

11. Induce fit theory is introduced by,


A) Buchner
B) Fisher
C) Koshland
D) Kuhne
E) Menten

12. Insulin is composed of,


A) Primary, secondary, tertiary, quaternary
B) Primary, secondary, quaternary
C) Primary
D) Primary, secondary, tertiary
E) Primary, secondary

13. Which of following technique is primarily used to amplify DNA?


A) Microarrays
B) Northern blotting
C) Polymerase chain reaction
D) Restriction fragment length polymorphism
E) Southern blotting

14. Which one of the following do not generate free radicals?


A) Electron transport chain
B) Environmental contaminants
C) Excessive consuming of processed food
D) Consuming large number of fruits and vegetables
E) Ionizing and ultraviolet radiation

15. What is the equation that represents the cellular aerobic respiration?
A) glucose → pyruvate + 2ATP
B) 6CO2 +6O2 +energy → glucose
C) glucose + 6O2 → 6CO2 +6H2O +energy
D) glucose → 2 lactate + 2ATP
E) glucose → 2 alcohol + 2CO2 + 2ATP

Presented by 15th Batch 41 FHCS | EUSL


14TH BATCH EME – SEQ

1. Briefly describe on
1.1. Alpha helix structure of protein (25 marks)
1.2. Cloning (35 marks)
1.3. Enzymes in clinical diagnosis (40 marks)

2. Briefly describe the fatty acids in relation to the


2.1
2.1.1. General features
2.1.2. Classification
2.1.3. Functions (35 marks)
2.2 Briefly explain the importance of lecithin in the human body. (20 marks)
2.3 Outline the chemiosmosis theory of electron transport chain. (25 marks)
2.4 List five functions of glycoproteins in the human body with examples. (20 marks)

15TH BATCH EME – MCQ

1) What is/are aldoses?


A. Fructose
B. Galactose
C. Glucose
D. Glyceraldehyde
E. Ribulose

2) The active site of the enzyme,


A. Binds covalently with the substrate
B. Contains polypeptide chains
C. Is small relative to the total bulk of the enzyme.
D. Is a restricted region
E. Formed by folding of protein

3) Phosphatidyl choline (Lecithin) is mainly needed for,


A. Work as second messenger in cells
B. Formation of Lipoprotein
C. It makes lung surfactant
D. Structural component of cellular membrane
E. Inducing inflammatory response

4) Pyruvate convert into Acetyl CoA inhibit by,


A. Acetyl CoA
B. AMP
C. Ca2+
D. Fatty acids
E. NAD+

Presented by 15th Batch 42 FHCS | EUSL


5) A 28-year-old man urinalysis results shown an increase in neutral amino acids and he is diagnosed with Hartnup
disease. Which of the following amino acids would have been found in the urine sample?
A. Lysine
B. Phenylalanine
C. Arginine
D. Histidine
E. Serine

6) Denaturation of protein,
A. Change primary structure
B. Decrease viscosity
C. Usually irreversible
D. Loses biological activity
E. Facilitate the digestion

7) The active site of the enzyme,


A. Binds covalently with the substrate
B. Contains polypeptide chains
C. Is small relative to the total bulk of the enzyme
D. Is a restricted region
E. Formed by folding of protein

8) Isozymes differ in,


A. Km
B. Amino acid sequence
C. Tissue location
D. Electrophoretic mobility
E. Catalyzing different reactions

9) Thymidine,
A. Is equal to the adenosine in double stranded DNA
B. Is a pyrimidine
C. Replaced by uracil in RNA
D. Made 3 hydrogen bonds with Guanine
E. Participate in hydrophobic interactions due to its methyl group

10) Polymerase chain reaction is,


A. Is an in vivo DNA amplification procedure
B. Is designed to amplify entire DNA at once
C. Is catalyzed by DNA polymerase
D. A pair of DNA primers needed to the initiation
E. Requires DNA ligase to line two DNA Okazaki fragments

SBR
11) which is following enzyme that require NAD+ as cofactor?
A. alpha ketoglutarate dehydrogenase
B. citrate synthase
C. fumarase
D. succinate dehydrogenase
E. succinyl CoA synthetase

Presented by 15th Batch 43 FHCS | EUSL


12) Lipoprotein core is formed by?
A. Triacylglycerol
B. TAG and Cholesterol esters
C. Triacylglycerol, cholesterol, phospholipids
D. Free fatty acid
E. Phospholipid, lipoprotein

13) Which is the amino acid important for buffering action of protein at physiological pH,
A. Tyrosine
B. Histidine
C. Lysine
D. Arginine
E. Alanine

14) The anticodon is a structure of,


A. mRNA
B. tRNA
C. Ribosome
D. rRNA
E. hnRNA

15) In enzyme kinetics, Vmax denotes,


A. The amount of an active enzyme
B. Substrate concentration
C. Substrate concentration that produces half the maximum velocity of time
D. Quantity of enzyme substrate complex
E. Dissociated constant Vmax

15TH BATCH EME – SEQ

1.
1.1 Briefly describe the importance of homopolysaccharides with examples. (30 marks)
1.2 State the functions of cholesterol with examples. (25 marks)
1.3 Cyanide is one of the most rapidly acting poisons in human.
Briefly explain, what effect cyanide would have on ATP synthesis. (25 marks)
1.4 Briefly describe on Alpha helix structure of a protein (20 marks)

2.
2.1. Write short notes on
2.1.1. DNA replication in prokaryotes (35 marks)
2.1.2. Restriction enzyme (30 marks)

2.2.
2.2.1. Draw the Michaelis-Menton and Lineweaver-Burk plots (20 marks)
2.1.2. Explain the significance of Michaelis-Menton constants (15 marks)

answers
Presented by 15th Batch 44 FHCS | EUSL
1. A-False
B-True
C-True
D -True
E-False

2. A-False
Substrates are bound to enzymes by multiple weak attractions.
(Binding is thought to cause a conformational change in the enzyme (induced fit model) that allows a rapid
conversion of the ES to enzyme–product (EP) complex that subsequently dissociates to free enzyme and product.)
B-True
Active Site is formed from Amino acids sequences in the polypeptide chain (3-5)
C-True
D-True
Active site is a restricted region (3D) or special pocket or cleft of an enzyme molecule which binds to the
substrate
E-True
Enzyme molecules contain a special pocket or cleft called the active site which is formed by folding of the
protein.

3. A-False
Phosphotidyl inositols act as second messengers
B-True
C-True
D-True
E-(False)

Presented by 15th Batch 45 FHCS | EUSL


Platelet activating factor triggers potent thrombotic and acute inflammatory events

4. A-True
B-False
AMP activate PDH to produce more acetyl coA for krebs cycle
C-False
Calcium (Ca2+ ) is a strong activator of PDH phosphatase, stimulating E1 activity. This is particularly important
in skeletal muscle, where Ca2+ release during contraction stimulates the PDHC and, thus, energy production.
D-True
The acetyl CoA from FA oxidation indirectly inhibits PDH [by activation of PDH kinase]
E-False
NAD is a co substrate of PDH

5. Non-polar side chains (hydrophobic) Ex: ala, val, leu, Ile , met, Phenyl, Try, Pro, Gly
Polar (hydrophilic) Uncharged side chains: Ser, Thr, Tyr, Asn, Gln, Cys
Charged Side Chains
Acidic side chains: Glu, Asp
Basic side chains: lys, arg, His

A-False
B-True
C-False
D-False
E-True
Presented by 15th Batch 46 FHCS | EUSL
6. A-False
Denaturation results in the loss of secondary, tertiary and quaternary structure of proteins.
The primary structure of a protein with peptide linkages remains intact i.e., peptide bonds are not hydrolysed.
B-False
The viscosity of denatured protein (solution) increases (while its surface tension decreases.)
C-True
Denaturation is usually irreversible.
Careful denaturation is sometimes reversible (known as renaturation).
lrreversible denaturation results in coagulation.
D-True
E-True
Denatured protein is more easily digested .This is due to increased exposure of peptide

7. -Question 2 repeated-

8.
A-True
B-True
C-True
D-True
E-False

9. The addition of a pentose sugar to a base through an N-glycosidic bond (see p. 94) produces a nucleoside. If the
sugar is ribose, a ribonucleoside is produced, and if the sugar is 2-deoxyribose, a deoxyribonucleoside is produced
Ribonucleosides of A, G, C, U are adenosine, guanosine, cytidine, uridine
Deoxyribonucleosides of A, G, C, T have the prefix deoxy- added. (Deoxyadenosine,....)

Thymidine+phosphate=Thymine

Presented by 15th Batch 47 FHCS | EUSL


A-True.....(if the amount is concerned)
The specific base-pairing in DNA leads to the Chargaff rule, which states that in any sample of dsDNA, the amount
of A equals the amount of T, the amount of G equals the amount of C, and the total amount of purines [A + G]
equals the total amount of pyrimidines [T + C].
B-True
C-False
Uridine is the equivalent of thymidine. While thymine is replaced by uracil
D-False
A binds with T by 2H bonds, C binds with G by 3 H bonds
E-
Thymine has a methyl group

10.

A-False(in vitro)
B-False(selected DNA)
C-True
D-True
E-False

Presented by 15th Batch 48 FHCS | EUSL


SBR

11. A

12. B

Presented by 15th Batch 49 FHCS | EUSL


13. B
Histidine is the only amino acid with a side chain that can ionize within the physiologic pH range (7.35 to 7.45).

14. B

15.

Presented by 15th Batch 50 FHCS | EUSL


SEQ

1.1 Briefly describe the importance of homo polysaccharides with examples. (30 marks)

Polysaccharides (or simply glycans) consist of repeat units of monosaccharides or their derivatives, held together by
glycosidic bonds. They are linear as well as branched polymers. Polysaccharides are of two types. They are homo
polysaccharides and hetero polysaccharides.

Homopolysaccharides on hydrolysis yield only a single type of monosaccharide. Examples for homopolysaccharides
are starch, glycogen, cellulose, inulin, dextrin, mannan, galactan

Starch is the carbohydrate reserve of plants which is the most important dietary source for higher animals,
including man. It is a homopolymer composed of D-glucose units held by E-glycosidic bonds.

Glycogen is the carbohydrate reserve in animals, hence often referred to as animal starch. It is present in high
concentration in liver, followed by muscle, brain etc. Stored glycogen serves as a fuel reserve. Because glycogen can
be rapidly metabolized, can generate energy in absence of oxygen, brain depends on continuous glucose supply.

Cellulose occurs exclusively in plants and it is the most abundant organic substance in plant kingdom. It is a
predominant constituent of plant cell wall. Cellulose is totally absent in animal body. Cellulose is composed of B-D-
glucose units linked by B (1-4) glycosidic bonds. Cellulose cannot be digested by mammals including man due to lack
of the enzyme that cleaves Beta glycosidic bonds. Cellulose, though not digested, has great importance in human
nutrition. It is a major constituent of fiber, the non-digestible carbohydrate. The functions of dietary fiber include
decreasing the absorption of glucose and cholesterol from the intestine, besides increasing the bulk of feces.

Inulin is a polymer of fructose. Inulin is not utilized by the body. It is used for assessing kidney function through
measurement of glomerular filtration rate (GFR).

1.2 State the functions of cholesterol with examples. (25 marks)

Cholesterol is a steroid alcohol of animal tissues. It is widely distributed in all cells of body. There are many
functions of cholesterol.

- It serves as a component of membranes of cells. Cholesterol in the membrane increases or moderates


membrane fluidity.

- It serves as a component of plasma lipoprotein.

- It is precursor of synthesis of
~ steroid hormones: progesterone, estrogen, androgen, glucocorticoid, mineralocorticoids
~ bile acids and bile salts
~ vitamin D

Presented by 15th Batch 51 FHCS | EUSL


- Regulates cell signaling

- Nerve conduction & important or brain development & function

- Intracellular transport (endocytosis)

1.3 Cyanide is one of the most rapidly acting poisons in human.


Briefly explain, what effect cyanide would have on ATP synthesis. (25 marks)

Cyanide is a respiratory chain inhibitor which block respiration’s in the presence of either ADP or uncouplers. It
blocks the electron flow in complex IV. Cyanide ion combines tightly with Fe3+ of cytochrome oxidase. In complex
lV 2 protons pumped from matrix to intermembrane space and water is formed. Since this is is inhibited the
consequent action of ATP synthase will be interrupted, leading to inhibition of electron transport chain. Utilization
of O2 is stopped in cyanide poisoning.

The synthesis of ATP (phosphorylation) is dependent on electron transport. Hence, all the site-specific inhibitors of
ETC also inhibit ATP formation.

1.4 Briefly describe on Alpha helix structure of a protein (20 marks)

There are 2 dimensional secondary structures of proteins. Those are Alpha helix & Beta sheet. The most common
type is the Alpha helix

Amino acids in an Alpha helix are arranged in a right handed helical structure. The polypeptide bonds form the
back-bone and the side chains of amino acids extend outward from the central axis.

Stabilised by H bond formed between H atom attached to peptide N , & O atom attached to peptide C. Inter chain
hydrogen bond is parallel to the peptide backbone.

Each turn is formed by 3.6 residues. The distance between each amino acid residue is 1.5 Å.

. E.g. Keratin(hair & skin) and myoglobin

Amino acids with bulky side chains less common in alpha helix. Glycine & Proline destabilizes alpha helix.

2.1. Write short notes on

2.1.1. DNA replication in prokaryotes (35 marks)

• DNA replication is semi conservative

• Replication occurs in 3 stages


1. Initiation
Presented by 15th Batch 52 FHCS | EUSL
2. Elongation
3. Termination

• DNA synthesis begins at specific nucleotide sequences, called replication origins, and proceeds in both directions
(i.e., bidirectional action).

a. Replication is semiconservative; that is, each new DNA double helix contains one parental (old copy) strand and
one daughter (new copy) strand.

b. DNA is exposed to replication enzymes by action of DNA gyrase (topoisomerase II); DNA is relaxed with negative
supercoils and unwound.

• Multiple proteins participate in initiation and elongation of daughter DNA strands at the replication fork.

a. Helicase separates the two strands of the relaxed DNA helix, and single strand–binding proteins attach to each
strand; the point of separation is the replication fork.

b. Helicase unwinding causes a physical strain (positive supercoiling) immediately ahead of the moving replication
fork; topoisomerase I relieves this strain by breaking one of the DNA strands and allowing it to rotate around the
unbroken strand before rejoining the broken ends.

c. DNA polymerases catalyze the addition of 5’-deoxyribonucleoside triphosphates to the free 3’-OH end of the
growing strands with a loss of pyrophosphate (PPi ); this always leaves a free 3’-hydroxy group to accept the next
addition.

(1) The parental (template) strand is always read in the 3’—5’ direction.

(2) The daughter (new) strand is always synthesized in the 5’—3’ direction.

(3) Because of the directionality of DNA synthesis, one daughter strand is synthesized continuously (i.e., leading
strand), and the other is synthesized discontinuously (i.e., lagging strand)

• Different processes are involved in the synthesis of leading and lagging strands.

a. Leading-strand synthesis, catalyzed primarily by DNA polymerase d, proceeds continuously from a single RNA
primer synthesized by DNA polymerase a.

b. Lagging-strand synthesis proceeds discontinuously from multiple RNA primers synthesized by DNA polymerase a,
forming short, 1000 base pair Okazaki fragments.

(1) The Okazaki fragments are extended by DNA polymerase , as in leading-strand elongation.

(2) Primer is removed from the fragments by a 5’—3’ exonuclease, and gaps are filled by DNA polymerase, which
also functions in repair.

DNA polymerases always synthesize daughter strands in the 5’—3’ direction.

Creates fragmented, lagging strand; Okazaki fragments linked together with DNA ligase

(3) DNA ligase connects the ends of the adjoining fragments.

Presented by 15th Batch 53 FHCS | EUSL


• The proofreading activity of DNA polymerase usually detects and corrects base-pairing errors that occur during
DNA synthesis (e.g., formation of an A-C pair rather than the correct A-T pair).

a. The 3’—5’ exonuclease activity of the DNA polymerases removes a mismatched nucleotide, and synthesis then
continues with insertion of the correct nucleotide

2.1.2. Restriction enzyme (30 marks)

One of the major molecular analysis of genomic DNA is the immense size of molecules involved. The discovery of a
special group of bacterial enzymes, called restriction endonucleases (restriction enzymes), which cleave double-
stranded DNA (dsDNA) into smaller, more manageable fragments, opened the way for DNA analysis.

Each enzyme cleaves dsDNA at a specific nucleotide sequence (restriction site) and produce restriction fragments.

These enzymes are naturally found in bacteria to protect bacteria from invasion of foreign DNA.

Restriction enzymes recognizes short stretch of DNA( 4-8 bp) that contain s nucleotide sequences. These
recognition sites are called Palindromes. Palindromes exhibit twofold rational symmetry.

These enzymes are named according to the organism from which it was isolated.
Example: EcoRI, Haelll

They may produce either cohesive ends or blind ends after the cleavage.

Presented by 15th Batch 54 FHCS | EUSL


Restriction endonucleases cleave dsDNA into fragments of different sizes (restriction fragments) depending upon
the size of the restriction site sequence.

2.2.

2.2.1. Draw the Michaelis-Menton and Lineweaver-Burk plots (20 marks)

2.1.2. Explain the significance of Michaelis-Menton constants (15 marks)

Michelis constant is the substrate concentration that produces half maximum velocity of enzyme.

Enzymes with low Km: have high affinity to the substrate i.e. they act at maximal velocity at low substrate
concentration
E.g. Hexokinase acts on glucose at low concentration (fasting state)

Enzymes with high Km: they have low affinity to the substrate i.e. they act at maximal velocity at high substrate
Concentration
E.g. Glucokinase enzyme acts on glucose at high concentration (fed state)

16TH BATCH EME – MCQ

01. Regarding starch


A. Amylopectin makes around 70-80% of starch.
B. Increase diet of refined starch increase the risk of diabetes
C. Polymers of fructose has 1-4 glycosidic bond

Presented by 15th Batch 55 FHCS | EUSL


D. This is high in leafy vegetable
E. They show positive results for Molish’s test

02. Match the specific inhibitor for electron transport chain complex
A. ATP synthase- Antimycin
B. Cytochrome C oxidase- cyanide
C. Cytochrome C reductase- Oligomycin
D. NADH dehydrogenase- Rotenone
E. Succinate dehydrogenase - Carbon monoxide

03. Regarding lipid


A. Arachidonic acid is a derivative of linolenic acid
B. Lauric acid predominant in coconut oil
C. Aspirin inhibits the cyclooxygenase pathway
D. Oleic acid is an omega-3 fatty acid
E. Phosphatidyl choline is component of the cellular membrane

04. Regarding glycoproteins


A. Albumin is protective glycoprotein
B. Lysosomal acid hydrolases are responsible for the degradation of
glycoproteins
C. The glycan attached to the protein only through O- link glycosidic bond
D. Act as a cell surface antigenicy
E. They are proteins to which small oligosaccharide chains are attached

05. The stability of primary structure of protein is supported by


A. Hydrogen bond
B. Peptide bond
C. Disulfide bond
D. Ionic bond
E. Ester bond

06. Regarding peptide bonds in proteins,


A. It has partial double bond character
B. Ionized at physiological PH
C. Cleaved by the agent that denature protein
D. Stable in heat with strong acids
E. Occurs mainly in trans configuration

07. The active site of an enzyme


A. Covalently binds with substrate
B. Contain polypeptide chain
C. Is small relative to the total bulk of enzyme
D. Is a restricted region
E. Formed by folding of proteins

08. True/ False


A. DNA Double strands are antiparallel
B. Messenger RNA provide blue print for the protein synthesis
C. Messenger RNA has anticodon

Presented by 15th Batch 56 FHCS | EUSL


D. Transfer RNA has codon
E. RNA mostly found in cytoplasm

09. Which of the following is a purine?


A. Uracil
B. Cytosine
C. Thymine
D. Adenine
E. Adenosine

10. Regarding Restriction endonuclease enzyme,


A. Cut the nucleotide in a special region
B. Are named according to bacteria from which organism is isolated
C. Recognize the DNA sequence in palindromic nature
D. Cut the DNA randomly
E. Found in human body

SBR

11. Precursor of eicosanoids


A. C18-C20 saturated fatty acid
B. C18-C20 polyunsaturated fatty acid
C. C20-C22 saturated fatty acid
D. C20-C22 polyunsaturated fatty acid
E. C20-C22 saturated fatty acid

12. Examples for epimers


A. Glucose & Ribose
B. Glucose & Galactose
C. Galactose, mannose & glucose
D. Glucose, Ribose & Mannose
E. Glucose & fructose

13. What is the Krebs’s cycle enzyme found in inter membrane space?
A. Citrate synthase
B. Malate dehydrogenase
C. Iso-citrate dehydrogenase
D. Succinate dehydrogenase
E. α- ketoglutarate dehydrogenase

14. Production and secretion of trypsin is inhibited in pancreas disease. Hydrolysis which compound?
A. Proteins
B. Lipids
C. Carbohydrate
D. Nucleic acid
E. Phospholipid

15. Enzyme that helps to join Okazaki fragments


A. DNA polymerase
B. Ligase

Presented by 15th Batch 57 FHCS | EUSL


C. RNA Polymerase
D. Peptidyl transferase
E. Topoisomerase

16TH BATCH EME – MCQ

1.
1.1. Outline the mechanism of oxidative phosphorylation (25 marks)
1.2. State the function of different types of lipoproteins (15 marks)
1.3. List three (03) consequences occurred due to deficiency of disaccharidases enzymes in human (10 marks)
1.4. List the functions of any 5 enzymes or protein required for DNA replication (15 marks)
1.5. Briefly describe the beta pleated sheet structure of protein (20 marks)
1.6. Mention five (05) enzymes used as therapeutic agent (15 marks)

Presented by 15th Batch 58 FHCS | EUSL


PHASE 01 QUESTIONS

8TH BATCH
8TH BATCH PROPER MCQ

2) Regarding amino acid


A) Not absorb visual light
B) Glycine has D & L isomers
C) Serine does not have phenolic –OH
D) It bears not net charge at its isoelectrical PH
E) R group determines unique biochemical function

3) Trytophanyl - tRNA synthatase will charge a tRNA leaving the correct anti-codon with L- tryptophan, but no D-
tryptophan or any other amino acid in protein biosynthesis. Which of the following enzymatic characteristic is seen
in above example?
A) Catalytic power
B) Specificity
C) Transformation of energy
D) Regulation
E) Competitive inhibition

4) Glycosaminoglycan are,
A) Heteropolysaccharides
B) Associated with proteins
C) Branched molecule
D) Having the ability to bind large amount of water
E) Are found in connective tissue

5) About competitive inhibitor


A) Similar to structural analogue of the substrate,
B) Increase Vmax
C) Increase Kmax
D) Alter the shape of the enzyme
E) Competes with the substrate

6) Prostaglandins
A) Supports gastric suppression
B) Elevate body temperature
C) Promote cervical dilatation
D) Are involved in inflammatory response
E) Increase blood pressure

7) Regarding RNA
A) tRNA comprise 5% of total RNA
B) mRNA is the smallest RNA
C) tRNA transfers amino acid to a growing polypeptide chain
D) tRNA is the abundant RNA
E) rRNA carries genetic information from DNA

8) Regarding Okazaki fragments


Presented by 15th Batch 59 FHCS | EUSL
A) Contain 100 nucleotides
B)
C) Allow replisome to move outer end
D) Bound with DNA polymerase
E) Leading to double strand formation

49) Genetic information is coded in DNA by


A) Sequence of nucleotides
B) Pentose sugar
C) Regular alteration of sugar & phosphate molecules
D) Structure of proteins
E) Pattern of coiling of double helix

8TH BATCH PROPER SEQ

1.0
. 1.1. Outline the basic structure of DNA & list the characteristics of the WatsonCrick model (45 marks)
1.2. 1.2.1. Define secondary, tertiary & quaternary structure of proteins (15 marks)
1.2.2. With the aid of a diagram, illustrate the main types of secondary structure. (20 marks)
1.3. Explain why maltose positively answers the benedicts test but sucrose does not. (20 marks)

8TH BATCH REPEAT SEQ

1)Write short notes on


a. Secondary structure of a protein (25 marks)
b. Lipoprotiens (25 marks)
c. Polymerace chain reaction and its application (25 marks)
d. Glycoseaminoglycans (25 mark)

Answers
2)
A) T All aromatic amino acids absorb UV rays.
B) F Glycine has 2 H atoms attached to the chiral C, therefore no optical activity and no D and L isomers.
C) T Lippincott pg. 3 – fig. 1.3
D) T Isoelectric pH; the pH at which the no. of negative charges on a population of molecules is equal; no net
charge.
E) T

3)
A) T
B) T
C)
D)
E) F

4)
A) T Heteropolysaccharide made up of the repeating unit [amino sugar + acidic sugar]
Presented by 15th Batch 60 FHCS | EUSL
B) T All GAGs except hyaluronic acid are found covalently attached to protein, forming proteoglycan monomers.
C) F Unbranched
D) T Binds large amounts of water producing the gel like matrix that forms the basis of the body's ground
substance.
E) T

5)
A) T Compete with substrate molecules for the active site.
B) F Vmax - unchanged
C) T Km - increased
D) F Competitive inhibitors compete with substrate molecules foe the active site.
Allosteric inhibitors bind irreversibly or reversibly to the enzyme not at the active site, they function by altering the
shape of the active site.
E) T

6)
A) T PGE2 - Reduce secretion of HCl from stomach
B) T PGE2 – Fever response
C) T PGE2/PGF2 – induce labour
D) T PGE2/Leukotriene
E) T PGF2/Thromboxane A2 - vasoconstriction

7)
A) F rRNA – 85% tRNA – 15% mRNA – 5%
B) F tRNA
C) T
D) F rRNA
E) F mRNA

8)
A) T Okazaki fragments; short sequences of DNA nucleotides containing approx. 150-200 base pairs.
B)
C) F Done by topoisomerase.
D) T
E) T

49)Answer – A

8TH BATCH PROPER SEQ - Answers

1.

1.1.
● There are 4 bases as adenine,guanine,cytosine and thymine.
● Adenine and guanine are purines which contain 2 cycles.
● Cytosine and thymine are pyrimidines which contain only 1 cycle.
● Deoxyribo nucleotide consist of purine or pyrimidine base,phosphate group and pentose sugar.

Presented by 15th Batch 61 FHCS | EUSL


● 3 prime 5 prime phosphodiester bonds link nucleotides together to form polynucleotide chains.
● 2 strands of polynucleotide of DNA run in opposite directions(antiparallel).
● The 2 strands are joined by bases.
● Each base is paired with a specific partner.
● Adenine is paired with thymine by 2 hydrogen bonds.
● Guanine is paired with cytosine by 3 hydrogen bonds.
● The 2 strands are complementary but not identical.
● The deoxyribose sugar and phosphate group make the hydrophilic backbone of the DNA outside.
● Base pairs and H bonds stacked one above other inside the 2 strands.
● 2 strands of DNA rotate around the same axis to form a right handed double helix.
● Double helix contain 2 grooves called major groove and minor groove.

1.2.
1.2.1.
● Secondary structures are ordered 2 dimensional structures formed due to H binding between peptide
groups which occurs between H of amino group and O of the carbonyl groups.
● These H bonds involve only the peptide backbone, not the side chains.
● Classified into 2 classes as alpha helix and beta sheet.
● Tertiary structure is a 3 dimensional folding of a protein due to interactions of amino acid side chains.
● A globular structure.
● Interactions that stabilize tertiary structure are ionic bonds,van der waals,hydrophobic interactions ,and
disulfide bonds.
● Quaternary structure is the association of 2 or more polypeptides into one functional protein.
● Sub units are held together primarily by noncovalent interactions such as H bonds , ionic bonds and
hydrophobic interactions.

1.2.2.

a)α-helix
b)β-sheets
c)β-Bends
d)Loops & Coils
e)Supersecondary structures (motifs)

Presented by 15th Batch 62 FHCS | EUSL


1.3.
● Carbohydrates with hemiacetal groups have the ability to reduce solutions of various metallic ions (Known
as reducing sugars)
● When reducing sugars are heated in the presence of alkali, they get converted into powerful reducing
species known as enediols (An enediol is a compound with
● a hydroxyl group attached to both carbon atoms of the carbon double bond )
● Enediols reduce Cu2+ to Cu+
● CU+ ions combine with OH- ions to form yellow CU2O which upon heating is converted into brick red
precipitate.
● Therefore as a reducing sugar maltose positively answers Benedict’s test
● However sucrose is a non-reducing sugar( The two monosaccharide units are held together by a glycosidic
linkage between C1 of α-glucose and C2 of β-fructose.
● No hemiacetal groups present in this disaccharide), since it does not have free aldehyde or ketone group to
cause reduction, hence it gives a negative reaction with Benedict's reagent.

8TH BATCH REPEAT SEQ - Answers

1.

1.1.

Regular folding and twisting of the polypeptide chain by hydrogen bonds.
●3 important kind of secondary structure
Alpha – helix
Beta – pleated sheet
Beta- bend
Alpha – helix
● Is the most common
● Spiral structure
● Consist tightly packed coiled polypeptide backbone core.
● Side chains of the amino acid extend outward from the central axis.
Ex ; keratine , myoglobin
Beta – pleated sheet
● Surface appear pleated
● Contain two or more polypeptide chains or segments of polypeptide chain
● Hydrogen bonds are perpendicular to polypeptide backbone.
● Polypeptide chains arranged in parrellel and anti-parrellel .
B –bend
● Most of proline compound in b –bend.

1.2.
● Complex of lipids and specific protein.
● Consist of chylomicron
VLDL

Presented by 15th Batch 63 FHCS | EUSL


LDL
HDL
● Differ in protein and lipid composition, size ,density, site
of origin
● Composition neutral lipid core – triglyceride , cholesterol
esters , hydrophobic layer .
● Amphipathic apolipoprotein , phospholipid ,free
cholestesterol are in periphery. hydrophilic layer
● Contains 5 types of apolipoproteins
● It provide recognition sites for cell surface
● As an activator or coenzyme for lipoprotein metabolism.
● It required as essential structural component of particles.

Chylomicrone
● TAG (90%) >Cholesterol/ cholesterol ester > phospholipid
>protein
● Need APO B -48 , APO C 2, APO E
VLDL
● TAG (60%) > Cholesterol /cholesterol ester> phospholipid
> protein
LDL
● Cholesterol/ cholesterol ester > phospholipid > protein > TAG
HDL
● Protein > phospholipid > cholesterol /cholesterol esters >
TAG
● Need APO A – 1 C – 2 and E.

● Size of lipoprotein
Chylomicrone> VLDL>LDL>HDL

● Density
HDL > LDL > VLDL > chylomicrone

● Electrophoretic mobility of plasma lipoprotein


- Chylomicrone < LDL <VLDL < HDL

● Chylomicrone deliver dietary TAG to peripheral tissues VLDL deliver endogenous TAG to peripheral tissue.
● LDL deliver cholesterol to peripheral tissues on back to
liver
● Increase risk for coronary heart disease.
● HDL deliver cholesterol from peripheral tissue to liver Good type
● Decrease risk for heart disease.

1.3.
● Test tube method
● For amplifying a selected DNA sequence
● Synthesize million of copies of specific nucleotide sequence in few hours.

Presented by 15th Batch 64 FHCS | EUSL


● Steps
Primer construction
Denature of DNA – heat to 95 c to separate double
stranded DNA
Bind primers to single stranded DNA – cool to 55 c
Chain extension
Elongate the chain using DNA polymerase 3
Elevate temperature to 72 c
Repeat this cycle more time
● Material needed for PCR – target DNA , free nucleotide,primer (DNA primer ) ,heat stable DNA polymerase
,Mg2 ( cofactor) ,buffer ,thermo cycler machine
● Applications
For rapid prenatal diagnosis and carrier testing of several
inherited disorders.
Tissue typing
Genetic finger printing
DNA cloning
Genetic mapping
DNA sequencing.

1.4.
● Long, un branch ,negatively charged ,heteropolysaccharide
● Composed of repeating disaccharide unit
● Acidic and amino sugar
● But keratan sulphate not contain acidic sugar, contain galactose
● Large complex
● All GAGs have sulphate group except hyaluronic acid
● Has special ability o bind large amount of water
● GAGs are associate with large amount of protein except hyaluronic acid
● Types of GAGs , 6 types

hyaluronic acid - synovial fluid , vitreous humor of eye , loose connective tissue , cartilage
Lubricant , shock absorber
heparin – present in mast cells, anticoagulant, present sulphate large amounts .
heparin sulphate – extracellular GAG, present in basement membrane.
Dermatan sulphate – Skin , blood vessel , heart valve,Need for coagulation
chondroitin sulphate – cartilage , tendon ,ligament, arota Most abundant GAG in the body
Forms proteoglycans aggregates with hyaluronic acid
keratan sulphate – cornea , loose connective tissue,most heterogeneous GAG
Absorb mechanical shock

● Functions of GAGs
Binding and packing of tissue
Connect, anchor , support the body and its organ
Transport of metabolites between capillary and tissue
Defence against infection
Repair in injury

Presented by 15th Batch 65 FHCS | EUSL


9TH BATCH

9TH BATCH PROPER MCQ

2) Hyaluronic acid
A) Linear polymer
B) Moisturizer
C) Found in vitreous humor of eye
D) Sulphated
E) Backbone of proteoglycan

3) Regarding plasma membrane of human cells


A) Major component of plasma membrane is lipid
B) All membrane carbohydrate is covalently attached to proteins or lipids
C) Cholesterol helps to regulate fluidity of the membrane
D) Integral membrane proteins act as receptors for hormones and neurotransmitters
E) Saturated and unsaturated fatty acids are present in the membrane

4) Regarding lipids
A) Sphingomyaline contains a phospholipids
B) Cholesterol is precursor of bile salt and vitamin D
C) Phospholipids contains nitrogenous residues
D) All fatty acids synthesized in body saturated
E) Lipid can be tested by Seliwanoff’s test

5) Essential amino acids include


A) Cysteine
B) Glutamate
C) Glycine
D) Methionine
E) Serine

6) The compounds derived from 20 carbon polyunsaturated fatty acids are


A) Docosahexaenoic acid
B) Eicosapentaenoic acid
C) Leukotrienes
D) Linoleic acid
E) Thromboxane

7) RNA
A) Has short half-life than DNA
B) Not contain bind with base pairs
C) Has different purine base that DNA has
D) Functions as catalytic enzymes in same form
E) Purine: pyrimidine is not similar
Presented by 15th Batch 66 FHCS | EUSL
8) Chromatin
A) Has double helix DNA
B) Has basic proteins
C) Has acidic proteins
D) Has small amount of proteins
E) Is the basic unit of nucleosome

9) Polymerase chain action


A) Is an invitro DNA amplification procedure
B) Is design to amplify entire DNA at one
C) Catalyzed by DNA polymerase
D) Need a pair of DNA primases to initiate the reaction
E) Requires DNA ligase to link 2 DNA fragments

10) Plasmids are


A) Single strand DNA
B) Smaller than the chromosomal DNA
C) Self-replicating DNA
D) Presence multiple copies in DNA
E) Circular DNA

11) Isoenzymes differ in


A) Km
B) Amino acid sequence
C) The types of reaction catalyzed
D) Tissue location
E) Electrophorectic mobility

SBR
41) Molish’s test is done to identify
A) Cryohydrates
B) Proteins
C) Phospholipids
D) Bile salts
E) Bilirubin

42) Heparin is a
A) Carbohydrate
B) Proteoglycan
C) Glycosaminoglycan
D) Lipoprotein
E) Glycolipid

43) Enzyme
A) Activity increases with temperature
B) Increases activation energy
C) Active site is formed by assembly of protein domain
D) Changes the equilibrium
E) Activity is measured during maximal enzymatic reaction

Presented by 15th Batch 67 FHCS | EUSL


48) A peptide bond
A) Has a partial double-bond character
B) Is ionized at physiologic PH
C) Is cleaved by agents that denature proteins, such as organic solvents and high concentrations of urea
D) Is stable to heating in strong acids
E) Occurs most commonly in the cis configuration

49) ADH requires NAD+ for catalytic activity. In the reaction catalyzed by ADH, an alcohol is oxidized to an aldehyde
as NAD+ is reduced to NADH and dissociates from the enzyme. The NAD+ is functioning as a (an)
A) Apo enzyme
B) coenzyme-co substrate
C) coenzyme-prosthetic group
D) Cofactor
E) Heterotropic effector

9TH BATCH PROPER SEQ

(01)
1.1 Briefly describe the α helix secondary structure of proteins (15 marks)
1.2 State the different types of inhibition (10 marks)
1.3 Describe competitive enzyme inhibition kinetics and other features (30 marks)
1.4 Give 2 example of competitive enzyme inhibition in usefulness in therapeutics (20 marks)

(02).Write short notes on,


2.1.Okaski fragment (30 mark)
2.2.Structural polysaccharides (25 mark)
2.3.Translation of protein synthesis (35 mark)

9TH BATCH REPEAT SEQ

01.
1.1.Briefly explain how the constituent molecules of the plasma membrane contribute to fluidity. (35marks)
1.2. Differentiate between starch and glycogen on the basis of structure and function. (30 marks)
1.3. Describe how DNA is packaged in the chromosomes with necessary diagrams. (35 marks)

02
2.1
a) Outline the levels of organization of protein structures. (25 marks)
b) Describe the bonds that stabilize each level of organization. (25 marks)
2.2
a) Define isoenzymes. (10 marks)
b) Briefly describe the diagnostic significance of the following isoenzymes. (2x20=40 marks)
i) Lactate dehydrogenase (LDH)
ii) Creatine phosphokinase (CPK)

Presented by 15th Batch 68 FHCS | EUSL


Answers

9TH BATCH PROPER MCQ - Answers

2)
A) T Long, unbranched polysaccharide
B) T Serve as lubricant and shock absorber.
C) T Synovial fluid in joints, vitreous humor of the eye, umbilical cord, loose connective tissue and
cartilage
D) F Unsulphated
E) T Lippincott pg. 160 – fig. 14.7

3)
A) T
B) T
C) T
Fluidity due to motility of lipids and proteins
It mainly depends on lipid composition and temperature. D) T
E) T

4)
A) T Phospholipids derived from alcohol sphingosine instead of glycerol.
B) T Synthesis of bile acids Lippincott pg. 225 fig. 18.9
Synthesis of vitamin D Lippincott pg. 387 fig.28.23
C) T Glycerol + 2 fatty acids + phosphate group
Also considered as derivative of phosphatic acid (phosphoric acid, N containing base and other
substituent)
D) F
E) F Seliwanoff’s test Ketose sugar test
Sudan III test Lipid test

5)
A) F Mnemonic for essential amino acids; PVT TIM HALL
B) F
C) F
D) T
E) F

6)
Presented by 15th Batch 69 FHCS | EUSL
A) F C18 – linolenic acid
B) F C18
C) T C20
D) F C18
E) T C20

7)
A) T
B) F Single stranded but has base pairing in t RNA
C) T Uracil instead of thymine
D) T Ribozyme – 28s r RNA catalyze the synthesis of peptide bonds. I
E) T

8)
A) T Chromatin DNA + histone protein
B) T Histone protein is a basic protein
C) F Proteins are basic; Arginine and Lysine
D) T
E) F Basic unit of chromatin is nucleosome

9)
A) T PCR allows scientists to amplify small, specific segments of DNA and make millions of copies of
segment.
B) F Small specific segments
C) T DNA polymerase III (Taq Polymerase)
D) T
E) F Temperature differences

10)
A) T
B) T
C) T
D) T
E) T

11)
A)T Isoenymes are enzymes that catalyze the same reaction but have different physical properties.
B)T Different physical properties.
C) F
D)T Eg; Creatine kinase (Lippincott pg.65)
E)T Lippincott pg.65 - Fig. 5.21

41) ANSWER-A

42) ANSWER-C

43) ANSWER-C

48) ANSWER – A

Presented by 15th Batch 70 FHCS | EUSL


The peptide bond has a partial double-bond character. Unlike its components, the α-amino and α-carboxyl
groups, the NH and –C =O of the peptide bond do not accept or give off protons. The peptide bond is not
cleaved by organic solvents or urea, but is labile to strong acids. It is usually in the trans configuration.

49) ANSWER – B
Coenzymes- cosubstrates are small organic molecules that associate transiently with an enzyme and
leave the enzyme in a changed form. Coenzyme-prosthetic groups are small organic molecules that
associate permanently with an enzyme and are returned to their original form on the enzyme. Cofactors
are metal ions. Heterotropic effectors are not substrates.

9TH BATCH REPEAT SEQ - Answers

1.
1.1
● There are 2 dimensional secondary structures of proteins.
● Those are Alpha helix & Beta sheet.
● The most common type is the Alpha helix.
● Amino acids in an Alpha helix are arranged in a right handed helical structure.
● All H-bonds in the alpha helix are oriented in the same direction.
● The N-terminus will have a positive charge & the C-terminus will get a negative charge.
● Side chain groups point outwards from the helix.
● Amino acids with bulky side chains are less common in alpha helix.
● Glycine & Proline destabilizes the alpha helix.

1.2
● Irreversible
● Reversible
Competitive
Non-Competitive

1.3
Because of the similar structure of the substrate and inhibitor, the inhibitor has to compete with the
substrate to bind with the active site of the enzyme. So it is called “Competitive Inhibition”. That binding
blocks the active site available for the substrate. So there will be no Enzyme-Substrate complex formation.
No enzymatic activity. No Enzyme-Product complex formation. Km will be increased and Vmax constant.

1.4
Inhibitor is a substance that reduces/stops the enzyme reaction & thereby interferes with catalysis.
Competitive inhibitor is a structural analogue of the substrate which competes with the substrate to bind to
the active site & inhibit the reaction. In this inhibition, Vmax is not changed. But Km increased.

Methotrexate

Methotrexate is an analogue of Dihydrofolate(DHF). So it competes with DHF to bind with DHF Reductase
enzyme. Methotrexate binds to the active site of enzymes. So, Ensyme-Substrate complex formation is
reduced resulting in decreased formation of Tetrahydrofolate(THF). Conversion of DHF to THF is essential

Presented by 15th Batch 71 FHCS | EUSL


in pyrimidine synthesis. So in cancer, Methotrexate is used as an anti-cancer drug which reduces the
synthesis of DNA. Therefore decreases the cell growth

Statin drugs – Simvastatin

Simvastatin is an analogue of HMG CoA. So it competes with HMG CoA to bind with the active site of
HMG CoA Reductase enzyme. As simvastatin binds with enzymes, the active site is not free for the
formation of Enzyme-Substrate complexes. Therefore the product; Mevalonate, is reduced. This reaction
is the regulatory step in cholesterol synthesis. By inhibiting this reaction, cholesterol synthesis is reduced.
So it is used in the treatment of Atherosclerosis.

2.
2.1.
Okazaki Fragments
An Okazaki fragment is a relatively short fragment of DNA. Those are created on the lagging strand during
DNA replication.

On the leading strand, DNA replication proceeds continuously along the DNA molecule as parent double
stranded DNA is unwound. But on the lagging strand, the new DNA is made in installments which are later
joined together by a DNA ligase enzyme. This is because the enzyme that synthesises the new DNA can
only work in one direction along the parent DNA molecule & the two strands are anti- parallel.

2.2.
Structural polysaccharides
Structural polysaccharides are,
Cellulose
Chitin
Gums & Pectin
GAG

Cellulose
Major component of plant cell walls.
Polymer of beta glucose units.
Linked by β-1,4 glycosidic bonds
Provides protection & support.
Presented by 15th Batch 72 FHCS | EUSL
GAG
Unbranched polysaccharides
Polymers of uronic acid & amino sugar
Heteropolysaccharides.
Amino sugars
- D-Glucosamine
- D-Galactosamine
Uronic acids
- L-Glucoronic acid
- L-Iduronic acid
GAGs attached covalently to extracellular proteins except hyaluronic
acid.
Forms proteoglycans.
GAGs can be found
- Synovial fluid of joints.
- Vitreous humour of the eye.
- Arterial walls.
- Bones & cartilages.
Major component of extracellular matrix.
Functions
1. As a cushion against mechanical shocks.
2. Act as molecular sieves.
3. Give elasticity to substances.
4. Lubricate joints both of the surface of cartilage & in synovial fluid.

Types of GAGs
1.Hyaluronic acid
Synovial joints
Loose connective tissue
2.Chondroitin sulphate
Tendons
Ligaments
3.Keratan sulphate
Cornea
Cartilage
4.Dermatan sulphate
Skin
Blood vessels
5.Heparin
Lining of arteries of liver
6. Heparan sulphate
Skin
Fibroblast

2.3
Protein synthesis has 2 steps.
1. Transcription
2. Translation
Translation takes place in the cytoplasm.
Presented by 15th Batch 73 FHCS | EUSL
It is the process of decoding the m-RNA into a polypeptide chain.
Ribosomes read m-RNA 3 bases or one codon at a time & construct the proteins.
Transcripted m-RNA enter the cytoplasm.
m-RNA transcript start codon AVG attaches to the small ribosomal sub unit.
Small sub unit attaches to large ribosomal sub unit. 05 stages
1) Activation of amino acids
2) Initiation - start codon (AVG)
3) Elongation
4) Termination – stop codon (UAG)
5) Folding & processing

9TH BATCH REPEAT SEQ - Answers

1.
1.2.

starch glycogen

α-D glucose is monomer α-D glucose is monomer

Branched & unbranched Highly branched

Contain amylose , amylopectin similar to amylopectin

Presented by 15th Batch 74 FHCS | EUSL


Amylose- branched
Amylopectin - unbranched

1-4 glycosylic bonds in Amylose 1-4 and 1-6 glycosylic bonds


1-6 glycosylic bonds in Amylopectin

Amylopectin molecule wrap around the


Amylose molecules

Storage carbohydrate in plant storage carbohydrate in animals (liver


&skel;mus)

1.3.
● Found in the forms of chromosomes
● Associate with the tightly bound basic protein – ( histone, + charged , so they bind with – charged
DNA)
● Histones H1 , H2A , H2B , H3 , H4
● Two molecules from H1 , H2A , H2B , H3 , H4 form the structural core of individual nucleosome
“beads”
● A segment of DNA double helix is twisted (wound) around the core.
● Nucleosome are joined by linker DNA and H1
● Nucleosomes are tightly packed to form a nucleofilament ( polynucleosomes )- a coil 30 nm fiber.
● The fibres are organised into loops and anchored to scaffold proteins.

2.0

2.1
a)Four organizational levels of protein structure
Primary

Presented by 15th Batch 75 FHCS | EUSL


Secondary
Tertiary
Quaternary

Primary structure

Specified sequence of amino acids in a polypeptide chain/protein.


Ex: Lysozyme- an enzyme that attacks bacteria, consists of a polypeptide chain of 129 a.a.
The precise primary structure of a protein is determined by inherited genetic information.

Secondary structure

Regular arrangements of amino acids that are located near to each other in the linear
sequence.
α-helix
β-sheet
Β-bend/ reverse
Loop & coils
Super secondary structure
Secondary structural elements are stabilized by extensive hydrogen bonding.

Tertiary structure

Refers to the three dimensional shape of a polypeptide chain.


Is stabilized by a variety of interactions (bond formation) among R groups and between R groups and the
polypeptide backbone.

Quaternary structure

The arrangement of polypeptide subunits is called the quaternary structure of the protein
results from the aggregation (combination) of two or more polypeptide subunits held together by non-
covalent interaction like H- bonds, ionic or hydrophobic interactions.
Subunits may either function independently of each other , or may work cooperatively , as in hemoglobin.
Ex- Hemoglobin is a globular protein with four polypeptide chains (tetrameric)

b)
Primary structure
peptide bond
Secondary structure
peptide bond
hydrogen bond
Tertiary structure
Hydrogen bonds
ionic bonds
van der waals
hydrophobic interactions
disulfide bonds
Quaternary structure
non-covalent interaction like H- bonds, ionic or hydrophobic interactions.

Presented by 15th Batch 76 FHCS | EUSL


2.2

a) The several enzymes that catalyse the same reaction.


b) i)LDH
● Lactate dehydrogenase (LDH) is a protein that helps produce energy in the body. An LDH test
measures the amount of LDH in the blood.
● LDH is most often measured to check for tissue damage. LDH is in many body tissues, especially
the heart, liver, kidney, muscles, brain, blood cells, and lungs.
● Isoenzymes of skeletal muscle and liver, M4, has four muscle (M) subunits, while H4 is the main
isoenzymes
● For heart muscle in most species, containing four heart (H) subunits.
○ LDH-1 (4H)—in the heart and in RBC (red blood cells), as well as the brain
○ LDH-2 (3H1M)—in the reticuloendothelial system
○ LDH-3 (2H2M)—in the lungs
○ LDH-4 (1H3M)—in the kidneys, placenta, and pancreas
○ LDH-5 (4M)—in the liver and striated muscle[4]
● LDH-2 is usually the predominant form in the serum.
● An LDH-1 level higher than the LDH-2 level (a "flipped pattern") suggests myocardial infarction
(damage to heart tissues releases heart LDH, which is rich in LDH- 1, into the bloodstream).
● The use of this phenomenon to diagnose infarction has been largely superseded by the use of
Troponin I or T measurement.
● Other conditions for which the test may be done include:
○ Low red blood cell count (anemia) Cancer, including blood cancer (leukemia) or lymph
cancer (lymphoma)
● Normal value range is 105 to 333 international units per liter (IU/L).
● A higher-than-normal level may indicate:
Blood flow deficiency (ischemia)
Heart attack
Hemolytic anemia
Infectious mononucleosis
Leukemia or lymphoma
Liver disease (for example, hepatitis)
Low blood pressure
Muscle injury
Muscle weakness and loss of muscle tissue (muscular dystrophy)
New abnormal tissue formation (usually cancer)
Pancreatitis
Stroke
Tissue death

ii) CK

● Creatine kinase is an enzyme expressed by various tissues and cell types.


● CK catalyses the conversion of creatine and utilises adenosine triphosphate (ATP) to create
phosphocreatine (PCr) and adenosine diphosphate (ADP). This CK enzyme reaction is reversible
and thus ATP can be generated from PC4 and ADP.
● In tissue cells that consume ATP rapidly,especially skeletal muscle, but also Brain, photoreceptor
cells in retina, hair cells of the inner ear, spermatozoa and smooth muscle.

Presented by 15th Batch 77 FHCS | EUSL


● Clinically creatine kinase is assayed in blood tests as a marker of damage of CK-rich tissue such
as in myocardial infarction (heart attack), rhabdomyolysis (severe muscle breakdown), muscular
dystrophy, autoimmune myositides, and acute kidney injury.

10TH BATCH
10TH BATCH PROPER MCQ

1) Which of the following are not disaccharides


A) Hyaluronic acid
B) Maltose
C) Lactose
D) Sucrose
E) Erythrose

2) Glycogen
A) Degradation results mainly glucose -1-phosphate
B) Formed from fatty acid
C) A steroid hormone
D) It is major type of storage carbohydrate
E) Directly use as GTP in muscle tissue

3) Buffer is the mixture of weak acid and its conjugate base. Regarding weak acid
A) it’s protonated at the pH below pka value
B) Acetic acid is an example
C) It has conjugate acid
D) It completely dissociate in the water
E) It behaves according to the Henderson Hasselbalch equation

4) 27yr old woman urinolysis high concentration neutral amino acid and has hartnup disease. Which are found in
urine
A) Arginine
B) Histidine
C) Lysine
D) Phenylalanine
E) Serine

5) Regarding peptides
A) Polymer of Amino acid
B) Positive to biuret reaction
C) Aspartame is a commercial dipeptide
D) Glutathione is dipeptide
E) When written nitrogen terminal on left hand and hydroxyl on right hand

6) Regarding coenzymes
A) Are precursor forms of enzymes
B) Inorganic substances
C) Not bound to the enzyme
D) Thermo stable
Presented by 15th Batch 78 FHCS | EUSL
E) Usually regenerate

-6 fatty acid
- linoleic acid
B) α-linolenic acid
C) Oleic acid
D) Arachnoid acid
E) Gamma-linolenic acid

8) All nucleotides
A) Contain a pentose monosaccharide
B) Function as co–enzymes
C) Have 3 phosphate groups
D) Contains a nitrogenous base
E) Are stored in the cell nucleus

9) Regarding DNA replication


A) Helicase enzyme unwind the double strand
B) DNA polymerase 1 lay down new sequence in place of the RNA primer
C) DNA polymerase 3 initiate Okazaki fragment synthesis
D) In bacteria DNA polymerase 3 synthesis DNA on both strand
E) Leading strand synthesis is discontinuous

41) A young black man entered his physician’s office complaining of bloating and diarrhea. His eyes were sunken
and the physician noted additional signs of dehydration. The patient’s temperature was normal. He explained that
the episode had occurred following a birthday party at which he had participated in an ice cream eating contest.
The patient reported prior episodes of a similar nature following ingestion of a significant amount of dairy products.
This clinical picture is most probably due to a deficiency in
A) Salivary α-amylase
B) Isomaltase
C) Pancreatic α-amylase
D) Sucrase
E) Lactase

42) Which one is the nucleoside?


A) Adenonine
B) Adenosine
C) AMP
D) ADP
E) GTP

43) Which one of the following statements is correct?


A) The α-helix can be composed of more than one polypeptide chain
B) β-Sheets exist only in the antiparallel form
C) β-Bends often contain proline
D) Domains are a type of secondary structure
E) The α-helix is stabilized primarily by ionic interactions between the side chains of amino acids

44) Which of the following give rise to prostaglandin?


A) Acetyl co A

Presented by 15th Batch 79 FHCS | EUSL


B) Arachidonic acid
C) Glucose
D) Leukotriene
E) Fatty Acid

10TH BATCH PROPER SEQ

01. Bio molecules play extremely important roles in the functioning of all body tissues.
1.1 Briefly describe the functions of carbohydrates. (35 marks)
1.2 Outline the process of protein denaturation and factors influencing it. (30 marks)
1.3 Briefly describe the types of plasma lipoproteins and their role in human body. (35 marks)

02 Write short notes on the following.


2.1 Mechanism of competitive inhibition and their use in drug therapy. (35 marks)
2.2 Transcription of DNA. (35 marks)

Answers

10TH BATCH PROPER MCQ - Answers

1)
A) T
B) F
C) F
D) F
E) T

2)
A) T glycogen is converted to glucose-1-phosphate by glycogen phosphorylase
B) F a glucose polymer
C) F is a storage polysaccharide
D) T other storage polysaccharide, starch, inulin
E) T

3)
A) T At pH values less than the pKa, the protonated acid from (CH3-COOH) is predominant.
B) T CH3-COOH
C) F
D) F 𝐶𝐻3𝐶𝑂𝑂𝐻 ⇔ 𝐶𝐻3𝐶𝑂𝑂− + 𝐻+
E) T pH = pKa + log [A-]/[HA]

4)
A) F Hartnup disease – characterised by defective absorption and defective transport of tryptophan.
B) F
C) F Neutral amino acid- Glycine, Alanine, Valine, Leucine, Glutamine, serine, Threonine, proline, tyrosine,
Asparagine, Isoleucine, cysteine, methionine, phenylalanine, tryptophan
D) T basic- Histidine, Arginine, lysine
Presented by 15th Batch 80 FHCS | EUSL
E) T acidic-Aspartate, Glutamate

5)
A) T peptides are polymers of 2-50 amino acids bound by peptide bonds. (Proteins > 50)
B) T biuret test -;
Reagent’s CuSO4 / KOH,
Original colour – blue
Turns into violet in the presence of peptide bonds.
C) T aspartate-phenylalanine dipeptide, Non-saccharide sweeter.
D) F Glutathione is a tripeptide, an antioxidant
E) T

6)
A)F coenzymes are small organic molecules that help enzymes in their reactions.
1. Co-substrate – transiently bound
2. Prosthetic group – permanently bound
B) F if the non-protein moiety is a mental ion such as Zn2+ or Fe3+ (inorganic) co-factor
C) F
D) T because it is a non-protein thermo stable.
E) T NAD+ ⇔ NADH

7)
A) T
B) F
C) F
D) T
E) T

8)
A) T
B) F NAD, FAD function as coenzymes.
C) F 2P-ADP, 1P-AMP.
D) T
E) F Found in other organelles such as mitochondria

9)
A) F Topoisomerase - unwinds DNA
Helicase- breaks H bonds
Polymerase- forms complementary DNA strand in 5’-3’ direction.
B) T DNA polymerase 1 – remove RNA primers and replace them with DNA.
C) T DNA polymerase 3 – add nucleotides to the 3' end and build a new stand in 5’-3’ direction.
D) F 5’-3’ direction
E) F polymerase 3 –synthesis in 5’-3’ direction – leading stand

Presented by 15th Batch 81 FHCS | EUSL


Lagging strand – discontinuous

41) ANSWER – E
The physical symptoms suggest a deficiency in an enzyme responsible for carbohydrate degradation. The
symptoms observed following the ingestion of dairy products suggest that the patient is deficient in
lactase.

42) ANSWER – B

43) ANSWER – C
β-Bends often contain proline, which provides a kink. The α-helix differs from the β-sheet in that it always
involves the coiling of a single polypeptide chain. The β-sheet occurs in both parallel and antiparallel
forms. Domains are elements of tertiary structure. The α-helix is stabilised primarily by hydrogen bonds
between the –C =O and –NH– groups of peptide bonds.

44) ANSWER – B
Arachnoid acid, (ω-6 acid contains 20 carbon & double bones) is immediate precursor of predominant type
of prostaglandin.

10TH BATCH PROPER SEQ - Answers

1.
1.1
● Carbohydrates act as a storage form of energy.
● They are glycogen, starch and insulin.
● Starch is the main storage polysaccharide of plants.
● It is a polymer of α glucose and consists of 2 glucose units called amylose and amylopectin.
● Glycogen is the storage polysaccharide of animals.
● Inulin acts as the storage polysaccharide in some plants like onion, garlic etc.
● It is a polymer of D- fructose units.
● Carbohydrates act as structural components.
● Glycosaminoglycans serve as structural components in humans.
● They are long unbranched polysaccharides.
● Functions of glycosaminoglycans are binding and packing of tissues repair of injury, defence
against infections, transport of metabolites between capillaries and tissues and connect, anchor
and support the body and its organs.
● Cellulose serves as a structural component of plants.

Presented by 15th Batch 82 FHCS | EUSL


● It is insoluble and consists of polymers of β glucose units.
● Chitin serves as a structural component in insects.
● It is a polymer of N- acetylglucosamine joined by β 1 to 4 glycosidic linkages.
● Carbohydrates are constituents of nucleotides.
● They serve as ribose in RNA and deoxyribose in DNA.
● Carbohydrates also serve as dietary fibres such as cellulose, agar, gum and pectin.
● It also plays a role in lubrication, cellular intercommunication and immunity.
● They are glycoprotein and glycolipids.
● Carbohydrates such as glucuronic acid are involved in detoxification.
● Carbohydrates serve as a source of energy in all organisms

1.2.
● Protein denaturation results in the unfolding and disorganisation of a protein’s secondary, tertiary
structures and quaternary structures without the hydrolysis of peptide bonds.
● Denaturing agents include
Heat
Organic solvents
Strong acids or bases
Detergents
Ions of heavy metals such as lead
● Denaturation may under ideal conditions be reversible such that the protein refolds into its original
native structure when the denaturing agent is removed.
● However most proteins, once denatured, remain permanently disordered.
● Denatured proteins are often insoluble and precipitate from solution.

1.3.
● Lipoproteins are molecules which contain lipids, proteins and phospholipids.
● They are water soluble molecules.
● Outer Shell of lipoprotein is made of a phospholipid layer.
● Apolipoproteins which are embedded in layers stabilise the structure. Lipoproteins are classified
into chylomicrons, very low density lipoproteins, intermediate density lipoproteins, low density
lipoproteins, high density lipoproteins .
● Chylomricrones consist of apo B-48 apo E, apo Cii.
● Chylomicrons carry dietary triacylglycerol, cholesterol, fat soluble vitamins, and cholesterol esters
to peripheral tissues.
● LDL consists of apo B-100, apo E .
● Low density lipoproteins contain much less TAG than VLDL. And have a high concentration of
cholesteryl esters.
● They provide cholesterol to peripheral tissues.
● High density lipoproteins consist of apo A-1, apo Cii, apo E.
● They supply apolipoproteins for chylomicron an VLDL metabolism. They uptake the unesterified
cholesterol.
● HDL also reverses the cholesterol transport.

2.
2.1.
● Competitive inhibitors compete with substrate molecules for active sites of enzymes.
● Bind reversibly.
● Inhibitory action is proportional to its concentration.
Presented by 15th Batch 83 FHCS | EUSL
● So, the effect of the inhibitor is reversed by increasing substrate concentration.
● At high substrate concentration, velocity reaches Vmax observed in
absence of inhibitors.
● So, Vmax is constant.
● Increase Km for a given substrate.
● So, more substrate is needed to achieve 1⁄2 Vmax.

Drug therapy
● Statin drugs competitively inhibit rate limiting steps in cholesterol biosynthesis.
Inhibit HMG- CO A reductase
Are structural analogues of natural substrate.
So, lower plasma cholesterol level
Ex: atorvastatin
Pravastatin

2.2.
● DNA transcription is the 1st stage of protein synthesis.
● It is the process of copying the sequence of one strand of DNA to form mRNA.
● Require RNA polymerase.
Initiation.
● RNA polymerase binds to a region of DNA known as promoter.
● It signals the start of a gene.
● RNA polymerase does not need a primer and there is no error correcting ability .
● Transcription factor assemble at promoter to form a transcription initiation complex
Elongation
● RNA polymerase unwinds the DNA by breaking the H bonds.
● Separate two strands and form a transcription bubble.
● Base pairing occur between incoming RNA nucleotide and DNA
nucleotides of genes.
● RNA uses uracil instead of thymine.
● RNA polymerase catalyses the bond formation between nucleotides.
● Bubbles move forward and rewarding behind the DNA strand.
Termination
● Terminator region signals the stop of the gene.
● RNA polymerase releases the mRNA and DNA.
mRNA editing
● a post transcriptional process.
Introns - non functional segments
They cut off
Exons - coding region of DNA
They region by ligase
● A guanosine triphosphate cap is added to 5’end of the new Mrna.
● A poly A tail is added to 3’end of mRNA.
● mRNA leaves through pores into cytoplasm.

Presented by 15th Batch 84 FHCS | EUSL


11TH BATCH
11TH BATCH PROPER MCQ

01.Alpha helix
A) Left hand
B) Disturb by proline
C) Is stabilize by H bond between the C=O in main chain
D) All globular proteins at same percentage
E) Play mechanical role in stiff bonding of fibres in keratin.

02. Protein denaturation


A. May be reversible
B. Increases the solubility of the protein
C. Can't take place if strengthened by disulphide bonds
D. Biological activity is lost; not the primary structure
E. Extremities of pH can cause this

03.
04.Irreversible enzyme inhibitors are
a) CN
b) snake venom
c)nerve gas
d)organophosphates
e) statin drugs

05. Regarding DNA double helix


A). Left-handed helix
B). Has parallel strands
C). Comprise major and minor grooves
D). Bonds between purine & pyrimidine
E). Denatured at low temperature

07.Regarding glycolysis
A. Occurs in cytoplasm
B. Lactate is an intermediate
C. hexokinase enzyme is used for the committed step
D. pyruvate kinase reaction require ATP
E. high level of citrate inhibits glycolysis

08. Which of the following are antioxidants?


1. Carotenoids
2. Citric acid
3. Glutathione reductase
4. Thiamine
5. Zinc

Presented by 15th Batch 85 FHCS | EUSL


09.Physiological uncouplers of electron transport chain include,
A. Calcitonin
B. Calcium ion
C. Phospholipid
D. Thermogenin
E. Thyroxine

42. Regarding m-RNA which is the incorrect statement,


A. 5'-cap protect m-RNA from exonuclease enzyme
B. Has alternative splicing mechanism
C. Is Monocistronic.
D. Exons are removed during postranslation modification
E. soluble

43)What is basic amino acid


A. arginine
B. glutamate
C. serine
D. leucine
E. proline

45.rancidity of butter can be prevented by adding,


A) cu
B) lead
C) biotin
D) vitamin D
E) tocopherol

11TH BATCH PROPER SEQ

01.
1.1. Outline the role of glycosaminoglycans in the human body. (25 Marks)
1.2. Briefly describe the functions of biologically active di and tri peptides. (20 Marks)
1.3. State the functions of kreb’s cycle. (25 Marks)
1.4. Outline the role of different types of RNA in a eukaryotic cell. (30 Marks)

Answers

11TH BATCH PROPER MCQ - Answers

1)
A)F right handed spiral structure
B)T
C)T
D)T
E)T

Presented by 15th Batch 86 FHCS | EUSL


2)
A)T
B)F
C)F
D)T
E)T

3)
A)F Mnemonic-PVT TIM HALL
B)F
C)F
D)T
E)F

4)
A)T
B)
C)T
D)T
E)F

5)
A)F right handed double helix
B)T
C)T
D)T
E)

6)
A)F same reaction
B)
C)T
D)T
E)T

7)
A)T
B)F
C)F fructose 6-phosphate → fructose 1,6- bisphosphate.
D)F
E)T

8)
A)T
B)T
C)T
D)F
E)T

9)
Presented by 15th Batch 87 FHCS | EUSL
A)F
B)T
C)
D)T
E)T

42) ANSWER D

43) ANSWER A

45) ANSWER E

11TH BATCH PROPER SEQ - Answers

1.
1.1
• GAGs are highly polar and can attract a large amount of water. Thereby produce a gel like matrix which
forms the basis of ground substance. Fibrous proteins make up an extracellular matrix.
• GAGs are highly negatively charged with extended conformation that gives high viscosity to the solution.
• Their low compressibility makes these molecules ideal for lubricating fluid in the joints(synovial).
• Rigidity provides structural integrity to cells and passageways for cell migration.

1.2
• Dipeptides are two amino acids joined by a peptide bond. For example Aspartame- aspartic acid with
phenyl alanine. Used as a sweetening agent replacing sugarcane.
• Tripeptide is three amino acids joined by two peptide bonds. Example Glutathione formed with glutamic
acid, cysteine and glycine. It helps in absorption of amino acids, protects against haemolysis of RBC by
breaking H2O2 which causes cell damage.

1.3
• Provides final common pathway for the oxidation of carbohydrate, fat and
protein.(Integration of major metabolic pathways.)
• Produce energy either directly as ATP or as reducing agents NADH or FADH2 oxidized in ETC
• Provides substrates for the ETC
• Source of biosynthetic precursors. Example –porphyrin from succinyl CoA, amino acids from OAA and
alpha ketoglutarate
• Some intermediates exert regulatory effects on other pathways. Example- citrate inhibits PFK-1 in
glycolysis.

1.4
1. mRNA–function as messengers which carry information in a gene to the protein synthesising ribosomal
RNA
2. rRNA- structural component of ribosome. Necessary for ribosomal assembly and play a key role in
binding of mRNA to ribosomes and its translation.
3. tRNA- carry the specific amino acid to the ribosome.

Presented by 15th Batch 88 FHCS | EUSL


12TH BATCH
12TH BATCH PROPER MCQ

1. Glycosaminoglycans,
a. expand in nature due to negativity
b. unbranched heteropolysaccharides
c. keratan sulfate softens vitreous
d. chondroitin sulphate is the highest in ECM
e. dermatan sulphate lubricates synovial fluid

2. What is / are the true matchings with clinical enzymes


a. Alpha amylase – pancreatitis
b. LDH – myocardial infraction
c. Alkaline phosphatase – prostate cancer
d. Alkaline transaminase – cholestasis
e. Troponin T – STEMI

3. Covalent modification,
a. Alter the quantity of enzyme
b. Mediate by hormones
c. Occur in amino acid residues contain hydroxyl residues
d. Of digestive enzymes by proteolytic cleavage
e. Of glucagon synthase regulate glycogen synthesis

4. Eukaryotic protein synthesis,


a. Aminoacyl t-RNA synthetase is responsible for attachment of amino acid to corresponding t-RNA
b. RNA is synthesized parallel to DNA template in 3’ – 5’ direction
c. Termination region of synthesis RNA is recognized by sigma factor of RNA polymerase
d. 3 different stop codons for termination
e. Transcription start by RNA polymerase

5. Alpha helix,
a. H bonds occur between electro negative atoms of side R chains of amino acid
b. H bonds are perpendicular to axis
c. The structure is stabilized by salt linkage
d. Branches of amino acids are exterior
e. Proline stabilize the helix

6. T form of Hb
a. Deoxy form of Hb
b. Has high oxygen affinity form
c. Has low oxygen affinity form
d. H bonds and ionic bonds limit the movement of monomer
e. The binding of oxygen destabilizes some of the H bonds and ionic bonds particularly between alpha – beta dimers

7. Regarding the cholesterols


a. Main sterol in human
b. Present all cell membranes
c. Excrete by liver as un modified cholesterol used to form vit D precursors
Presented by 15th Batch 89 FHCS | EUSL
d. Used to synthesize steroid hormones

8. Regarding leukotrienes
a. Increase vascular permeability
b. Induce chemotaxis
c. Destroyed neutrophils
d. Induce muscle contraction
e. Synthesis from arachnoid acid

9. Cyanide ion and carbon monoxide inhibit


a. Cytochrome C oxidase
b. ATP production
c. Antibiotic activity
d. Co-Q formation
e. Co-A formation

10. What reaction produce NADH


a. α-keto glutaric acid to succinyl Co-A
b. Malate to Oxaloacetate
c. Succinate to fumarate
d. Oxaloacetate to citrate
e. Isocitrate to α-keto glutarate

11. Regarding enzyme


a. Enzyme molecule is larger than substrate
b. Optimum pH in human body is 7.4
c. Enzyme act by lowering the changes of free energy
d. Competitive inhibition can be overcome by increasing substrate concentration
e. Glucokinase action on glucose is an example for zero kinetic reaction

12. Regarding proteoglycans,


a. Prostaglandins have a very short life
b. Prostaglandins are synthesized only in liver and adrenal cortex
c. Prostaglandins generally act locally
d. The common precursor of prostaglandin is arachidonic acid
e. Prostaglandins synthesis can be inhibited by a number of unrelated compounds, including cortisol and aspirin

39. Southern hybridization using for


a. Specific DNA
b. Specific RNA
c. Protein
d. DNA and RNA
e. RNA and protein

40. Coenzyme of alfa-ketoglutarate dehydrogenase except


a. Co enzyme A
b. Biotin
c. Riboflavin
d. TPP
e. Niacin

Presented by 15th Batch 90 FHCS | EUSL


46. Which of the following is non reducing sugar
a. Fructose
b. Glucose
c. Mannose
d. Sucrose
e. Lactose

12TH BATCH PROPER SEQ

13TH BATCH
13TH BATCH PROPER MCQ

1. Starch
A. Pink colour appearance with iodine
B. its a homopolysaccharide
C. high in leafy vegetables
D. predominantly it contains amylopectin
E. Structure support by disulphide bond

2. Secondary structure of protein


A. Beta bend contain proline
B. Motifs predict biological function
C. Alpha helix composed of more than polypeptide chain
D. Alpha helix stabilize primary by H bonds
E. Beta sheet exist only in antiparallel form

3. PDH complex
A. Catalyze the oxidative decarboxylation of pyruvate
B. Activated by acetyl co A
C. Activated by insulin
D. Vitamin B12 need for action.
E. Produce 1 NADH molecule as a product

4. Uncouples of the respiratory chain include,


A. Carbon monoxide
B. 2,4 dinitrophenol
C. rotenone
D. thermogenine
E. cyanides

5. Regarding high energy compound


A. Possess high energy phosphate bonds
B. supply energy to endogenous metabolism
C. Are synthesized via substrate level phosphorylation
D. Phosphoenol pyruvate is an example
Presented by 15th Batch 91 FHCS | EUSL
E. NADH is a reduced form of high energy molecule

6. Regarding Isoenzymes
A. identify by electrophoresis
B. catalyze same reactions
C. have different kinetics
D. present in different subcellular organs
E. genetically determined Primary stucture are same

7. Regarding DNA,
A. Containglycosidic bonds between nucle.
B. H formation occur between purine & pyramids .
C. Hydrophobic bases locate inside the helix.
D. Uracil present in DNA.
E. Denature in high temperature.

8. Match the component of the cell membrane with function


A. cholesterol - maintain fluidity
B. glycolipid - cell recognition
C. intergral protein – enzyme
D. peripheral protein - transfering substance
E. Sphingolipid - signal transmission

20. Mitochondrial DNA


A. Consists over 100 coding sequences
B. Act as human genome
C. Has double stranded circular DNA
D. Has TAG sequence for coding tryptophan

21. In protein synthesis,


A. Degeneracy of genetic codes denotes existence of multiple codons
B. Initiatory codon is UAG.
C. Peptidyl transferase involves initiation.
D. Aminoacyl synthetase requires ATP.
E. There are 3 different stop codons for termination.

41. Triple helix in protein is an example of


A. a peptide
B. Primary structure
C. quaternary structure
D. Secondary structure
E. Tertiary structure

42. Incorrect regarding Prostaglandin,


A. Derived from COX pathway
B. Inhibits uterin contraction
C. Synthesis is inhibited by anti inflammatory drugs
D. induce synthesis of cAM
E. gsatric protection

Presented by 15th Batch 92 FHCS | EUSL


43. Which of the following is not the reason for free radical formation
A. Electron transportation chain
B. UV radiation
C. Green vegetables and fruits
D.
E.

44. A person recently diagnosed with a urinary tract infection was prescribed with sulfa drug which alters Km value.
What is the type of inhibition?
A. Allosteric
B. Competitive
C. Feedback
D. Non competitive

45. Select the "incorrect" statement regarding cholesterol


A. All the C atoms of cholesterol are derived from acetyl COA
B. Cholic acid is a precursor in biosynthesis of cholesterol
C. HMG COA reductase is main regulatory enzyme of cholesterol synthesis
D. It is essential for the production of glucocorticoids
E. It is the most abundant sterol form in human

13TH BATCH PROPER SEQ

5.
5.1. Write short notes on restriction endonucleases (25 marks)
5.2. 5.2.1. Briefly describe the general structure of hemoglobin (15 marks)
5.2.2. Give the biochemical basis for sickle cell anaemia (30 marks)
5.3 5.3.1 What is genetic code ? (10 marks)
5.3.2 Explain the characteristic features of genetic code (20 marks)

6.
6.1. Describe the functions of carbohydrates that are important to humans (35 marks)
6.2. Outline the structure, location and functions of collagen in the human body (30 marks)
6.3. List posttranscriptional ad posttranslational modifications (15 marks)
6.4. State the main functions of Kreb’s cycle (20 marks)

14TH BATCH
14TH BATCH PROPER MCQ

02. Glycogen,
A) Degradation result mainly glucose 1 phosphate
B) Formed from fatty acids
C) A steroid hormone
D) It is a major type of storage carbohydrate
E) Directly used as GTP in muscle tissue

Presented by 15th Batch 93 FHCS | EUSL


03. Regarding amino acids,
A) Proline interrupts the alpha helical structure
B) Serine is a S containing amino acids
C) Glutamate transports ammonia in the body
D) Cysteine acts as a buffer
E) Tyrosine is a both ketogenic and glucogenic amino acid

13. Regarding lipoproteins,


A) HDL has highest volume.
B) Apo B-48 is unique for chylomicrons.
C) APO C-2 activates lipoprotein lipase.
D) LDLs are synthesized by liver directly.
E) Exogenous TAGs are the main component of VLDL.

28. Match correctly regarding amino acids,


A) Alanine - non polar
B) Glutamate - contain carboxylic group in side chain
C) Glycine - contain shortest side chain
D) Methionine - contain sulfhydryl group
E) Tyrosine - contain hydroxyl group in side chain

29. True/false
A) Disulfide bond require cysteine to be adjacent in the primary structure
B) Protein consisting of a single polypeptide chain has quaternary structure.
C) The peptide bonds are always trans
D) The primary driving force for protein is the hydrophobic effect.
E) Beta sheets are antiparallel.

30. Prosthetic group,


A) Loosely bound to protein
B) No protein parts
C) Organic molecule
D) Are ribozymes
E) Converts original form after reaction

31. In competitive inhibition,


A) Inhibitors are irreversibly bound to active site of enzyme
B) Inhibitors bind enzyme - substrate complex
C) Km is increased
D) Substrate analog binds to the enzyme
E) Vmax is reduced

34. Regarding lipids


A) Arachidonic acid is derived from linoleic acid
B) Aspirin inhibit cyclooxygenase pathway
C) Lauric acid is found in coconut oil
D) Oleic acid is an omega 3 fatty acid
E) Phosphatidylcholine is a component of cell membrane

35. Regarding antioxidant agent

Presented by 15th Batch 94 FHCS | EUSL


A) Ascorbic
B) Beta carotene
C) Glycerol
D) Folic Acid
E) Alpha tocopherol

39. Correctly matched electron transport chain inhibitors are,


A) ATP Synthase - Antimycin A
B) Cytochrome Oxidase – Cyanide
C) Cytochrome Reductase – Oligomycin
D) NADH Dehydrogenase - Rotenone
E) Succinyl CoA Dehydrogenase - Carbon Monoxide

54. Restriction Endonuclease


A) Used to join DNA to cloning vector
B) Used to separate DNA strands
C) Cleaves DNA at specific sequences
D) Cleaves DNA randomly
E) Digest DNA molecule from either end

55. Example of reversible covalent modification


A) Allosteric modulation
B) Competitive inhibition
C) Non-competitive inhibition
D) Hydroxylation of serine group of an enzyme
E) Suicide inhibitor.

56. The iron in heme is linked through globulin


A) Lysine
B) Arginine
C) Valine
D) Histidine
E) Glycine

57. The retained segment of RNA when pre mRNA becomes a mature mRNA
A) Codon
B) Exon
C) Intron
D) Nucleotide
E) Splice boundaries

58. Incorrect about GAGs


A) Chondroitin sulphate - tensile strength to cartilage
B) Dermatan sulphate - resistance to infection
C) Heparin – anticoagulant
D) Hyaluronic acid - lubricating synovial fluid
E) Keratan sulfate - consistency to vitreous humor

59. In citric acid cycle substrate level phosphorylation occurs at the conversions of?
A) Alpha ketoglutarate to succinyl co A

Presented by 15th Batch 95 FHCS | EUSL


B) Citrate to iso citrate
C) Fumarate to malate
D) Succinate to fumarate
E) Succinyl co A to succinate

14TH BATCH PROPER SEQ

3.
3.1. Briefly describe the functions phospholipids with examples. (20 marks)
3.2. Write a note on
3.2.1. Protein denaturation. (25 marks)
3.2.2. Clinically important isozymes. (35 marks)
3.2.3. Pyruvate kinase deficiency. (20 marks)
4.
4.1. List the functions of proteins and enzymes involved in DNA replication. (20 marks)
4.2. Briefly describe the role of eukaryotic RNA polymerases in transcription. (30 marks)
4.3. Briefly describe on regulation heme biosynthesis. (30 marks)
4.4. State 5 conditions that will decrease the plasma albumin level (20 marks)

15TH BATCH
15TH BATCH PROPER MCQ

01) Regarding carbohydrates


A. Adults should obtain 45-65% of their total calories from carbohydrates
B. Dihydroxyacetone is an aldose sugar
C. Higher risk of diabetes is associated with diets high in whole grain cereals.
D. Produce positive outcome in molish test.
E. Recommended dietary allowance of carbohydrates for adult is 130g/day

02) Regarding citric acid cycle


A. carboxylation of pyruvate to oxaloacetate is an anaplerotic reaction
B. Isocitrate dehydrogenase is one of regulatory enzyme
C. NADPH is produced in this
D. It takes place in mitochondrial matrix
E. Vitamin A act as co enzyme in this pathway

03) amino acids ketogenic but not glucogenic


A. leucine
B. isoleucine
C. lysine
D. phenyl alanine
E. tyrosine

04) Enzyme regarding


A. activity affected by substrate shape
Presented by 15th Batch 96 FHCS | EUSL
B. catalytic activity depends on kinetic energy of reactants
C. Concentration alter the equilibrium
D. Increases the rate of reaction
E. Substrate complex is formed by multiple interactions

05) Correctly matched with function


A. Immunoglobulin- defense
B. Insulin- regulate blood sugar
C. Myoglobin- transport O2
D. Transferrin- Fe2+
E. Tubulin - muscle contraction

06) Creatine kinase isozymes differ in


A. amino acid sequence
B. corresponding gene
C. intracellular location
D. km
E. Sensitivity to inhibitors

07) Regarding double helical DNA molecule


A. All hydroxyl group of pentose are in linkage
B. Based are perpendicular to the axis
C. Each strand repeat itself
D. Each strand is antiparallel
E. Has Equal number is purine and pyrimidine

08) Eukaryotic RNA


A. Has a net positive charge at nucleic phase
B. Has equal number of uridine and adenine
C. A single strand
D. Derived antiparallel Hydrogen complements
E. Ratio of ribose base is equal

09)Derived lipids include


A. Cholesterol
B. Fatty acids
C. Phospholipid
D. TAG
E. Vitamin A

53) The highest concentration of cysteine can be found in,


A. Chondroitin sulfate
B. Collagen
C. Keratin
D. Melanin
E. Myosin

54) In the study of enzymes, a sigmoidal plot of substrate concentration versus reactions velocity may indicate?
A. Competitive inhibitors
B. Cooperative binding

Presented by 15th Batch 97 FHCS | EUSL


C. Michaelis Menten Kinetics
D. Myoglobin binding to O2
E. Noncompetitive inhibitors

55) which of the following involve to Heme synthesize?


A. Asparagine
B. Glutamate
C. Glycine
D. Lysine
E. Histidine

56) High concentration of G6P is inhibitory to?


A. Enolase
B. Glucokinase
C. Hexokinase
D. Phosphofructokinase1
E. Pyruvate kinase

57) Fructose and glucose can be distinguished by


A. Barford test
B. Benedict test
C. Burette test
D. Fehling test
E. Seliwanoff test

15TH BATCH PROPER SEQ

2.3 State the inhibitors of each of the enzyme complexes of electron transport chain (10 marks)
2.4 Briefly describe the functions of the electron transport mechanism (15 marks)
2.5 Briefly discuss the importance of essential fatty acids in human with examples (25 marks)

3.
3.1. Mention the functions of amino acids in human (15 marks)
3.2. Briefly describe 3.2.1. the regulation of enzyme activity (35 marks)
3.2.2. DNA replication (30 marks)
3.2.3. the basic steps in DNA cloning with plasmids (20 marks)

Presented by 15th Batch 98 FHCS | EUSL


HB 02

Presented by 15th Batch 99 FHCS | EUSL


EME QUESTIONS

10TH BATCH EME - MCQs

(01) Regarding composition of human body


a. Water contitutes about 65% of body weight in average adult males.
b. Infants have a lower % of water compare to adult.
c. Males have higher % of fat than females.
d. Water % decrease with advancing age.
e. Variation in body weight is mainly due to variation in protein content.

(02) Major forces facilitating filtration in the formation of tissue fluid


a. Crystalloid osmotic pressure in capillary.
b. Hydrostatic pressure in the capillary.
c. Oncotic pressure in the capillary.
d. Hemotocrit of capillary blood.
e. Osmotic pressure in the interstitial fluid.

(03) When compare lymph with blood, the lymp,


a. Is formed from interstitial fluid.
b. Has no protein.
c. Is intracellular fluid.
d. Has no blood cells.
e. Carries chylomicrons absorption from the small intestine.

(04) The following substances can act as ligand


a. Testosterone
b. Toxine
c. Acetyl Choline
d. G-protein
e. Prophanal

SBR
11. In which phace of the cell cycle will you find chromosomal condensation inactivation & not transcribed to
mRNA.
A. G_0 Phase
B. G_1 Phase
C. G_2 Phase
D. S phase
E. M phase

12. Most important function of plasma albumin is,


A. Transport steroids.
B. Transporting bile pigments.
C. Coloid osmotic pressure.
D. Contribute in blood viscosity.
E. Providing immunity.

13. Important buffering activity is provided by,


A. Bicarbonate ion in blood.
Presented by 15th Batch 100 FHCS | EUSL
B. Haemoglobin
C. Ammonia in urine
D. Phosphate in urine.
E. Plasma protein

10TH BATCH EME – SEQ

No SEQs for HB 02 on this year

11TH BATCH EME – MCQ

11TH BATCH EME – SEQ


01.
1.1. Briefly describe the defects of cell division that causes Down syndrome. (30 marks)
1.2. Describe the fluid mosaic model of the Eukariyotic cell membrane structure (40 marks)
1.3. Outline the stages of cell signalling mechanism (30 marks)

02. Urine was collected in a subject at 30 minutes interval for 2 hours after drinking one litre of water rapidly and its
volume and specific gravity were measured. Results are shown below:

2.1. Comment on the above readings. (20 marks) 2.2. Explain the Physiological basis of the deviations from the
normal (80 marks)

11TH BATCH EME – SEQ ANSWERS

1.
1.1 Briefly describe the defect of cell division that causes Down syndrome (30)
A genetic disorder causing developmental and intellectual delays. Due to abnormal cell division resulting extra
chromosome 21, trisomy of 21 chromosome.
Two types,
1. Trisomy 21 (96%)- Extra copy of 21 in every cell. Maybe due to either nondisjunction of homologous
chromosomes in meiosis 1 or nondisjunction of sister chromatids in meiosis 11
2. Translocation (4%) – Due to Robertsonian Translocation. Children have only extra part of chromosome 21. 46
total chromosomes but 13/14/15/21/22 chromosomes has an extra chromosome 21 attached. Down’s syndrome
causes distinct facial appearances, intellectual disability and developmental delays. Maybe associated with heart
and thyroids.

1.2 Describe the fluid mosaic model of the eukaryotic cell membrane structure (40)
• Resemble a continually moving sea of fluid lipids that contain a mosaic of many different proteins. 7.5- 10nm
thick.
• Basic structural framework is the phospholipid bilayer.
• Thin, pliable, elastic structure.
Presented by 15th Batch 101 FHCS | EUSL
• 2 back-to-back layers made up of three types of lipid molecules—phospholipids (75%), cholesterol (20%), and
glycolipids (5%) .
• Amphipathic- have both,
→ Polar hydrophilic head (outward)
→ Nonpolar hydrophobic tail (inner)
• Membrane fluidity depends both o on the number of double bonds in the fatty acid tails of the lipids that make
up the bilayer (Each double bond puts a “kink” in the fatty acid tail) , and o on the amount of cholesterol present.
• Different proteins responsible for uniqueness of different membranes
• Proteins can be just on the surface,
1. peripheral proteins
→ Loosely bound to surface, often attached to integral proteins (eg.- enzymes) • Or embedded in the membrane,
2. integral proteins,
→ A few integral proteins are tightly attached to one side of the bilayer by covalent bonding to fatty acids. Like
membrane lipids, integral membrane proteins are amphipathic.
→ Some transmembrane proteins which span the thickness of bilayer.(channel pores, carrier proteins)
• Carbohydrates makeup about 3 %
• Combined with proteins or lipids (glycolipids, Proteoglycans)
• Form glycocalyxes

1.3 Outline the stages of cell signalling mechanism (30)


Three stages;
1. Reception
2. Transduction
3. Response

1. Reception;
two components ‐ Receptor is a protein molecule that receive chemical signal. They trigger signalling cascade. Can
be upregulated or downregulated. Trigger signalling cascade.
I. Intracellular receptors
II. Cell surface receptors; Channel- linked receptors Enzyme linked receptors G- protein coupled receptors ‐
Ligand is the molecule that bind to receptor.(Eg.-Neurotransmitter)
2. Transduction ‐ Relay molecules resulting from signal receptor binding ‐ Second messengers pass the signal
initiated by the first messenger (Eg.- Ca2+, IP3 )
3. Response Can be,
l. Gene expression
ll. Cellular metabolism
‐ Gene expression is the process by which information from a gene used to produce functional protein
‐ Example for cellular metabolism is Adrenalin preparing the body for short term emergencies

12TH BATCH (EME) - MCQ

1) Following lipids are major components of Eukaryotic cell membrane,


a. Cholesterol
b. Glycoprotein
c. Phospholipids
d. Prostaglandins
e. Spingolipids
Presented by 15th Batch 102 FHCS | EUSL
2) Mitochondria,
a. Found in hepatocytes less in number
b. Generate ATP through aerobic respiration
c. have smooth internal membrane
d. have ability for self-replication
e. transmit the inheritance diseases from mother

3) Cyclin dependant protein kinases (CDK’s) regulate the cell cycle. That check,
a. Microtubule attached to the centromere
b. DNA damage
c. Cell size and grow
d. Number of times the cell can divide
e. Cell size and nutrients

4) Carbonic Anhydrase present in,


a. Gastric peri interstitium cells
b. Interstitial cells
c. Plasma
d. Red blood cell
e. Colleting duct cell

5) What happens when the osmolarity of ECF increased?


a. In fall in intercellular volume
b. Decrease water absorption from kidney
c. Stimulate ADH secretion
d. Stimulate Osmoreceptors
e. Thirst

6) Which one of following brings the body temperature to normal when it is high?
a. Raising of skin hair
b. Redistribution of blood flow to periphery
c. Shivering
d. Sweating
e. Thermogenesis

7) Cellular transport mechanisms,


a. Active transport mechanisms occur against concentration gradient
b. Antiporters involve in secondary active transport
c. Diffusion across human cell membrane is an energy dependant process
d. Facilitated diffusion aids in transport of polar molecules
e. Osmosis is the movement of solutes across the membrane

8) Regarding intracellular communication,


a. cAMP activates G protein, leading and exchange GTP to GDP
b. cAMP inactivates protein kinase
c. G proteins have two identical subunits
d. Majority of the enzyme link receptors are protein kinases
e. Signal transduction pathway serve to amplify an effect on the extracellular signaling

Presented by 15th Batch 103 FHCS | EUSL


9) Regarding extracellular fluid,
a. Volume is higher than intracellular fluid
b. Tonicity is greater than intracellular fluid
c. Ph is greater than intracellular fluid
d. Na+/ K+ ratio is higher than intercellular fluid
e. Anions are mainly inorganic than intracellular fluid

10) Regarding to the acid base state,


a. An acid is a substance which can dissociate and produce H+
b. Bases accept protons
c. Weak bases are completely dissociate in high Ph
d. Buffering is the process by which a strong acid is replaced by a base
e. Most of the metabolic intermediate are week acids

SBR
11) What is the predominant buffer in interstitial fluid,
a. Cl-
b. HCO3-
c. PO43-
d. Proteins
e. Sulfate ion

12) The largest fluid volume found in the,


a. Extra cellular fluid (ECF) compartment
b. Interstitial fluid space
c. Intra cellular fluid( ICF) compartment
d. Plasma
e. Third space

13) Plasma and interstitial fluid are included in extra cellular fluid. Most significant feature seen in plasma
compared to interstitial fluid,
a. Plasma contain high amount of sodium ion than interstitial fluid.
b. Plasma has higher buffering capacity
c. Plasma has more protein than ISF
d. Plasma is electrically more negative
e. Plasma is more viscous than that of interstitial fluid

14) A patient is admitted to the hospital emergency treatment ETU.A blood gas analysis was performed the results
are follow
P O2 12.4 (11- 13)
P CO2 55(47-60)
Ph 7.29 (7.35- 7.45)
HCO3- 15 (22-36)
a. Compensated metabolic acidosis
b. Metabolic acidosis
c. Metabolic alkalosis
d. Respiratory acidosis
e. Respiratory alkalosis

15) Which of the following statement best describe homeostasis?

Presented by 15th Batch 104 FHCS | EUSL


a. A positive feed back mechanism
b. Constantly achieving dynamic equilibrium
c. Altering the external environment according to bodies need
d. Keeping the body in a fixed unaltered state
e. Maintaining a near constant internal environment

12TH BATCH EME – SEQ

01)
1.1 Briefly describe the formation of Trisomy 21 during the meiosis. (30)
1.2 Briefly describe the role of protein in plasma membrane
1.3 Outline the types of cell surface receptor with example(15)
1.4 briefly explain the role of free radicals in cellular aging (25)
2. 2.1 List the four starling forces
2.2 Using diagram, illustrate the role of these forces in environment
2.3 Cardiac failure results in increased capillary venous pressures
2.4 Outline the clinical features of dehydration on physiological basis

12TH BATCH EME – SEQ ANSWERS

1.2 Briefly describe the role of protein in plasma membrane


➢ Transport, Transport the molecule across the membrane
• Channel proteins – transport certain polar molecules & ions Eg – Aquaporins (allow only passive transport)
• Carrier protein – holding on to their passenger & change the shape in a way that shuttles them across the
membrane. (allow for passive & active)
➢ Enzyme, Some protein in the membrane may expose their active site to speed up a chemical reaction. ➢
Receptor Eg – act as some hormone receptors (Insulin receptors)
➢ Cell to cell recognition, Glycoprotein in cell surface
➢ Intra cellular joining
➢ Attachment to the cytoskeleton Intra and extra cellular matrix

1.3 Outline the types of cell surface receptor with example(15)


➢ Channel link receptor o Eg – neurotransmitters
➢ Enzyme link receptors o Protein kinases
➢ G protein coupled receptors o Beta adrenergic receptor

1.4 briefly explain the role of free radicals in cellular aging (25)
Many changes that occur in our body are caused by free radicals. Free radicals have a free electron which bind to
other essential molecules to stabilize themselves. This cause cells not to perform their normal functions. They can
damage DNA, proteins, lipids, cell membranes and other parts of human cells. Damage to DNA results in production
of ineffective proteins and cell death. Damage to proteins causes disease directly. Therefore it accelerate the aging
process.

2. 2.1 List the four starling forces


➢ Hydrostatic pressure of the capillaries = Pc
➢ Hydrostatic pressure of the ECF (interstitial fluid) = Pi
Presented by 15th Batch 105 FHCS | EUSL
➢ Oncotic pressure of capillaries = πc
➢ Oncotic pressure of ISF = πi

2.2 Using diagram, illustrate the role of these forces in environment

2.3 Cardiac failure results in increased capillary venous pressures


Cardiac failure
Right heart failure
Blood pooling in right atrium
Pooling in SVC /IVC
Blood pooling in venular end
Increase Hydrostatic pressure in venular end

2.4 Outline the clinical features of dehydration on physiological basis

13TH BATCH EME – MCQ

1) Cell Membrane
A. Consist of lipid bilayer
B. A constant composition of protein throughout life of the cell
C. Cell membrane is a dynamic structure
D. Is freely permeable to protein
E. Protein structure and enzyme content is varies cell to cell

2) Integral protein in the cell membrane can function as


A. Carrier protein
B. Channel protein
C. Enzyme
D. Hormones
E. Receptors

3) Rough endoplasmic reticulum


A. Most abundant in enzyme synthesis cells
B. Continue with nuclear membrane
C. Detoxification of alcohol in the liver
D. Ribosomes in cytoplasmic membrane
E. Is involved to breakdown of glycogen

4) Nucleus
A. Controls activities in the cell
B. Has a nuclear organizing region to reform the nucleolus in telophase
C. Has more euchromatin in metabolically active cells
D. There are pores in nuclear membrane to transport ribosomes
E. Present in mature RBC

Presented by 15th Batch 106 FHCS | EUSL


5) Neuronal ligand molecule/s include/s
A. Acetylcholine
B. Growth factors
C. Histamine
D. Insulin
E. Serotonin ration

6) Regarding Body Fluid


A. Blood has no ECF
B. In 70Kg adult man, Total body water about 60% of total body weight
C. Intra cellular fluid contain 40% out of total body weight
D. Plasma is the largest compartment of ECF
E. Volume of fluid compartment can be measured by indicator diluted method

7) What are the composition of ORS?


A. HCO3 –
B. Citrate
C. Glucose
D. Na+
E. K +

8) Examples for the hypertonic dehydration


A. Burn
B. Diabetes incipits
C. Fever
D. Hemorrhage
E. Sweating

9) T/F regarding extra cellular oedema


A. Blockage of lymph
B. Burn
C. Chronic renal failure
D. Heart failure
E. Hyponatremia

10) Examples for the second messengers


A. Ca2+
B. cAMP
C. Diacylglycerol
D. Nitric Oxide
E. CO2

SBR
11) What Contributes least to the osmolarity of plasma
A. Cl
B. Glucose
C. Protein
D. Sodium
E. Urea

Presented by 15th Batch 107 FHCS | EUSL


12) A patient was transfused with a high volume of a solution which made his RBC lysis This solution is mostly like,
A. Hypertonic NaCl
B. Hypertonic urea
C. Hypotonic Urea
D. Isotonic Mannitol
E. Isotonic NaCl

13) Which from the followings is incorrect about the total body water
A. Is measured by the calculating the dilution of source in the compartment
B. Constitutes about 55% of the body weight in adult male
C. Mainly in intracellular compartments
D. Proportionally high in infants than elders
E. Proportionally low in women than men

14) which of the following is not a ligand?


A. Aspirin
B. Glycogen phosphorylase
C. Growth hormone
D. Neuronal Transmitters
E. Viruses

15) Which following method that loss water without losing solutes?
A. Faces loss
B. Gastric juice
C. Insensible perspiration
D. Pancreatic juice
E. Sweat

13TH BATCH EME – SEQ

1.
1.1. Write an Account on the cell cycle including the check points (30 Marks)
1.2.
1.2.1. Briefly describe the functions of eukaryotic cell membrane. (25 Marks)
1.2.2. outline the G protein coupled mechanism in the signal transduction pathway with an example (25 Marks)
1.2.3. state the Factors Affecting cellular ageing process (20 marks)

2.
2.1. Explain he physiological basis of formation of oedema in heart failure (50 marks)
2.2. Explain briefly the effect of adding following solutions to the extra cellular fluid
2.2.1. Isotonic saline (25 marks)
2.2.2. Hypotonic saline (25 marks)

13TH BATCH EME – MCQ ANSWERS

01)
AT
Presented by 15th Batch 108 FHCS | EUSL
BF
CT
DF
ET

02)
AT
Transporting substances that could not penetrate the layer
BT
Water, water soluble substances, ions can diffuse between the cells. Also have selective properties .
CT
DF
ET
Receptors for peptide hormones.

03)
AT
BT
CF
Smooth endoplasmic reticulum contains enzymes for metabolism and detoxification of substances in the
hepatocytes such as drugs, hormones and alcohol.
DT
Presence of polyribosomes on the cytoplasmic surface give them a rough granular appearance.
EF
SER of hepatocytes contains glucose-6-phosphatase enzyme which convert the glycogen in to glucose in liver cells .

04)
AT
BT
During the cell division the nucleoli disappear and around the nuclear organizing region, nucleoli reform during
telophase of cell division.
CT
Euchromatin is metabolically active and involve in protein synthesis.
D
EF

05)

AT
BF
CT
DF
ET

06)
AF
Total body water -: 60% of body weight ➢ Intra cellular fluid: 2/3 of TBW ➢ Extra cellular fluid: 1/3 of TBW:
interstitial tissue 75% of ECF : plasma 25% of ECF
BT
CT
Presented by 15th Batch 109 FHCS | EUSL
DF
ET

07)
A F Content of ORS: ➢ Sodium chloride ➢ Glucose ➢ Potassium chloride ➢ Trisodium citrate
BT
CT
DT
ET

08)
AF
❖ Isotonic dehydration Loss of water and electrolytes in equal proportion.
EG-: burns, vomiting, haemorrhage, diarrhea Treatment -> replace the loss of isotonic solution. 0.9 % saline ,
Hartmann’s solution , 5%dextrose
❖ Hypertonic dehydration Loss of water excess of solutes.
EG-: fever , severe sweating , diabetes insipidus Treatment -> infusion of hypotonic solutions. 5% dextrose (later
become hypotonic) , 0.45% saline
❖ Hypotonic dehydration Loss of electrolytes in excess water . water moves in to the cells . EG-: excessive use of
diuretics , hyperglycemia Treatment -> hypertonic saline
BT
CT
DF
ET

09)
AT
Blockage of lymph return cause for oedema.
BT
Increase capillary permeability
CT
DT
Increase capillary pressure due to high venous pressure.
E

10)
AT
BT
CT
DT
EF

11)
Answer E

12)
Answer C
fluid moves in to the cells – swelling of cells causes for lysis .

13)
Presented by 15th Batch 110 FHCS | EUSL
Answer B
Total body water – 60% of body weight Intracellular fluid 2/3 of TBW Extra cellular fluid 1/3 of TBW
❖ PHYSIOLOGICAL VARIATIONS IN TBW
Age – TBW as a percentage of body weight decrease with age. At birth TBW – 80-85% of body weight. Male >
female
Fat content – greater fat content lesser the TBW as a % of bodyweight. HYDROPHOBIC

14) Answer B
A molecule that bind to a receptor is called a ligand , and can be a peptide ( short – protein) or another small
molecule such as a neurotransmitter, hormone, oharmaceutical- drug, toxin or parts of the outside of a virus or
microbe.

15) Answer E
Loss of water excess of solute – Hypertonic dehydration ECF Water moves from ICF to ECF ICF Volume decreased
decreased osmolality increased increased

14TH BATCH EME – MCQ

01. Smooth endoplasmic reticulum,


A) Abundant in steroid hormones synthesizing cells.
B) Continuous with the nuclear membrane.
C) Detoxification of alcohol in the liver.
D) Has ribosomes on its surfaces.
E) Involve Ca2+ regulation of skeletal muscles.

02. Daughter cells of mitosis,


A) Identical to each other.
B) Small in size.
C) Half number of chromosomes in parent cells.
D) Prepare to enter the next cell cycle at G1 phase.
E) Will replace dead cells.

03. Regarding characteristics of secondary active transport,


A) In symport, the solute in opposite direction across the cell membrane.
B) Na+ glucose transport represents antiport.
C) Na+ / K+ ATPase inhibit Na+ across the cell membrane.
D) Solute transported downhill will provide energy for uphill transport of other solutes.
E) Transport of two or more solute are coupled.

04. Compare electrolytes & fluid in the body,


A) Intracellular have more protein anions than plasma.
B) Percentage of EC body water in infants is greater than in 5 years old child.
C) Percentage of total body water high in obesity than in normal healthy person.
D) The most abundant cation in ICF is K+
Presented by 15th Batch 111 FHCS | EUSL
E) Total anion concentration is higher in ICF than plasma.

05. Regarding plasma osmolarity,


A) Change in osmolarity will stimulate aortic & carotid bodies.
B) Hypoalbuminemia results hypo osmolar plasma.
C) Increased osmolarity of plasma results concentrated urine in a healthy person.
D) Infusing 2L of 0.45% NaCl will increase plasma osmolarity.
E) Plasma osmolarity decrease will increase the release of ADH.

06. After administration of hypertonic solution nurses should concern about,


A) Fluid volume deficit.
B) Formation of pulmonary oedema.
C) Increasing blood oedema pressure.
D) Increasing lactate level.
E) Sign of dehydration.

07. In a patient with metabolic acidosis, a decreasing can be observed in?


A) Plasma PH
B) Pco2 of arterial blood.
C) Plasma HCO3-concentration.
D) Respiratory rate
E) Reabsorption of HCO3- in renal tubules.

08. Physiological buffers is/are,


A) Bicarbonate
B) Hemoglobin
C) Phosphate
D) Sodium ion
E) Triglycerides

09. Intracellular receptors,


A) Are activates by lipophilic signaling molecules.
B) Receptors lead to the activation of signaling cascades.
C) More abundant in mitochondria.
D) Are located only in the cytoplasm.
E) Mostly comprise a receptor protein bound to an inhibitory protein complex.

10. Correct the match components of cellular membrane and their role,
A) Cholesterol-control fluidity of cell membrane.
B) Glycolipid-cell recognition.
C) Integral proteins-enzyme
D) Peripheral protein-control transport of substances.
E) Sphingolipid-signal transmission

SBR.
11. Because of the respiratory obstruction disease, the Pco2 level of the plasma increases (40-60) and plasma PH
reduced from 7.4-7.0. What could be the result of the
condition?
A) Decrease plasma HCO3
B) H+ Concentration in the urine is increased.

Presented by 15th Batch 112 FHCS | EUSL


C) Ammonium secretion of the urine is decreased.
D) Arterial chemoreceptors are inhibited.
E) H+ions are buffered by non-bicarbonate buffer systems.

12. Which process is involved, when transporting glucose from the intestinal tube to the epithelial cell is inhibited,
and the Na+ transportation across the cell membrane along the concentration gradient is also disturbed?
A) Co transport
B) Counter transport
C) Facilitated diffusion
D) Secondary active transport
E) Simple diffusion

13. Homeostasis is achieved through feedback mechanism. Which is not true regarding positive feedback
mechanism?
A) Positive feedback mechanism results in amplification of output signals.
B) Involve in regulation of blood clot formation pathway.
C) Involve in regulation of homeostasis which constrained in time frame.
D) Involve with minimizing the initial intensification of stress.
E) It involves in breakdown of homeostasis of the system.

14. Which one is not a G – protein coupled receptor


A) Adrenergic
B) Glutamate
C) Glycine
D) Muscarinic
E) Receptors for peptide hormone

15. What are the factors that does not allow formation of free radicals.
A) Environment contamination.
B) Electron transport chain.
C) Excessive consumption of processed foods.
D) High intake of fruits and vegetables.
E) Ionizing and ultraviolet radiation

14TH BATCH EME – SEQ

1.
1.1. Briefly describe the possible errors occur during the cell division. (30 marks)
1.2. Describe the role of second messengers in cell signaling pathway with examples.
(30 marks)
1.3. Explain the biochemical basis of cellular aging process. (40 marks)
2. Disturbance in Starling forces lead to accumulation of fluid in extracellular compartment.
Explain the physiological basis of following condition in relation to disturbance in the
above forces.
2.1. Dyspnea (shortness of breath) and limb oedema with patient suffer with congestive
heart failure. (40 marks)
2.2. Generalized oedema with patient suffered with end stage liver. (cirrhosis) (20 marks)
2.3. Explain the physiological basis of using oral rehydration solution in Diarrhea.
(40 marks
Presented by 15th Batch 113 FHCS | EUSL
14TH BATCH EME – MCQ ANSWERS
01) Answer - T, F, T, F, T

a) mainly synthesizes steroids, cholesterol.

b) RER is continuous with nuclear membrane

c) theory

d)RER has ribosomes.

e) main store of calcium in muscles forming sarcoplasmic reticulum.

02) Answer - T, T, F, T, T

a) same with identical genetic material and shape, size.

b) small in size comparing with parent cell initially after mitosis due to equal division of cytoplasm.

c) chromosome number isn't halved.

a) d) main function of mitosis. helps to replace dead cells and wound healing.

03) Answer F, F, Nil, T, T

a) Transported in same direction.

b) Actually, it's a symport.

c) Question redundancy. (Either pumps or something should be there)

d) This is why secondary active is called as an active process though it doesn't use energy directly. Concentration
gradient of particular ion which is produced by active ion pumping is used to transport another substance against
its own concentration gradient.

e) 2 substance Ex: Na+ and glucose

b) 3 substances Ex: Na+/K+/2cl- co transport.

04) Answer T, T, F, T, F

a) More proteins in intracellular than plasma.

b) Initially its higher in infants than childhood but later ICF becomes dominates.

a) ECF>ICF infant

b) ECF< ICF childhood

c) so, infants need more concentration of fluid in treating dehydration.

d) water content is inversely proportional to body fat content. so high fat in obesity results in low water content.

e) theory

Presented by 15th Batch 114 FHCS | EUSL


f) e) If the total anion concentration defers within intra and extracellular compartments then there'll be a net
charge. But the polarized nature of plasma membrane is a minute fraction based. Not a net change in charge.

05) Answer F, F, T, F, F

a) Osmoreceptors are stimulated under such condition which are present in hypothalamus.

b) Contribution of proteins to osmolarity is less than 2 m osmoles. So, there won't be much change.

c) Due to the secretion of ADH in response to increased osmolarity will result in reabsorption water in renal tubules
in an increased manner so low water more substances ---> concentrated urine.

d) Already 0.45% is a hypotonic solution. Since it's taken as 2l almost it'll be more hypotonic in nature. Also,
retention of NaCl solution in blood will result in hypo-osmolar plasma.

a) e) Decrease in osmolarity will result in reduced release of ADH to elevate the osmolarity by excreting a dilute
urine (water>solute).

06) Answer F, T, T, T, F

a) In case of a solution of NaCl, it'll result in increasing blood osmolarity hence increasing ADH secretion which will
eventually lead to fluid excess.

b) fluid overload----> increased hydrostatic pressure and leads to edema. (pulmonary)

c) fluid overload and changes in net filtration pressure could lead to edema.

d) In case of glucose administration, it could lead to overload of glucose in cell and pyruvate via glycolysis and could
result in increased lactic acid formation.

e) e) unlikely to cause dehydration in fluid overloaded condition.

07) Answers T, T, T, F, F

a) In case of metabolic acidosis large acids are added or loss of base will result in excess of H+. So, the pH of blood
will be reduced.

b) In order to neutralize the excess H+, HCO3- which presents in blood will accept H+ and forms H2CO3. Hence it
will continue toform CO2 and water at lungs. So, CO2 will be exhaled via expiration. So, this will reduce the PCO2 of
arterial blood after circulation via lungs.

c) Since neutralizing H+ via HCO3- will be reduced and will be compensated from bicarbonate reservoir later.

d) Reduced plasma pH will simulate the respiratory centers and will result in hyperventilation or increased
respiratory rate and depth to remove H+ via CO2.

e) Here's a condition to conserve HCO3- to compensate the loss of HCO3- during the neutralization process occurs
in blood. So, for that renal tubular absorption of HCO3- will be enhanced.

08) Answer: T, T, T, F, F

Other than E all are physiological buffers present in body as intrinsic and extrinsic buffers. though triglycerides can
function as a buffer invitro it’s not a buffer available in vivo.
Presented by 15th Batch 115 FHCS | EUSL
09) Answer T, F, F, F, T

a) Lipophilic molecules are able to diffuse through the plasma membrane easily unlike water soluble molecules
which requires special transport carriers.

b) Binding of a signaling molecule to the receptor results in activation in extracellular receptors. But in case of
intracellular they're mostly accompanied by release of molecules which alters gene expression.

c) More abundant in nucleus.

d) Located in cytoplasm and nucleus.

e) e) upon binding release of inhibitory complex and that receptor protein binds with genome and alters gene
expression.

10) Answer T, T, T, T, T

a) main fluidity regulator temperature and cholesterol.

b) being components of receptors for polypeptide hormones

c) enzyme linked receptors (protein kinases)

d) theory

e) e) sphingolipid is a composition of myelin which function as an electrical insulator of nerve axons.

11) B is the answer.

because this is a condition called respiratory acidosis due to the obstruction of air way and reduced removal of CO2.
will result in favoring the bicarbonate buffer equation in forward direction hence will increase more HCO3- and
arterial chemoreceptors will remain stimulated despite the inability to inhale breath due to the obstruction. H+ will
be excreted via kidney and HCO3- will be preserved. And H+ are not buffered by non-bicarbonate buffer system at
this movement.

12) Answer A

The reason is since the both Na+ and glucose transported in same direction and transport occurs only if glucose is
present without any inhibition refers as co-transport.pumping activity of Na/K ATPase pump forms low intracellular
Na+ concentration hence Na+ concentration gradient across the membrane towards inside (+). By using this
gradient Na+ is transported following its concentration gradient (passive) but glucose is transported against its own
gradient, so whole process is active since the required Na+ gradient achieved by the active pumping of Na/K ATPase
pump.

13) A is the answer.

Because positive feedback mechanism amplifies output signals, involves in blood clot formation by releasing more
and more serotonin, and involves in homeostasis which should happen with a constrained time frame. also, it

Presented by 15th Batch 116 FHCS | EUSL


breaks down the homeostasis due to increasing the initial intensification of stress rather than minimizing Ex:
parturition, Lactation, blood clot formation

14 )Answer is C

15) Answer-D

a) environmental contamination means chemicals pesticide provokes damages of bio molecules and yielding free
radical

b) free electrons reacting with molecular oxygen in absence of enough H could result in ROS formation.

c) fruits and vegetables comprise antioxidants hence they help to get rid of free radicals.

b) d) radiation causes damages to cells and results in formation of free radical

14th batch EME seq

Presented by 15th Batch 117 FHCS | EUSL


Presented by 15th Batch 118 FHCS | EUSL
Presented by 15th Batch 119 FHCS | EUSL
Presented by 15th Batch 120 FHCS | EUSL
Presented by 15th Batch 121 FHCS | EUSL
Presented by 15th Batch 122 FHCS | EUSL
Presented by 15th Batch 123 FHCS | EUSL
15TH BATCH EME – MCQ

1) Regarding the functions of lysosome,


A. It carries out self-destruction in response to cell injury
B. Involved to bile secretion
C. It is involved in cellular signaling
D. It involved to breakdown of long chain fatty acid
E. It serves in defense by engulfing the pathogen

2) Regarding meiosis,
A. It generates genetic diversity
B. it is a cell division that produce haploid(n) sex cells
C. It occurs after DNA replication
D. Produce 2 daughter cells with same genetic materials
E. Responsible for subsequent growth and development of organisms

3) Golgi bodies / Golgi apparatus,


A. Are membrane bound flattened sacks which connects nucleus
B. Distribute secretory products as vesicles
C. Modifies protein into glycoproteins
D. Involved in protein synthesis
E. Involved in synthesis of lysosomes

4) Regarding cell membrane fluidity,


A. Cholesterol act as a fluidity buffer
B. Depend on unsaturated fatty acids on cell membrane
C. Facilitate the movement of the cellular materials during the cell division
D. Decreases with temperature
E. Lipid bilayer regulates fluidity bidirectionally

5) Simple diffusion,
A. Facilitates the movement of blood gasses through the cell membrane
B. Facilitates the movement of fat-soluble vitamins across the cell membrane
C. Is an active process
D. Moves the substances through the electrochemical gradient
E. Rate decreases in higher temperature

6) Plasma osmolarity,
A. Increase will decrease the release of ADH by pituitary gland
B. Increase with hypoproteinemia
C. Mainly determined by blood Na+, glucose and urea
D. Measures the hydration status of the body
E. Range between 280-300mOs/kg

7) Regarding dehydration,
A. It is caused by hyperventilation
B. Can occur hypovolemic shock leads to dysfunction

Presented by 15th Batch 124 FHCS | EUSL


C. It results in depletion of both intracellular and extracellular fluid
D. Results in hypotonic extra cellular fluid
E. Increase osmolarity leads to oliguria

8) Following conditions could lead to generalized edema,


A. Congestive heart disease
B. Liver cirrhosis
C. Local inflammation
D. Lymphatic Obstructing
E. Malnutrition

9) Signal molecules interact with cell surface receptors,


A. Cortisol
B. Gastrin
C. Glucagon
D. Insulin
E. Testosterone

10) Regarding to G Proteins,


A. G protein involves in signal cascade
B. They are peripheral membrane Proteins
C. Beta sub unit bind with G protein
D. Guanine nucleotides regulate G proteins
E. Adrenergic receptors act through the G Proteins

SBR
11) What is the following are important for glucose absorption into the intestine?
A. Cotransport
B. Counter transport
C. Facilitated diffusion
D. Primary active transport
E. Simple diffusion

12) Which of the following is a cause of metabolic alkalosis?


A. Chronic renal failure
B. Diarrhea
C. Hyperventilation
D. Respiratory tract disorder
E. Vomiting

13) Capillary filtration,


A. Decreases when interstitial oncotic pressure increases
B. Increase lymphatic obstruction
C. Is increase in protein malnutrition
D. Cause of hypoproteinemia
E. Decrease when hydrostatic pressure in Capillary increase

14) Which of the statement is not true regarding composition of body fluid?

Presented by 15th Batch 125 FHCS | EUSL


A. Clis a major anion in extra cellular
B. Intra cellular anion concentration is lower than plasma anion concentration
C. k+is major cation in intracellular fluid
D. Body water in infants greater than adults.
E. Most abundant intracellular cation is K+

15) Which is the correct order of cell signaling pathway,


1- Transport of signals to a target
2- Start of transduction pathway
3- Secrete the signaling molecule
4- Ligand bind with specific receptor

A. 1,3,2,4
B. 1,4,2,3
C. 3,2,1,4
D. 3,1,4,2
E. 4,2,3,1

15TH BATCH EME – SEQ

1.
1.1 State the function of microtubules in cell division. (15marks)
1.2 Briefly describe the different composition of eukaryotic cell membrane and their role. (25 marks)
1.3 State the importance of telomeres on cellular aging process. (10 marks)
1.4 Explain the physiology behind the conditions
1.4.1 Person having higher ADH secretion due to Syndrome of Inappropriate Antidiuretic Hormone
secretion (SIDAH) develops seizures due to acute oedema in brain cells. (25marks)
1.4.2 A patient admitted with severe episodes of vomiting was found to have high arterial HCO3
and PCO2

15TH BATCH EME – MCQ ANSWERS

01) True – autolysis


False – peroxisome
True
False – peroxisome
True – phagocytosis

Presented by 15th Batch 126 FHCS | EUSL


02) True
True
True – whether meiosis or mitosis occurs after DNA replication
False – it happens in mitosis , in meiosis 4 daughter cells with different genetic material
False – mitosis is responsible for growth and development

03) False – membrane bound flattened sacs which connects nucleus is RER
True
True- modification of protein ( glycoprotein, lipoprotein)
False – protein synthesis occurs in ribosome
True- lysosome is secreted by golgi

04) True
True- fluidity depends on number of unsaturated fatty acids, amount of cholesterol,
temperature
True- fluidity allows interaction with plasma membrane, enables the movement, self steal role
False- increase with temperature
False- cholesterol

05) True – blood gases can cross lipid bilayer


True- fat soluble vitamins can cross lipid bilayer
False- simple diffusion is passive process
True- simple diffusion occurs through high electrochemical gradient to low gradient
False- higher temperature increases the rate of diffusion

06) False- if osmolarity increased water should be reabsorbed , so ADH release increases
False- low level of protein decrease the osmolarity
True- osmolarity is determined by electro chemicals , glucose and urea
True
True
07) True – hyperventilation can cause dehydration
False- hypovolemic shock leads to hypovolemia not dehydration
True- during dehydration ECF water reduced , so osmolarity increased so water flows from ICF to ECF , so
both the ECF,ICF volume will be reduced
False-hypertonic extra cellular fluid
True- increased osmolarity cause increased ADH , so high amount of water will reabsorbed, so
concentrated urine . so it will lead to oliguria .
Oliguria means low urine output

08) True
True
False – localized edema
False- localized edema
True

09) False- steroid hormone ( intracellular receptor )


True
True
True
False- steroid ( intracellular receptor )

Presented by 15th Batch 127 FHCS | EUSL


Except steroid hormones and thyroxine others have cell surface receptor
Thyroxine has receptor in nucleus

10) True
False- G protein is an integral protein
False- alpha subunit binds with G protein
True
True – adrenergic , muscarinic ,metabotropic glutamate receptors and receptors for peptide hormone act
through G protein receptors

11) Answer A
In the intestine glucose is absorbed by co transport

12) Answer E
Vomiting cause depletion of H + ions , it cause metabolic alkalosis

13) Answer C
In protein malnutrition capillary oncotic pressure is decreased . so fluid reabsorption will decrease , so
filtration is increased

14) Answer B
Intracellular anion concentration is higher than plasma , so intracellular is more negative.

15) Answer D

15TH BATCH EME – SEQ ANSWERS

1.1.

Microtubules play a vital role in cell division .


In the prophase microtubule form the spindle apparatus and ensure the proper segregation of duplicated
chromosome.
Microtubules bind with chromosome and establish the cellular polarity.
In the metaphase of cell division microtubule binds with kinetochore of the chromosome and make the
chromosome lie in the metaphase plate .
Some microtubules not attached to chromosome and they just lie end to end of the cell and they prevent the
collapsing of the chromosome.
In the anaphase microtubule pull the each sister chromatids to the edge of the cell.
Finally in telophase microtubules and spindle disappear .

1.2 .
Composition of cell membrane
Protein -55%
Phospholipid -25%
Cholesterol -13%
Other lipids -4%
Carbohydrates -3%

Presented by 15th Batch 128 FHCS | EUSL


Phospholipid –
Lipid bilayer act as a barrier to the passage of molecules and ions into and out of the cell.
It is impermeable to water soluble molecules and permeable to lipid soluble molecules .

Sphingolipid –
Protect from harmful environmental factor
Signal transmission
Adhesion site for extra cellular proteins

Cholesterol _
Controls and maintain the membrane fluidity

Proteins-
Act as channel proteins and carrier proteins
Act as receptors for peptide hormones
Act as enzymes
Support in signal transduction
Cell cell recognition

Carbohydrates –
Play a key role in cell cell recognition
Act as antigen
Important in organ and tissue development

1.3.
Telomeres are protective structures present at the end of the chromosome .
Telomeres shorten as a result of cellular replication .
When the telomeres become too short , the cell enters the senescence stage .
Telomeres regulate how many times an individual cell can divide.
Telomeric sequences shorten each time the DNA replicates.
Once the telomere shrinks to a certain level , the cell can no longer divide.
It metabolism slows down .
It ages and dies .

1.4.
1.4.1.
This person has higher ADH secretion .
Normaly ADH reabsorp water in distal convoluted tubule and collecting duct in nephron.
Higher ADH secretion will lead high reabsorption of water in kidney.
So water level in blood will increase.
Blood osmolarity is decreased than normal level.
So the water will shift into cell and interstitial fluid.
So there is an abnormal accumulation of fluid in interstitial fluid and intracellular fluid.
It will lead to edema.
This occurs in brain cells also in SIADH .
So the brain cells will swallow and cause edema in brain cells and develops seizures.

1.4.2.
Patient admitted with severe episodes of vomiting .

Presented by 15th Batch 129 FHCS | EUSL


He lost H+ ions in gastric acid .
So he has low level of H+ ions and developed metabolic alkalosis.

CO2+ H2O >>>>H2CO3 >>>>HCO3- + H+

To compensate it , this equation will shift forward to increase H+ level.


To compensate it HCO3- will be reabsorbed in kidney.
So HC03- in body will increase more than normal level.

Due to this patient has high arterial blood HCO3-

Also , to compensate it the patient will develop hypoventilation .


So CO2 can’t efficiently throw out.
So there will be high level of CO2 in blood .
So the Pco2 is high than normal level.
So this patient has high PCO2

16TH BATCH EME – MCQ

01. Regarding starch


A. Amylopectin makes around 70-80% of starch.
B. Increase diet of refined starch increase the risk of diabetes
C. Polymers of fructose has 1-4 glycosidic bond
D. This is high in leafy vegetable
E. They show positive results for Molish’s test

02. Match the specific inhibitor for electron transport chain complex
A. ATP synthase- Antimycin
B. Cytochrome C oxidase- cyanide
C. Cytochrome C reductase- Oligomycin
D. NADH dehydrogenase- Rotenone
E. Succinate dehydrogenase - Carbon monoxide

03. Regarding lipid


A. Arachidonic acid is a derivative of linolenic acid
B. Lauric acid predominant in coconut oil
C. Aspirin inhibits the cyclooxygenase pathway
D. Oleic acid is an omega-3 fatty acid
E. Phosphatidyl choline is component of the cellular membrane

04. Regarding glycoproteins


A. Albumin is protective glycoprotein
B. Lysosomal acid hydrolases are responsible for the degradation of glycoproteins
C. The glycan attached to the protein only through O- link glycosidic bond
D. Act as a cell surface antigenicy
E. They are proteins to which small oligosaccharide chains are attached
Presented by 15th Batch 130 FHCS | EUSL
05. The stability of primary structure of protein is supported by
A. Hydrogen bond
B. Peptide bond
C. Disulfide bond
D. Ionic bond
E. Ester bond

06. Regarding peptide bonds in proteins,


A. It has partial double bond character
B. Ionized at physiological PH
C. Cleaved by the agent that denature protein
D. Stable in heat with strong acids
E. Occurs mainly in trans configuration

07. The active site of an enzyme


A. Covalently binds with substrate
B. Contain polypeptide chain
C. Is small relative to the total bulk of enzyme
D. Is a restricted region
E. Formed by folding of proteins

08. True/ False


A. DNA Double strands are antiparallel
B. Messenger RNA provide blue print for the protein synthesis
C. Messenger RNA has anticodon
D. Transfer RNA has codon
E. RNA mostly found in cytoplasm

09. Which of the following is a purine?


A. Uracil
B. Cytosine
C. Thymine
D. Adenine
E. Adenosine

10. Regarding Restriction endonuclease enzyme,


A. Cut the nucleotide in a special region
B. Are named according to bacteria from which organism is isolated
C. Recognize the DNA sequence in palindromic nature
D. Cut the DNA randomly
E. Found in human body

SBR
11. Precursor of eicosanoids
A. C18-C20 saturated fatty acid
B. C18-C20 polyunsaturated fatty acid
C. C20-C22 saturated fatty acid
D. C20-C22 polyunsaturated fatty acid
E. C20-C22 saturated fatty acid

Presented by 15th Batch 131 FHCS | EUSL


12. Examples for epimers
A. Glucose & Ribose
B. Glucose & Galactose
C. Galactose, mannose & glucose
D. Glucose, Ribose & Mannose
E. Glucose & fructose

13. What is the Krebs’s cycle enzyme found in inter membrane space?
A. Citrate synthase
B. Malate dehydrogenase
C. Iso-citrate dehydrogenase
D. Succinate dehydrogenase
E. α- ketoglutarate dehydrogenase

14. Production and secretion of trypsin is inhibited in pancreas disease. Hydrolysis which compound?
A. Proteins
B. Lipids
C. Carbohydrate
D. Nucleic acid
E. Phospholipid

15. Enzyme that helps to join Okazaki fragments


A. DNA polymerase
B. Ligase
C. RNA Polymerase
D. Peptidyl transferase
E. Topoisomerase

16TH BATCH EME – SEQ

1.
1.1. Outline the mechanism of oxidative phosphorylation (25 marks)
1.2. State the function of different types of lipoproteins (15 marks)
1.3. List three (03) consequences occurred due to deficiency of disaccharidases enzymes in human (10 marks)
1.4. List the functions of any 5 enzymes or protein required for DNA replication (15 marks)
1.5. Briefly describe the beta pleated sheet structure of protein (20 marks)
1.6. Mention five (05) enzymes used as therapeutic agent (15 marks)

Answers
01.
A. (T) - Leads to heavy loss of water through expiration
B.
C. (F) - More water loss than solute. So it increase plasma osmolarity
D. (T) - Both ICF & ECF depletion leads to dehydration
E.

02.

Presented by 15th Batch 132 FHCS | EUSL


A. (F) - Form similar daughter cells
B. (F) - Produce diploid cells
C. (T) - Mitosis is similar to meiosis 2
D. (T) - It produces genetically identical cells
E. (T) - Mitosis responsible for growth and development of organism.

03.
A. (T) - It has mitochondrial DNA
B. (F) - Glycosylation of proteins happen intracellularly
C. (T) - It has ribosomes. It synthesis protein for its own growth
D. (F) - Golgi body synthesis lysosomes
E. (T) - It’s a double membranous organelle

04.
A. (T) - Macromolecules are captured in vesicle on one side of the cell, drawn across the cell & ejected on the other
side
B. (F) - Gases pass through diffusion
C. (T) - Secondary active transport (Bulk transport)
D. (T)
E. (F) - Against the electrochemical transport (Active transport)

05.
A. (T)
B. (T)
C. (T)
D. (F) - When it’s cold they are found closer together. So decrease in low temperature.
E. F - cholesterol regulates the fluid bidirectionaly

06.
A. (T) - In our body
Solids- 40%
Liquid -60%
ECF – 20% Here Interstitial fluid -80%
Plasma-20%
ICF-40%
B. (F) - Na+ and k+ is high in the interstitial fluid but protein is very low so cl- is very low
C. (F) - In plasma, plasma proteins are there. But Interstitial fluid low Protein are there (low Protein is right
D.
E. (F) - proteins are high in the cell not ECF .

07.
A. (F) - congestive heart failure – It leads to increase increase capillary hydrostatic pressure mainly in the venous
fluid movement is increased Into interstitial fluid ( But here it is generalized edema ,not localized
B. (T) - When flea bites swelling is happen on that area
C. (T)
D. (F) - generalized edema … reduced capillary oncotic pressure
E. (T) - fluid accumulation of interstitial fluid is high

08. plasma oncotic pressure is determined by plasma proteins


A. F

Presented by 15th Batch 133 FHCS | EUSL


B. T
C. T
D. T
E. F

09.
A. collagen proteins
B.F
C.T
D.T
E.T

10.
A. T
B. F
C. T
D. T
E. T

11. answer B

12. (D)
Protein malabsorption leads to Decrease plasma oncotic pressure so capillary filtration will increase

13. (D)
When we Intake Isotonic solution which increase the cardiac output, Increase the flow rate of kidney, filtrate
increase,excreate urine will increase

14. (B)
Testosterone is a hydrophobic signaling molecule and Adrenaline is example for hydrophilic molecule.
Adrenaline actives G-Protein coupled receptors in many different tissues.

15. (A)
Cytokinesis is the physical process of cell division, which divides the cytoplasm of a parent cell into two daughter
cells.Karyokinesis is the step during cell division where the nucleus divides to form 2 daughter nuclei.

16th batch seq EME


01.)
1.1 ) Function of lipids in cell membrane
The cell membrane is continuous over the entire cell surface. Lipid bilayer composed of
1. Phospholipid
2. Sphingolipids
3. Cholesterol

Role of phospholipid
lipid bilayer acts as a barrier to the passage of molecules and ions into and out of the cell.
The lipid layer in the middle of membrane is impermeable to water soluble substances (ions, glucose and urea) 2 &
permeable to lipid soluble substance.
Role of Sphingolipids on CM
Presented by 15th Batch 134 FHCS | EUSL
• Structural components
• Protection from harmful environmental factors
• Signal transmission
• Adhesion sites for extracellular proteins

Role of cholesterol in CM
It helps in determine the degree of permeability of the bilayer to water soluble constituents of body fluid and
Controls much of the fluidity of the membrane

1.2) Role of calcium ion in signal transduction pathway


Second messengers are short lived intercellular signaling molecules which become active after the binding of first
messengers and cause a cellular response after a cascade of events.
Calcium is a hydrophilic second messenger. It is stored in endoplasmic reticulum or is present in the extra cellular
environment.
G-protein couple receptors(GPCRs) mediate the majority of cellular responses to external stimuli. Upon activation
by a ligand, the receptor binds to a partner heterotrimeric Gprotein and promotes exchange of GTP for GDP,
leading to dissociation of the G-protein into
alpha and beta gamma subunits that mediate downstream signals.
On binding of a ligand and after the conformation of GPCR and dissociation of the subunits and activation of GTP
by phosphorylation and attachment of alpha subunit to Phospholipase C leads to dissociation of PIP2 (
Phosphatidylinositol 4,5-bisphosphate) into IP3(Inositol triphosphate) and DAG ( Diacylglycerol).
IP3 acts as an intracellular ligand binding onto the receptors on calcium releasing channels on the membranes of
endoplasmic reticulum, releasing calcium ions from within the endoplasmic reticulum which is the site of the
temporary storage to the cytoplasm. This calcium ion causes intracellular cell response such as cell contraction,
secretion, metabolism etc.

1.3) Proto-oncogene
An oncogene is a gene that has the potential to cause cancer. In tumor cells, these genes are often mutated, or
expressed at high levels. Most normal cells will undergo a programmed form of rapid cell death when critical
functions are altered and malfunctioning.Proto-oncogenes normally regulate cell division, but can be changed into
oncogenes through mutation, which may cause cancers to form.
• gene mutation causes a change that increases the activity of a positive regulator.

1.4) A Patient diagnosed with chronic obstructive lung disease has the following arterial
blood values;
PH = 7
PCO2 = 50mm Hg [Normal range (35-45 mm Hg )]
[HCO3- =Normal range (22-26 mEq/L)]
Identify the acid base imbalance and explain the reason for observing above changes in the
blood test.
Chronic obstructive lung disease is a group of disease that cause airflow blockage and breathing related problems. It
includes emphysema, chronic bronchitis and asthma.
Normal arterial blood, PH = 7.35 -7.45
PCO2 = 35-45mmHg
[HCO3-]= 22-26 mEq/E
According to the patient’s blood test arterial blood pH is lower than the normal level. Therefore it is an acidosis
condition. But PCO2 and [HCO3-] are higher than the normal level. Therefore it can identify as a Respiratory
Acidosis.
The reasons for observing this changes are can’t expel CO2 properly, rate of CO2 generation exceeds than the rate
of CO2 removal, homeostasis disturbed by increasing PCO2 caused by hypoventilation.

Presented by 15th Batch 135 FHCS | EUSL


Then it becomes hypercapnia. It shifts carbonic acid- bicarbonate buffersystem to the right, generating more
carbonic acid and it dissolves in to
H+ and HCO3- . HCO3- goes in to bicarbonate reservoir. And also excess
H+ causes to decrease blood pH .Therefore from this patient can observe low blood pH in the blood test. Arterial
and CSF
chemoreceptors stimulated and the compensatory mechanisms are activated. Increased H+ secret by kidney and
reabsorb HCO3- ions. And also other buffer systems accept H+ ions more. But HCO3- more and more generates by
dissolving carbonic acid and reabsorbing from kidney. Therefore HCO3- concentration is higher than normal level.
CO2 can’t expel out because the lung disease. Therefore PCO2 level also higher than normal range.

1.5) Brain edema is caused by excess accumulation of fluid in intracellular or extracellular spaces of brain.
Hypertonic fluids have a greater osmotic pressure than the cell. When infused it raises the serum osmolarity
pressure which pulls fluid from cells and interstitial tissues into vascular space. It will reduce swelling of brain tissue
and will correct cerebral edema

PHASE 01 QUESTIONS

8th MCQ- Proper

1) Which of the following are true or false regarding buffers


A) Appearance by mixing weak acid with strong base
B) Main blood barrier is PO4
C) Urine buffered by HCO3
D) Hemoglobin acts as a buffer
E) A 0.5M buffer on 1:1 dilution changes PH

9) Cell membrane
A) Selectively permeable
B) Discharges particles by a process of vacuolization
C) is flexible due to presence of saturated fatty acids
D) Process Trans membrane proteins extending between cytosol and ECF
E) Consists of cholesterol molecules exposed to ECF

10) With regards to nucleus


A) During the interphase of cell cycle, chromatin exits as a tangled mass
B) Euchromatin is an electron lucent nuclear material
C) Nucleoli provide, site for messenger RNA synthesis
D) Nuclear envelope is a single membrane with multiple pores
E) Eosin is stained by binding DNA to make the nucleus prominent

11) Regarding cell division


A) Mitotic phase occupies most of cells cycle than interphase
B) Chromosome become visible at the end of prophase of mitosis
C) Oogonia multiply by mitosis during fetal development
D) In meiotic cell division of female, one gamete matures from a single diploid cell
Presented by 15th Batch 136 FHCS | EUSL
E) Barr body is produced by “Y” chromosome during meiosis

12) Na+-K+ ATPase pump


A) Maintains the electrical polarity of the cell membrane
B) All cells have in the body
C) Involves in secondary active transport
D) Involves in the Na+ absorption when there is aldosterone in the DCT
E) Active when two K+ in and one Na+ out

13) Examples of facilitated diffusion include


A) Transport of glucose into adipose tissue
B) Transport of amino acid into muscles
C) Excretion H+ in DCT
D) Water transport through aquaporin channel
E) Absorption of glucose in gut

14) Which of the following are correctly matched regarding body fluid compartment?
A) ECF- radioactive chromium
B) ECF- Inulin
C) ICF- sucrose
D) Plasma- radioactive iodine
E) Total body water- Deuterium

15) Normal Saline


A) Contains 0.9 M NaCl
B) is inferior to 5% dextrose in treating dehydration due to vomiting
C) ІV is inferior to “Jeevani” in mild diarrhea
D) ІV therapy may leads to edema new borns
E) is not suitable as an oral drug

SBR
46) Mitochondria
A) Are immobile organelles
B) Receives more protein from cytosol
C) Are abundant in hepatocytes
D) Are spare in muscles
E) No enzymes in its outer membrane

47) Function of basement membrane is


A) Supply blood to epithelial tissue
B) Breach and allow cells to grow inside the tissue
C) Anchor epithelial tissue to underlying connective tissue
D) Secrete intra cellular matrix
E) Stores metabolites within it

54) Edema is a feature in lymphatic obstruction. What is the main disturbance to starling forces in lymphatic
edema?
A) Decrease hydrostatic pressure interstitial space
B) Increase capillary hydrostatic pressure at arteriolar end
C) Increase capillary hydrostatic pressure at venular end

Presented by 15th Batch 137 FHCS | EUSL


D) Decrease oncotic pressure in interstitial space
E) Decrease plasma oncotic pressure

55) Interstitial fluid is different to plasma though both are belongs to ECF. What is the most significant feature seen
in plasma?
A) Contain more Na+ than interstitial fluid
B) Higher buffering capacity than interstitial fluid
C) Has more protein than interstitial fluid
D) Electrically more negative than interstitial fluid
E) More viscous than interstitial fluid

8th SEQ- Proper

2.2.3. Write a short account on mitochondrial inheritance (20 marks)


3.Write short notes on,
3.3. Cell membrane (40 marks)
5.2. Describe the role of plasma protein in, 5.2.1. Formation of edema (50 marks)

8th SEQ -Repeat


2.
2.1.1. Describe the Electron microscopic features of a mitochondrian in
a eukaryotic cell (20 marks)
2.1.2. List 5 functions of a mitochondria (10 marks
5.
5.1. Explain the terms osmosis and facilitated diffusion (20 marks)
5.2. Explain why polyuria occurs when ADH do not act on collecting ducts of renal tubule (40 marks)
5.3. Explain why oedema is a feature in nephrotic syndrome (40 marks)

8th MCQ- Proper Answer

1) A) T
Buffer is a mixture of a weak acid with a strong base or a strong acid with a weak base.
B) F HCO3
C) T
3 important buffers in urine; bicarbonate, di-basic acid phosphate and ammonia.
D) T is the major blood buffer
E)

9)
A-T
B-T vesicular transport-active
C-F Due to the presence of unsaturated fatty acids.
D-T
E-F

10)
A-T

Presented by 15th Batch 138 FHCS | EUSL


B- T Euchromatin; transcriptionally active deep condensed chromatin, is electron lucent
Heterochromatin; more condensed inactive, not electron lucent
C) F Nucleus
D) F Double membrane
E) F
To stain cytoplasmic structures such as mitochondria. Nuclear stain is hematoxylin.

11)
A-F

B) T Chromosomes condensed, get coiled, shortened and get visible.


C) T
D) T
E) F Barr body; a small densely staining structure in the cell nuclei of the females consisting of a condensed X
chromosome.

12)
A)T
B) T
C) F Primary active
D) T
E) F 3Na+ and 2K+

13)
A) T GLUT 4
B) T Through channel proteins.
C) F Secondary active
D) T
E) F SGLT 1 – secondary active.

14)
A)F ECF – inulin
RBC volume – radioactive Cr
B)T
C)F Cannot be measured (calculated as TBW – extracellular fluid volume)
D)T Radioactive I labeled protein, Evans blue dye
E)T

15)
A)T
B) F

Presented by 15th Batch 139 FHCS | EUSL


C) T
D) T
E) T

46)ANSWER – C
Abundant in liver, muscle and kidney

47) ANSWER – C
Provide physical binding of the epithelium to the underlying tissue and physical support.

54) ANSWER – E
In lymphatic obstruction, the filtered proteins do not enter the blood reducing the plasma oncotic pressure.

55) ANSWER – C

9th MCQ -proper


1) Acid base balance
A) Kidney excrete acids as NH4Cl
B) Albumin is more effective in maintaining blood PH than hemoglobin
C) Carbonic anhydrase is absent in plasma
D) The plasma PH is below 7 in a normal healthy adult
E) Normal arterial HCO3 concentration is about 45mmol/l

8) Chromatin
A) Has double helix DNA
B) Has basic proteins
C) Has acidic proteins
D) Has small amount of proteins
E) Is the basic unit of nucleosome

12) Small endoplasmic reticulum


A) Generate energy to other biochemical process
B) Is involved in steroid synthesis
C) Is involved in phagocytosis
D) Is the main site of modification of alcohol in liver
E) Is the main site of modification of proteins after translation

17) Regarding formation of edema


A) R/ventricular failure – increase hydrostatic pressure in venous end
B) Nephrotic syndrome – increase oncotic pressure in capillary bed
C) Vasodilator – increase hydrostatic pressure in arteriolar end of capillary bed
D) Fillaria lymphatics – increase oncotic pressure in capillary bed
E) Cirrhosis in liver – decrease oncotic pressure in capillary bed

18) Regarding transporters


A) H+ transport in DCT - active
B) I2 transporters in thyroid – secondary active
Presented by 15th Batch 140 FHCS | EUSL
C) O2 transport through alveoli – simple diffusion
D) Glucose transport through RBC – facilitated diffusion
E) NH3 transport – non in renal tubules ionic diffusion

19) Factors contributing to negative in inner side of the cell membrane include
A) Presence of Na/K channel
B) Semi permeability of cell membrane
C) Relative permeable Na/K across leaky channels
D) Secondary active transport of Na+
E) Presence of more PO3- inside the cell

SBR
4) Functions of SER except
A) Glycoprotein production
B) Phospholipid synthesis
C) Remove phosphate from glucose 6 phosphate
D) Detoxification of harmful chemicals
E) Synthesize enzymes

50) Important buffering activity is provided by


A) Bicarbonate ion in blood
B) Hemoglobin
C) Ammonia in urine
D) Phosphate in urine
E) Plasma protein

9th SEQ- proper


1.
1.1. Describe the lipid bilayer and its function in plasma membrane (25 marks)
3.
3.1.1. Error in meiosis leading to Down’s syndrome (30 marks)

9th SEQ-Repeat
1.
1.1. Briefly explain how the constituent molecules of the plasma membrane contribute to fluidity. (35 marks)

3.Write a brief account on,


3.1 Golgi apparatus. (25 marks)
3.2 Meiotic cell division. (40 marks)

5.A 7 year-old child came with a history of diarrhea. It was found that child was moderately dehydrate and
breathing rapidly. The child was treated with Jeevani. (ORS made according to WHO formula)
5.1 State the expected acid-base disturbance and explain the mechanisms (15 marks)
5.2 Explain the Physiological basis for rapid breathing in this child? (15 marks)
5.3 What are the basis for having Na+ and glucose in ORS. (35 marks)
5.4 What are the limitations of ORS as a rehydration solution (35 marks)

Presented by 15th Batch 141 FHCS | EUSL


10th MCQ-Proper

3) Buffer is the mixture of weak acid and its conjugate base. Regarding weak acid
A) it’s protonated at the pH below pka value
B) Acetic acid is an example
C) It has conjugate acid
D) It completely dissociate in the water
E) It behaves according to the Henderson Hasselbalch equation

11) Regarding cellular organelles


A) Centrosomes form the mitotic spindles during mitosis
B) RER is responsible for synthesis of enzymes
C) Ribosomes are produced from Golgi body
D) Microvilli are supported by microfilaments
E) Heterochromatin is more abundant in active nucleus

12) Regarding nucleus of human cells


A) DNA replication occur in the nucleus
B) Nucleus envelope is a single membrane
C) Chromatin in the nucleus is dense mass of DNA
D) RNA polymerase is found in the nucleoplasm
E) DNA polymerase is found in nucleolus

13) Regarding Sarcoplasmic Reticulum


A) Continuation of nucleus membrane form it
B) Has ribosome in cytoplasmic membrane
C) Steroid hormone synthesis
D) Detoxify the alcohol in liver
E) Glycogen degradation

15) Following transport mechanisms are energy depended


A) Diffusion
B) Osmosis
C) Filtration
D) Endocytosis
E) Exocytosis

SBR
45) What is the most likely acid-base disturbance caused in a patient with insulin dependent diabetes mellitus .His
plasma PH=7.2 HCO3 - =17mmol/l Pco2 = 2-20mmhg
A) Metabolic acidosis
B) Metabolic alkalosis
C) RD) Respiratory alkalosis
E) None of the above

46) Reduce chromosome number in


A) Anaphase 1
Presented by 15th Batch 142 FHCS | EUSL
B) Anaphase 2
C) Metaphase 1
D) Prophase 1
E) Telophase 1

47)Daughter cells are genetically different during meiosis because


A) Sister chromatids exchange their genetic materials
B) Homologous chromatids exchange their genetic materials
C) Cytosol division
D) Meiosis has meiosis 1& 2
E) Chromosomes are separated in anaphase

48) Which substance easily diffuse through the plasma membrane


A) Lipid soluble substance
B) Water soluble substance
C) Cations
D) Anions
E) Proteinsespiratory acidosis

10th SEQ-Proper

02. Write short notes on the following.


2.1 Fluid mosaic model of eukaryotic cell membrane (30 marks)
03. Write an account on
3.1 Cell cycle

10th MCQ-Proper ANSWER

3)
A) T
At pH values less than the pKa, the protonated acid from (CH3- COOH) is predominant.
B) T
CH3-COOH
C) F
D) F 𝐶𝐻3𝐶𝑂𝑂𝐻 ⇔ 𝐶𝐻3𝐶𝑂𝑂 − + 𝐻 +
E) T
pH = pKa + log [A~ ]/ [HA]

11)
A) T
centromere
a pair of centrioles
Centrosome matrix
B) T
RER — synthesis proteins.
OuteC) F
ribosomes consists of 2 subunits (large, small). These sub units are produced in nucleolus and exit together to
cytoplasm.
Presented by 15th Batch 143 FHCS | EUSL
D) T
microfilaments from micro villi.

E) F
in inactive state—chromatin appears as beads of nucleosomes referred as heterochromatin. For transcription
occur, DNA uncoil and uncoiled state (active) is called Euchromatin.

12)
A) T
B) F
C) T
D) T
E) F

13)
A) F

B) F

C) T
atty acids and steroid hormones.
D) F
sarcoplasmic reticula not found in liver.
E) F
takes place in the cytosol.

15)
A) F
passive (along the concentration gradient
B) F
-permeable membrane from a less concentrated solution into a
more concentrated one.
C) F
passive (along the pressure gradient)
D) T
E) T

45) ANSWER – A
Normal pH – 7.35- 7.45 HCO3 - = 2 Pco2 = 44 mm/Hg (venous) 40mm/Hg (arterial)

47) ANSWER – B
Genetic variability enhanced
- Crossover which redistributes genetic material.
- Random distribution of homologues chromosome to Daughter cells.

48) ANSWER – A
Plasma membrane is a phospholipid bilayer.so lipid soluble substance can easily diffuse through it.

Presented by 15th Batch 144 FHCS | EUSL


10th SEQ-Proper

2.1.
• Cell membrane is surrounded the cell from its outside.
• Thickness 5-8nm.
• Selectively permeable.
• Composed of a) Lipid phospholipids (75%) Cholesterol (20%) Glycolipid (5%) b) Protein integral protein (
transmembrane) Extrinsic protein ( peripheral) c) Carbohydrates
• Singer and Nicholson proposed the fluid mosaic model.
• Phospholipid molecules
Amphipathic
Hydrophilic
– polar head
Hydrophobic
– non polar tail
• Phospholipid arrange as a bilayer, facing their head outward.
• Fluidity and flexibility of membrane is increased by UFA.
• Cholesterol molecule among phospholipid molecules.
• They stabilize and regulate the fluidity of bilayer ( reduce)
• Transmembrane protein have a hydrophobic central zone amd hydrophilic zone.
• Peripheral protein attach to inner and outer membrane.
• Mosaic model of membrane ids due to protein.
• Protein make up almost ½ of total mass of membrane.
• Function of transmembrane protein,
I. Communication
II. II. Cell matrix adhesion
III. III. Formation of pores
• Short chain polysaccharide contact with protein and lipid molecule forming glycocalyx.

• It face outside of the cell.

3.1. Cell cycle


• time interval between 2 miotic divisions is the cell cycle of a cell
• 24 hours
• Consist of two major periods.
• They are interphase and mitosis ( M phase)
• Interphase is the interval between cell division and it consist of 3 phases.

• Interphase is in a high metabolic activity and does most of its growing and synthesis of organelles.
• In M phase ( mitosis) duplicated chromosomes and organelles goes to daughter cells
• M phase is the periods ofcell division.
• M phase is consist of 4 phases.
• They are

Presented by 15th Batch 145 FHCS | EUSL


11th MCQ-Proper

10. Mitosis
A) Causes growth of tissues
B) Has 2 cell divisions
C) Regulated by cyclin dependent protein kinases
D) Occurs in secondary spermatocytes
E) Gives rise to 2 identical daughter cells

11. Integral proteins in the cell membrane function as


A) Carrier proteins
B) Channel proteins
C) Enzyme
D) Hormone
E) Receptors

12. Diffusion,
A) Accounts for the greatest amount of substance transported across the cell membrane
B) Is an active transport
C) Is the process by which gases move across alveolocapillary membrane.
D) Takes place against concentration gradient.
E) Takes place due to random movement of molecules.

13. T/F regarding internal environment


A) Is in constant exchange with external environment
B) Is in equilibrium with intracellular environment
C) In equilibrium with plasma
D) Can be voluntarily controlled
E) Is referred to interstitial space

14. True or false regarding ICF


A) Bicarbonate is the major buffer.
B) Chloride is the major anion.
C) It's Ph is lower than that of ECF.
D) It's osmolality is higher than that of ECF.
E) It accounts about 2/3 of total body weight

35. which one is following not a second messenger?


A) Ca+2
B) cAmp
C) Co2
D) Diacylglycerol
E) Nitric oxide

SBR
41. Important buffering activity is provided by
Presented by 15th Batch 146 FHCS | EUSL
A) Bicarbonate ion in blood
B) Hemoglobin
C) Ammonia in urine
D) phosphate in urine
E) plasma protein

46. Nucleus
A) disappear in prophase of meiosis 1
B) increase Heterochromatin in metabolic active cells
C) continue as SER
D) found in mature RBC
E) in osteoclast cells as single structure

50. A patient was transfused with a high volume of a solution which made his RBC lysis, this solution is mostly like?
A) Hypertonic Nacl
B) Hypertonic Urea
C) Hypotonic urea
D) Isotonic mannitol
E) Isotonic Nacl

59. Edema is a feature in lymphatic obstruction. What is the main disturbance to starling forces in lymphatic
edema?
A) Decrease hydrostatic pressure in interstitial space
B) Increase capillary hydrostatic pressure at arteriolar end
C) Increase capillary hydrostatic pressure at venular end
D) Decrease oncotic pressure in interstitial space
E) Decrease plasma oncotic pressure

11th SEQ-Proper

02. 2.1. List the types of cell surface receptors with examples. (15 Marks)
1. Ligandgated ion channels. Example- neurotransmitter receptor
2. Enzymelinkedreceptors- receptors for growth factors
3. G-proteincoupledreceptors- beta adrenergic receptor

12th Proper-MCQ
15. Mitosis,
a. Give non identical daughter cells
b. Repair damaged tissues
c. High rate in tumor cells
d. Can cause mutations
e. May cause loss of chromosome parts

20. Endocytosis
a. Active method of cell transport
b. Phagocytosis
c. Intra cellular vesicles fuse with cell membrane
Presented by 15th Batch 147 FHCS | EUSL
d. Remove cell debris
e. Intracellular substances are removed to extra cellular area

SBR
44. Which of the following is responsible for the seasonal allergic reactions such as runny nose, nasal congestion
and wheezing
a. Monocytes
b. Neutrophils
c. Mast cells
d. Lymphocytes
e. Basophils

50. Most abundant intracellular ion


a. Na
b. K
c. Cl
d. Mg
e. Ca

12th Batch SEQ-Proper

1.1 Mechanisms by which chemical messengers (hormones or neurotransmitters) modify the cellular activities in
their target tissues
1.2 Functions of different components of eukaryotic cell membrane
1.3 Events of Meiosis occur in ovary
1.4 Defects of meiosis that cause Down syndrome

12th Batch MCQ-Proper ANSWER

15
A.F give identical cells
B.T replace and growth tissues divide from mitotic
C.T
D.F mutation mostly occur in meosis
E.F in meosis

20)
A.T
B.T endocytosis include pinocytosis and phagocytosis
C.T
D.F from exocytosis
E.F extracellular substance take into intracellular

44. C
Mainly allergies are due to mast cells which releases allergic inducing substances.

50. B
Abundant positive ion – K+
Presented by 15th Batch 148 FHCS | EUSL
Abundant negative ion – HPO4-2
K+ is higher than HPO4-2

13th Batch MCQ- Proper

8. Match the component of the cell membrane with function


A. cholesterol - maintain fluidity
B. glycolipid - cell recognition
C. intergralprotien - enzyme
D. peripheralprotien - transfering substance
E. Sphingolipid - signal transmission

9. Regarding signal transduction pathway,


A. G protein is bound to cell membrane receptor
B. G protein has 3 sub units
C. G protein is inactive when it is bound to GTP
D. Alpha sub unit activates adenylcyclase
E. C.AMP inactivates protein kinase

11. interstitial fluid


A. contain waste and nutrition
B. flitrate from lymphatic
C. 60% of the total body weight
D. contains high amounts of protein
E. carried out by lymph to circulating blood

12. Serum is different from plasma. What is the main feature of serum?
A. it has higher osmolarity.
B. It is devoid of fibrinogen.
C. it has increased calcium concentration.
D. It has no antibodies.
E. It contains platelets.

SBR
47. Which of the following is wrong about mitosis?
A.It is regulated by the cyclin dependant protein kinases.
Presented by 15th Batch 149 FHCS | EUSL
B. It leads to formation of genetic variations.
C. It occurs in repair and replacement of celss.
D. It occurs rapidly in keratinocytes.
E. Rapid mitosis leads to occurance of mutations in daughter cells

54. Which of the following capillaries have the highest hydrastatic pressure ?
A. Pulmonary capillaries
B. Glomerular capillaries
C. capillaries in Liver
D. capillaries in Skeletal muscle
E. Capillary in intestinal villi

13th Proper-SEQ
1)
1.4. State 3 genetic importance of meiosis (15 marks)

13th MCQ-Proper ANSWER

08
A) True
Cholesterol – maintain the fluidity
B) False
Cell recognition - glycoproteins (ABO antigens in RBC membrane –contain galactose)
C) True
Integeral proteins act as- channel proteins ,carrier proteins, enzymes, Receptors
D) True
Peripheral protein functions
1.enzymes
2.control the transport of substances through cell membrane across the pores
E) True
Sphingophospholipids –ex.Sphingomyelin Myelin predominantly found in neuron cell membrane.

09
A) True
B) True
Trimeric protein – Alpha, beeta and gamma sub units
C) False
G-protein active when bind to GTP
D)
E) False In many times cAMP activates protein kinase

11
A) True
B) False
Filtrate from blood capillaries , excess interstitial fluid drain by lymphatic system.
C) False
ECF+ICF = 60% of Total body weight
D) False
Low protein . (consider as 0mmHG oncotic pressure in interstitial fluid) E True
Presented by 15th Batch 150 FHCS | EUSL
47. B Cross over in meiosis –genetic variant formation

54. B

14th Proper-Mcq

01. Regarding cytoskeleton,


A) Intermediate filaments are important to identify the origin of cancer
B) Microfilaments support microvilli
C) Microfilament stabilize the position of cell organelle.
D) Microtubule are structural unit for centrosome
E) Mitotic spindles composed of tubulin protein

14. Passive transport method in cell membrane,


A) Pinocytosis
B) Phagocytosis
C) Exocytosis
D) Simple diffusion
E) Osmosis

15. Ionic movements in cell membrane,


A) Na ion move down the concentration gradient
B) Solutions across the membrane are neutral
C) Permeability is equal across the membrane
D) The gradient is generated by the distribution of ions across the cell membrane.
E) RBC in hypotonic saline solution shows shrinking appearance due to this.

36. Regarding telomere,


A) Telomerase prevent shortening of chromosomes
B) Each replication increases the length of DNA
C) Situated at middle of linear chromosome
D) It prevents sticking of one chromosome to another
E) After becoming too short, reach senescence stage

SBR
41. p53 protein is a key regulator of the progression of the cell cycle. It provides opportunity for own repair to occur
via the arrest of the following phase. which arrested for DNA repair before DNA replication by p53
A) G1 phase
B) G2 phase
C) S phase
D) Metaphase
E) Prophase

42. Which organelles have the ability to self replicate for the energy demand?
A) Centrosome
B) Lysosome
Presented by 15th Batch 151 FHCS | EUSL
C) Mitochondria
D) Peroxisome
E) Ribosome

48. Which of the following involves energy expenditure?


A) Diffusion
B) Osmosis
C) Proton pump
D) Exocytosis
E) Filtration

49. Non diffusible particles with in cell


A) Sodium
B) Glucose
C) Chlorine
D) Protein
E) Potassium

50. Mainly responsible for oncotic pressure


A) Sodium
B) Potassium
C) Albumin
D) Globulin
E) Fibrinogen

14th Proper-SEQ

2.4. Outline the role of G protein coupled receptors on cell signaling pathway with illustration. (25 Marks)
5.
5.1. Hypoalbuminemia leads to:
5.1.1. hyponatremia
5.1.2. hypovolemia
Explain the physiological basis of the abnormalities stated in 5.1.1 and 5.1.2. following hypoalbuminemia.(50marks)

14th Proper-MCQ ANSWERS

01)
AT
Different cell types express specific kinds of intermediate filaments. They are composed by different proteins. (
Nuclear lamins protein forms the intermediate filaments in Nucleus ) Specific type of proteins is used to determine
cells of origin of metastatic tumors
BT
Finger like cell extensions = microvilli → Microfilaments provide mechanical support
CF
Position of cell organelle → Intermediate filaments
D T Centrosome = Pair of centrioles + Pericentriolar matrix Composed of microtubules
E T Spindle fibers consist of microtubules synthesized from the protein Tubulin
Presented by 15th Batch 152 FHCS | EUSL
14)
AF
Endocytosis (pinocytosis, phagocytosis) and exocytosis are active transport methods
BF
CF
DT
ET

15)
AT
Na+ can transport through channels. It is passive
BF
Inside the membrane is negative and outside the membrane is positive
CF
Permeability depends on the substance. So permeability is not equal for different ions.
DT
Gradient is made from high to low concentration of ions
EF
Only move water. Water move from solution to cell.so cells swell.

36)
AT
Prevent the shortening of telomeres
BF
C F Presents at the ends of linear chromosomes
DT
ET

41) A
DNA replication is occurred in S phase. G1 is the phase before S phase

42) C
Mitochondria and Chloroplasts are two self-replicating cell organelles. Main function of mitochondria is ATP
production.

48) D
Exocytosis is an active process. It uses energy

49) D
Protein is not lipophilic. It is not diffusible. Ions transport through channels and glucose is transported through
facilitated diffusion

50) C

Presented by 15th Batch 153 FHCS | EUSL


15th Proper-MCQ

16) Regarding endoplasmic reticulum,


A. Self-replicating
B. has double membrane
C. Connected to prenuclear space via lumen of RER
D. give rise to sarcoplasmic reticulum
E. Involve in synthesis of other organelles

17) Regarding cell division,


A. Mitosis can be arrested by colchicine drug
B. Centrosomes are duplicated during prophase stage of mitosis
C. Chiasma formation occurs in meiosis
D. Neuroglial cells of CNS exhibit meiosis
E. Uncontrolled mitosis is seen in tumor tissue

28) Regarding the substance moving by facilitated diffusion


A. Amino acids through plasma membrane
B. Calcium from sarcoplasm into sarcoplasmic reticulum
C. Glucose from gut lumen into intestinal epithelium
D. Glucose from plasma to RBC
E. Na from neuronal cytoplasm to extracellular space

30) Cause for alkalemia


A. Hyperaldosteronism
B. Hyperventilation
C. Persistent diarrhea
D.
E. Persistent vomiting

SBR
44) A woman has plasma osmolarity of 290mos/L & urine osmolarity of 1100mosm/L. What is the reason?
A. Central diabetes insipidus
B. Dehydration
C. Drinking large volumes of water
D. Nephrogenic diabetes insipidus
E. SIADH

45) normal cell function is dependent on folding of the cell membrane. which one of the following increases the
membrane fluidity?
A. Increased body cold stress
B. Increased integral membrane protein content
C. Increased peripheral membrane protein
D. Increased saturate phospholipid content
E. Increased unsaturated phospholipid content

46)Nucleus
A. contains nucleoproteins synthesized inside it
B. contains ribosomes
Presented by 15th Batch 154 FHCS | EUSL
C. is bounded by single layered membrane
D. is surrounded by a membrane showing the identical features of plasma membrane
E. nucleoli which produce mitotic spindles

15th Proper-SEQ

3.2.2. DNA replication


4.3.
4.3.1. What is cellular aging? (05 marks)
4.3.2. State five factors contributing to cellular aging process (15 marks)
5.
5.1. Following are the results of a patient with pituitary tumor;
Serum Na+: 120mmol/L (Normal range: 136-145mmol/L) (15 marks)
Serum osmolality: 140 mosmol/kg (10 marks)
Explain the finding using physiology behind the condition.

5.2. Explain the physiological basis of using oral rehydration solution (ORS) in chronic diarrheic patient (25 marks)

Presented by 15th Batch 155 FHCS | EUSL


HB 03

Presented by 15th Batch 156 FHCS | EUSL


TISSUES OF HUMAN BODY
EME QUESTIONS

10th batch EME MCQ

(05) Epithelium tissue


a. Has abundant extracellular materials.
b. Lower mitotic index.
c. Lack of blood vessels.
d. Cells are anchored to a basement membrane.
e. Lines the body surface & cavities

(06) Fibroblast,
a. Has an extensive RER & free ribosomes.
b. Release histamine & associated with allergies & inflammation.
c. Secretes proteins that become fibers in the connective tissue matrix.
d. Has the ability to provide repair after injury in connective tissue.
e. Is responsible for scar tissue formation.

(07) Regarding cartilage,


a. Crystalloid osmotic pressure in capillaries.
b. Resilience of cartilage is given by collagen fibers.
c. Fetal hyaline cartilage is replaced by endochondral
ossification.
d. Menisci of synovial joint are formed by fibrous cartilage.
e. Elastic cartilage lacks in distinct perichondrium.

(08) Bone,
a. Rich in blood supply.
b. Serve as a source of mineral ions for homeostasis.
c. For growth in thickness has growth plate.
d. Has a good regenerative potential to regenerate through out life.
e. Disorder caused by vit D deficiency leads to reduced strength.

(09) Skin,
a. Is 15% of total body weight in normal adults.
b. Is the storage organ for vitamin D.
c. Always has an acidic pH.
d. Is impermeable to drugs.
e. Epidermis is continuously replaced.

Presented by 15th Batch 157 FHCS | EUSL


(10) Skeletal muscles,
a. Multinucleate.
b. Are capable to contract spontaneously.
c. Have a refractory period longer than that of the cardiac
muscle.
d. It can be tetanized.
e. Are innervated by alpha motor nerves.

(14) Resting membrane potential is depend on,


a. Intracellular K ions
b. Extracellular Na ions
c. Intracellular phosphate ions
d. Extracellular Cl ions
e. All of the above

(15) Lamela Granules are rich in glycolipid & released by


keratinocyte from a barrier to matrix. Which mostly epidermis?
a. Stratum Corneum
b. Stratum Lucidum
c. Stratum Granulosum
d. Stratum Spinosum
e. Stratum Basale

10th batch EME SEQ

1 .Briefly describe the sequence of event at the neuromuscular junction when a motor nerve is stimulated with
reference to ,
1.1 Event at the terminal buttons (35 marks)
1.2 Nature of transmission from nerve to muscle (35 marks)
1.3 Replace of motor end plate (30 marks)

11th batch EME MCQ

11th batch EME SEQ

1.3 Compare and contrast the structure of skeletal and cardiac muscle(30 marks)
1.4 Describe the process of ossification (40 marks)
2.2.1 Briefly describe the importance of ‘Oxygen debt’ after exercise(20 marks)
2.2 Repeated stimuli at short intervals produces tetanus in skeletal muscles but not in cardiac muscle
2.2.1 Define the term tetanus(20 marks)
2.2.2 Briefly explain the physiological basis of the above difference

Presented by 15th Batch 158 FHCS | EUSL


12th batch EME MCQ

1) Regarding cell junction,


a. Desmosome resist mechanical stress exert on the epithelium
b. Desmosomes contain connexins to connect intermediate filaments
c. Gap junctions can be seen between intercalated discs in cardiac muscles
d. Hemi desmosomes can be seen between lateral surfaces of cells
e. Tight junctions act as permeability barriers

2) Voltage gated Sodium channels,


a. Found in high concentration at axon hillock
b. Found in high concentration at terminal cisternae of sarcoplasmic reticulum
c. Responsible in generating action potential in nervous tissue
d. Open at the voltage of +30mv
e. Transmit Na+ from the inside to outside

3) Membrane potentials,
a. Is an unique feature of excitable tissues
b. Development to form skeletal system
c. Induce the neural tube forming
d. Formed from mesoderm
e. Net effect of concentration gradient and electrical gradient between plasma membrane

4) Regarding nerve regeneration,


a. Distant from the effector organ
b. Extent of damage to neuron
c. Extent of damage to neurolemanocyte
d. Damage to the cell body
e. Extent from myelin sheath formation

5) Smooth muscle,
a. include tropomycine
b. single nucleated
c. contain sarcomere
d. contain satellite cells
e. Have unstable membrane potential

6) Regarding muscles,
a. Refractory period of cardiac muscles is longer than skeletal muscles
b. Cardiac muscles show fatigability
c. In cardiac muscles force of contraction is inversely proportional to the end diastolic volume
d. Skeletal muscle velocity great at resisting membrane potential
e. Membrane potential of smooth muscle is unstable

7) Long term effects of muscle exercise,


a. Increase in muscle size
b. Increase muscle coordination
c. Increase the quantity of blood vessels
d. Muscle damage
Presented by 15th Batch 159 FHCS | EUSL
e. Muscle fatigue

8) Endochondral ossification,
a. Ends at birth
b. Create medullary cavity
c. Can be seen in clavicle
d. Need the formation of hyaline cartilage
e. Spongy bone firstly form

9) Regarding vitamins in bone growth and remodelling,


a. Vit A ----- stimulate osteoblastic activity
b. B12-protein synthesis
c. Vit C-Differentiation of osteoblast in osteoclast
d. Reduce Ca2+ absorption in the intestine
e. Vitamin K=stimulates osteoclast activity

10) Dermis,
a. Apocrine sweat gland involving thermogenesis
b. Contain collagen fibers
c. More cells than epidermis
d. Is damaged by 1st degree burn
e. Is vascular

11) What is the common feature of epithelial tissue & connective tissue?
a. Cells are closely packed
b. Cells lies on basement membrane
c. Have rich blood supply
d. Occur throughout the body
e. Possesses underlining intracellular materials

12) Incorrect about collagen,


a. Basic unit of a mature collagen is a tropocollagen
b. Mutation in collagen causes Marfan syndrome
c. Proline stabilize the helix confirmation of alpha chain
d. They are made by connective tissue cell
e. They are most abundant protein in mammalian

13) Plasma and interstitial fluid are included in extra cellular fluid. Most significant feature seen in plasma
compared to interstitial fluid,
a. Plasma contain high amount of sodium ion than interstitial fluid
b. Physostigmine drug increase the action potential of sarcomere
c. Physostigmine drug increase the acetylcholine in synaptic cleft
d. Physostigmine increase ca+2 removal from t-tubules
e. Physostigmine increase speed of impulse

14) A tough sheet of dense irregular tissue, that surround bone surface ,bone growth assist in bone repair,
nourish bone tissue and serve attachment site of tendon, which structure,
a. Epiphysis
b. Endosteum
c. Metaphysis
d. Diaphysis
e. Periosteum

Presented by 15th Batch 160 FHCS | EUSL


15) A 65 years old person has diabetes .when he touches the heat object, he haven't feel .after that can be seen
marks in his hand .what is the reason for that ?
a. His dermis become thin
b. The ability to feel pain is reduced in his hand
c. The ability to sensation is reduced in his hand
d. The ability to feel temperature is reduced in his hands
e. The blood supply to his hands is reduced

12th batch EME SEQ

1.
1.1. State the components of the Haversian system. (20 Marks)
1.2. Outline the characteristic of the epidermis of thick skin. (25 Marks)
1.3. Illustrate the structure of a sarcomere. (20 Marks)
1.4. Describe the growth of cartilage. (35 Marks)

2.
2.1. Generation an action potential is a feature of excitable tissues
2.1.1. Illustrate the action potential in a neuron using a diagram. (40 Marks)
2.1.2. Explain the events that take place during the action potential in a neuron. (40 Marks)
2.2. Muscle contraction requires enormous amounts of energy in the form of ATP.
Outline the sources of energy for the muscle contraction in the human body with examples. (20 Marks)

13th batch EME MCQ

1) Regarding epithelium tissues except,


A. Cells arranged tightly
B. Resting on basement membrane
C. May have extensions on the apical surface of the cell
D. Has abundant extracellular materials
E. Is vascularized

2) Regarding the serous glands


A. Cells have abundant RER
B. Goblet cells are single serous glands
C. It develops from surface epithelium cells
D. It secrets viscous glycoproteins
E. Secretory granules dominate the apical cytoplasm of serous cells

3) Functions of areolar connective tissues


A. Causing movement of the body
B. Deposit of fat
C. Defend the body from microorganisms
D. Holding body fluids
E. Support other tissues

4) Regarding Elastin protein


A. Hydrophilic
B. Non glycosylated
C. Rich in proline and lysine
D. Precursor is proelasting
Presented by 15th Batch 161 FHCS | EUSL
E. It is triple stranded

5) Acetylcholine released to the neuromuscular junction


A. Causes activation of presynaptic calcium channels
B. Cause motor end plate potential
C. Enters sarcoplasmic reticulum
D. It broken down enzymatically after its post synaptic action
E. Is transport from the cell body of the neurone to nerve terminal

6) Regarding electrical potential of skeletal muscle fibers


A. Action potential is produced by threshold stimules
B. Cell membrane has gap junctions
C. Repeated stimulation can cause tetany
D. Duration of action potential is 200ms
E. Resting membrane potential is +20mv

7) Smooth muscle differs from cardiac muscles from it’s


A.
B.
C. It has long contraction period
D. multinucleated
E. action potential takes longer time

8) Regarding phosphagen system


A. Creatin consists of pyridine
B. Creatin phosphate store in the liver
C. Used after completion of stored stored ATP
D. Can generate ATP during anaerobic condition
E. Serum creatinine is a sensitive indicator for kidney diseases

9) Fibro cartilage
A. Has abundant collagen type I
B. Has multiple chondrocytes in a lacunae
C. Is highly flexible
D. Lacks perichondrium
E. Provides support to the joints

10) Regarding hair follicle


A. It is made up of dead keratinized epidermal cells
B. Hair shaft is connected with sweat gland
C. Colour of the hair is due to the action of melanocytes
D. Damage to hair follicle leads to baldness
E. Emotion cause to goose bumps

11) Mostly adapted cell for transport substances across cell


A. Pseudo stratified columnar epithelium
B. Simple columnar epithelium
C. Simple cuboidal epithelium
D. Simple squamous epithelium
E. Transitional epithelium

12) Which of the following feature Is mostly likely responsible for transitional epithelium to prevent the leakage of
urine from bladder to underlying tissues
A. Lies on the basement membrane
Presented by 15th Batch 162 FHCS | EUSL
B. Multi-layer
C. Desmosome junctions between cells
D. Tight junctions between cells
E. Umbrella cells

13) Cardiac muscles differ from skeletal muscle is


A.
B.
C. Mechanical response is faster than skeletal muscles
D. Summation can be seen
E. Respond to stimuli like skeletal muscle

14) Which of the nerve has high conduction velocity


A. Alpha motor neurones
B. Autonomous fibres
C. Gamma motor neurones
D. Type B fibres
E. Type C fibres

15) What is incorrectly paired


A. Vitamin A- Stimulate osteoblast activity
B. Vitamin B12- Collagen synthesis
C. Vitamin C – hydroxylation
D. Vitamin D – decrease blood calcium level
E. Vitamin K – Protein synthesis

13th batch EME SEQ

1.
1.1. State the importance of tension lines in skin (15 Marks)
1.2. Outline the process of nerve regeneration (20 Marks)
1.3. Describe the growth of a long bone (35 Marks)
1.4. Compare and contrast the structure of skeletal and cardiac muscle (30 Marks)

2.
2.1. Outline the disease related to collagen synthesis (20 marks)
2.2. Define nerve impulse (20 marks)
2.3. Explain the ionic basis of propagation of nerve impulse (20 marks)
2.4. Explain briefly how receptor potential differs from action potential (20 marks)
2.5 Explain briefly how myelination increase velocity of conduction (20 marks)

Answers

01.
A. False
B. False
C. False
D. True
E. True

Presented by 15th Batch 163 FHCS | EUSL


Features of epithelial tissue,
✓ Arranged in continuous sheets
✓ Closely packed, tightly held

✓ Covers or lines the body parts

✓ Has free surface


✓ Apical surface has extensions(cilia, villi)
✓ Little or no extracellular matrix
✓ Act as an interface tissue
✓ Exhibit polarity
✓ Avascular but innervated
✓ Supported by connective tissue
✓ Has regenerating ability

✓ Cellularity

02. (Serous glands)


A.
B.
C. T
D. T

E.

03.
A. F
B. F
C. T

D. T
E. T

Functions of areolar connective tissue


✓ Wraps and cushions organs
✓ Phagocytosis
✓ Inflammatory role
✓ Holds & conveys tissue fluids

04.
A. F
B. T
C. T
D. F
E. F

Presented by 15th Batch 164 FHCS | EUSL


Elastin protein
• Composed of elastin & glycoprotein (fibrillin)
• Insoluble polypeptide,so hydrophobic.

• Rich in proline & lysine (scant hydroxyproline & hydroxylysine)


• Precursor is tropoelastin
✓ Small
✓ Soluble

✓ Non polar
• Mutation in fibrillin 2 results Marfan’s syndrome

• Lungs, walls of large arteries, elastic ligaments

Collagen protein
• Long, rigid
• Three alpha polypeptides/triple stranded /triple helix
• Collagen related diseases :scurvy, ehler’s danlos syndrome
• ECM, vitreous humor in eye, tendons, cornea of eye

05.

1- Neurotransmitter = made in the soma + stored in the vesicle.

2- The action potential arrives at the terminal button.

3- causes a depolarization of the terminal button, which causes calcium channels to open.

4- Ca2+ enters the terminal button through the calcium channels.

5- Ca2+ makes the vesicles to fuse with the surface of the presynaptic cell.

6- Neurotransmitter = released into synaptic cleft.

7- Neurotransmitter binds to receptor molecules in the postsynaptic cell.

8- This opens or closes postsynaptic channels.

9- This creates excitatory or inhibitory postsynaptic potential.

10- Excess NT = removed

Theories on how this happens:

*excess NT is uptaken by either the glial cells or the presynaptic cell's terminal buttons.

*NT removed by enzymatic degradation when it's broken down by enzymes.

11- The left over shells from the vesicles are fatehed back from the presynaptic cell's plasma.

Vesicles are then reconstructed.

A. False (Electrically stimulated Presynaptic membrane depolarizes. It results in activation of calcium channels)

B. True
C. False (Ach is stored in vesicles. Not in the sarcoplasmic reticulum. Calcium ions stores in sarcoplasmic reticulum)

Presented by 15th Batch 165 FHCS | EUSL


D. True (By acetyl choline esterase enzyme)
E. True (Neurotransmitters synthesis In cell organelles. They are in the cell body of neuron)

06.
A. True
B. False
C. True
D. False
E.
Electrical potential of skeletal muscle fibres

07.
A.

B.
C. True
D. False
E. True

08.
A.

B. False
C. True
D. True
E. True
*Consists of glycine and arginine
*Stored in skeletal muscle

*Synthesis :liver & kidney


*Elimination :kidney

Free ATP -
Low levels of ATP exist within the muscle fibers and can immediately provide energy for contraction. However, the
pool is very small and after a few muscle twitches will be exhausted.
Phosphocreatine -
Phosphocreatine, also known as creatine phosphate, can rapidly donate a phosphate group to ADP to form ATP and
creatine under anaerobic conditions. Enough phosphocreatine is present in the muscle to provide ATP for up to 15
seconds of contraction. The reaction of phosphocreatine + ADP to ATP + creatine is reversible. During periods of
rest, the store of phosphocreatine is regenerated from ATP.
Glycolysis -
Glycolysis is the metabolic reaction which produces two molecules of ATP through the conversion of glucose into
pyruvate, water, and NADH in the absence of oxygen. The glucose for glycolysis can be provided by the blood
supply, but is more often converted from glycogen in the muscle fibers. If glycogen stores in the muscle fibers are
expended, glucose can be created from fats and proteins. However, this conversion is not as efficient. Pyruvate is
continually processed into lactic acid. With pyruvate accumulation, the amount of lactic acid produced is also
increased. This lactic acid accumulation in the muscle tissue reduces the pH, making it more acidic and producing
the stinging feeling in muscles when exercising. This inhibits further anaerobic respiration, inducing fatigue.
Glycolysis alone can provide energy to the muscle for approximately 30 seconds, although this interval can be
Presented by 15th Batch 166 FHCS | EUSL
increased with muscle conditioning.
Cellular Respiration -
While the pyruvate generated through glycolysis can accumulate to form lactic acid, it can also be used to generate
further molecules of ATP. Mitochondria in the muscle fibers can convert pyruvate into ATP in the presence of
oxygen via the Krebs Cycle, generating an additional 30 molecules of ATP. Cellular respiration is not as rapid as the
above mechanisms; however, it is required for exercise periods longer than 30 seconds. Cellular respiration is
limited by oxygen availability, so lactic acid can still build up if pyruvate in the Krebs Cycle is insufficient. Cellular
respiration plays a key role in returning the muscles to normal after exercise, converting the excess pyruvate into
ATP and regenerating the stores of ATP, phosphocreatine, and glycogen in the muscle that are required for more
rapid contractions.

09.
A. True
B. False
C. False
D. True
E. True
Hyaline cartilage

✓ Consists of Collagen type two


✓ Multiple chondrocytes in one lacunae
✓ Flexible
✓ Provide support
✓ Allows movement at joints
✓ Reduces friction
✓ Absorbs shock

Elastic cartilage
✓ Consists of Collagen type two
✓ Single chondrocyte in one lacunae

✓ Gives support
✓ Maintain shape
✓ Allows flexibility (elasticity)

Fibro cartilage
✓ Consists of Collagen type one
✓ Parallel rows of chondrocytes alternating with collagen fibres
✓ Highly compressible
✓ Great tensile strength

✓ Contains collagen bundles in matrix


✓ Lacks perichondrium
✓ Support and fusion
✓ Absorbs shocks

10.
A.

Presented by 15th Batch 167 FHCS | EUSL


B. True
C. True
D. True
E. True

11. D
Epithelial type
1) Simple squamous - Diffusion, Filtration, Secretion(serosa), form Blood brain barrier

2) Simple cuboidal - Secretion, Absorptions, Protection


3) Simple columnar
I.Ciliated - Propulsion
II.Non ciliated - Absorption, Secretion

4) Pseudo stratified columnar

I.Ciliated - Propulsion
II.Non ciliated - Protection, Absorption, Secretion
5) Stratified squamous - Protection
6) Stratified cuboidal - Protection
7) Stratified columnar - Protection, Secretion
8) Transitional - Stretch, Permits distension of sacs

12. D
Tight junctions – Inhibit passage of substances between cells. Prevent contents of organ from leaking into blood or
surrounding tissues.

Adherence junctions – Resist separation during various contractile activities.


Desmosomes – Prevent separation under tension & cardiac muscle cells from pulling apart during contraction.
Hemidesmosomes – Anchor cells at basement membrane

Gap junctions – Allows cell communication

13.
A.

B.
C. False
D. False

E. False

14. A
Nerve conduction velocity (CV) is the speed at which an Allows cell electrochemical impulse propagates down a
neural pathway.

Nerve conduction velocity depends on,

1. Myelination amount
Myelination results in soltatary propagation which is more faster than continuous propagation. Therefore velocity
increases with the amount of Myelination
2. Axon diameter
Presented by 15th Batch 168 FHCS | EUSL
Velocity increases with the increasing of axon diameter.
3. Temperature
Velocity increases with temperature.
❖ A fibres – thick myelinated
❖ B fibres – thin myelinated

❖ C fibres-unmyelinated
❖ Preganglionic fibres – myelinated

❖ Postganglionic fibres – unmyelinated

15. B/D
Factors affecting bone growth and remodelling
A. Nutrition
a) Minerals
Calcium ions /Phosphate ions – Need vitamin D calcitriol to absorb
Fluoride ions

Magnesium ions
Manganese ions

Ferric ions
b) Vitamins
Vitamin C- Collagen synthesis
Vitamin D-Absorb Calcium & Phosphate
- Increase blood calcium level
Vitamin K & B 12-Protein synthesis
Vitamin A-Stimulation of osteoblastic activity

B. Diet changes
C. Lifestyle

Presented by 15th Batch 169 FHCS | EUSL


D. Exercise
E. Hormones
I. Growth hormone
II. Insulin
III. Thyroid hormone
IV. Glucocorticoids
V. Calcitriol
VI. Sex hormones

13th - EME SEQ

1.
1.1. (15)

Surgical cuts and incisions make along tension lines because they heal and repair scars quickly
1.2. (20)
Axons and dendrites are associated with neurolemma. Following the damage, Nissls granules break into granular
masses (chromatolysis). Even though the axon and myelin sheath degenerate, the neurolemma remains.
Degeneration of the distal portion of the axon and myelin sheath is called Wallerian degeneration. Following
chromatolysis, signs of recovery in the cell body become evident. Macrophages phagocytize the debris. Synthesis of
RNA and protein accelerates, which favours rebuilding/regeneration of the axon. The Schwann cells on either side
of the injured site multiply by mitosis, grow toward each other, and may form a regeneration tube across the
injured area. The tube guides growth of a new axon from the proximal area across the injured area into the distal
area previously occupied by the original axon. New axons cannot grow if the gap at the site of injury is too large or if
the gap becomes filled with collagen fibers. During the first few days following damage, buds of regenerating axons
begin to invade the tube formed by the Schwann cells. Axons from the proximal area grow at a rate of about 1.5
mm (0.06 in.) per day across the area of damage, find their way into the distal regeneration tubes, and grow toward
the distally located receptors and effectors. Thus, some sensory and motor connections are reestablished and some
functions restored. In time, the Schwann cells form a new myelin sheath.
1.3. (35)
• Long bone growth by endochondral ossification.

• First, a cartilage model of the future bone is developed by the mesenchymal cells. Cells convert to chondroblasts,
secrete cartilage ECM and perichondrium developed.

• Cartilage grow in length by interstitial growth and in thickness by appositional growth.


• Chondrocytes in the mid-region hypertrophy and surrounding ECM calcify. Cells inner to it die forming lacunae
spaces.
• Primary ossification develops inward. Nutrient artery penetrate perichondrium and the osteogenic cells in it
differentiate into osteoblasts. Then perichondrium known to be periosteum.
• Primary ossification center develops in the middle. Spreads towards both ends. Osteoblasts deposit bone matrix
forming spongy bone
• Osteoclasts breaksome spongy bone trabeculae forming medullary cavity in diaphysis. Most its world replaces by
compact bone.

• When branches of epiphyseal artery enter epiphysis, secondary ossification centre develops outward (around
time of birth). Spongy bone remains and no medullary cavity.
• Hyaline cartilage surrounding epiphysis become articular cartilage.

Presented by 15th Batch 170 FHCS | EUSL


• Prior to adulthood, epiphyseal growth plate remains.
2)2.1.

Scurvy
Due to vit.C deficiency
Vit.C coenzyme for hydroxylation
Collagen synthesis impaired
Osteogenisis imperfecta
Genetic disorder
Problem in mutation of genes responsible for production of collagen type
Brittle bones
Ehlers danlos syndrome
Genetic disorder
Affect production of collagen in blood vessels,skin
2.2.
Propagation of action potential
2.3.
Due to strong stimulus Na+ leaky channels open RMP increase upto threshold due to Na+ influx Voltage gated Na+
channels open,Na+ influx-depolarization. Then K+ channels open, K+efflux -repolarization. At threshold K+ channels
start to close. Still some K+ efflux- hyperpolarization. Na+/K+ ATPase action regenerate RMP. Electrical circuits
between nearby areas lead to propagation of AP
2.4.
Action potential generate if stimulus reach firing level, receptor potential (RP) is a membrane potential AP generate
simplistically, RP is a graded potential
2.5.
In myelinated nerves impulses transduced from one Ranvier nodes to next Ranvier node. Because myelinated nerve
impulses jump over areas myelinated, they are faster than unmyelinated ones

14th batch EME MCQ

01.Regarding the adaptation of epithelium for its function,


A) Adaptation is facilitated by the presence of microvilli in the apical surface of the cell.
B) Presence of tight junctions in the epithelium acts as a permeability barrier.
C) Simple epithelium enables the filtration of substances.
D) Stratified epithelium protects the underlying tissue from injury.
E) The epithelium is less sensitive due to lack of innervation.

02. Exocrine glands,


A) Can be unicellular.
B) Are developed from surface epithelium.
C) Cells have higher regeneration ability.
D) Might develop adenocarcinoma.
E) Release the secretions directly to the bloodstream.

03. Regarding adipose tissue,


A) Brown adipose tissue has single lipid droplets.
B) Brown adipose tissue involves in generation of heat.
Presented by 15th Batch 171 FHCS | EUSL
C) Numerous mitochondria present in white adipose tissue than in brown type.
D) Weight gain is due to excess deposition of white adipose tissue.
E) White adipose tissue presents in the yellow bone marrow.

04. Membrane potential,


A) Is always conducted from one cell to another.
B) Is the property seen only in nerves and muscles.
C) Is caused by the uneven distribution of charged particles
D) Is energy dependent
E) Is possible, because of the semi permeability of the membrane.

05. Regarding action potential,


A) It can be initiated by any stimulus.
B) It can be recorded in all tissues when stimulated.
C) It initiates the contraction process in a muscle
D) It is propagated only in the nervous tissues.
E) Opening of K+ channels mark the beginning of action potential.

06. Hyaline cartilage,


A) Disorder causes osteoarthritis.
B) Has a rich blood supply.
C) Models undergo endochondral ossification.
D) Occurs in pubic symphysis.
E) Provide flexibility.

07. Compact bone,


A) Accommodates red bone marrow.
B) Consists of osteons
C) Increases the surface area to improve the metabolic rate
D) Resists the stress in multiple directions
E) Thickness determines the strength of bone

08. Regarding neuromuscular junction,


A) It is the Site of transmitting action potential from nerve to muscle.
B) It is the site where Nerve and muscle cells are separated by synaptic cleft.
C) It is the site where nerve transforms to muscle cell.
D) Motor endplate has nicotinic receptors.
E) The transmitter released at the junction is noradrenalin.

09.Contractile proteins in the skeletal muscle,


A) Actin
B) Myoglobin
C) Myosin
D) Tropomyosin
E) Troponin

10. A trained 400m sprinter obtain energy for the event by,
A) Anaerobic glycolysis
B) Kreb's cycle
C) Oxidation of fatty acids
D) Phosphagen system
E) Stored ATP

SBR
Presented by 15th Batch 172 FHCS | EUSL
11. Which gland secretes sticky and waterproof secretion to prevent entry of foreign particles and insects to ear
canal?
A) Apocrine gland
B) Ceruminous gland
C) Eccrine sweat gland
D) Goblet or unicellular glands
E) Sebaceous glands

12. Which of the following cells regulate the exchange of nutrients and metabolites from the neuron cell bodies in
ganglia?
A) Astrocytes
B) Ependymal cell
C) Microglial cell
D) Satellite cell
E) Schwann cell

13. Which of the following responsible for saltatory conduction in nerve?


A) Diameter of nerve.
B) Inter nodal segments.
C) Ion channels in the fibre.
D) Length of the fibre.
E) Nodes of Ranvier.

14. Type of contraction in a skeletal muscle in any mechanical work is


A) Clonus
B) Eccentric
C) Isometric
D) Twitch
E) Wave of summation

15. When the epiphyseal plate is replaced by bone,


A) Appositional bone growth begins.
B) Interstitial bone growth begins
C) Long bone has reached their adult length
D) Puberty begins
E) The bone becomes more brittle.

14th batch EME SEQ

1.
1.1. Compare and contrast the structure of both involuntary type of muscles. (30 Marks)
1.2. Outline the characteristic features of the epidermis of thin skin. (30 Marks)
1.3. Describe the process of ossification of humerus. (40 Marks)
2.
2.1. Compare and contrast the structure of collagen and elastin proteins in the extracellular matrix. (20 Marks)
2.2. Briefly describe the role of skin in
3.2.1. Thermoregulation (40 Marks)
3.2.2. Immunity

Presented by 15th Batch 173 FHCS | EUSL


Answers
14th - EME MCQ

01.
A- T it’s the specialization of epithelial surface to increase surface are for the absorption.
B- T prevents free movement of substances and forms tissue blood barriers: blood brain barrier.
C- T made of single layers of cells facilitates easy diffusion.
D- T due to abrasion lost cells replaced by epithelial cells with high mitotic index.
E- F has abundant nerve supply and nerve endings.

02.
A- T unicellular glands are presents. Goblet cells.

B- T developed from surface epithelium and the connection maintains to form exocrine ducts.
C- F if having high generating ability could result in cancer.
D- F adenocarcinoma is a cancer arises from endocrine tissues.
E- F via ducts secretions is released.

03.
A- T
B- T since having mitochondria which have UCP- (uncoupler protein) results in heat generation rather than ATP
production.
C- F
D- T
E- T refer wheatear’s histology

04.
A- F cell to cell or cell to receptor.
B- F present in all cells.
C- T theory
D- T energy needed for Na/K ATPase pump to form resting membrane potential.

E- T presents of channels and phospholipid bilayer enables to do this.

05.
A- F only a stimulus with intensity higher than RMP
B- F easy to record in nerves and muscles.
C- T trigger of muscle contraction.
D- F also in muscles as well.

Presented by 15th Batch 174 FHCS | EUSL


E- F theory

06.
A- T loss of cartilages causes abrasion of bone by friction and arthritis
B- F blood supply by diffusion.
C- T theory
D- F fibrocartilages
E- T presence of collagen fibers provides considerable flexibility.

07.
A- F red bone marrow present in spongy bone.

B- T theory
C- F refer to the spongy bone. Refer histology of wheatear’s
D- F in single longitudinal direction.

E- T thickness refers to the amount of boney minerals in bone.

08.
A- T theory
B- T theory
C- F
D- ?
E- ?

09.
A- T
B- F

C- T
D- F
E- F

10.
A- T
B- T
C- F
D- T
E- T

11. B is the answer refer wheatear’s histology.

12. A is the answer. By forming blood brain barrier with perivascular end feet of astrocytes.

13. E is the answer presence of Ranvier forms myelin cover only at those locations. Nerve impulse transmitted only
at Ranvier nodes via jumping AP.

Presented by 15th Batch 175 FHCS | EUSL


14. E is the answer. During a mechanical work tone is constant. Only possible during a remain contracted stage.
(Wave summation) length reduces.

15. C is the answer theory.

14th - EME SEQ

1.

1.1.

Cardiac muscles Smooth muscles

Branched cylindrical fibres with one centrally Fibres thickest in middle, tapered at each end and
located nucleus with one centrally positioned nucleus

Straited Not striated

Sarcomeres present Sarcomeres present: but the structure is different

Fibers diameter is large (10-20 micrometer) Fiber diameter is small

Fiber length is large (50-100 micro meter) Fiber length is intermediate

Connective tissue components are endomysium Connective tissue component is endomysium


and perimysium

Actin and myosin are arranged in a sarcomere Actin and myosin are not organized to a
sarcomere

Sarcoplasmic reticulum is attached to one side of Caveolae present


T-tubule

Have moderately developed sarcoplasmic reticula Have poorly developed sarcoplasmic reticula and
and tranverse tubule no transverse tubules

Intercalated discs contain gap junctions and Gap junctions in viscera smooth muscle, none in
desmosomes multi-unit smooth muscle

Myoglobin is present Myoglobin is absent

Have high no of mitochondria Have low no of mitochondria

Regular protiens - troponin and tropomyosin Regular protiens - calmodulin and myosin light
chain kinase

High Ca2+ storage and Ca2+ binds to troponin Low Ca2+ storage and Ca2+ binds to calmodulin

Present in heart Present in walls of visera, airways, blood vessels,


iris and ciliary body of eye, errector pilli muscles

1.2.

Presented by 15th Batch 176 FHCS | EUSL


Skin is consisted of 3 major layers. They are epidermis, dermis and hypodermis(subcutaneous tissue). Epidermis is
the most superficial layer of the skin. It contains 4 major types of cells. Keratinocytes are the most abundant cell
type in are the epidermis. It produces keratin. Melanocytes produce melanin to protect the skin from the ultraviolet
radiation. Langerhan cells are involved in immune responses. Merkel cells function for the sence of touch. The
epidermis contains 5 layers in thick skin and 4 layers in thin skin. Stratum lucidum can only be found thick skin such
as skin of fingertips, palms and soles.

The epidermis of thin skin is consisted of 4 major layers from bottom to top, Stratum basale, stratum spinosum,
Stratum granulosum and stratum corneum.

The deepest layer; stratum basale is the layer where continuous cell division occurs and make all other cell layers.
Stratum spinosum includes 8-10 layers of keratinocytes which includes keratohyalin. Stratum granulosum includes
lemellar granules which are rich in glycolipids. Stratum corneum is composed of many sublayers of flat, dead
keratinocytes that are continuously shed and replaced by cells from deeper strata. (Stratum spinosum + stratum
basale --> stratum germinativum). Stratum spinosum --> in the process of protein synthesis.

1.3.

Dermis - papillary layer -> meissner's corpuscles -> touch

- reticular layer ->pacinian corpuscles -> pressure

The process of bone formation is reffered as ossification. Bone formation occurs in 4 situations, bone formation in
an embryo, growth of bones until adulthood, remodeling of bone, repaire of fractures. There are 2 types of
ossification, they are membranousossification and endochondrial ossification. Long bones as humerus follow the
endochondrial ossification. It uses hyaline cartilage models and thus requires breakdown of hyaline cartilage prior
to ossification.

Cartilage template is formed with the shape the future bone. Primarily offification centers appear in the shaft or
diaphysis. Thin bone collar appears around the diaphysis by intramembranous ossification. So, that the
perichondrium of cartilage become periosteum. Deep to newly formed collar cartilage matrix begins to calcify.
Chondrocytes hypertrophy and die. Blood vessels invade the diaphysis interior and bring in associated
mesenchymal and osteoprogenitor cells. Then the erosich of cartilage in the centre is occured and primitive marrow
cavity or medullary cavity is formed. Secondary ossification centres appear. And subsequently articular cartilage
and the epiphyseal plate is formed. Interstitial proliferation of remaining chondrocytes cause the two ends to grow
longitudinally. Condrocytes are arranged in columns on both sides of central region. Thus forms the epiphyseal
plate at junction between epiphysis and diaphysis.

At the primary ossification centre, spongy bones are forms,other osteoblasts below the periosteum makes compact
bone. Sondary occifications centres appear as diaphysis. The growth of long bones in length involves 2 major
events. They are the growth of cartilage on the epiphyseal plate and replacement of cartilage by bone tissue in
epiphyseal plate. Until the epiphyseal plate is there the growth of long bones continues. But after puberty at a
certain age they dissapear and replaced by bone tissue. Then the lengthening stops. Epiphyseal plate cartilage
organises into 5 functional zones.

- zone of resting- Small clusters of flattened, rounded and randomly arranged, quiescend chondrocytes.

- zone of proliferation - Chondrocytes undergo rapid mitosis under influence of growth hormone.

- zone of hypertrophy - Miroses ceases and cells and their lacunae enlarge.

- zone of calcification - dead chondrocytes are resorbed, lacunae erode.

- Richly vascularized primary marrow extends into the newly opened spaces and osteoblasts
differentiate from mesenchymal cells in the marrow.

Presented by 15th Batch 177 FHCS | EUSL


- zone of ossification- osteoblasts secret osteoids, which becomes mineralized and form new diaphysis.

2.

2.1.

Collagen Elastin

Main structural protien found in skin and other Major structural protien in the connective tissue
connective tissue. of elastic structures.

25 types are there. Only 1 type is found

Long, stiff, triple stranded, helical structure Rubber like structure, highly hydrophobic, not
glycosylated.

3 collagen polypeptide chains (alpha chains) are Contain little hydroxyproline and hydroxylysine.
wound around one another in a roperike super Insoluble protien.
helix by inter chain H bonds.

Basic unit is tropocollagen Precursor is tropoelastin

Extremely rich in proline and glycine. Hydroxy Composed of 2 large types as hydrophobic
proline is present. Hydroxylysin is absent segments and alanine and lysine rich alpha-
helical segments

2.2.

2.1.1.

Skin plays a major role in thermoregulation. Normal body temperature of a healthy individual is 37°C (369°C). When
the body temperature is varied than than the body metabolic processes alter their action. So, It is very important to
keep the temperature in their proper range. Skin is playing important role there. When the body is 37°C, it should
be increased by heat gain. Shivering, releasing of epinephrine and norephrine and vasoconstriction of cutaneous
blood vessels make it happen. When the body is exposed to a cold environment, heat loss from the body should be
decreased cutaneous vasoconstriction decreases cutaneous blood circulation and decreased the heat loss.
Goosebumps in a cold weather occurs due to constriction of arrector pili muscle. That also create some heat by
muscle action but it is very small to consider. But the erection of hair In the skin makes a heat protecting shield over
the skin by the air trapped among the hair shafts. When the body temperature is increased, it should get back to
normal level. It is normal to have increased body temperature during excercises as the muscle action is increased
during exercises. But apart from exercising having high temperature is pathological. To loss the heat Skin has many
mechanisms. Evaporation of sweat, radiation, conduction and convection, cutaneous vasodialation etc. Sweet
glands secretes increased amount of sweat when there is increased body temperature. By the increased heat sweat
Presented by 15th Batch 178 FHCS | EUSL
on the skin surface is evaporated and heat is loss by that action. Cutaneous vasodilation increased the cutaneous
blood supply and make more blood to come near the surface to loss heat by conduction and convection. Radiation
from skin also can loss heat. Receptors for temperature changes are located in the skin mainly. Warmth and cold
receptors from skin takes signals to heat loss centre and heat gain centre in hypothalamus.

Heat loss centre: preoptic and anterior hypothalamic nuclei.

Heat gain centre: posterior hypothalamus.

When the skin receptors are stimulated by increased heat, preoptic and anterior hypothalamic nuclei are
Stimulated and it inhibit sympathetic nervous system to decrease heat production and increase heat loss via
posterior hypothalamus by the actions mentioned before. When the cold receptors in the skin are stimulated by
decreased body temperature, preoptic and anterior hypothalamic nuclei are inhibited and posterior hypothalamus
activates sympathetic nervous system to activate heat gain mechanisms of skin.

2.2.2.

If any foreign substance to enter the body, it should first contact with the skin. So, the skin plays a major role as a
first line barrier for pathogens to enter. It does the 1st line defense by being a barrier to the pathogens to enter.
Skin takes out innate immunity. Skin has many layers. Top most layer is dead cells and keratin. They are tightly
packed. They are continually shedding. It prevents body fluids to leak through the skin. So that makes a mechanical
barrier along with the flushing of sweat to get away from pathogens. Sweat glands and sebaceous glands also helps
to make the skin moist to prevent it from mechanical attacks and pathogens. Skin has sweat by sweat glands and
sebum by sebaceous glands to work as chemical barriers. Sebum contains of fatty acid and it lowers the pH to
inhibit fungi and some bacteria. Sweat by sweat glands contain lysozyme and dermicidin lysozyme attacks
peptidoglycans which make the bacterial cell membrane. If any how the pathogens enter the skin, 2nd line defence
is taken place by cellular barriers to kill the microbes. Dendritic cells in the skin are phagocytic and intergrated
among epidermal cells. Langerhan cells present in epidermis are a unique populatation of tissue resident
macrophages that form a network of cells across the epidermis, but which have the ability to migrate from the
epidermis to draining lymph nodes. Their location in skin barrier also help to conduct immune functions. After
microbe is infected, inflammatory responses are occurs. The inflammatory responses are also give signals to that an
inflammation has started due to infection. Redness, pain, heat and swelling are almost always can be observed
from the skin. So, it is important to take steps to prevent further infection.

*Skin

-physical barrier

-chemical barrier - antimicrobial proteins

-biological barrier - cells

15th batch EME MCQ

01) The velocity of conduction of action potential along a nerve is determined by,
A. Body temperature
B. Diameter
C. Length of nerve fiber
D. Myelination of axon
Presented by 15th Batch 179 FHCS | EUSL
E. Stimulation of Na+, K+ pump

02) Inhibitory post synaptic potential


A. depolarizes the post synaptic membrane by detours opening K channels.
B. hyperpolarizes the postsynaptic membrane by opening Ca2+ channels
C. Hyperpolarizes the postsynaptic membrane by opening Cl- channels
D. Is a potential that depolarize the post synaptic neuron
E. Is a synaptic potential that less likely generate action potential in post synaptic neuron

03) Cardiac muscles are different from skeletal muscles with,


A. Elevation of intracellular Ca2+ for excitation - contraction coupling
B. High degree of electrical coupling between cells through syncytium
C. Spontaneous depolarization occurs in the membrane
D. Spontaneous depolarization of the membrane potential
E. Thick and thin filaments are arranged in sarcomeres

04) Multi-unit smooth muscles,


A. Behave as separate motor unit
B. Exhibit pacemaker activity
C. Have high degree of electrical coupling
D. Have own innervation
E. Found in intrinsic muscle of eye, piloerector and large blood vessels

05) Following is/are an inhibitory neurotransmitter in the central nervous system (CNS)?
A. Glycine
B. Histamine
C. Norepinephrine
D. Serotonin
E. Aminobutyric acid (GABA)

06) Zone of proliferation of epiphyseal plate,


A. Cells are arranged in longitudinal columns
B. Consist of small scattered cells
C. Has calcified extracellular matrix
D. Involves in proteoglycan rich matrix production
E. Is a true germinal layer

7) Regarding Elastin,
A. During biosynthesis, cross linking takes place via glycine
B. It is highly hydrophobic protein
C. It is protected by elastase enzymes
D. It is a dominant extra cellular matrix protein in arteries
E. Undergoes hydroxylation reaction as post translational modification

08) Which one of the following matches correctly with the function?
A. Ciliated columnar - Transportation
B. Simple cuboidal - secretion
C. Simple squamous - Protection
D. Stratified Cuboidal - Secretion
E. Transitional – Stretch

Presented by 15th Batch 180 FHCS | EUSL


09) Cardiac muscle is,
A. Multinucleated
B. has gap junction in intercalated discs
C. Has Calmodulin receptor for Ca2+ binding
D. has satellite cell for regeneration
E. Produce involuntary contraction

SBR
10) Neurogila cells its function correct Mach,
A. Astrocyte - Absorption of neurotransmitters
B. Ependymal cells - Secrete of celebrospinal fluid
C. Microglia cells – Phagocytosis
D. Oligodendrocytes - Facilitate the rapid nerve conducting
E. Satellite cells - Myelin sheath formation

11) The correct temporal sequence for event of the neuromuscular junction is?
A. Action potential in motor neuron, depolarized muscle end plate, uptake of Ca2+ in to presynaptic nerve terminal
B. Release of acetylcholine, action potential in the motor neuron, action potential in the muscle.
C. Release of acetylcholine, action potential in the muscle end plate, action potential in the muscle.
D. Uptake of Ca2+ into motor end plate, action potential in the motor end plate, action potential in the muscle.
E. Uptake of Ca2+ into the presynaptic terminal, release of Ach, repolarization of muscle end plate.

12) Repeated stimulation of skeletal muscle fiber cause a sustained contraction (tetanus) accumulation of which
solute in intracellular fluid is responsible for the tetanus.
A. ATP
B. Ca2+
C. Na+
D. Tropomyosin
E. Troponin

13) Which is the wrong pair regarding the effects of the hormone during the bone remodeling and growth?
A. Parathyroid hormone - Increasing bone resorption
B. Gluco corticoids - Stimulates osteoclast activity
C. GH hormone - Stimulates protein synthesis
D. Insulin like growth factor - Promote cell division in the epiphyseal plate
E. Oestrogen - Increase osteoblast activity

14) A location where you find the fibro cartilage would be?
A. Intervertebral disc
B. Nasal septum
C. Tracheal rings
D. Ear canal
E. Epiglottis

15) A cross sectional view of a skeletal muscle fiber through the H-zone would reveal the presence of,
A. Actin
B. Actin and troponin
C. Dystrophin
D. Myosin

Presented by 15th Batch 181 FHCS | EUSL


E. Actin and myosin

15th batch EME SEQ

1.
1.1.Explain the ionic basis of action potential on a neuron. (30 marks)
1.2. 5-Year old woman diagnosed with myasthenia gravis has been found with increased muscle stregth when she
is treated with an acetylcholinesterase (AChE) inhibitor. Explain the physiological basis of the above condition. (40
marks)
1.3. Outline the biochemical basis of
1.3.1. Muscle cramp occurs during sprinting (15 marks)
1.3.2. Ontogenesis imperfect (15 marks)

Answers

01)

Factors determine the speed of AP propagation

1.Amount of myelination

AP propagate more rapidly along myelinated axons than along unmyelinated axons.

2.Axon diameter

Larger diameter axons propagate action potentials faster than smaller ones.

Larger – diameter axons have larger surface area, short absolute refractory period of about
0.4msec(1000impulses/s)

Propreseptive sensation, Motor function, touch, pressure

Small – diameter axons have absolute refractory period as long as 4msec(250 impulses/s)

Pain, thermal sensation

3. Temperature

Axons propagate action potentials at lower speeds when cooled.

4. Pressure

Larger diameter fibers are susceptible, Saturday morning paralysis??

5. Hypoxia/ Anesthesia

A) T

B) T

C) F

D) T

E) F

Presented by 15th Batch 182 FHCS | EUSL


02) IPSP (Inhibitory Post Synaptic Potential)

•Hyperpolarization of post synaptic membrane, due to

•Opening of Cl- channels

•Opening of K+ channels

•Closure of Na+ channels

•Closure of Ca2+ channels

•GABA & glycine produces IPSPs in postsynaptic membrane

A) F

B) F

C) T

D) F

E) T

03)

A) F initially from ECF then intracellular Ca2+

B) T

C)

D)

E) F

04)

Tunica media of blood vessels has both visceral and multi-unit smooth muscle types.

Presented by 15th Batch 183 FHCS | EUSL


A) T

B) F – pacemaker activity present in single unit smooth muscle

C) F unlike unitary smooth muscle, multi unit smooth muscle is non syncytial and contractions do not spread
widely through it.

D) T

E) T

05)

A) T

B) F

C)

D) T

E) T

06)

Zone of Proliferation (P) :-

- Slightly larger chondrocytes

- Cells are flattened, arranged like stacks of coins / longitudinal columns

- True germinal layer of growth plate

- ↑ sed Type II collagen synthesis & mRNA expression

- Functions - proteoglycan rich matrix production

Presented by 15th Batch 184 FHCS | EUSL


- mitotic cell division → longitudinal growth

A) T

B) F – small scattered cells present in Resting zone.

C) F

D) T

E) T

07)

A) F – crosslinking via Lysines

B) T

C) F – Alpha – 1 antitrypsin protects elastin in lungs

D) T

E) T

08)

A) T Simple Columnar Ciliated Epithelium

Functions : Transportation & Secretion

Location: Lining of uterine tube/ uterus/ endo-cervix

B) T Simple Cuboidal Epithelium

Function: Secretion and absorption

Location: Glands and ducts (pancreas & salivary), kidney tubules, covers ovaries

C) F Simple Squamous Epithelium

Functions : • Absorption, and filtration • Not effective protection – single layer of cells.

Location : Walls of capillaries, air sacs in lungs, Form serous membranes in body cavity

D) F Stratified Cuboidal Epithelium

Function: Protection

Location: largest ducts of salivary gland, mammary gland and sweat gland

E) T Transitional epithelium

Function: • Stretches readily and permits distension of urinary organ by contained urine.

• acts as a permeability barrier and protects tissues from noxious effects of urine Location: Lines the
ureters, urinary bladder, and part of the urethra.

Presented by 15th Batch 185 FHCS | EUSL


09)

A) F – one or two central nuclei

B) T

C) F- calmodulin present in smooth muscles

D) – in skeletal muscle

• Satellite cells

• lies between sarcolemma and external lamina

• retain the capacity to regenerate damaged muscle fibers

E) T

10) Astrocytes

Functions:

• Provide metabolic and physical support for neurons

• Provide the appropriate chemical environment for the generation of nerve impulses

• Blood brain barrier

Ependymal cell

• Simple cuboidal to columnar epithelium

• line ventricles and central cavities of brain and spinal

• prevents passage of potentially neurotoxic substances to the brain

• bears apical microvilli to increase surface area

• has motile cilia that project into the ventricular lumen

• sweep foreign particles

• metabolite exchange between CSF and extracellular spaces of the brain and spinal cord

• Choroid plexus

• Lined by modified ependymal cells

• Secrete components of the CSF

Microglia

• Thorny bushes in appearance and the smallest glia

• Phagocytic function in CNS

• Constitute the brain’s immune system

• Originate from monocytes

Oligodendrocytes

• Form myelin sheath in CNS


Presented by 15th Batch 186 FHCS | EUSL
• one oligodendrocyte can myelinate up to 60 axons

11)

Neuromuscular Transmission

-Action potential reaches the motor end plate

- Increase the permeability of Ca 2+ in to the presynaptic axon terminal

-Trigger the release of acetylcholine contain vesicle through exocytosis

-Acetylcholine diffuse through synaptic cleft and bind with cholinergic receptors on motor end plate

-Opening of ligand gated channels and increased Na+ & K + conductance.

-End plate potential reach to firing level(Generation of action potential in muscle fiber)

12) B

13) B - Glucocorticoids inhibit osteoclast activity.

14) A Fibrocartilage is found in thick pad -like structures

• Menisci of the knee

• Intervertebral discs of the vertebral column

• Pubic symphysis

Hyaline cartilage

• Articular - covers the end of bones

• Epiphyseal plates

• Costal - connect ribs to sternum

• Laryngeal - skeleton of larynx

• Tracheal & bronchial - reinforce the respiratory passages

• Nasal - support the external nose

Elastic cartilage

• external ear

auditory tubes

epiglottis

15) D

Presented by 15th Batch 187 FHCS | EUSL


15th - EME SEQ

1.1.

A. Depolarization

( Stimulus generates a graded potential in the nerve fibre

( Some voltage gated Na+ channels open.

( Na+ INFLUX

( Membrane potential rises to threshold level (-55mV)

( More voltage gated Na+ channels open

( Rapid influx of Na+ makes the membrane potential more positive.


Presented by 15th Batch 188 FHCS | EUSL
( Membrane potential overshoots zero potential.

( This is a positive feedback cycle

B. Repolarization

( Resistance for Na+ influx increases as membrane potential becomes more positive, reversing the electrical
gradient for Na+ .

( Voltage gated Na+ channels close 1/104 s after they open.

( Na+ INFLUX stops.

( When MP rises from -70mV to 0mV, voltage gated K+ channels open. – They are slow to open and slow to close.

( Opening of K+ channels coincide with the closure of Na+ channels.

( Resulting K+ EFFLUX makes the membrane potential negative again.

( After depolarization, K+ voltage gated channels close via a negative feedback mechanism

C. Hyperpolarization

( K + efflux lasts longer than needed to restore membrane potential to RMP.

( Therefore, the membrane is hyperpolarized.

( When K+ channels close, K+ EFFLUX stops.

( Membrane stops becoming more negative.

( RMP is restored. But a large amount of Na+ remains within the cell & a large amount of K+ has escaped.

( Na+ / K+ ATPase corrects the ionic distribution

Key points for the answers (should be included)

What is AP?

Threshold stimulation

Depolarization – voltage gated Na+ channels open….Na+ influx

Firing level – positive feed back mechanism

Attain maximum diffusion potential of Na+/closing Na+ channels

Repolarization – opening K+ voltage gated channels….K+ efflux

Hyperpolarization – slowly closing K+ channels – negative feed back mechanism

RMP - Na+ K+ ATPase pump

Presented by 15th Batch 189 FHCS | EUSL


1.2.

- Myasthenia Gravis results by formation of circulating antibodies against muscle type of Nicotine cholinergic
receptors.

- These antibodies destroy some of the receptors & bind others to neighboring receptors triggering their removal by
endocytosis.

- In a normal person repetitive stimulation at neuromuscular junction causes decline in number of acetyl choline
molecules.

- Additional receptors are translocated to post synaptic membrane to up regulate the receptors.

- So no significant loss of function.

- In myasthenia gravis Ig G type antibodies are produced which can bind with ACh receptors.

- Down regulation of receptors (Decline number of receptors)

- ACh molecules decline with successive stimuli.

- But no up regulation of receptors.

- Loss of functions at neuromuscular junction.

- Muscle weakness & easy fatigability.

Cholinesterase catalyse the hydrolysis of the neurotransmitter acetylcholine into choline and acetic acid.

Cholinesterase inhibitors prevent breakdown of acetylcholine.

Compensate for the normal decline in released neurotransmitters during repeated stimulation.

1.3.

1.3.1.

Muscle contraction requires huge amounts of energy in the form of ATP. The amount of ATP stored in the cell only
last for 5 seconds. The rest of the ATP must be regenerated using one of three pathways.

Creatine phosphate

Anaerobic glycolysis

Aerobic respiration

Creatine phosphate provides energy up to 10 seconds. After that muscles get energy from anaerobic glycolysis.
Anaerobic glycolysis produces 2 ATP and lactic acid as a waste product. Accumulation of lactic acid causes muscle
fatigue and cramp.

1.3.2.

• Genetic disorder of bone fragility

• Increased bone fracture with minor or no trauma

• 80% cases due to mutation in alpha 1 or 2 chains in type II

• Most common mutation causes the replacement of glycine with bulky side chain (prevent triple helix structure)

• Type 1 most common; mild bone fragility, hearing loss and blue sclera
Presented by 15th Batch 190 FHCS | EUSL
• Type II ; most severe form, lethal in perinatal period

16th batch EME MCQ

01. Special features of simple columnar epithelium include


A. Goblet cells
B. Keratin
C. Microvilli
D. Stereocilia
E. Umbrella cells

02. Which cells of supporting tissue are responsible for secreting extra cellular matrix
A. Adipocytes
B. Chondrocytes
C. Fibroblast
D. Myofibroblast
E. Osteoclast

03. Neuroglial cells and its functions correctly match


A. Astrocyte- Blood brain barrier
B. Ependymal cells- Absorption
C. Oligodendrocyte- Metabolites exchange
D. Microglia cell- Phagocytosis
E. Satellite cells- Myelin sheath formation

04. Regarding cardiac muscle contraction,


A. ATP hydrolyze to supply energy
B. Binding of Ca2+ to troponin C exposes myosin binding sites of actin
C. Increased Na+ conduction generates an action potential in adjacent myocytes
D. Wave summation cause tetanus formed in cardiac myocytes
E. Uses extracellular Ca2+ for cross bridge formation

05. Following is/are an excitatory neurotransmitter in the central nervous system


A. Glycine
B. Histamine
C. Norepinephrine
D. Serotonin
E. Gamma amino butyric acid (GABA)

06. Excitatory post synaptic potential


A. Depolarizes the post synaptic membrane by detours opening K+ channels
B. Depolarize the post synaptic neuron by opening the Na+ ion channels
C. Hyperpolarizes the postsynaptic membrane by opening Cl- channels
D. Synaptic potential that depolarizes post synaptic membrane
E. Decrease action potential by moving membrane potential further away from threshold

Presented by 15th Batch 191 FHCS | EUSL


07. A
A.
B. Binding of neuro transmitter to post synaptic membrane opens voltage gated Ca2+ channels
C.
D.
E.

08. Regarding collagen


A. Collagen fibers are rich in alanine and leucine
B. It is a glycoprotein
C. It maintains the integrity of tissues
D. Marfan’s syndrome occurs due to defect in collagen fibers
E. Basic unit is tropocollagen

09. Regarding proliferative zone in bone


A. Anchors the epiphysial plate to the epiphysis of the bone
B. Anchor the epiphysial plate to the epiphysis of the bone
C. Is involved in proteoglycan rich matrix production
D. Is the true germinal layer of growth plate
E. Is composed of calcified matrix

10. Regarding creatinine phosphate,


A. Is formed from amino acid, glycine, arginine
B. Creatin phosphate mainly stored in liver
C. In creatinine phosphate system, the ATP production require oxygen
D. By measuring urine creatinine phosphate level can calculate total muscle mass
E.

SBR
11. Which of the following sports, training program for increasing maximal muscle glycogenolytic enzyme activity
would be less important?
A. 200m track
B. 800m track
C. Basketball
D. High jump
E. Swimming

12. At node of Ranvier of myelinated axon, voltage gated Na+ channels are inactive, voltage gated K+ channels are
active and the membrane potential is in fall. What phase of neural action potential is occurring here?
A.
B. Depolarization
C. Hyperpolarization
D. Repolarization
E. Resting potential

13. O
A. Decreases the action potential velocity
B. Depolarize the neuron
C.

Presented by 15th Batch 192 FHCS | EUSL


D. Hyperpolarization of neuron
E. Increases action potential velocity

14. A tough sheath of dense irregular connective tissue the surround the bone surface, function bone growth, assist
in fracture repair, help to nourish the bone tissue and serve as an attachment site for tendons, what is the most
possible structure mentioned in the above statement?
A. Diaphysis
B. Endosteum
C. Epiphysis
D. Metaphysis
E. Periosteum

15. In cardiac cells nerve transmission is conducted through


A.
B. Desmosomes
C. Gap junction
D. Hemidesmosome
E. Tight junction

16th batch EME SEQ

1.
1.1. A 42-year-old woman presents with the history of episodes of weakness in limbs. MRI reveals multiple sclerosis,
a demyelinating disease damages the myelin sheath of the nerve cells in the brain and spinal cord.
Explain how the demyelination causes muscle weakness in the above condition. (30 marks)
1.2. Cardiac muscles functions as syncytium. Explain the phenomenon in relates to synaptic transmission (15 marks)
1.3. Briefly describe the factors affecting bone growth and remodeling. (15 marks)
1.4. List all the cells that are found in the bone matrix. (10 marks)
1.5. State one function of each of the cells mentioned in 1.4 (15 marks)

Answers

01.

A- T

B- F

C- T

D- T

E- F( in urinary bladder, ureters and renal pelvis- transitional epithelium)

02.

A- T(mainly collagen)

B- T
Presented by 15th Batch 193 FHCS | EUSL
C- T

D- T(replace the myochardium of the scar by secreting new ECM)

E- F

03.

Neuroglial cells and functions

1. Astrocytes- most numerous


- Star shaped, long branched processors
Function: - Mechanical support
Mediate exchange of materials between neurons &vascular system
Form part of blood-brain barrier
Repair of CNS tissue after damage
2. Oligodendrocytes – Responsible for myelination of axon
- Absent in Gray matter/organized in tracts
3. Microglia – Small cells
- Nucleus—small, elongated / Cytoplasm- Scanty Branched processes
Function: - Represent the monocyte- macrophage system
4. Ependyma- Make the specialized epithelium of ventricle & spinal cord
- Cuboidal cells

A- T

B- T(reabsorb the CSF)

C- T(facilitates the transfer of metabolites to fuel neurons and support the integrity of myelinated axons)

D- T

E- F(provide nutrient support and protection in PNS)

04.

A- T

B- T

C- T

D- F(cardiac muscle cells have long refractory period,so no summation or tetanus seen in cardiac muscles)

E- T

05.

Presented by 15th Batch 194 FHCS | EUSL


A- F

B- T (both inhibitory via GABA and excitory)

C- T

D- F

E- F

06.

Arrival of action potential in axon terminal

Opening of calcium channels in presynaptic membrane

Influx of calcium ions from ECF into the axon terminal

Opening of vesicles and release of ACh via exocytosis

Passage of ACh through synaptic cleft

Formation of neurotransmitter receptor complex

Opening of sodium channels and influx of sodium ions from ECF

Development of exitatory postsynaptic potential

A- F

Presented by 15th Batch 195 FHCS | EUSL


B- T

C- T

D- T

E- T

07. ?

A-

B- F (sodium channels)

C-

D-

E-

08.

Collagen

• High amount in supporting structures


• Most abundant protein (25% of proteins in mammals)
• Rigid
• insoluble
• Several types depend on structural role
• Usual amino acid composition
➢ 33% Glyicine – enables easy packing
➢ 25% Proline /Hydroxy Proline
➢ Also contain Lysine, Hydroxy Lysine
• Low aromatic amino acids (Try, Tyr)
• Gly – X-Y repeating sequence
• X= Pro or other, Y= OH-Pro, OH-Lys or other
• Pro α chain forms a left handed kinked helix with 3 AA per turn
• Triple helical structure with 3 α chains (not α helixes) intertwined in right handed manner to form pro collagen
• Opposite twist gives added strength
❖ Type 1 is the most common

❖ Types;
➢ Type 1 – skin, bone, tendon, blood vessels, cornea
➢ Type 2 – cartilage, intervertebral disc mineralized
➢ Type 3 – blood vessels, foetal skin
➢ Type 4 – basement membrane

A- F (low level of leucine)

B- F

C- T

D-
Presented by 15th Batch 196 FHCS | EUSL
E- T

09.

A- F (resting zone is the one which anchors)

B- F

C- T (in the proliferative stage, the osteoblast synthesis a type 1 collagen-rich matrix, termed osteoid, to support
ECM formation)

D- T (germinal layer- contains chondrocytes in a relatively quiet state that will serve as the source of cells for
growth)

E- F

10.

Phosphagen system
o Creatine phosphate (CP) is stored in muscles
o CP contsins a high energy bond.
o Bond is cleaved reversibly to reconstitute ATP.

o CK has different isoforms in different muscle types and it is useful as a cardiac biomarker.

A- T (glycine+arginine+methionine)

B- F (stored in skeletal muscle, brain and heart)

C- T

D- T ( serum creatinine levels are influence by muscle mass. And creatinine is a derivative of creatinine phosphate)

E-

Presented by 15th Batch 197 FHCS | EUSL


11. D (Explosive events such as jumping or throwing derive almost all of their ATP from resting muscle stores and
from PCr hydrolysis. There would therefore be little to gain from an increase in muscle glycogenolytic activity)

12. D

13. ?

14. E

15. C

16th - EME SEQ

1.

1.1.

Demyelination interrupts the flow of nerve impulses in the spinal cord or (less frequently) in the brain that keep a
muscle or group of muscles functioning properly. As a result, muscles lose strength.

Myelin allows nerve fibers to transmit signals very rapidly (40-60 meters/second). Loss or damage to myelin can
cause slowing or blockage of the nerve signals and can lead to loss of nerve fibers. This causes weakness, paralysis
and/or impairment in motor function, especially of the arms and legs.
Presented by 15th Batch 198 FHCS | EUSL
1.2.

Synaptic transmission is the biological process by which a neuron communicate with the target cell across a
synapse. Chemical synaptic transmission involves the release of a neurotransmitter from the pre-synaptic neuron,
and neurotransmitter binding to specific post-synaptic receptors. Electrical synapse transmission involves the
transfer of electrical signals through gab junctions. Cardiac muscle act as a syncytium through gab junctions. They
conduct signals by electrical synapse transmission.

1.3.

♠Nutrition

*Calcium (Ca2+)
*Phosphate (PO43-)
*Fluoride (F-)
*Magnesium (Mg2+)
*Iron (Fe3+)
*Manganese (Mn2+)
2. Vitamins

*Vitamin C
*Vitamin D

*Vitamin K & B12


*Vitamin A

♠Hormones

1. GH

2. Sex hormones (Estrogens and Androgens)

3. Insulin like growth factors (IGFs)

4. Thyroid hormones (T3 & T4)

5. Insulin

6. Parathyroid hormone

7. Glucocorticoids

8. Calcitriol
♠Exercise
♠Life style
♠Changes in diet

1.4.

1.5.

Presented by 15th Batch 199 FHCS | EUSL


Presented by 15th Batch 200 FHCS | EUSL
PHASE 01 QUESTIONS

8TH BATCH PROPER MCQ

16) Features limited to epithelial cells


A) Avascularity
B) Cellularity
C) Non polarity
D) Degeneration
E) Innervation

17) Collagen
A) Is synthesized by fibroblasts
B) Rich in glycine and proline
C) Type ІІІ contributes to form the basement membrane
D) Type ІV is hyaline
E) Has 4 types

18) Osteoblast
A) Involves in bone formation
B) Stimulated by calcitonin
C) Secrete inorganic calcitonin
D) Multinucleated
E) Concern with bone resorption

19) Regarding the bone tissue


A) Osteocytes synthesis osteoid and mediate its mineralization
B) Osteocytes represent largely inactive osteoblasts trapped within formed bone
C) Osteocytes assist in nutrition of bone
D) Osteoclasts are phagocytic cells capable of eroding bone
E) Osteoblasts are derived from osteoprogenitor cells

20) Cardiac muscle fibers


A) It has nucleus peripherally
B) Has transverse as well as longitudinal striations
C) Has multiple nucleus
D) Are tightly bound each other so that contraction of one cell spreads to the next cell
E) Are branched cells

22) Neuron cells generally have


A) Well developed cytoskeleton
B) Poorly organized vesicles
C) Few lysosomal elements
D) Prominent RER in all types
E) Small nucleus

24) Conducting system for contraction of muscle fiber A) Motor end plates innervate two to three muscle fibers
B) The neurotransmitter of somatic neuromuscular is acetylcholine
Presented by 15th Batch 201 FHCS | EUSL
C) Each myofibril is surrounded by sarcoplasmic reticulum and T tubules into it
D) Depolarization release Ca2+ from sarcoplasm into the sarcoplasm surrounding myofilaments
E) Ca2+ activate the sliding filament mechanism of myofibril

25) Regarding functioning of skeletal muscle fibers


A) In duchenne muscular atrophy, muscle has deficient link protein called dystrophin
B) Some myofibrils of a muscle fiber are arranged in irregular pattern
C) Electron dens Z bands divide each myofibril into sarcomeres
D) Thin actin filaments are attached to Z line
E) In sliding theory, sliding of myosin and actin filaments shortens the sarcomere

44) Regarding neurotransmitters, following statements are true, EXCEPT


A) GABA brings inhibition
B) Serotonin inhibit pain pathways
C) Acetylcholine activate sweat glands
D) Pre ganglionic neurons of sympathetic ganglia produce acetylcholine
E) Dopamine possesses excitatory action

45) Adipose tissue has the following features EXCEPT? A) Has sympathetic nerve endings
B) Bear receptors for insulin
C) Act as an insulator, by serving as a heat generator
D) Has cushioning effect in soles and palms
E) Has the capacity for regeneration than hypertrophy in mature state

48) Elastic cartilages are absence in


A) Pinna
B) Corniculate cartilage
C) Medial part of the auditory tube
D) Arytenoid cartilage
E) Epiglottis

56) Tetani is seen in skeletal muscles but not in cardiac muscles) Best explanation for that is
A) Mechanical event on cardiac muscle is almost complete in refractory period
B) Refractory period of action potential in cardiac muscles longer than skeletal Muscles
C) Slow opening Ca2+ channels in action potential of cardiac muscles
D) There is a plateau phase in cardiac muscle
E) Tetani of cardiac muscles will lead to death

08TH BATCH PROPER SEQ

3.0. Write short notes on


3.1. Cardiac muscle (30 marks)
3.2. Neuroglia of the nervous system (30 marks)

Presented by 15th Batch 202 FHCS | EUSL


Answers

16.

A) F - Cartilages are also avascular.

B) F

C) F - Epithelial cells have apical and basal poles.

D) F

E) F

17.

A) T

B) T

C) T - Both type III and IV

D) F - Type II

E) F - about 25

18.

A) T

B) T

C) F - Calcitonin secreted by para follicular cells of thyroid.

D) F - Single nucleus

E) F- Bone mineralization

19.

A) F - Osteoclast

B. T

C. T

D. T

E. T

20.

A) F - Central nucleus

B) F - Vertical

C) F - Single nucleus

D) F - Through gap junction

E) T

22.

A) T
Presented by 15th Batch 203 FHCS | EUSL
B) F - Nissle’s granules are RER

C) T

D) T

E) F - Prominent large nucleus

24.

A) T

B) T

C) T

D) T

E) T

25.

A) Absent link protein

B) F - Regular patterns

C) T- Sarcomere is the functional unit of a muscle fiber. It is the

repeating unit between two Z lines

D) T

E) T

44) E

Dopamine is an inhibitory neurotransmitter secreted by

substantia nigra

45) E

48) B

Elastic cartilage is found only in the external ear, auditory tubes,

epiglottis and parts of laryngeal cartilages.

Arytenoids cartilage is formed by hyaline cartilage

08TH BATCH PROPER SEQ

3.0. Write short notes on

3.1. Cardiac muscle (30 marks)

3.2. Neuroglia of the nervous system (30 marks)

Answers

3.

3.1
Presented by 15th Batch 204 FHCS | EUSL
located in myocardium of heart.

 Muscle fibres are long,cylindrical,branched and have 1 or 2 nuclei which


 is centrally located.
 End of fibres are spit longitudinally into small number of branches.
 Actin , myosin fibres arranged in sarcomere,so have striations.
 Has intercalated discs. Contain desmosomes.
 Have gap junctions which allow action potential to spread one muscle to
 other (function syncytium)
 Cardiac muscle tissue has endomysium and perimysium but lack of
 epimysium,
 Contract when stimulated by it’s own autorythmic muscle fibres.
 Mitochondria in muscle fibres are larger and numerous than skeletal
 muscle.
 Has tubule system and sarcoplasmic reticulum.
 Involuntary.
 Has automaticity feature.
 Cannot be tetanized because it has large absolute refractory period.
3.2

Presented by 15th Batch 205 FHCS | EUSL


2 types in PNS

 Schwann cells – envelope nerve fiber in PNS


 Wind repeatedly around a nerve fiber
 Produce myelin sheath

Presented by 15th Batch 206 FHCS | EUSL


Satellite cells
 Surround the neuro soma in the ganglia of PNS
 Provide electrical insulation around the soma
Regulate the chemical

09TH BATCH PROPER MCQ

13) Connective tissue


A) Bind and support body
B) Line the body cavity and surface
C) Store lipid
D) Store minerals
E) Dissolve substances and transporters

14) Regarding cartilage


A) Avascular and innervated
B) Osteoarthritis is disease of articular cartilage
C) Collagen fibers provide a tonsil strength
D) The matrix of cartilage include calcium salt
E) Original skeleton of human embryo is made up of cartilage

15) Neuroglia cells


A) Cause phagocytosis
B) Have ability to reproduce
C) Play a role in cell to cell communication
D) Carry nervous signals
E) Support and hold nervous signals

16) Myaline sheath around the neuron


A) Astrocyte
B) Microglia
C) Oligodendrocyte
D) Satellites
E) Schwan

22) Conduction of nerve fiber


A) C fibers have slow conduction
B) When impulse enter in to the nerve middle spontaneously it goes right away
C) A and B fibers are myelinated
D) Demyelination diseases reduce conduction
E) Nerve fibers re saltatory

23) T or F regarding a neuron


A) The resting cell membrane is in a depolarized state B) Sub-threshold stimulus does not cause an action potential
C) Reestablishment of Na+ and K+ gradients after an action potential needs energy
D) An action potential can be propagated in both directions along the nerve fibe
Presented by 15th Batch 207 FHCS | EUSL
E) The conduction rate is highest in unmyelinated fibers

24) The osteoblasts


A) Have many nuclei
B) Are responsible for the synthesis of bone collagen C) Are capable of removing cartilage remnants on
endochondrial ossification
D) Have a basophilic nucleus
E) Develop from undifferentiated mesenchymal cells

25) Regarding skin


A) Secretion of sweat gland is under neural control
B) All nerve endings are pain receptors
C) Skin is the site of vitamin D production
D) Leptin is secreted by subcutaneous fat
E) Cutaneous vessels contribute to skin color

26) Epithelium tissue


A) More extensive extracellular matrix
B) Tight junctions are more
C) Avascular but it has nerve supply
D) Are present in visceral organs
E) Origin from all 3 germ layers

27) Regarding skeletal muscles


A) Has largest muscle fibers and multinucleated
B) Under voluntary control
C) Satellite cells retain the capacity to generate damage muscle fibers
D) Well-developed t tubules
E) Striated muscles have bands of actin and myosin filaments

28) Bone
A) Rich in blood supply
B) Serve as a source of mineral ions for homeostasis
C) Thickness has growth plate
D) Has a good regenerative potential to regenerate throughout life
E) Disorder caused by vit D deficiency leads to reduced strength

35) Sarcotubuler system in muscles


A) Include sarcoplasmic reticulum
B) Transmit action potential
C) Connected with ECF
D) Contain high levels of Ca2+
E) Connected to terminal cistern

52) Lamella Granules are rich in glycolipid & released by keratinocyte from a barrier to matrix. Which are mostly
epidermis?
A) Stratum Corneum
B) Stratum Lucidum
C) Stratum Granulosum
D) Stratum Spinosum

Presented by 15th Batch 208 FHCS | EUSL


E) Stratum Basale

09TH BATCH PROPER SEQ


01.
a.
1.1 Describe the following ,
ossification of humerus (35 marks)
1.2 Write a brief account on microscopic feature of skeletal muscle fiber (35 marks)
b.
Explain the physiology basis for,
1.3 Giving anticholinesterase in myasthenia gravis (50 marks)
1.4 Reduced excitability of cardiac muscle in hypokelamia

Answers

A) T

B) F - Epithelial tissues

C) T - Storage of energy in the form of lipids.

D) T- Bones tissue

E) T - Blood

14)

A) F

Cartilage – avascular and no innervation

Epithelium – avascular but innervated

B) T

C) T

D) F - In GAGs

E) T

15)

A) T

 Microglia Cells
 Neuroglia cells are supporting cells in
 Astrocytes
 Oligodendrocytes
 Microglia
 Ependymal
 Schwann cells
 Satelite cell
B) T - Astrocytes

C) T - Astrocytes Blood brain barrier

Presented by 15th Batch 209 FHCS | EUSL


D) F

E) T

16)

A)F

B) F

C) T

D) F

E) T

17)

A) T -Ganong pg. 94 – table 4.1

B) T

2 impulses traveling in the opposite directions are set up

C) T

D) T

E) T

Only A & B myelinated nerve fibres

Saltatory conduction; propagation of the action potential by

jumping from node to node.

23)

A) F

It is in a polarized state

B) T

But after depolarization stage even a sub threshold 2nd stimuli can elicit a 2nd action potential (supernormal
period)

C) T

Under the influence of Na+

/K+ ATPase pump

D) T

But unidirectional flow across a synapse

E) F

Type A and type B fibers have the highest conductive velocity

since they are myelinated. C fibers are not myelinated, hence,

their conductive velocity is low.


Presented by 15th Batch 210 FHCS | EUSL
24)

A) F -it is related with osteoclasts

B) T -Osteoblasts secrete bone matrix

C) F- it is done by phagocytic cells

D) T -Purplish color

E) T- Bone is a type of connective tissue

25)

A) T- Sympathetic cholinergic

B) T

C) T -Lippincott pg. 387 – fig. 28.23

D) T- Secreted from adipose tissues.

E) T

26)

A) F -less or no extracellular matrix

B) T

C) T

D) T

E) T

27)

A) T

B) T

C) T -Satellite cells retain the capacity to regenerate damaged muscle

fibers.

D) T -Muscle action potentials travel along it.

E) T

28)

A) T -Through the haversian system

B) T - Ca2+ & PO43-

C) F

D) F

Presented by 15th Batch 211 FHCS | EUSL


E) T

Vit D deficiency causes osteomalacia and rickets

35)

A) T

Sarcotubular system is made up of T system and sarcoplasmic

reticulum.

B) T

C) T The space between the two layers of the T system is an extension

of the extracellular space.

D) T

Important store of Ca2+

E) T - Has enlarged terminal cisterns in close contact with the T system at the junction between the A and I bands.

52) C

09TH BATCH PROPER SEQ

01.

a.

1.1 Describe the following ,

ossification of humerus (35 marks)

1.2 Write a brief account on microscopic feature of skeletal muscle fiber (35 marks)

b.

Explain the physiology basis for,

1.3 Giving anticholinesterase in myasthenia gravis (50 marks)

1.4 Reduced excitability of cardiac muscle in hypokelamia

Answers

1.

a)

1.1



Presented by 15th Batch 212 FHCS | EUSL
Small model of the long bone is formed by solid hyaline cartilage.

1.2

 mely elongated & cylindrical shaped.



 – striations of alternating light & dark
 bands.


 a dark transverse line, the “Z” line.

 extends from “Z” line to “Z” line.


 sarcomeres.
 & thin filaments lie parallel to long axis of the myofibrils
 in a symmetric pattern.

 form the “A” band.

 attached to the “Z” line.

 overlap thick filaments.

 of thin filaments that overlap thick filaments.
 nter of the “A” band & it
 is called as “H” band.
Presented by 15th Batch 213 FHCS | EUSL

 connections are made between adjacent.

 hexagon.
 proteins.
1. Actin

2. Troponin

3. Tropomyosin

1. Myosin

globular.

-binding

sites.

nin has 03 subunits which are,

1. TnT – strongly attach to tropomyosin.

2. TnC – bind to Ca2+

3. TnI – inhibits actin-myosin interaction.

pair of light chains.

b.

1.3

Myasthenia Gravis results by formation of circulating antibodies

against muscle type of Nicotine cholinergic receptors.


 to neighboring receptors triggering their removal by
 endocytosis.

 junction causes decline in number of acetyl choline molecules.

 membrane to up regulate the receptors.


 can bind with ACh receptors.


Presented by 15th Batch 214 FHCS | EUSL
 ut no up regulation of receptors.



 ( Ach esterase break down Ach into acetyl & choline)
 evel in neuromuscular junction (NMJ)

 NMJ is enough to normal function.

1.4

Phase 01 – Na+ Channels opening followed by inactivation/ Na+ influx

- Phase 01 - Na+ Channels inactivation & K+ channels activation/ K+

efflux

- Phase 03 – Opening of slow inward Ca2+ channels/Ca2+ influx

- Phase 0 - Establishment of RMP by Na+/K+ ATPase pump

- Hypokalemia – decreased K+ concentration in the extracellular fluid

than normal level



 the cell

 electronegative
 ent


 energy
 -) charge in the cell

 gradient for K+ increases.

 gradient (Normally K+ concentration in intracellular fluid >
 extracellular fluid)

 become more negative
 reach the threshold level easily.

 increased.

Presented by 15th Batch 215 FHCS | EUSL


10TH BATCH PROPER MCQ

10) Collagen
A) Synthesized in fibroblast
B) Has triple helix structure
C) Rich in lysine
D) May be enzymatically glycosylated
E) Synthesis is affected by ascorbic acid deficiency

14) Epithelium tissue


A) Has abundant extracellular materials
Presented by 15th Batch 216 FHCS | EUSL
B) Lower mitotic index
C) Lack of blood vessels
D) Cells are anchored to a basement membrane
E) Lines the body surface & cavities

16) Skin
A) Is 15% of total body weight in normal adults
B) Is the storage organ for vitamin D
C) Always has an acidic PH
D) is impermeable to drugs
E) Epidermis is continuously replaced

17) Match the epithelium with their functions


A) Simple squamous - secretion
B) Simple cuboidal- Excretion
C) Simple columnar- secretion
D) Stratified squamous - protection
E) Stratified cuboidal- Secretion

18) Fibroblast
A) Has an extensive RER & free ribosomes
B) Release histamine & associated with allergies & inflammation
C) Secretes proteins that become fibers in the connective tissue matrix
D) Has the ability to provide repair after injury in connective tissue
E) Is responsible for scar tissue formation

19) When compare lymph with blood, the lymph


A) Is formed from interstitial fluid
B) Has no protein
C) Is intracellular fluid
D) Has no blood cells
E) Carries chylomicrons absorption from the small intestine

20) Bone tissue


A) No nerve innervation
B) Reduce strength by vitamin D deficiency
C) Has good regenerative power throughout life
D) Increase thickness by growth plate
E) Osteoblasts mediate bone mineralization

21) Hyaline cartilage in bone


A) Disease cause Osteoarthritis
B) Rich in blood supply
C) Undergo endochondral ossification
D) Found in External ear
E) Give flexibility to structure

22) All or non-rule is followed by


A) One skeletal muscle fiber
B) One nerve fiber

Presented by 15th Batch 217 FHCS | EUSL


C) All skeletal muscle
D) All cardiac muscle
E) Nerve bundle

23) The type of muscular contraction in which the muscle develop


tension but not length
A) Isometric contraction
B) Isotonic contraction
C) Tetanic contraction
D) Treppe
E) Twitching

24) An action potential in a skeletal muscle cell


A) Has a prolonged plateau phase
B) Spreads inwards via T tubules
C) Initiates binding of Ca2+ to calmodulin
D) is likely to be generated within absolute refractory period of another
action potential.
E) Rapidly move to another muscle cell through gap junctions

25) Myelination
A) Found in neurons inside the CNS
B) Complete at end of the fetal life
C) Swan cells involved in myelination
D) It slow down the nerve conduction velocity
E) Nerve conduction velocity inversely proportional to diameter of the
cell

35) Factors help in Bone growth


A) Vitamin k
B) Phosphate iron
C) Exercise
D) Vitamin E
E) Insulin like growth factor

49) Which of the following features of transitional epithelium is most


likely responsible for the distension of the urinary bladder
A) Lies in basement membrane
B) Multiple layers
C) Presence of desmosomes
D) Tight junction
E) Umbrella cells

50) A tough layer of dense irregular connective tissue that surround the bone surface functioning in bone growth
assist in fracture repair nourish, attachment for tendons, what's the structure
A) Periosteum
B) Diaphysis
C) Epiphysis
D) Endosteum
E) Metaphysis

Presented by 15th Batch 218 FHCS | EUSL


51) Regarding bipolar neurons
A) Most Abundant type
B) Consists of multiple dendrites
C) In Olfactory Epithelium
D) Only in Moto neurons
E) Can’t find in retina

52) Resting membrane potential is depend on


A) Intracellular K ions
B) Extracellular Na ions
C) Intracellular phosphate ions
D) Extracellular Cl ions
E) All of the above

54) What are the iron need in muscle contraction?


A) PO43-
B) Ca2+
C) K+
D) Na+
E) HCO3-

10TH BATCH PROPER SEQ


02. Write an account on
2.1 Cell cycle (25 marks)
2.2 Endochondral ossification (30 marks)
2.3 Epithelium of gastrointestinal tract (45 marks)
03. Regarding neuromuscular junction
3.1 Briefly describe the structure at the neuro-muscular junction. (40 marks)
3.2 Briefly describe the sequence of changes during excitation – contraction coupling. (60 marks)

10)

A) T -

B) T - 3 polypeptide chains (α chains) wound around one another in

rope like triple helix.

C) F rich in proline and glycine.

D) T the hydroxyl group of hydroxyl lysine residues of collagen may be

enzymatically glycosylated.

E) T ascorbic acid required for post translational modification.

14)

A) F - less extracellular matrix


Presented by 15th Batch 219 FHCS | EUSL
B) F- high ability to regenerate

C) T - avascular but innervated. Only blood supply to basement

membrane nutrients reach the cells through diffusion.

D) T

E) T - lines body surfaces, cavities and tubes

16)

A) T- largest organ of body in both weight and surface area. (16% of

BW)

B) F - only synthesize vit. D3 from UV light

C) T - for antibacterial action

D) F - skin is a selectively permeable barrier. Is permeable to water

soluble substances

E) T - composed of keratinized stratified squamous epithelium

17)

A) F - simple squamous—filtration, diffusion.

B) T - simple cuboidal—excretion, secretion, absorption.

C) T - simple columnar—secretion, absorption.

D) T- stratified squamous—protection, eg: - skin

E) T- cuboidal—secretion

18) Fibroblasts – supporting cell in connective tissue. Synthesize,

secret, maintain and recycle the extra cellular matrix.

A) T - has prominent RER mitochondria and Golgi apparatus

B) F - mast cells – abundant in sides of blood vessels supplying to

connective tissue, secrete histamine and associated in allergic

and inflammation

C) T - secrete GAG, collagen and all other extra cellular constituents

D) T - because it secretes collagen

19)

A) T- lymph is formed as an ultra-filtrate of capillary micro circulation.

They are formed from interstitial fluid collected via lymph

capillaries.

B) F- composition of lymph is similar to ECF except the presence of

high concentration of protein in lymph.


Presented by 15th Batch 220 FHCS | EUSL
C) F - is extracellular

D) F-

glucose, fats, water

E) T - functions of lymph,

tissues

 Recirculation of lymphocytes
 Exposure of antigens to the immune system
 Formation of lymphocytes
 Absorption and transport of fatty acids and fats as
 chylomicrons from digestive system
20)

A) F- blood vessels, nerves and lymph are carried in neurovascular

channels called Haversian channels.

B) T -

Deficiency causes rickets and osteoporosis.

C) F- after menopause bone strength

D) F -

Increase in thickness by appositional growth at the periosteal surface

E) T -

blood

Ca2+ homeostasis

21)

A) T - osteoarthritis is a degenerative disease of synovial joint due to

excessive wear and tear.it is a degenerative change in articular

cartilage. Articular cartilage is a hyaline cartilage.

B) F- devoid of blood supply. Blood supply is only up to ground

substance and diffuse from there.

C) T - endochondrial ossification -

- Hyaline cartilage forms a small model of the long bone

and it is progressively replaced by bone tissue.

D) F - elastic cartilage – external auditory canal, external ear, epiglottis,

Presented by 15th Batch 221 FHCS | EUSL


parts of laryngeal cartilage, walls of Eustachian tube.

Hyaline cartilage – nasal septum, larynx, tracheal ring, most

articular surfaces, sternal edge of ribs.

Fibro cartilage – intervertebral disc, some articular cartilage, pubic

symphysis, joint capsule, ligaments, tendons.

E) T - hyaline is smooth, firm but slightly flexible.

22) All or non-law:-

Action potential occur with constant amplitude and form

regardless of the strength of the stimulus, if the stimulus is at or

above threshold intensity.

Individual fibers of both skeletal muscle and nerve respond to

stimulations according to all or none law.

A) T

B) T

C) F

D) F

E) F

23)

A) T -

B) F -

C) F- tetany = summed muscular contractions, sustained contraction.

D) T-reppe = graduated series of increasingly vigorous contractions

that results when a

Corresponding series of identical stimuli is applied to a rested

muscle.

Twitching = the response of a muscle to a single brief threshold

stimulus.

E)

24)

A) F - There is no plateau phase. A prolonged plateau phase is seen in

cardiac muscles.

B) T - the depolarization spreads from the nerve to the sarcolemma and

then to all the fibrils in the fiber via T tubules.


Presented by 15th Batch 222 FHCS | EUSL
C) F- Ca2+ binds to troponin C in skeletal muscles and in smooth

muscles it binds to calmodulin.

D) F - Even a supra threshold stimuli cannot elicit a second action

potential during this absolute r refractory period.

E) F- Gap junctions are low resistant bridges for the impulse

transmission which are seen in cardiac muscles & smooth

muscles but not in skeletal muscles

25)

A) T -

B) F - begins – 4 months of gestation

Final myelin sheath thickness achieved by the time of physical

maturity.

C) T

D) F - depolarization in myelinated axons travels from one node of

Ranvier to the next. This jumping of depolarization from node to

node is called “saltatory conduction”. Myelination allows axons to

conduct up to 50 times faster than the fastest unmyelinated fibers.

E) F- diameter α velocity.

35)

A) T

B) T - bone matrix is composed of Ca2+& PO4-3

C) T- bone density

D) F

E) T

49)D

50)A

51)C

52) E

10TH BATCH PROPER SEQ


Presented by 15th Batch 223 FHCS | EUSL
02. Write an account on

2.1 Cell cycle (25 marks)

2.2 Endochondral ossification (30 marks)

2.3 Epithelium of gastrointestinal tract (45 marks)

03. Regarding neuromuscular junction

3.1 Briefly describe the structure at the neuro-muscular junction. (40 marks)

3.2 Briefly describe the sequence of changes during excitation – contraction coupling. (60 marks)

Answers

2.1

Cell cycle

 time interval between 2 miotic divisions is the cell cycle of a cell


 24 hours
 Consist of two major periods.
 They are interphase and mitosis ( M phase)
 Interphase is the interval between cell division and it consist of 3
 phases.



 Interphase is in a high metabolic activity and does most of its growing
 and synthesis of organelles.
 In M phase ( mitosis) duplicated chromosomes and organelles goes to
 daughter cells
 M phase is the periods ofcell division.
 M phase is consist of 4 phases.
 They are




2.2

Endochondral ossification

 Replacement of hyaline cartilage by bone is called endochondral


 ossification.
 Mesenchymal cells cluster together in shape of bones and develop into
 chondroblasts.
 Chondroblasts secrete cartilage, extracellular matrix, producing
 cartilage model.
 Chondroblasts become deeply buried in cartilage extracellular matrix
 and called chondrocytes.
 Continual cell division of chondrocytes with further extracellular matrix
 occur.
 It leads to interstitial growth leading to increase in length.
Presented by 15th Batch 224 FHCS | EUSL
 Growth of cartilage in thickness due to addition of more extracellular
 matrix material.
 When the extracellular matrix is deposited on cartilage surface leading
 to appositional growth.
 Development of primary ossification centers occur in diaphysis which
 replace the most of cartilage.
 Osteoblast begin to deposit bone extracellular matrix over remnant of
 calcified cartilage, forming spongy trabeculae.
 Osteoclast breakdown some newly formed spongy bone trabeculae
 leaving medullary cavity in diaphysis.
 Most of diaphysis replaced by compact bones.
 Development of secondary ossification centers occurs in epiphysis of
 bone.
 Finally formation of articular cartilage and epiphyseal plate which
 consist of hyaline cartilage occur.
2.3.

Epithelium of GI tract

 The epithelial lining, covering the oral cavity divided into 2 types.
 Non keratinized stratified squamous epithelium covers the inner lining
 of lips, cheeks, soft palate, floor of mouth and undersurface of the
 tounge.
 Rest of the tongue, gingivae and hard palate is covered by keratinized
 stratified squamous epithelium.
 Oesophagus is lined by non keratinized stratified squamous epithelium.
 In stomach-oesophageal junction the abrupt change from non
 keratinized stratified squamous epithelium of oesophagus to simple
 columnar epithelium of stomach.
 Duodenum, jejunum, ileum of small intestine is lined by simple
 columnar eoithelium covering the villi.
 Glands are lined by columnar cells, paneth cells and enteroendocrine
 cells and the crypt of liberkuhn can be seen.
 Large intestine consist of ceacum, appendix, ascending, transverse,
 descending, sigmoid colons, rectum and anal canal.
 Except middle and lower parts of anal canal , the other parts are lined
 by simple columnar epithelium.
 Villi and plicae circularis are absent in mucosa and consist of straight
 interstinal glands (crypts),goblet cells and paneth cells are only present
 in ceacum and appendix.
 Upper part of anal canal consist of simple columnar epithelium, middle
 part of tha anal canal consist of stratified squamous non keratinized
 epithelium and lower part covered by true skin.
3.

3.1

Each presynaptic terminal ( axon terminals of motor neurons) is separated from

the post synaptic structures ( muscle fibers) by synaptic cleft.

Presented by 15th Batch 225 FHCS | EUSL


Repeat campaign 2K18 presented by 10th wms 9

membrane enclosed

vesicles which contain Ach.

sarcolemma.

surface area for

additional Ach receptors.

3.2

Excitation- contraction coupling

contraction.

1. An action potential travels through the axon terminal and

eventually reaches the synaptic terminal.

2. The depolarization of the synaptic terminal from the action

potential includes opening of Ca2+ voltage gated channels.

3. Opening of these channels allow flux of Ca2+ ions inside the neuron.

4. Increase the intracellular Ca2+ concentration introduces

conformational changes of the microtubular component of the

neuronal synaptic terminal cytoskeleton.

5. These changes lead the exocytotic process concerning the synaptic

vesicles containing Ach.

6. Ach diffuses From the synaptic terminal membrane towards the

motor end plate membrane.

7. Ach bind with ligand gated ion channel coupled receptors specific

for Ach on the sarcolemma.

8. Allow the opening of the Na+

channels ( flux of Na+

into muscle

fibers occurs)

9. Generate action potential in the muscle fibers.

10. Spreads throughout the membrane of the fiber and within the T
Presented by 15th Batch 226 FHCS | EUSL
tubules of the muscle fibers deep inside the fiber.

11. Activate Ca2+ voltage gated channels located on the plasma

membrane and T tubules.

12. Increases Ca2+ influx inside cell. Elevate intracellular Ca2+

concentration. This cause for opening of Ca2+ voltage gated

channels of the sarcoplasmic reticuli near the T tubules allowing

even greater increase of intracellular Ca2+.

Contraction sequence of skeletal Muscles.

1. Free Ca2+ binds with the troponine C protein component of the

thin actine filaments introducing the active calcium- troponin

complex.

2. This binding causes the conformational changes of the troponin C.

3. This includes the alteration of conformation of tropomysin protein

component of thin actin filaments.

4. These changes promote the exposure of the acting binding sites in

order to provide anchoring of th the myosin filament head in order

to induce ineteraction between the thick and thin filaments and

elicit contractions.

5. Myosin heads bind to the uncovered binding sites on the thin

filaments. ADP and Pi released. Power stroke occurs.

6. ATP binds myosin, allowing it to release actin and be in the weak

binding state.

7. The myosin then hydrolyses the ATP and uses the energy to move

into the cocked back conformation.

11TH BATCH PROPER MCQ

Regarding growth plate of long bone


A) Replace by bone tissue after puberty
B) Growth is influenced by testosterone in male
C) Elastic cartilaginous plate

Presented by 15th Batch 227 FHCS | EUSL


D) Promote growth in thickness
E) Chondrocytes undergo rapid mitosis in proliferation zone

16. Nerve
A) Has blood nerve barrier
B) Covered by connective tissue layer
C) Has regeneration capability in CNS
D) Collection of axons of neurons
E) Supported by satellite cells

17. Areolar connective tissue


A) Allow the movements of the body
B) Defend the body against the infections
C) Store fat
D) holds the body fluid
E) supports other tissues

18. skeletal muscle fiber


A) can be tetanised by repeated stimulation
B)contraction trigger by extracellular Ca+2
C) has a refractory period longer than the contraction phase
D) multinucleated
E) utilizes creatinine phosphate

19.True regarding smooth muscles


A) Smooth muscle in blood vessels controlled only by sympathetic system
B) smooth muscles in gut show spontaneous depolarization
C) Neuromuscular junction is always same as in skeletal muscles
D) has a more negative potential than the skeletal muscle
E) No actin and myosin

21. Phosphagen energy system of the body,


A) Creatine amount represent the muscle bulk
B) Creatinine is made up from glutathione
C) Need o2
D) Most reedy source of energy in muscle
E) Give energy for 30 min of high strength work

48. Elastic cartilage are absence in


A) pinna
B) Corniculate cartilage
C) Medial part of the auditory tube
D) Arytenoid cartilage
E) Epiglottis

49. What is best described electron mechanical coupling?


A) action potential last long than contraction
B) contraction last shorter than action potential
C) action potential precede to contraction
D)action potential and contraction simultaneous

Presented by 15th Batch 228 FHCS | EUSL


E) repetitive action potential is summated

11TH BATCH PROPER SEQ

2.2 Briefly describe the factors affecting bone remodeling(25 marks)


3.3.1 Explain the term ‘tension lines and state their importance (15 marks)
3.2 Compare and contrast the structure and functions of involuntary types of muscles.(30 marks)
3.3 Describe the growth in cartilage with illustrations(30 marks)

Answer

15.

A. True

B .True

C .False -Hyaline cartilage

D. False -Increase in length

E -True

16 )

A .True -Physiological space with in the axons,

Schwann cells & other associated cells

B .True -Dense connective tissue layer epineurium

C. False -Regeneration in PNS, bcz of Schwan cell

D .True

E .True

17)

A.

B. True - Phagocyte the bacteria

C. False - Fat store in adipose loose connective tissue

D. True- Hold & conveys the tissue fluid

E.

18)

A.

B.

C.

D. True

Presented by 15th Batch 229 FHCS | EUSL


E. True

19)

A. True
B.
C.
D. False - RMP -20 to -65mV

E. False - Has actin & myosin, but differs from skeletal

Muscle

21)

A. True
B. False
C. False - No O2 require
D. True
E. False - Give energy only for 8 to 10 seconds
48)

49)

A. false- Mechanical response last longer than action potential

B.

C. True - Mechanical event start 2ms after depolarization

D. False - Both did not occur at same time

E. False- Mechanical events are summated.

11TH BATCH PROPER SEQ

2.2 Briefly describe the factors affecting bone remodeling(25 marks)

3.1 Explain the term ‘tension lines and state their importance (15 marks)

3.2 Compare and contrast the structure and functions of involuntary types of muscles.(30 marks)

3.3 Describe the growth in cartilage with illustrations(30 marks)

Answers

2.2

Factors affecting bone remodelling are nutrition, hormones, exercise and lifestyle.

• We need minerals like calcium, phosphate salts.

• Also need many vitamins for regulation of the process. Vit C to synthesize collagen,

Vit D ; calcitriol helps in absorption of calcium and phosphate ions and Vit A

Presented by 15th Batch 230 FHCS | EUSL


stimulate activity of osteoblasts.

• Hormones also play a great role; Growth hormone stimulates protein synthesis and

cell growth, sex hormones responsible for increased osteoblastic activity and

synthesis of bone ECM. Thyroid and Insulin also increase bone formation and

glucocorticoids inhibit osteoclastic activity. Mainly Parathyroid and calcitonin act

antagonistically in bone resorption and deposition.

• According to exercise, bone adjusts its strength in proportion to the degree of bone

stress. Rearrange shape to proper support of mechanical forces.

3.1

Tension lines are due to arrangement of collagen fibres in the dermis.

• Incision made along the skin tension lines heal with a minimum scarring. They result

in less gapping, faster healing and less scar tissue.

Incision made across them can gap and increase the time needed for healing and Tension lines are due to
arrangement of collagen fibres in the dermis.

• Incision made along the skin tension lines heal with a minimum scarring. They result

in less gapping, faster healing and less scar tissue.

Incision made across them can gap and increase the time needed for healing and resulting increased scar tissue
formation.

3.2

3.3

Presented by 15th Batch 231 FHCS | EUSL


12TH BATCH PROPER MCQ

16. Stratified squamous epithelium,


a. Present in smooth wall of vagina
b. Present in mucosa of esophagus
c. Present in absorptive surfaces
d. Basal layer is highly proliferative
e. Contain different shapes of cells

17. Regarding nails


a. They are dead epidermal cells
b. Nails of fingers grow faster than toes
c. They are made up with protein called collagen
Presented by 15th Batch 232 FHCS | EUSL
d. Size and the length of nails are determined by thickness of matrix
e. The nail bed is the skin beneath the nail plate

18. Regarding adipose tissue


a. Brown adipose tissue present more than white adipose tissue
b. Main role or the white adipose tissue is store energy in the form of lipids
c. Produce hormone called leptin
d. White adipose tissue has protein called thermogenin
e. They produce sex hormones

41. elastic cartilages are absence in


a. Pinna
b. Corniculate cartilage
c. Medial part of the auditory tube
d. Arytenoid cartilage
e. Epiglottis

45. Which of the following cell regulate the exchange of nutrient metabolite from the neuron cell bodies in dorsal
root ganglia
a. Astrocyte
b. Ependymal cells
c. Microglia
d. Satellite cells
e. Schwann cells

48. Heart act as syncytium due to presence of?


a. Sarcomere
b. Intercalated disc
c. Z line
d. Branches
e. Neuromuscular junction

51. Prolonged refractory period has


a. Muscle spindle
b. Gamma fiber
c. Alpha fiber
d. Cardiac muscle
e. C fibers

52. The most characteristic feature of skeletal muscle


a. Excitability
b. Conductivity
c. Rhythmicity
d. Elasticity
e. Voluntary

57. an action potential in a nerve fiber,


a. occurs when its membrane potential is hyperpolarized to a critical level
b. is associated with a transient increase in membrane permeability to sodium
c. is associated with a transient decrease in membrane permeability to potassium

Presented by 15th Batch 233 FHCS | EUSL


d. induces local electrical currents in adjacent segments of the fiber
e. has an amplitude which varies directly with the strength of stimulus

58. regarding cell junctions,


a. desmosomes use occludins as trans membrane protein
b. in stratified squamous epithelium adhering junctions are very rare
c. in tissues subjected to shear and stress, hemidesmosomes can be seen
d. vertebrates have septate junctions
e. gap junctions contain anchoring proteins but not contain transmembrane channels

12TH BATCH PROPER SEQ

1.3. Describe the features and growth of femur (30 Marks).


1.4. Compare and contrast the skeletal muscle and smooth muscle. (20 Marks)

3.2.
3.2.L. Define the term "motor unit". (10 marks)
3.2.2. When a single stimulus is applied to a nerve bundle it records several action potentials
(compound action potential) at different intervals. Explain the physiological basis of the
compound action potential in response to a single stimulus. (40 marks)

Answers

16.
A. T
the wall of vagina consists of
- a mucosal layer lined by stratified squamous epithelium
- a layer of smooth muscle
- an outer adventitial layer.

B.T upper half of the esophagus

C.F present in sites what need mechanical abrasion, stress and tissue protect

D.T Basal cells are active in mitosis and produce the cells of the more superficial layers

E.F Basal cells are cuboidal or columnar. Surface cells are flattened

17.
A.T Nails are composed of hard, keratinized epidermal cells located over the dorsal
surfaces of the ends of fingers and toes

B.T

Presented by 15th Batch 234 FHCS | EUSL


C.T

D.T width, thickness pf nail plate is determined by the size, length and thickness of the
matrix

E.T
Each nail consists of:
• free edge
• transparent nail body (plate) with a whitish lunula at its base
• nail root embedded in a fold of skin
• nail matrix

18.
A.F white is more abundant

B.T

C.T

D.F thermogenin is an uncoupling protein found in the mitochondria of brown adipose


tissue

E.T Estradiol ,Testestorone


41. D
Pinna – Elastic
Corniculate – Elastic
Auditory tube – Elastic
Arytenoid – Hyaline
Epiglottis – Elastic

45. D
Astrocytes – Support CNS & BBB
Ependymal – Like epithelia
Microglia – Immunity
Satellite – Nutrient exchange & Protection
Schwann – Myelination

48. B
Cardiac muscle behaves as a functional syncytium due to presence of intercalated discs,
although it is composed of individual cells.

51. D
Prolong refractory period prevent the tetanus formation of cardiac muscle

52. E
Presented by 15th Batch 235 FHCS | EUSL
Skeletal muscle contract voluntary ...but smooth and cardiac muscles are involuntary
A,B,D are not only present in the skeletal muscle
C is a feature of cardiac muscle

57. B

58. C
A. desmosome contain Cadherine protein
B. abundant
C. hemidesmosome present in skin where tend to friction
D. in invertebrate
E. transmembrane channels present

12TH BATCH PROPER SEQ

1.3 Describe the features and growth of femur (30 marks)

Femur is a long bone. It grow in thickness by appositional growth and lengthen by the
addition of bone material on the diaphysial side of the epiphyseal growth by Interstitial
growth.

Interstitial growth
Epiphyseal cartilage (close to the epiphysis) of the epiphyseal plate divides to create more
cartilage, while the diaphysial cartilage (close to the diaphysis) of the epiphyseal plate is
transformed into bone. This increases the length of the shaft.

Growth in length continues until puberty under the influence of growth hormone, thyroid
hormone, parathyroid hormone, and androgens.

The growth in length of long bones involves two major events:


1) growth of cartilage on the epiphyseal plate
2) replacement of cartilage by bone tissue in the epiphyseal plate

Epiphyseal plate cartilage organizes into important functional zones:


• Zone of resting
-consists of small clusters of flattened or rounded, randomly arranged, quiescent
chondrocytes.

• Zone of proliferation
-chondrocytes undergo rapid mitosis under influence of growth hormone.
-daughter cells are stacked into columns, resembling stacks of coins, and are parallel to the
long axis of the future bone.

• Zone of hypertrophy
Presented by 15th Batch 236 FHCS | EUSL
-mitosis ceases, and cells and their lacunae enlarge.
-Mature chondrocytes arranged in columns.

• Zone of calcification
-Dead chondrocytes are resorbed; lacunae erode.
-Richly vascularized primary marrow extends into the newly opened spaces, and osteoblasts
differentiate from mesenchymal cells in the marrow.

• Zone of ossification
-Osteoblasts secrete osteoid, which becomes mineralized.
-Forms new diaphysis.

Appositional growth
Increase in thickness and remodeling of all bones by osteoblasts and osteoclasts on bone
surfaces.

Osteoblasts beneath the periosteum secrete bone matrix on the external surface of the bone.
This obviously makes the bone thicker. At the same time, osteoclasts on the endosteum
break down bone and thus widen the medullary cavity. This results in an increase in shaft
diameter even though the actual amount of bone in the shaft is relatively unchanged.

1.4 Compare and contrast skeletal muscle and smooth muscle (20 marks)

Skeletal muscle Smooth muscle

Contractions are voluntary Contractions are involuntary

Cells with obvious striations, due to organization of Lack striations, myofibril meshwork anchored to
myofibrils dense bodies

multinucleate cells One central nucleus

Fibers Spindle shaped cells

Innervated by alpha motor neuron Autonomic innervation

Sliding filament mechanism of contraction Cells shorten and broaden on contraction

Presented by 15th Batch 237 FHCS | EUSL


3.2.1 Define the term motor unit (10 marks)

Single motor neuron and all the muscle fibers innervated by it is called motor unit. It is the
functional unit of muscle

3.2.2 When a single stimulus is applied to a nerve bundle it record several action potentials (
compound action potential) at different intervals. Explain the physiological basis of the
compound action potential in response to a single stimulus (40 marks)

Action potential recorded from a group of nerve fibers (e.g. sciatic nerves) or a nerve trunk
is called compound action potential as it is the summated action potentials of different types
of nerve fibers having different conduction velocities. The multi-peaked shape of the
compound action potential is due to the activity of the different nerve fibers of varying
conduction velocity. Most nerves are composed of myelinated nerve fibers of various
diameter and also unmyelinated fibers of quite large number.

*[With a large shock, the action potential appears as,

It is conventionally split into three waves called A, B, C. The A wave itself is divided into
alpha, beta, gamma and delta sections.

The A wave represents activity in myelinated axons and the C wave that in unmyelinated
axons. The B wave may represent both types. The reason for the spreading out of the waves
lies in the differing conduction velocities of the fibers. If the recording electrode is close to
the stimulating electrode, only a single wave can be detected. It is possible to activate
selectively the axons of different sizes. With small shocks, only the wave is apparent. As the
Presented by 15th Batch 238 FHCS | EUSL
shock strength is increased, so other components of the A wave appear, followed by the
Band C waves. The largest, fastest-conducting fibres thus have the lowest threshold to
stimulations. If a maximum shock (one which fires all the fibers) is used, and pressure is
carefully applied between the stimulating and recording electrodes, another phenomenon
becomes apparent. The pressure blocks, conduction in the largest fibers first. The alpha
wave is the first to go followed by beta, gamma, delta and B until only the C wave is left.]*

13TH BATCH PROPER MCQ

15. Regarding adipose tissue


A. Brown adipose tissue has single large TAG droplet
B. Brown adipose tissue involve in heat production
C. Number of mitochondria high in white adipose tissue
D. White adipose tissue maintain body mass
E. White adipose tissue present in yellow bone marrow

16. Compact bone


A. Accomadate the red bone marrow
B. Consist of osteon
C. Is located in the medulla of bone
D. Provide support and protection
E. Resist the stress in multiple directions

17. Myaline sheath


A. Derivative of lipd
B. Nerve coduction speed
C. Insulator for the axon
D. Made by shwan cells in CNS
E. Multiple axons are myalinated by oligodandrocytes

19. True or false regard wave summation for in skeletal muscle


A. it seems single supratreshold stimuli
B. it can be produced in repeated treshold stimuli
C. it is caused merge at all action potential
D. it is caused by merge of wave contraction
E. it is normal occur at contraction in skeletal muscle fiber

49. Which of the following cell junctions allow the passage of small molecules between adjacent cells to coordinate
and synchronize cardiac muscle contraction
A..Adherens junction
B. Desmosome
C. Gap junction
D..Hemidesmosome
E. .Tight junction

Presented by 15th Batch 239 FHCS | EUSL


50. What is the gland that secrete the sticky and waterproof secretion that impede the entry of foriegn body in to
ear
A. Apocrine sweat gland
B. Ceruminous gland
C. Eccrine sweat gland
D. Sebaceous gland
E. Goblet or unicellular gland

13TH BATCH PROPER SEQ

1.
1.1. Compare the microscopic features of the type of cartilage present in costal cartilage and
intervertebral disc. (25 marks)
1.2. Briefly describe the structural arrangement of a sarcomere in smooth muscle (35 marks)
1.3. Describe how the meiosis differs in male and female (25 marks)
1.4. State 3 genetic importance of meiosis (15 marks)

3.2. Denervation of the skeletal muscle leads to


3.2.1 Fasciculation / fibrillation of muscle fibers during early stages
3.2.2 Later followed by atrophy of muscle

Explain the physiological basis of the above changes after denervation. (2 * 25 marks)

Answers

15.
A. F

B. T
Contains more mitochondria

C. F

D. T
Fat storage

E. T

16.
A. F
Spongy bone

B. T

C. F

Presented by 15th Batch 240 FHCS | EUSL


D. T

E. F
It is the function of spongy bone

17.
A. T

B. T
increase

C. T

D. F
Oligodendrocytes in CNS
Shwann cells in PNS

E. T
oligodendrocyte can myelinate up to 60 axons

19.
A. T
Need repeated stimulation. But seems like a single stimuli

B. T

C. F
No merging of action potentials

D. T
Wave merging can be seen

E. T

49. C
Gap junction

50. B
Specialized sweat glads- found in external auditory canal (outer 1/3) Classed as apocrine
gland

13TH BATCH PROPER SEQ


Presented by 15th Batch 241 FHCS | EUSL
1.1. Compare the microscopic features of the type of cartilage present in costal cartilage and
intervertebral disc. (25 marks)

Costal cartilage (Hyaline) Intervertebral disc (Fibrous)

weakest type of cartilage and can be fractured. It is highly compressible and has great tensile
strength

Fine collagen fibers (Type II)embedded in a gel Contains alternating layers of hyaline cartilage
matrix matrix with thick layers of dense collagen fibres,
orientated in the direction of the functional stresses.

Chondrocytes lie in lacunae (cluster of 2-4 cells) Chondrocytes are arranged in rows between the
dense collagen layers within lacunae in the
glycoprotein matrix

Contains perichondrium Lacks perichondrium

1.2. Briefly describe the structural arrangement of a sarcomere in smooth muscle (35 marks)

• In smooth muscle actin and myosin-II are present, they slide on each other to produce
contraction

• But they are not arranged in regular arrays, so lacks striations

• Like Z lines in skeletal muscle, the contain dense bodies in cytoplasm, attached to cell
membrane.

• Actin filaments are attached to dense bodies by α-actinin

• Smooth muscle contains tropomyosin but lacks in troponin

• Less extensive sarcomere and few mitochondria present, compared to skeletal muscle and
cardiac muscle

1.3. Describe how the meiosis differs in male and female


Spermatogenesis oogenesis

Begins after puberty Begins before birth

Not get arrested in prophase-1 Get arrested in prophase 1

Deve 4 sperms from single meiosis Develop only one egg from single meiosis

Doesn’t produce polar bodies Produce 3 polar bodies

Presented by 15th Batch 242 FHCS | EUSL


Spermatogenesis oogenesis

Won’t get arrested in metaphase-II Arrested in metaphase 1 until the arrival of sperm

Highly mobile cells produced Nearly immobile cells produced

1.4. State 3 genetic importance of meiosis

• Maintains the chromosome numbers constant from generation to generation

• Assures a different genetic make up for the next generation as a result of crossing over and
new combination of genes

3.2. Denervation of the skeletal muscle leads to


3.2.1 Fasciculation / fibrillation of muscle fibers during early stages
3.2.2 Later followed by atrophy of muscle
Explain the physiological basis of the above changes after denervation. (2*25 marks)

3.2.1

• Usually signals from CNS smoothes the reflexes on skeletal muscle.

• When denervation occurs, this control will be lost

• When the muscle start to stretch, it will activate muscle spindles, start to contract intrafusal
fibres.

• Due to less contractile force of intrafusal fibres, fibrillation occurs

3.2.2

• When a muscle or any organ cease its action for a long period, it will enter atrophy state

• Denervated muscle cannot be contracted, which is the action of the muscle.

• So the nutritional supply (blood supply) will start to reduce for the muscle

• That is not enough to rebuild the muscle protein destructed during normal wear and tear

• Ultimately the muscle will start to shrink, which is called atrophy

Presented by 15th Batch 243 FHCS | EUSL


14TH BATCH PROPER MCQ

04. Intramembranous ossification,


A) Fuses the fontanelles of the skull
B) Forms medullary cavity
C) Leads to the development of bones without cartilaginous skeleton
D) Requires hyaline cartilage
E) Produce spongy bone first

05. Regarding elastic cartilage,


A) Single chondrocyte in lacuna
B) Lacks perichondrium
C) Maintain the shape of organs
D) Epiglottis
E) Withstands repeating bending

16. Regarding epithelia,


A) All types of epithelia lie on a basement membrane
B) Has a rich capillary bed surrounding the cell.
C) Simple epithelia are found on surfaces involved in filtration
D) Stereocilia are present in stratified epithelium
E) Cytoskeleton of adjacent cells joined by adhesion junctions.

17. Regarding skeletal muscle fibre,


A) It is multinucleated.
B) Each fibre is innervated by a single motor nerve.
C) It is a functional syncytium.
D) Its normal contraction is a twitch.
E) Myoglobin is the contractile protein.

18. Cardiac muscle characteristics,


A) Involuntary
B) Contraction initiated by autonomic nervous system.
C) Spontaneous action potential generate.
D) Shorter refractory period.
E) Longer contraction phase

20. Creatinine,
A) Level points out urinary function
B) Converted to creatine phosphate by creatine kinase
C) High molecular compound
D) Found in plant food
E) Depends on muscle mass in the body

32. Conduction of nerve fiber


A) C fibers have slow Conduction
B) When impulse enter in to the nerve middle spontaneously it goes right away
Presented by 15th Batch 244 FHCS | EUSL
C) A and B fibers are myelinated
D) Demyelination diseases reduce conduction velocity
E) Peripheral nerve fibers can regenerate

37. Regarding collagen,


A) Collagen fibers are rich in alanine and leucine
B) It is a glycoprotein
C) The basic unit is tropocollagen
D) It maintains structural integrity of tissues
E) Mafan’s syndrome occurs due to defect in collagen fibers

SBR
43. Which type of connective tissue present in aorta for recoiling adaptation?
A) Dense elastic CT
B) Dense irregular
C) Dense regular D) Loose areolar
E) Loose reticular

44. Which gland secrete the oil like substance that prevent dehydration of the hair and skin?
A) Apocrine sweat gland
B) Ceruminous gland
C) Eccrine sweat gland
D) Goblet cell or uni cellular gland
E) Sebaceous gland

60. False statement of regarding bone remodeling


A) Adjust bone architecture to meet changing mechanical needs
B) Increase bone growth
C) Maintenance the plasma Ca to hemostasis
D) Is most active in early stage of life
E) Involves osteoblasts in bone resorption

14TH BATCH PROPER SEQ

1.
1.2. State the components of the Haversian system. (20 Marks)
1.3. Compare and contrast the structure of skeletal and cardiac muscle. (30 Marks)
1.4. Explain the adaptations of alveolar epithelium for its functions (20 Marks)
2.
2.5. Strenuous exercise eventually builds up a significant oxygen debt that forces an athlete to terminate the
exercise session.
2.5.1. What is oxygen debt? (10 marks)
2.5.2. State the importance of repaying oxygen after a strenuous exercise. (15 marks)
5.
5.2. High frequency stimulation of skeletal muscle produces tetany (summation / sustained contraction). But not
in cardiac muscle.Explain the physiological basis of
5.2.1. tetany in skeletal muscle on high frequency stimulation. (25 marks)
5.2.2. inability to produce tetany in cardiac muscle (25 marks)

Presented by 15th Batch 245 FHCS | EUSL


Answers

04.
A. T

B. F
Endochondral ossification

C. T

D. F
Endochondral ossification

E. T

05.
A. T
Single chondrocyte in lacuna

B. F
Has a perichondrium [ Lacks perichondrium→fibro cartilage ]

C. T

D. T
Found in the external ear, auditory tubes and the epiglottis

E. T
Elastic fibers enable it to withstand repeated bending

16.
A. T
Cells of epithelial tissue are attached to a basement membrane. Supported by connective
tissue.

B. F
Epithelial tissues are avascular but innervated.

C. T
Simple epithelium has single cell layer Allows passage of materials by diffusion and
filtration

D. F
Stereocilia are the projections in the pseudostratified columnar epithelium cells. They are
cylindrical non-motile. [ Epididymis and vas deferens ]
Presented by 15th Batch 246 FHCS | EUSL
E. T
Adhesion junctions contain plaque on the inside of the plasma membrane Attaches both to
membrane proteins and to microfilaments of the cytoskeleton

17.
A. T
Skeletal – multinucleated
Smooth and cardiac – mono nucleated

B. F
Axon of a motor neuron branches many times.
Each branch extending to a different skeletal muscle fiber.
[ MOTOR UNIT = somatic motor neuron and all the skeletal muscle fibers it innervates ]

C. F
Functional syncytium→cardiac muscles [ gap junctions located in intercalated disc ]

D. T

E. F
Contractile proteins → actin, myosin

18.
A. T
Involuntary – cardiac and smooth
voluntary - skeletal

B. F
Cardiac muscle contraction is initiated by SA node [ pacemaker cells ] Autonomic nervous
system can control the cardiac contraction

C. T
Automaticity/Autorhythmicity [ pacemaker cells undergo spontaneous depolarization ]

D. F
Longer refractory period→250ms

E.

20.
A. T
Creatinine is a breakdown product of creatine phosphate from muscle and protein
metabolism

Presented by 15th Batch 247 FHCS | EUSL


B. F
Creatine kinase
Creatine phosphate ——————-> Creatinine

C. F
Both creatinine and creatine are small molecules D

D. F
Normally creatine found in animal tissues

E. T
Creatine exists as creatine phosphate in skeletal muscles.

32.
A. T
Conduction velocity→A>B>C

B. T

C. T
Groups A and B are myelinated, and group C are unmyelinated

D. T
Myelination increase the conduction velocity of nerve fiber. [ saltatory conduction ]

E. T
Peripheral nerve fibers can regenerate. Central tracts can’t regenerate

37.
A. F
Rich in glycine, proline and hydroxyproline

B. T
Contains glucose and galactose

C. T

D. T

E. F
Marfan syndrome due to defect in elastin fibers.

43. A
Allows recoil of tissue following stretching

Presented by 15th Batch 248 FHCS | EUSL


44. E

60. E
Osteoclasts involve in bone resorption

14TH BATCH PROPER SEQ

1.2 State the components of the harvesian system. (20 marks)

Compact bones are also known as cortical bones which makes the dense outer layer. It is
important to the resistance of the stress and supportve protection functions.

The major components of the compact bones are harvesian systems. They are also known as
osteons.

Osteons are made up of,


- Central canal
- Concentric lamellae
- Interstitial lamellae
- Circumferential lamellae
- Canaliculi
- Volkmann’s canal
Central canals are longitudinally located canals containing blood vessels and nerves.
Around the central canals rings of calcified matrix with lacunae with osteocytes and
canaliculi are seen. This structure is known as concentric lamellae.

Canaliculi filled with extracellular fluid forming a system of interconnected canals. It is


important for nutrient and oxygen supply for the maintenance of osteocytes. Intersttal
lamellae are present connecting neighboring osteons.

Circumferential lamellae near to periosteum is known as outer circumferential lamellae.


Circumferential lamellae near to medullary cavity known as inner circumferential lamellae.

Connecting each central canals, volkman’s canals are present horizontal manner.

1.3. Compare and contrast the structure of skeletal and cardiac muscles.

Cardiac muscle Skeletal muscle

Mono nucleated. Centrally located nucleus. Multi nucleated. Peripherally located nucleus.

Presented by 15th Batch 249 FHCS | EUSL


Cardiac muscle Skeletal muscle

Branched Unbranched

There are intercalated discs. No intercalated discs.

Many gap junctions between fibers No gap junctions .

Fibers have large diameter. Fibers have very large diameter.

Only endomysium and perimysium present as Endomysium, perimysium and epimysium are
connective tissue components. present.

Sarcoplasmic reticulum are fewer. Sarcoplasmic reticulum are abundant.

Similarities

Both muscles are striated.

Both muscles have troponin and tropomyosin as their regulatory proteins.

Fibers arranged as sarcomere.

1.4) explains the adaptations of alveolar epithelium for its functions.

Alveolar epithelium is composed of simple squamous epithelium. Its mean there is single
layer of alveolar cells present in alveolar epithelium. Main function of the alveolar
epithelium is gas exchanging. So single layer of alveolar cells reduces thickness of the gas
exchanging epithelium. This leads to facilitate gas exchange across the alveolar epithelium
efficiently. There are 2 types of alveolar cells present in alveolar epithelium. Type 1
alveolar cells participate in gas exchanging. Type 2 alveolar cells produce surfactant which
reduces the surface tension. So surfactant prevents lungs from collapsing. Also alveolar
epithelium is surrounded by network of blood capillaries. This network of blood capillaries
bring sufficient amount of blood for gas exchanging. We all know simple squamous
epithelium has flattened cells like fried egg. So surface area of the simple squamous
epithelium is high. As I said above alveolar epithelium consists of simple squamous
epithelium. Large surface area facilitates gas exchange more efficiently.

2.5)strenuous exercise eventually builds up a significant oxygen debt that forces a athlete to
terminate the exercise session.

2.5.1) what is oxygen debt?

Presented by 15th Batch 250 FHCS | EUSL


Extra amount of oxygen that must be taken during strenuous exercises. After anaerobic
respiration the body is in recovery and must get rid of lactic acid in that case we must get
extra oxygen for pay oxygen debt. That’s why athletes breathe heavily after their events.

2.5.2) Importance of repaying oxygen debt after a strenuous exercise.

Oxygen is needed for conversion of lactic acid in to H2O + CO2 80% of lactic acid enter in
to the blood stream and converted in to pyuruvic acid in kidneys ,liver and cardiac muscles.
Also oxygen is needed for the replenishing of the phosphagen system. Repaying of oxygen
debt also helps to remove the CO2 which accumulate in the blood during exercises

5.2. High frequency stmulaton of skeletal muscle produces tetany (summaton / sustained
contracton). But not in cardiac muscle.

Explain the physiological basis of

5.2.1. Tetany in skeletal muscle on high frequency stmulaton. (25 marks)

• Skeletal muscle has a refractory period “ Period of time during which a cell is incapable
of repeating an action potential ”

• There are 2
Absolute RP—> completely impossible for another acton potential to be activated
Relative RP —> high frequency stimulus can stimulate a second response.

• According to the electrical stimulation, there is a mechanical response in skeletal muscle


cells.

• In skeletal muscles,
Duration of acton potential and refractory period [ absolute ] is shorter than the duration of
contraction phase.

• When a certain frequency of acton potential is reached, the muscle contract before
relaxation
• So the muscle is in sustained contraction.
• This occurs when a muscle's motor unit is simulated by multiple impulses at a sufficiently
high frequency.

• Each stimulus causes a twitch.


Presented by 15th Batch 251 FHCS | EUSL
• If stimuli are delivered slowly enough, the tension in the muscle will relax between
successive twitches.
• If stimuli are delivered at high frequency, the twitches will overlap, resulting in tetanic
contraction

5.2.2. Inability to produce tetany in cardiac muscle (25 marks)

• Cardiac muscle has a refractory period


• “ Period of time during which a cell is incapable of repeating an acton potential ”

• There are 2
Absolute RP —> completely impossible for another acton potental to ee activated
Relative RP —> high frequency stimulus can simulate a second response.

• According to the electrical stimulation, there is a mechanical response in cardiac muscle


cells.
• Cardiac muscles, Have prolonged acton potential and refractory period due to plateau
phase.

• Therefore the muscle is always relaxing state when absolute refractory period is over.
• So cardiac muscles cannot be sustainly contracted or tetanioed.
Presented by 15th Batch 252 FHCS | EUSL
• This is a safety measure for cardiac muscles.

15TH BATCH PROPER MCQ

11) elastin
A. Is hydrophilic protein
B. Is glycosylated
C. Is protected by alpha 1 anti-trypsin in lung
D. Is the dominant extra cellular matrix protein in arteries
E. Mutation in gene results in narrowing of aorta

15) Regarding creatine phosphate,


A. Action of CPK on creatine phosphate produce molecule within use to estimate muscle mass.
B. Creatine is formed from Glycine, arginine
C. It produces mainly in liver
D. Provide for phosphatase synthesis
E. Use for ATP produce

18) skeletal muscle fiber


A. can be tetanized by repeated stimulation
B. voluntary
C. has a refractory period longer than the contraction phase
Presented by 15th Batch 253 FHCS | EUSL
D. multinucleated
E. utilizes creatinine phosphate

19) In typical neuron


A. well-developed cytoskeleton
B. poorly organized vesicles
C. few lysosomal elements
D. prominent rough endoplasmic reticulum
E. centrally located nucleus

20) Regarding cartilage,


A. Always arrange in a single layer of cells
B. Osteoarthritis is disease of articular cartilage
C. Collagen provide tensile strength
D. Matrix of cartilage includes Ca2+ salts
E. Receives blood supply

32) The following characteristics of axon most on its diameter


A. Activity of Na+/K+ pump
B. Conduction velocity of its action potential
C. duration of its refractory period
D. the magnitude of Action potential
E. magnitude of RMP

33) Regarding action potential of cardiac muscle


A. Not essential for contraction
B. It is longer than action potential of skeletal muscle
C. It has prolonged plateau phase
D. It is causes immediate uptake of ca2 into lateral surface of sarcoplasmic reticulum
E. Resting membrane potential is +20mv

47) Osteoclasts are


A. cuboidal in shape
B. derived from osteoprogenitor cells
C. have large Golgi apparatus
D. trapped within the formed bone
E. responsible for bone remodeling

60) For which of the following sport would training program for increasing maximal muscle gluconeogenetic
enzyme activity be less important
A. 200m, track
B. 800m, track
C. Basketball
D. High jump
E. Swimming

Presented by 15th Batch 254 FHCS | EUSL


15TH BATCH PROPER SEQ

1. Write short accounts on


1.1 Microscopic features of compact bone with the aid of a diagram (50 marks)
1.2 Supporting cells of nervous system (50 marks)
5.
5.3. A patient with muscle weakness is evaluated and his arm muscles become progressively weaker during
repeated lifting of a weight. Clinical nerve conduction tests indicate direct stimulation of the muscle produces
normal muscle action potentials and contractions and direct application of acetylcholine to the motor end plate
leads to normal muscle responses.
5.3.1. Based on these test results, what could be the reason for the muscle weakness in this patient? Justify
your answer (30 marks)
5.3.2. How could the cholinesterase inhibitors improvethis above condition? (20 marks)

Answers

11. elastin
B. F
Highly hydrophobic protein

B. F
Not glycosylated

C. T
Alpha 1 anti trypsin protects lungs from neutrophil elastase

D. T

E. T
Mutations in the elastin gene causing a deficiency of the protein resulting in narrowing of
aorta or other arteries as a result of the excessive proliferation of smooth muscle cells in the
arterial wall.

15) Regarding creatine phosphate,


A. F

B. T

C. T

D. F

E. T

Presented by 15th Batch 255 FHCS | EUSL


18) skeletal muscle fiber
A. T

B. T

C. F
contraction phase longer than refractory period so tetanus can occur in skeletal muscles

D. T

E. T
creatine phosphate is use to convert ADP to ATP in the muscles for muscle contraction

19) In typical neuron


C. T
cytoskeleton provide mechanical strength and stable cytoskeletal frame work in neurons

B. F
typical presynaptic terminal contain about 200synaptic vesicles

C. F
they are most abundant in neuronal cell bodies moderately present in dendrites and rare in
axon

D. T
Nissle substance is RER in neuronal cell body.it is primary site of protein synthesis

E. T
nucleus of the neuron is large, round and usually located in centrally

Presented by 15th Batch 256 FHCS | EUSL


20) Regarding cartilage,
A. F

B. T
It is associated with decreased glycosaminoglycan content of the matrix accompanied by
increased water content – Matrix degradations

C. T

D. F
Extracellular matrix contains
• Fibers – collagen, elastin fibers
• Ground substances – chondroitin sulphate
• Water - 70%-75

E. T

32) The following characteristics of axon most on its diameter


A. F

B. T diameter of axon affects to the conduction velocity of action potential

C. F

D. F

E. F

33) Regarding action potential of cardiac muscle


A. F

B. T because cardiac muscle has long plateau phase

C. T

D. F action potential of cardiac muscle causes Ca2+ release to the cell from the
sarcoplasmic reticulum

E. F -90mV

47) Answer E

Presented by 15th Batch 257 FHCS | EUSL


60) Answer D

15TH BATCH PROPER SEQ

1. Write short accounts on


1.1 Microscopic features of compact bone with the aid of a diagram (50 marks)

Compact bone is made up of parallel bony columns. Each column made up of a unit called
osteon or Haversian systems.

Osteons consist of a central (Haversian) canal and 4 – 20 concentrically arranged lamellae.

The osteocytes are thus arranged in concentric rings within the lamellae.
Haversian canal contain blood vessels, lymphatic vessels, nerves.

Surrounding central canal are concentric rings of osteocytes along with the calcified matrix.
It is called concentric lamellae. Diameters of rings increases from the central canal.
Between the lamellae are small spaces called lacunae which contain osteocytes.

Canaliculi are tiny canal like extension radiates in all directions from the lacunae. They are
filled with extracellular fluid and the plasma membrane extensions of the osteocyte. They
connect lacunae, forming a system of interconnected canals. And provides routes for
nutrients and oxygen to reach the osteocytes.

Interstitial lamellae is mineralized circular plates. They lie between the neighboring osteons.
They are fragments of older osteons. They have lacunae with osteocytes and canaliculi.

Circumferential lamellae is arranged around the entire outer and inner circumference of the
shaft of a long bone. They develop during initial bone formation
- lie deep to the periosteum - outer circumferential lamellae
- line the medullary cavity - inner circumferential lamellae

Presented by 15th Batch 258 FHCS | EUSL


Perforating canals or Volkmann’s canals accommodates vessels and nerves. They connect
with the medullary cavity, periosteum, and central canals.

1.2 Supporting cells of nervous system (50 marks)

Presented by 15th Batch 259 FHCS | EUSL


Nervous tissue also includes cells that do not transmit impulses, but instead support the
activities of the neurons. These are the glial cells (neuroglial cells), together termed the
neuroglia. Supporting, or glia, cells bind neurons together and insulate the neurons.there are
6 types of supporting cells
• CNS neuroglia:
1. astrocytes
2. oligodendrocytes
3. microglia
4. ependymal cells
• PNS neuroglia:
1. Schwann cells (neurolemmocytes)
2. satellite cells

• Astrocytes- Most numerous


• Star shaped with branched cell processes
• Perivascular end-feet form an intimate relationship with surfaces of small blood vessels,
with a Complete covering around capillaries.
#Functions:
• Provide metabolic and physical support for neurons
• Provide the appropriate chemical environment for the generation of nerve impulses
• Blood brain barrier

• Oligodendrogliocytes
• Fewer branches than astrocytes
• More numerous in white matter than in grey matter of the CNS
• Form myelin sheath in CNS
• one oligodendrocyte can myelinate up to 60 axons

• Microglia –
• Thorny bushes in appearance and the smallest glia
• Phagocytic function in CNS
• Constitute the brain’s immune system
• Originate from monocytes

• Ependymal cells
• Simple cuboidal to columnar epithelium line ventricles and central cavities of brain And
spinal
• prevents passage of potentially neurotoxic Substances to the brain
• bears apical microvilli to increase surface Area
• has motile cilia that project into the Ventricular lumen
a. sweep foreign particles.
b. metabolite exchange between CSF and Extracellular spaces of the brain and spinal cord

• Choroid plexus
Presented by 15th Batch 260 FHCS | EUSL
i. Lined by modified ependymal cells
ii. Secrete components of the CSF

• Schwann cells- They form myelin sheath around peripheral axon


o ensheath groups of several small axons in non-myelinating cells
o Look like jelly roll with neurolemma cove
o Node of Ranvier separates each Schwann cell
• Multiple layers of glial plasma membrane surround the axon, with the Schwann cell’s
• Cytoplasm and nucleus forming the outermost layer – neurolemma (sheath of Schwann)
• A neurolemma is found only around axons in the PNS.

5.3. A patient with muscle weakness is evaluated and his arm muscles become progressively
weaker during repeated lifting of a weight. Clinical nerve conduction tests indicate direct
stimulation of the muscle produces normal muscle action potentials and contractions and
direct application of acetylcholine to the motor end plate leads to normal muscle responses.

5.3.1. Based on these test results, what could be the reason for the muscle weakness in this
patient? Justify your answer (30 marks)

Direct stimulation of the muscle producing a normal action potential indicates that there is
no problem with the muscle. The problem lies at the neuromuscular junction. Based on the
symptoms and tests it can be suggested that the patient has myasthenia gravis. It is an auto-
immune condition in which the body produces antibodies against nicotinic acetylcholine
receptors on the motor end plate of the muscle. A reduction in the number of receptors is
seen. When acetylcholine binds to these receptors sites, it causes for the influx of Na+ ions
and an action potential is generated locally. With fewer receptor sites available, the muscles
receive fewer nerve signals, resulting in weakness. Further, Repetitive stimulation of nerves
leads to a reduction in quanta of acetylcholine therefore progressive weakness is seen due to
the number acetyl choline molecules getting reduced. At this state there are fewer receptors
as well as fewer acetyl choline molecules, thus further weakness is observed.

5.3.2. How could the cholinesterase inhibitors improve this above condition? (20 marks)

Acetylcholinesterase is an enzyme that is found in high concentrations at the NMJ. It


removes acetylcholine at the neuromuscular end plate. Acetylcholinesterase inhibitors bind
to acetylcholinesterase and prevent the breakdown of acetyl. It is a type of competitive
inhibition. This does not cause for the concentration of acetylcholine to drop after repeated

Presented by 15th Batch 261 FHCS | EUSL


stimulation. Since a higher number/quanta of acetylcholine is present it can act on the
available receptors at the NMJ and reduce the progressive muscle weakness that is see

Presented by 15th Batch 262 FHCS | EUSL


HB 04

Presented by 15th Batch 263 FHCS | EUSL


GENARAL HUMAN EMBRYOLOGY
EME QUESTIONS
11TH BATCH (EME) – MCQ

01) Zona pellucida,


a) Is a glycoprotein coat.
b) Is formed by primordial follicle.
c) Prevents polyspermy.
d)
e)

02) .
a) .
b) Sertoli cells assist in the release of the mature spermatozoa.
c) Abnormal sperm lack of normal motility.
d) It is influenced only LH.
e) The duration of the process is longer than oogenesis.

03) Regarding formation of neural tube


a) Ectoderm overlying notochord forms the neural plate.
b) Neural tube defect called spina bifida occurs in lumbar sacral region.
c) Entire neural tube formation occurs at the same time.
d) After neural tube formation, neural crest cells produce the spinal posterior ganglion.
e) Formation of neural tube is induced by ectoderm.

04) Invagination process leads to,


a) Formation of germ layers.
b) c) d) e)

05) Which embryonic tissue is correctly matched to ultimate product?


a) Epiblast – Amnion
b) Hypoblast – Endoderm
c) Endoderm – Intestinal villi
d) Mesoderm – Nail
e) Ectoderm – Nervous system

06) Regarding prenatal diagnostic test,


a) Fetal age can be calculated by doing ultrasonography.
b) c) d) e)

07) Multifactorial inheritance,


a) X – fragile syndrome
b) Hypercholesteremia
c) Huntington’s syndrome
d) Marten’s syndrome
e) Achondroplasia
Presented by 15th Batch 264 FHCS | EUSL
08) Characteristic features of genetic code,
a) Ambiguous
b) Comma less
c) Degenerate
d) Non overlapping
e) Written in circular form

09) According to the structure of chromosome,


a) Has single strand of DNA.
b) They are classified according to position of centromere.
c) Y chromosome is acrocentric.
d) In normal females one X chromosome is totally inactivated.
e) Telomeres involved in the aging.

10) Turner syndrome,


a) Trisomy
b) Shows genital dysenteries.
c) Survive after few months of born.
d) Present in both sex.
e) Can be detected by cytogenetic techniques.

SBR
11) The most common form of gene expression regulation in eukaryotes is,
a) Translation control
b) Transcription control
c) Post transcription control
d) Post translation control
e) Control of the passage from the nucleus

12) Human allantois,


a) Plays a significance role in exchange of gases with maternal blood.
b) Function in formation of gonads.
c) Degenerates to join pharynx and the rest of the GIT gut.
d) Incorporated to form the urinary and reproductive track.
e) Ultimately becomes part of the gallbladder.

13) Folding of embryo result in,


a) Neural tube
b) Connective stalk
c) Foregut
d) Definitive yolk sac
e) Stomodeum

14) What is the structure remaining active in the mature umbilical cord?
a) Vitelline duct
b) Umbilical vein
c) Allantois
d) Intestinal loop
e) Amniotic cavity

Presented by 15th Batch 265 FHCS | EUSL


15) Which of the following condition is an autosomal dominant inheritance?
a) Hemophilia
b) Sickle cell anemia
c) Homocystinuria
d) Thalassemia
e) Neurofibromatosis

11TH BATCH (EME) - SEQ

1. Placenta is a feto maternal organ which plays an important role in the maintenance of fetal growth and
development
1.1 Describe the formation of placenta (40)
1.2 Illustrate the structure of fully developed placenta(15)
1.3 State the composition of placental barrier and list 3 functions of it (25)
1.4 Explain the features of fetal membranes in monozygotic twins (20)
2.
2.1 Write an account on
2.2.1Autisomal recessive disorders(20marks)
2.2.2 Structural anomalies of choromosomes.(40marks)
2.2 Briefly describe the steps of Eukaryotic gene expression with illustration.

11th mcq
1)
A)T
B)F granulosa cells in primary follicle
C) T
D)
E)

2)
A)
B)T
C)T
D) F
E)T

3)
A)The appearance of notochord & the prechordal mesoderm
induces the overlying ectoderm to thicken to form neural
plate.
B)T
Neural tube defect
• It results when neural tube closure fails.
• If closure fails in the cranial part: anencephaly
Prevnted by termination of pregnancy
• If closure fails caudally at any part: spina bifida
• Common site lumbosacral region

Presented by 15th Batch 266 FHCS | EUSL


Prevented by oral intake of folic acid.
C)
D)
E)

4)
The epiblast, through the process of gastrulation, is the source of all
of the germ layers
A)
B)
C
D)
E)

5)
Ectodermal layer give rise to organs & structures that maintain contact
with outside world.
1. CNS
2. PNS
3. Sensory epithelium of ear, nose & eye
4. Epidermis including hair & nail
5. Sebaceous glands
6. Mammary glands
7. Pituitary gland
8. Enamel of teeth
A)T
B)F
C)
D)F
E)T

6)
A)T
B)
C)
D)
E)

7)A)
B)
C)
D)
E)

8)
A)F
B)T
C)T
D)T
E)F

9)
A)F

Presented by 15th Batch 267 FHCS | EUSL


B)T
metacentric ; p=q – chromosomes 1,3,16,19,& 20
-submetacentric ; p < q - chromosomes 2,4,5,6-12,17,18 & X
-acrocentric ; p < < q - chromosomes 13-15, 21,22 & Y
C)T
D)T
E)T

10)
A) F Monosomy 45,XO
B)T
C) F 96-98% do not survive to birth
D) F common in females
E)
Cytogenetics
• It is a branch of genetics is concerned with how the
chromosome relate to cell behaviour.
• The commonest laboratory investigations used is the
Karyotyping
11)B
genes are only expressed when they needed often regulated at transcription
12)D

13)A

14)B
When the allantois and the vitelline duct and its vessels are
also obliterated, all that remains in the cord are the
umbilical vessels surrounded by the jelly of Wharton.
15) E
others all are autosomal recessive

11TH BATCH (EME) – SEQ Answers

1. Placenta is a feto maternal organ which plays an important role in the maintenance of fetal growth and
development
1.1 Describe the formation of placenta (40)
‐ Placenta has maternal & fetal components.
‐ Fetal part derived from chorionic plate.
Maternal part from uterine endometrium.

→ Formation of Fetal part-chorionic plate (trophoblast+Extraembryonic mesoderm)


• Primary villi formation - by the beginning of 3rd week, appearance of cytotrophoblastic core covered with
syncytial layer.
• Secondary villi formation-During further development, mesodermal cells penetrate the core of primary villi &
grow towards decidua.
• Tertiary villi formation- mesodermal cells in the core of villi begin to differentiate into blood cells & small blood
vessels. They form villous capillary system.
• Capillaries in tertiary villi join with developing capillaries of chorionic plate & connecting stalk. Establishing
contact with intraembryonic circulatory system.
• Outer cytotrophoblastic shell formation
Presented by 15th Batch 268 FHCS | EUSL
-meanwhile cytotrophoblastic cells in villi penetrate into syncytium & cover it. This shell gradually surrounds
trophoblast entirely& attaches the chorionic sac firmly to the maternal endometrium.
• Anchoring villi/Stem villi – villi that extend from chorionic plate to decidua basalis of maternal endometrium. Stem
villi branch from the sides to form terminal/free villi in to intravillous spaces. Exchange of nutrients & other factors
occur through these free villi.
• By 19th-20th day embryo is attached to its trophoblast shell by connecting stalk.
• By the beginning of 2nd month trophoblast has a great number of secondary &tertiary villi.
• By the 4th month cytotrophoblastic & some connective tissue cells disappear in the villi, forming larger villi.
Thereby placental barrier formed by syncytium & endothelium.
• Villi covers entire surface of chirion.as pregnancy advanced, villi on embryonic pole grow & expand to form
chorion frundosum.villi on abembryinic pole degenerate.

→ Maternal part development-


• Spiral artery delivered maternal blood to placenta. They erode &release blood in to intervillous spaces.
• Invasion of maternal spiral arteries by cytotrophoblastic cells creating hybrid vessels containing both maternal &
fetal cells. This process transforms small diameter high resistance vessels to large diameter, low resistance vessels.
• The decidua of endometrium over chorionic frundosum is decidua basalis. This connects the chorion tightly.
• During 4th &5th months decidua forms a number of decidual septa, which project in to intervillous spaces divide
the placenta in to cotyledons.
• Together with decidua basalis, chorionic frundosum makes up the placenta

1.2 Illustrate the structure of fully developed placenta(15)

‐ 15-20 cotyledons on maternal side.


‐ Grooves separating cotyledons are sulci. formed by decidual septa.
‐ Fetal surface entirely covered by chorionic plate & Amniotic membrane.
‐ Blood vessels seen radiating from umbilical cord.
‐ Inserted umbilical cord in fetal surface. usually e acentric.

1.3 State the composition of placental barrier and list 3 functions of it (25)
Hemochorial barrier.
• Placental barrier initially has 4 layers.
1. Syncytiotrophoblast
2. Cytotrophoblast
3. Connective tissue
4. Endothelial lining of fetal blood vessels.
• From the 4th month, cytotrophoblast & connective tissue degenerate- endothelial lining comes in intimate
contact with the syncytiotrophoblast. Facilitating diffusion.
Presented by 15th Batch 269 FHCS | EUSL
• Placental barrier is not a true barrier-many substance pass through it freely.
• Passes nutrients from mother to fetus -Eg : AAs, free Fatty Acids, Carbohydrates, Vitamins
• Exchanges gases between mother & fetus . Eg:oxygen,CO2,CO
• Allow maternal antibodies to cross it-eg:IgG
• Separate fetal & maternal blood.

1.4 Explain the features of fetal membranes in monozygotic twins (20)


• Twins developed from single fertilized ovum are monozygotic twins.
Splitting of zygote occurs at various stages of development.
1. Separate at 02 cell stage-02 separate zygotes developed, blastocysts implant separately, Each embryo has its own
Placenta, chorionic sac,& amnion.
2. Splitting of zygote at early blastocyst stage. Inner cell mass splits in to 02 groups within same blastocyst cavity.02
embryos have a common placenta, common chorionic cavity But separate amniotic cavities
3. Splitting of inner cell mass at a late stage of development, embryos have a common placenta, a common
amniotic cavity & a common chorionic cavity.
2.
2.1 Write an account on
2.2.1Autisomal recessive disorders(20marks)

• Phenotyoe only in homozygous due to recessive nature.


• Parents may normal but with affected offsprings.
• This condition suspect in consanguineous parents.
• If both parents are affected,all children are affected.
• If parents are Normal-heterozygote, For each pregnancy 25% o offsprings are affected. 50% of heterozygotes are
carriers 25% of homozygotes are Normal.
• Both sexes are equally affected.
• Consanguineous parents increase the incident.
• Features-carriers usually normal Eg:Tay-sachs disease
• Screening programmes used to identify heterozygous carriers in a population to prevent them from mating.
Eg:Tay-sachs disease,Sickle cell anemia,Cystic fibrosis
• Intermediate inheritance:sickle cell disease carriers may show cell sickling& manifest symptoms as in
homozygotes under conditions of high Altitude,parachuting.

2.2.2 Structural anomalies of choromosomes.(40marks)


• Involve one or more chromosomes.

1. Deletion Loss of chromatin.caused by environmental factors such as viruses,radiation,drugs


Terminal deletion
-break at the end of arm of chromosome.
Eg:Cri du chat syndrome-tip of short arm of chromosome 5 deleted.cat like cry due to malformed larynx.low set
ears & Hypertelorism,congenital heart diseases,intellectual disability.
Presented by 15th Batch 270 FHCS | EUSL
Interstitial deletion
- two breaks occur within chromosome arms,broken segment lost,2 ends unite to form a short arm.
Microdeletion
-very small deletion.
Eg: parder willi syndrome(long arm of chromosome 15) Angelman syndrome (Maternal chromosome 15)

2. Duplication-Gain of chromosome material by doubling in a particulary region.


Direct /tandem duplication
Inverted /mirror duplication Eg: Pallister killian

3. Isochromosome-it is formwd when a chromosome with 2 chromatids splits at right angles to normal lengthwise
separation at cell division.Resulting chromosomes have either both short arms/both long arms .also net loss will be
an entire long/short arm

4. Inversion- chromosome break at 2points & broken segments turns round 180° to reverse the order of chromatin.

5. Translocation-exchange of chromosomal material between chromosomes.


03 types.
Centric fusion(Robertsonian translocation)
Fusion of whole arms of acrocentric chromosomes.short arm chromosome is lost ,long arm chromosome survive
Eg:5-8% of down syndrome(long arm of 13,14,15,22 translocate in to extra 21 chromosome on No 13,14,15,22)
Reciprocal- breakage & exchange of segments between chromosomes.exchange of segments with no loss of
chromosome material. Eg:chronic myeloid leukemia
Intertional translocation- Insertion of a deleted segment of a chromosome interstitially or inside another
chromosome following a breaking at that point.

2.2 Briefly describe the steps of Eukaryotic gene expression with illustration.
• Gene expression is the process by which information from a single gene is used to synthesis of a functional gene
product.
• These products are often proteins or functional RNAs.
• Multi steps
Transcription
RNA processing/Splicing
Translation
Post translational modification

1. Transcription- is RNA Production.,by copying part of DNA.


Has 03 steps-initiation,elongation & Termination.
started & contrlolled by RNA polymerase enzyme.
Initiation is started by this enzyme which binds to DNA double strand at promoter site.promotor site is recognized
by sigma factor of RNA polymerase.unwinding that region & reads the template DNA strand from 3’ to 5’
direction.RNA is synthesized antiparallel To DNA starand.RNA polymerase doesn’t need a primer to begin
synthesis.Termination region is recognized by rho factor of RNA polymerase.mRNA is synthesized.

2. RNA processing- Post transcriptional Modification.


introns are removed by splicing.Exones come together.
5’ capping by 7 methylguanosine Addition of poly A tail Product is Mature RNA molecule

3. Translation-synthesis of a protein molecule from m RNA.


03 parts.initiation,elongation & Termination.

Presented by 15th Batch 271 FHCS | EUSL


Initiation-a ribosome bind to mRNA molecule.
mRNA contains codons(3 bases on mRNA) T RNA contain anticodons.
they link with codon.
start codon is AUG.AUG is recognized by a special initiator tRNA which carry methionine as the 1st AA.
Elongation-another tRNA bring 2nd AA & its anticodon linked with the 2nd codon of mRNA.
A peptide bond formed between 2AAs.
1st tRNA release its aminoacid & move away.
ribosome move along the mRNA.
another tRNA comes with AA.a polypeptide chain is formed like this.
Termination-when stop codon is reached(UAG,UGA or UAA) they do not code for AA.
so stop the protein synthesis.end product is a primary structure of a protien.

4. Post translational modifications- trimming


Covalent attachment (phosphorylation,glycosylatiin,hydroxylation)
Protein folding
Protein degradation

12TH BATCH (EME) - MCQ

1) Regarding germinal period,


a. Oogonia develops from PGC’s
b. LH is responsible for the initiation of transformation of PCG to mature spermatozoa
c. It takes 4 days to complete the process of spermatogenesis inside male reproductive system
d. Primordial germ cells can be seen at the peripheral of the seminiferous tubules
e. Secondary oocyte complete meiosis 2 before ovulation

2) Primitive streak,
a. Initiate the formation of neural plate
b. Is thickening of ectoderm
c. Is located immediately caudal to precordal plate
d. Is needed for the formation of extra embryonic mesoderm
e. Involves in the of cells

3) Notochord,
a. Is formed during the third week of developed
b. Development to form skeletal system
c. Induce the neural tube forming
d. Formed from the mesoderm
e. Formation intravertible disc of axil skeletal

4) Regarding placenta,
a. It begin to form at 9th week development
b. It expel from body 30 minutes after birth
c. Maternal surface consist 30-35 cotyledons
d. It entire covered by chronic plate on the fetal side
e. Can allow the entry virus the fetus

5) Regarding somites,
Presented by 15th Batch 272 FHCS | EUSL
a. 1st pair appeared 20th day
b. Scelrotome form axial skeleton
c. At 20th day appear 1st pair of somites
d. At 4th week appear 44 somites
e. Somites are formed by notochord

6) Regarding gene expression,


a. Addition of poly A tail is a post transcriptional modification
b. It occurs in the cytoplasm
c. RNA is synthesized from 5'-3' direction
d. Promoter region is recognized by Rho factor of RNA polymerase
e. RNA polymerase does not need a primer to begin RNA synthesis

7) Regarding Down syndrome,


a. Risk increased with late maternal age
b. Most of times associated heart defects
c. Can be diagnosed prenatally
d. Having trisomy 21
e. New born babies are hypotonic

8) Example for multifactoral inheritance include,


a. Arterial septum
b. Downs syndrome
c. Hemophilia
d. determination of a height of an individual
e. Diabetes mellitus

9) Regarding structural anomalies of chromosomes,


a. Deletion is result in loss of chromatin
b. Isochromosomes are seen in turners syndrome
c. Robertsonial translocation produce loss of chromatin
d. Reciprocal translocation produce loss in chromatin
e. Angleman syndrome is an example for the duplication

10) corpus luteum,


a. is formed by stimulation of LH
b. secrete only progesterone
c. become a corpus luteum gravidities only its fertilized
d. Removal before 4th month usually leads to aborsh
e. Is found in the cortex of ovary

SBR
11) What isn't post transcriptional modification?
a. glycosylation
b. protein degradation
c. protein folding
d. removal on intron by splicing
e. trimming

12) What is the structure internally surrounding the oocyte immediately following ovulation?

Presented by 15th Batch 273 FHCS | EUSL


a. theca interna
b. corona radiata
c. proline stabilize the helix confirmation of alpha chain
d. zona pelucida
e. Cumulus oophorus

13) What is the correct statement about twins,


a. Dizygotic twins have similar genetic constitution
b. Monozygotic twins form only by separating of two cell stage
c. Monozygotic twins can have common amniotic cavity
d. Dizygotic can have common chorionic cavity
e. Monozygotic may not have separated chorionic cavity

14) A disease transmitted only from mother generation to generation,


a. Cystic fibrosis
b. Meningoencephalocele
c. Leber's hereditary optic neuropathy
d. meningomyelocele
e. homocystenuria

15) What is the most suitable answer regarding Y chromosome?


a. Male gamete has only Y chromosome.
b. It inhibits excessive generation formation in male.
c. Hemophilia gene is carried by its long arm.
d. It determines sex at fertilization
e. It has more coding sequences than the x chromosome

12TH BATCH (EME) - SEQ

1. 1.1. Write and account on


1.1.1.Formation,maturation and functions of umbilical cord (25 Marks)
1.1.2. Formation and fate of the definitive yolk sac (30 Marks)
1.1.3. Autosomal dominant inheritance (25Marks)
1.2. Briefly explain the biochemical basis of sickle cell disease with regard to mutation (20 Marks)

Answers

MCQ

1.
A. True
B. True spermatogenesis is regulated by LH porduction by pituitary gland.
C. False. 74days
D.True
E. False. If the oocyte fertilized, meiosis 2 is completed.

2.
Presented by 15th Batch 274 FHCS | EUSL
A. F notochord and prechordal mesoderm
B. F narrow groove with slightly bulging region on either side on the surface of ectoderm.
C. F. Notochord is there between.
D. T epiblast cells migrating through the caudalmost parr of streak+ primitive yolk sac

3.
A. T
B. T become nucleus pulposus of intervertebral disc
C. T. Notochord+prechordal mesoderm
D. F prenotochordal cells from epiblast
E. T

4.
A. T. Start of 9th week or end of 2nd month
B. The placenta should be delivered within 30 to 60 minutes after having the baby. C. F 15to 20
D.T
E. T
Maternal steroidal hormones,synthetic progestin, thyroxine(slow), synthetic estrogen,
viruses(rubella,cytomegalovirus,coxsackie,variola, varicella,measels, poliomyelitis virus) , most drugs

5.
A. T page 82
B. T. Vertebrae,ribs,tendon,cartilage
C. T
D. F 20th day - 1pair; 3pair each day; 28 day(4th week)- 25to 35pair approximately E. F. Paraaxial
mesoderm

6.
A. T
B. T. Cytoplasm and nucleus.
C. T RNA polymerase will read the information sequence on
DNA template from 3′→ 5′ direction, so RNA is synthesized antiparallel to DNA template i.e. from 5′→3′
direction.
D. F. Protomer region is recognized by sigma factor. Termination region is recognized by rho factor.
E. T

7.
A. T
B. T
C. T
D. T
E. T

8. Multifactorial disorders happen due to genetic defect and environmental influences such as nutrition,
medical condition.
A. T
B. F chromosomal disorder
C. T acquired hemophilia
D. T E. T

Presented by 15th Batch 275 FHCS | EUSL


9.
A. T
B. T 20% of turners syndrome is caused by this
C. T short arms of acrocentric chromosomes are lost. But, it doesn't lead to any consequences as these
short arms have genetically inert material or RNA genes.
D. F
E. F. Microdeletion disorder in maternal chromosome 15.

10.
A. T
B. F estrogen and progesterone
C. T
D. T luteal cells of corpus luteum continue to secrete progesterone which keeps the pregnancy until
end of 4th month. After 4th month, placenta secrete progesterone to keep pregnancy.
E. T

SBR
11. There is an error in the question. It should be post translational instead of transcriptional.
So, we can get the answer, D
12. Answer may be zona pellucida. But,they asked internally
13. C during late stage bilaminar disc separation
14. C In mitochondrial inheritance,Only mother can transmit traits to both son and daughter.
15. D

SEQ
1. Write an account on,

1.1.1 Formation, maturation and functions of Umbilical cord.(25marks)

At 13th day of embryonic development, the connecting stalk is formed by extra-embryonic Mesoderm.
At 3 week, Allantois, a diverticulum from yolk sac extends in to connecting stalk. With embryonic folding,
connecting stalk & allantois become ventral of embryo. From 19th day of development, embryonic blood
vessels appear. At the 5th week of development, an oval reflection between amnion & embryonic
ectoderm is the Primitive Umbilical ring.

Structures passing through the ring are,

1. Connecting stalk

• Allantois

• Umbilical vessels (2arteries & 1 vein)

2. Yolk sac stalk (Vitelline duct & Vitelline vessels)

3. Canal Connecting Intraembryonic & Extraembryonic Cavities.

Presented by 15th Batch 276 FHCS | EUSL


Formation of primitive Umbilical cord, as the further development of embryo occur as Amniotic cavity
enlarges rapidly, expense of chorionic cavity. Amnion envelop the connecting & Yolk stalks crowding them
together to form primitive umbilical cord. In this cord, distally-Yolk sac stalk& umbilical vessels more
proximally-Some intestinal loops & Allantois. Maturation of Umbilical cord occurs as the fetal growth.at
the end of 3rd month. Amnion comes in contact with chorion. Chorionic cavity obliterated, causing yolk sac
shrinking. Intestinal loops are withdrawn in to abdominal cavity. Yolk sac shrink & gradually obliterated.
Allantois also obliterated. Remaining in the umbilical cord is umbilical vessels surrounded by Wharton
jelly.

Functions of umbilical cord:


- Connecting the fetus to placenta.
- Carries nutrients, Oxygen from maternal blood to fetus by umbilical vein
- Carries wastes-metabolic wastes & CO2 by umbilical arteries, to mother.

1.1.2 Formation & Fate of Definitive Yolk sac (30 marks)

In 2nd week of the development of embryo, primitive yolk sac appears. Flat cells from Hypoblast form a
thin membrane (Heuser membrane). This membrane lines inner surface of cytotrophoblast forming the
lining of primitive yolk sac. This occurs about 9th day of development.
At 13th day Hypoblast provides an Additional cell layer. These cells migrate along the inside of exocoelomic
membrane /Heuser membrane. These cells proliferate & make a new cavity within exocoelomic cavity. It
is the secondary or definitive yolk sac.
Its’

• Roof formed by Hypoblast

• Wall formed by exocoelomic membrane + Splanchnic layer of Extra Embryonic Mesoderm.

At day 16, yolk sac send a diverticulum (allantois) in to the connecting stalk. As embryo folds, yolk sac
become ventral. Closure of lateral body wall except for the umbilical region, where connecting stalk & yolk
sac duct remain attached. Gut tube formed by incorporating a large portion of endoderm in to embryo’s
body. Definitive yolk sac remain outside of the embryo. Mid gut of the gut tube communicates with yolk
sac by broad vitelline duct/vitello inteatinal duct. With further embronic development this duct become
narrow. In the 5th week, yolk sac duct, allantois, with Umbilical vessels are in umbilical cord.
Vitellointestinal duct fibrosed & degenerated at the end of 6th week. Yolk sac detached from mid gut. In
2% of Adults ,Its proximal intraabdominal part persists as “Meckel diverticulum”. At 10th week definitive
yolk sac lies in chorionic cavity between amniotic & chorionic sacs. As pregnancy advances, definitive yolk
sac atrophies.

1.1.3
Autosomal dominant Inheritance (25 marks)

• Shows Mendelian pattern of inheritance.


• Dominant gene located on one of the autosomes.(Non sex chromosomes)
• Traits are expressed in any individual with at least one dominant allele.
• Individuals who are Homozygous or Heterozygous for trait will display the dominant Phenotype
• Doesn’t skip generations.
Presented by 15th Batch 277 FHCS | EUSL
• Males & females are equally affected.
• Disorders transmitted by both sexes.
• Every affected child has an affected parent, except for new mutations.
• Mating affected heterozygous with unaffected, each offspring has a 50% chance of inheriting the
affected phenotype.
Parent Aa aa
Gametes Aaaa
Offspring genotype Aa Aa aa aa
Phenotype Affected : Normal
50% : 50%
1:1

• Special features.

a. New mutations – increased paternal age may be associated with new mutations. Eg:
Achondroplasia

b. Reduced penetrance

c. Variable expressivity- degree of expression of a trait. Individuals in family may show mild to
severe forms of disease. Eg: Marfan syndrome

d. Variation in age-adult polycystic kidney disease, Huntington’s disease show features later in
life

e. Variation in severity may depend on sex

f. Phenocopy- an environmentally caused phenotype.it is similar to a mutation produced by


mutant gene

g. Influence of non-genetic factors. Eg: Diet can influence familial hypercholesterolemia

1.2 Briefly explain the biochemical basis of Sickle cell disease with regard to mutation(20 marks)

• Sickle cell anemia is a genetic disorder, caused by a point mutation/single gene mutation.

• Autosomal recessive inheritance.

• Mutant gene for beta globin of hemoglobin. Mutant B globin chain is designated HBs

• Hbs contains 02 normal alpha globin chains & 02 mutant B globin chains.

• Mutation changes the 06th codon “GTG”, which codes for glutamic acid in to “GAG” which codes for
Valine.

• Replacement of charged glutamate with nonpolar Valine.

Presented by 15th Batch 278 FHCS | EUSL


• Valine forms protrusion of Beta chain of another Hb molecule in the RBC.

• At the low oxygen tension, deoxy Hbs polymerize inside the RBC & form an insoluble fibrous
network .

• This stiffen & distort the RBC.

• RBC become rigid & misshapen.

• Sickled RBCs can block the blood flow in narrow capillaries& interrupt the oxygen supply to tissues
causing pain & eventually death of cell.

13TH BATCH (EME) - MCQ

1) Regarding spermatogenesis
A. It is beging before puberty.
B. Influence by LH
C. Leydig cells assist in the mature spermatozoa
D. Duration of the process is longer than oogenesis
E. It is a continuous process

2) Event occur during 11th & 12th day of development include,


A. Formation of sinusoids
B. Establish uteroplacental circulation
C. Formation of extraembryonic mesoderm
D. Desidua reaction occurs
E. Formation of definitive yolk sac

3) Notochord
A. Derived from mesoderm
B. Is form cranial to the prechordal plate
C. Is produce migrate epinblast through blastopore
D. Influences formation of neural tube
E. Derivatives in adult is the nucleus pulposes

4) Derivative of neural crest cells


A. Cranial nerves
B. Oodontoblast
C. Adrenal medulla
D. Neuroglia
E. Melanocytes

5) External features of embryo seen during the 2nd month of gestation include

Presented by 15th Batch 279 FHCS | EUSL


A. Larger head
B. Appearance of eye
C. Appearance of external genitalia
D. Pericardial swelling
E. Prominent somites

6) Derivatives of definitive yolk sac


A. Epithelium of respiratory tract
B. Epithelium of GI tract
C. Epithelium of urinary bladder
D. Epithelium of retina
E. Oogonia cells

7) Regarding the umbilical cord


A. Distally contain yolk sac stalk
B. Proximally umbilical vessels
C. Distally intestinal loops
D. Proximally amniotic fluid

8) Disorders regarding reduction of genetic material


A. Angelman syndrome
B. Downs syndrome
C. Acute lymphocytic leukemia
D. Philadephia syndrome
E. Cri-du-chat- syndrome

9) Regarding genetic code


A. Ambiguous
B. It is redundant
C. It is virtually universal
D. Overlapping
E. Particular codon always coded for same amino acid

10) Clinical features of patau syndrome


A. Scalp defect
B. Micrognathia
C. Lateral cleft lip
D. Missing eyes
E. Webbed neck

11) Which of following congenital malformation will most predictable result oligohydramnions
A. Anencephaly
B. Spinal brivida
C. Renal agenesis
D. Tracheoesophageal fi stula
E. Urethral atresia

12) Which of the following disorders result due to frameshift mutation


A. Down syndrome
B. Turner syndrome

Presented by 15th Batch 280 FHCS | EUSL


C. Klienfelter syndrome
D. Tay sach syndrome
E. Sickle cell anemia

13) Which of the following cross the placental barrier from mother to fetus in a slow rate
A. Maternal steroid hormones
B. Thyroxine
C. Cocaine
D. Megala virus
E. Drug metabolites

14) Which of following genetic disorders show reduce penetrance/Variable expressivity


A. X-linked recessive
B. Autosomal recessive
C. Autosomal dominant
D. Y-linked dominant
E. X-linked dominant

15) Which is following most suitable regarding twins


A. Dizygotic twins have same genetic constituents
B. Monozygotic twins are always separate during two cell stage
C. Dizygotic twins may have common chorionic cavity
D. Monozygotic twins may have separate chorionic cavity
E. Monozygotic twins can’t have common amniotic cavity

13TH BATCH (EME) - SEQ

1. Placenta is a fetomaternal organ


1.1. Describe the formation of placenta. (30 marks)
1.2. List 5 structural features of matured placenta (15 marks)
1.3. Explain how you would perform karyotyping to detect any chromosomal anomalies of a new born (30 marks)
1.4. Outline the various steps in the process of gene expression (25 marks)
Answers
MCQ
1.

A False Begins at puberty.

B True Regulated by LH. LH binds the receptors on Leydig cells stimulate testosterone production promote
spermatogenesis. Testosterone binds to sertoli cells
C False Sertoli cells support and protect the germ cells,supply nutrition, assist in the release of mature
spermatozoa.
D False Spermatogenesis duration=74 days approximately. Oogenesis duration=begins before birth. Will not
complete the Meiosis before fertilization.
E True

2. Event occur during 11th & 12th day of development include,


A True
• Blastocyst completely embedded in the endometrium.
Presented by 15th Batch 281 FHCS | EUSL
• Maternal capillaries congested, dilated and forms the sinusoids.
• Lacunae become continuous with the sinusoids.
• Uteroplacental circulation is begun.
• Extra embryonic mesoderm is formed.(extra embryonic somatic mesoderm+ extra embryonic splanchnic

mesoderm)

• Extra embryonic cavity/ chorionic cavity is formed.


• Decidua reaction occur.
B True
C True
D True
E False Occur during 13th day.

3 Notochord
A False Derived from endoderm
B False Prenotochordal cells move forwards cranially in the midline until they reach to the prechordal plate

C True
D True
E True

4 Derivative of neural crest cells


A True
• Dermis in the face and neck
• Odontoblast (the cells which produce tooth dentine.
• CT and bones of the face and skull
• C cells of thyroid gland
• Conotruncal septum in the heart
• Spinal(dorsal root) ganglia
• Sympathetic chain and preaortic ganglia
• Meninges (forebrain)
• Parasympathetic ganglia of the GIT
• Smooth muscle cells of blood vessels in the face and forebrain.
B True
C True
D True
E True

5 External features of embryo seen during the 2nd month of gestation include
A True
❖ Increase head in size.
❖ Formation of the limbs ,face ,ears, nose and eyes.,crest cells derivatives ❖ Counting of smites

becomes difficult. therefore the age of the embryo indicated as crown rump length (CRL) ❖ Paddle shaped

limb buds

Presented by 15th Batch 282 FHCS | EUSL


❖ Pericardial swelling
❖ Forelimbs innervated by brachial plexus.
❖ Formation of digits
B True
C false Appeared during the 3rd month of development.
D True
E false

6 Derivatives of definitive yolk sac


A True
B True
C True
D False
E True

7 Regarding the umbilical cord


A True Distally the cord contains the yolk sac stalk and the umbilical vessels.
B True Proximally contains some intestinal loops and the remnant of the allantois. But, contents of distal part
also can be included in proximal part.
C False
D False

8 Disorders regarding reduction of genetic material


A true Occur due to loss of function of a gene on chromosome no.15.
B False Trisomy 21
C
D Chromosome 9 and chromosome 22break and exchange portions. forms an abnormally small

chromosome 22.

E True Tip of the short arm of chromosome no.5 is deleted

9 Regarding genetic code


A False Characteristics of the genetic code
1. Written in linear form
2. Each word consist of 3 riboneuleotide letters.
3. The code is unambiguous/specific-a particular codon always code for same
AA
4. Degenerate/reduntant-some AA may have more than one codon.
5. The code contain 1 start and 3 stop codons.
6. The code is commaless.
7. Non overlapping
8. The code is (nearly) universal
B True
C True
D False

Presented by 15th Batch 283 FHCS | EUSL


E True

10 Clinical features of patau syndrome (trisomy 13)


A true
❖ Growth retardation
❖ Microcephally
❖ Mental retardation
❖ Midline abnormalities
❖ Scalp defects
❖ Brain lessions (holoprosencephally)
❖ Cardiac defects ❖ Omphaloceles

❖ Cryptochydism
❖ Post axial polydactyly
❖ Facial defects
• Micrognathia
• Central or unilateral clefts in lip and palate
• Eye defects (hypotelorism)
• Micropthalmia/ anopthalmia /cyclopia( single central eye)
B True
C True
D True
E False In turner syndrome.

SBR
11 C
❖ Oligohydramnios means decreased amount of amniotic fluid (<400ml) ❖ It is may result

from renal agenesis.

❖ Results= constrict the fetus ,lung hypoplasia


❖ Hydramnios/ polyhydramnios means excess of amniotic fluid. (1500 to 2000ml) ❖

Causes for hydramnios are idiopathic causes, maternal diabetics, congenital malformations [ CNS

disorders (anencephaly), GIT defects (atresias )]-they prevent the infant from swallowing the fluid.

12 D
• Inserting or deleting of one or more nucleotides.
• Changes the reading frame.-shift the reading frame of genetic message.
• Proteins built incorrectly.
• Ex:- Tay-sachs disease

13 B
❖ Steroidal hormones readily cross the placental barrier.
❖ Other hormones such as thyroxine do so only at a slow rate.

Presented by 15th Batch 284 FHCS | EUSL


❖ Rubella, cytomegalovirus, coxsackie, variola, varicella, measles, poliomyelitis viruses traverse the

placenta without difficulty.

❖ Most drugs and metabolites traverse the placenta without difficulty.


❖ Heroin and cocaine can traverse through placenta.

14 C
• Reduce penetrance means although only one mutated copy is needed, not all individuals who

inherit that mutation go on to develop the disease.

• Autosomal dominant conditions sometimes have reduce penetrance.

15 D
• Most common
• Simultaneous shedding of 2 oocytes and fertilized by different sperms.
• 2 zygotes have totally different genetic constitutions.
• The zygotes implant individually in the uterus.
• Each has its own placenta, amnion, chorionic sac
Monozygotic twins
• Monozygotic/ identical twins
• Developed from a single fertilized ovum
• Result from splitting of the zygote
❖ During Two cell stage-2 separate zygotes developed. Blastocysts implant separately.

Has its own placenta and chorionic sac. ❖ During early blastocyst stage- inner cell

mass split into 2 within the same blastocyst cavity. 2 embryos have common placenta,

common chorionic cavity but separate amniotic cavities. ❖ During bilaminar germ disc

stage.- have single placenta, common chorionic and amniotic sac.

SEQ

01
1.1 Describe the formation of the placenta (30 marks) Placenta has maternal & fetal components. Fetal part
derived from chorionic plate. Maternal part from uterine endometrium. Formation of Fetal partchorionic plate
(Trophoblast+Extraembryonic mesoderm) 11th batch Medicine. Primary villi formation - by the beginning of 3rd
week, appearance of cytotrophoblastic core covered with syncytial layer. Secondary villi formation- During
further development, mesodermal cells penetrate the core of primary villi & grow towards decidua. Tertiary
villi formation- mesodermal cells in the core of villi begin to differentiate into blood cells & small blood vessels.
They form villous capillary system. Capillaries in tertiary villi join with developing capillaries of chorionic plate &
connecting stalk. Establishing contact with intraembryonic circulatory system. Outer cytotrophoblastic shell
formation -meanwhile cytotrophoblastic cells in villi penetrate into syncytium & cover it. This shell gradually
surrounds trophoblast entirely& attaches the chorionic sac firmly to the maternal endometrium. Anchoring
villi/Stem villi – villi that extend from chorionic plate to decidua basalis of maternal endometrium. Stem villi
branch from the sides to form terminal/free villi in to intravillous spaces. Exchange of nutrients & other factors
occur through these free villi. By 19th-20th day embryo is attached to its trophoblast shell by connecting stalk.
Presented by 15th Batch 285 FHCS | EUSL
By the beginning of 2nd month trophoblast has a great number of secondary &tertiary villi. By the 4th month
cytotrophoblastic & some connective tissue cells disappear in the villi, forming larger villi. Thereby placental
barrier formed by syncytium & endothelium.

Villi covers entire surface of chirion.as pregnancy advanced, villi on embryonic pole grow & expand to
form chorion frundosum. Villi on abembryinic pole degenerate. Maternal part development-spiral
artery delivered maternal blood to placenta. They erode &release blood in to intervillous spaces.
Invasion of maternal spiral arteries by cytotrophoblastic cells. Creating hybrid vessels containing both
maternal & fetal cells. This process transforms small diameter high resistance vessels to large diameter,
low resistance vessels. The decidua of endometrium over chorionic frundosum is decidua basalis. This
connects the chorion tightly. During 4th &5th months decidua forms a number of decidual septa, which
project in to intervillous spaces divide the placenta in to cotyledons. Together with decidua basalis,
chorionic frundosum makes up the placenta.

1.2 List 05 structural features of mature placenta (15marks)


1. 15-20 cotyledons in maternal side.
2. Grooves separating cotyledons are sulci. Formed by decidual septa.
3. Fetal surface entirely covered by chorionic plate & Amniotic membrane.
4. Blood vessels seen radiating from umbilical cord.
5. Inserted umbilical cord in fetal surface. Usually eccentric.

1.3. Explain how you would perform karyotyping to detect any chromosomal anomalies of a new born.(30
marks)
• Karyotyping is a technique that uses a picture of an individual’s chromosomes to analyse & detect
chromosomal abnormalities.
• Can be used to analyse the size, structure & number of chromosomes in a sample.
• Chromosome cultures are performed in rapidly dividing cells from metaphase stage of Mitosis.
1. Collecting samples-Cells from new born. Eg:WBC
2. Separating cells
3. Culturing cells-adding culture medium, harvested cells are cultured, at body temperature
(37°) for 3 days.
4. Cell sample is treated with Colchicine to arrest the cells in Metaphase.
5. Centrifuge the cells. It concentrates the cells at the bottom of the tube.
6. Addition of hypotonic saline-to swell up & more apart the cells.
7. Lysed cells are prepared, fixed using alcohol & acetic acid. Placed on a microscopic slide.
8. Digest with trypsin & stain with Giemsa. 11th batch Medicine
9. Analyse metaphase spread. G banding appearance can be seen in Giemsa staining. Series of
lightly &darkly stained bands in chromosomes. Dark bandstend to be heterochromatic, late
replicating & AT rich Light bandseuchromatic, early replicating GC rich.
10.The good spreads are captured using photomicroscope.
11.Individual chromosomes cut & arranged in pairs in decreasing order of size. Numbered from 1-22 &
sex chromosomes separately.
12.The photographic representation of entire somatic chromosomes complement is karyotype.

Presented by 15th Batch 286 FHCS | EUSL


1.4 Outline the various steps in the process of gene expression.(25 marks) • Gene expression -synthesis
of a functional gene product by using genetic Information.
• These products are often proteins or functional RNAs.
• Multi steps
Transcription
RNA processing/Splicing
Translation
Post translational modification

1. Transcription- is RNA production, by copying part of DNA. Started & controlled by RNA polymerase
enzyme. RNA polymerase doesn’t need primers to begin synthesis of RNA. This enzyme binds to DNA
double strand at promoter site. Unwinding that region & reads the template DNA strand from 3’ to 5’
direction. RNA is synthesized antiparallel To DNA strand.
2. RNA processing- Post transcriptional Modification. Introns are removed by splicing. Exons come
together. 5’ capping by 7 methyl guanosine. Addition of poly A tail produces mature RNA molecule.
3. Translation- synthesis of a protein molecule from m RNA.;Ribosome attaches to mRNA molecule;
tRNA molecule arrives. It brings an amino acid to first three bases (codon) in mRNA; Start codon is AUG.
Anticodons in tRNA link up with codons on mRNA; Ribosome reads the mRNA; Another tRNA comes
with AA; Peptide bond forms between AAs;As Ribosome moves, tRNAs comes with AAs.
Elongation of polypeptide chain.
When stop codon (UAA,UGA,UAG) is reached, termination occurs.Result is polypeptide chain.
4. Post translational modifications- trimming Covalent attachment (phosphorylation, glycosylation,
hydroxylation) Protein folding Protein degradation

14TH BATCH (EME) - MCQ

01. Primordial germ cell,


A) Are haploid cells.
B) Derived from epiblast.
C) Located periphery of the gonads.
D) Can undergo both meiosis and mitosis cell division.
E) Migrate from the yolk sac to the developing gonads from 4th week.

02. As soon as spermatocyte has entered the oocyte?


A) The oocyte finishes its second meiotic division.
B) Zona pellucida become impenetrable to other spermatozoa.
C) Corticle granules of acrosome are released.
D) Metabolic activation of oocyte is initiated.
E) First polar body release.

Presented by 15th Batch 287 FHCS | EUSL


03. At the time of implantation,
A) Endometrium is highly vasculated.
B) Zona pellucida is removed.
C) An epithelial defect is formed in the endometrium.
D) Embryonic pole of blastocyst is oriented toward the endometrium.
E) The mucosa of uterus in the secretary phase.

04. During the foetal period,


A) The foetus is vulnerable to the effects of teratogenic agents.
B) There is rapid growth and change in body proportion.
C) Between 9th - 12th weeks, upper limb are more developed than lower limb.
D) At 3rd month, eyes and ears of the fetus have come to their definitive positions.
E) Movements of the fetus can be detected by mother after 13th week of gestation.

05. Parameter that can be obtained ultrasound investigation include,


A) Age of the fetus.
B) Biparietal diameter of the skull.
C) Chromosomal defects.
D) Length of the femur.
E) Heart defects.

06. Teratogens associated with human malformations are,


A) Thalidomide.
B) Iron.
C) Alcohol.
D) Cytomegalovirus.
E) X rays.

07. Karyotyping
A) Helps to detect chromosomal disorders.
B) Can determine Edward syndrome.
C) The cells that used are in the telophase of meiosis.
D) Can be performed from chorionic villi cells of the fetus.
E) Helps to identify the defect in the chorionic myeloid leukemia.

08. Regarding the autosomal recessive diseases,


A) Transmission of trait from generation to generation.
B) Females are more affected in this disease.
C) Consanguinity increases the incidence of the disease.
D) Certain racial groups are affected by this disease.
E) It can be caused by deletion of the short arm of chromosomes.

09. Turner syndrome is,


A) A triploidy.
B) Show gonadal dysgenesis.
C) May survive after few month of birth.
D) Can be observed in both sexes equally.
E) Cannot be detected at birth.

Presented by 15th Batch 288 FHCS | EUSL


10. Regarding X- linked dominant inheritance,
A) More males are affected than females.
B) Affected males transmit to both sons & daughters.
C) Affected heterozygous female transmit to half their sons.
D) Female show variable clinical pictures.
E) Rett syndrome is example for such mode of inheritance.
.
Find out the most appropriate answer from each question (11-15) and mark them inside the cages.
11. Human allantois,
A) Play a significant role in gas exchange with maternal blood.
B) Function in early formation of gonads.
C) Degenerates to join pharynx and rest of GI tract.
D) Incorporated to form urinary and reproductive tracts.
E) Ultimately become part of gall bladder.

12. On ultrasound examination, it was found that a fetus had well-developed facial and thoracic regions, but caudal
structures were abnormal. Lumbar and sacral vertebrae were missing, and the hind limbs were fused. What process
may have been distributed to cause such defects?
A) Neurulation.
B) Gastrulation.
C) Folding of embryo.
D) Notochord formation.
E) Placentation.
13. Placenta is the fetomaternal organ compensation the natural demand as the fetal grows. Which of the
following statement correctly define the placental barrier?
A) It acts as a barrier permeability harmful virus to enter the fetus.
B) It is matured by incorporating the cytotrophoblast.
C) It can allow the IgG immunoglobulins to escape from the maternal blood.
D) It can allow the maternal thyroxin to diffuse rapidly into the fetal circulation.
E) It is composed of endothelial lining of the maternal vessels with the syncytium.

14. A disease transmitted specially from mother to next generation would be


A) cystic fibrosis.
B) sickle cell anaemia.
C) Lebers hereditary optic neuropathy.
D) Meningomyelocele.
E) Homocystinuria.

15. Which of the following type of genetic heterogeneity in the affected population,
A) X linked recessive
B) Y linked dominant
C) X linked dominant
D) Autosomal dominant
E) Autosomal recessive

14TH BATCH (EME) - SEQ

1.
1.1. Describe the oogenesis process. (30 marks)

Presented by 15th Batch 289 FHCS | EUSL


1.2. State 3 genetic importance of meiosis. (15 marks)
1.3. Describe the embryonic changes that occur in the paraxial mesoderm. (30 marks)
1.4. Briefly describe the types of gene mutation with examples. (25 marks)

Answers

01
A) F = primordial germ cells are diploid cells.
B) T
C) T
D) F = It can undergo only mitotic cell division.
E) F = 5th week.
02
A) T
B) T
C) F = cortical oocyte granules are released.
D) T
E) F = 2nd polar body release.
03
A) T
B) F = before implantation, in the early blastocyst stage zona pellucida is removed.
C) F = epithelial defect during the menstrual phase. During the implantation
epithelium
has proliferated.
D) T
E) T
04
A) F = during the 3rd week of development. (during the embryonic period)
B) T
C) T
D) T
E) F = during the 5th month.
05
A) T
B) T
C) F
D) T
E) F
06
A) T = It causes cardiac defect & ear abnormalities.
B) F
C) T = It causes microcephaly.
D) T = It causes vision loss, hearing loss & anaemia.
E) T = It causes mental retardation & leukaemia.
07
A) T
B) T = 47,XY, +18 or 47,XX, +18.
C) F = metaphase of mitosis.
D) T
E) F
08
Presented by 15th Batch 290 FHCS | EUSL
A) T
B) F = both sees are equally affected.
C) T
D) T
E) F = It is a single gene disorder. Short arm deletion is chromosomal
mutation
09
A) F = 45,X.(monosomy)
B) T
C) T
D) F = can be seen in females (only).
E) F = detected through the karyotype.
10
A) F = more females are affected than males.
B) F = never transmit to sons, but to all daughters.
C) T
D) T
E) T
11
A) T = form the placenta
B) F
C) F
D) F
E) F
12
A) F
B) T = Sirenomelia.
C) F
D) F
E) F
13
A) F = harmful viruses can pass through the placenta to fetus.
B) F
C) T
D) F = thyroxine slowly cross & steroid hormones rapidly cross the placenta.
E) F
14
A) F
B) F
C) T = mitochondrial disorder (specially transmitted from mother).
D) F
E) F
15
A) F
B) F
C) F
D) T
E) F

14th seq

01)
1.1 )
•Oogenesis Is the sequence of events by which germ cells oogonia differentiate into mature oocytes.

Presented by 15th Batch 291 FHCS | EUSL


• Gametes are derived from PGCs.
• Formed in the epiblast during 2ndweek and then move to the wall of yolk sac.
• Then begin to migrate from the yolk sac in the 4th week and arrive in the gonads by end of 6th week.
•Process of the differentiation of oogonia into mature oocytes.

Maturation of oocyte continues at puberty


- Until puberty primary oocyte undergo resting stage /diplotene stage.
- Total primary oocytes at birth; 600,000-800,000
- Primary oocytes at puberty: 40,000
- Primary oocytes ovulate: 500
- Primary oocytes are vulnerable to damage as they age, results in

chromosomal abnormalities.
• overian cycle is changes in a female’s ovary
• From sexual maturation to menopause
• Usually lasts 28 days
• During cycle primordial follicle resumes maturation and prepared
for ovulation
this the growth follicle

Ovulation
• Due to stimulation of LH & FSH secondary follicle grows.
• With final development of 2ndry follicle a sudden increase of LH
occurs
• LH causes the completion of meiosis I and help to enter the pre-
ovulatory phase
• Meiosis II started but arrested in metaphase.
• Surface of ovary bulge and at the apex an avascular stigma appears.

Presented by 15th Batch 292 FHCS | EUSL


• LH causes (Increasing) collagenase activity(Increasing) which digest the collagen surrounding the follicle.
• Increasing of prostaglandins cause local muscle contraction in the ovarian wall.
• It expel oocyte together with its surrounding granulosa cells from the
cumulus oophorus.
• When oocytes breaks free & floats out of the ovary is called ovulation
• Some of the cum. Ooph. Cells re-arrange to form corona radiata.
•After ovulatin granulosa cells remaining in the wall of ruptured follicle
with cells of theca interna are vascularised.
• LH induce these cells to develop a yellowish pigment & change into
lutean cells.
• They form corpus lutem & secrete oestrogen & progesterone.
•If fertilization doesn’t occur corpus luteum grows to the maximum
upto 9 days after ovulation.
• Then it shrinks & luteolysis occurs
• A mass of fibrotic scar tissue produced & termed as corpus albicans.
• Decreasing Progesterone and precipitating menstrual bleeding.

1.2)
*It provides consistency of the chromosome number from generation to generation.
*It allows random assortment of maternal and paternal chromosome between gametes.
*It relocates segments of maternal and paternal chromosomes by crossing over of chromosomes segments and
produce a recombination of genetic material.

1.3)
*intra embryonic mesoderm seperates the ecto & endoderm except prochordal plate, claocal membrane & the
place occupied by notochord.
At the edges of the disc it continues as extra embryonic mesoderm
IEM has 3 parts are,
1. Para axial mesoderm:
2. Lateral plate mesoderm:
3. Intermediate mesoderm:
in the 3rd week Paraxial mesoderm
- Organized in to segments; They somatomeres.
- Formation proceeds cephalocaudally.
- Somatomeres of head region associated with the segmentation of
neural plate called neuromeres, which forms the mesodermal cells
of head.
- In the occipital region somatomeres organised into somites.
• First pair of somite arise in the occipital region
• New somites arise in craniocaudal direction in a rate of 3pairs/per day.
• Formation starts around 20th day and ends in 5th week.
• A total of 42- 44 pairs get formed.
- occipital: 4
- cervical: 8
- thoracic: 12
- lumbar: 5
- sacral: 5
- coccygeal : 10
• 1st occipital and last 5- 7 coccygeal somites later disappear.
• Remaining somites forms the axial skeleton.
• As the somites develops in a specific period, the age of the embryo can
be accurately determined by counting the somites
Differentiation of somites
- Initially somites exist as ball of mesoderm cells.

Presented by 15th Batch 293 FHCS | EUSL


- Then undergoes epithelialization with a lumen (donut shape)
- Cells located in the ventral & medial walls of the somite become
mesenchymal and shift to surround the neural tube & notochord: become
sclerotomes and forms the vertebra & ribs
- Cells from the dorsomedial & ventrolateral edges of somite forms muscle precursor cells.
- Remaining cells between the above 2 forms the dermatome.
-Cells of muscle precursor cells again become mesenchyme & migrate
beneath dermatome to become dermomyotome.
• Some cells from ventrolateral edge migrate to parietal mesoderm &
forms most the musculature such as abdominis muscles & limb muscles
• Cells in the dermomyotome forms the dermis of back skin, muscles of

Presented by 15th Batch 294 FHCS | EUSL


back, body wall & some limb muscles.
•Each somite forms it own sclerotome, myotome and its own dermatome with their own segmental nerve supply.
•Therefore each myotome & dermatome retain its innervation from its segment of origin.

1.4)

•Gene Mutations refers to Change in the nucleotide sequence of a gene


• May only involve a single nucleotide
• May be due to copying errors, chemicals, viruses,
Types of Gene Mutations
• Include:
– Point Mutations
– Other mutations
1) Trinucleotide repeat expansion
2) Splice site mutations
3) Frameshift

Point Mutation
• Sickle Cell disease is the result of one nucleotide substitution
• Occurs in the hemoglobin gene
Point mutation can be seen in 3 types
1.Silent mutation
*No aminoacid change * redundancy in code
2.Missense
*change amino acid
3.Nonsense
*change to stop codon

Trinucleotide repeat expansion

*A sequence of 3 bases repeated in tandem will become amplified in number causing Too many copies of triplet
*if happen in coding region
The protein will have extra copies of amino acid
resulting in Trinucleotide repeat expansion
Eg: Huntington disease

Presented by 15th Batch 295 FHCS | EUSL


Splice site mutations
• A genetic alteration in the DNA sequence that occurs at the
boundary of an exon and an intron (splice site) Alter the way in which introns are removed from pre-MRNA
it Produce aberrant proteins
Eg: Myotonic dystrophy

Frameshift Mutation
• Inserting or deleting one or more
nucleotides
• Changes the “reading frame” like
changing a sentence
• Proteins built incorrectly bcz Amino Acid Sequence is Changed Frameshift Mutation
Eg:- Tay sachs disease

15TH BATCH (EME) - MCQ

1) Regarding Graafian follicle,


A. It is about 25mm in diameter
B. It ruptures due to FSH surge
C. Theca interna has the ability to secrete steroids
D. After the ovulation only, the granulosa cells change to become lutein cells
E. After ovulation, secondary oocyte surrounded by remnant of granulosa cells

2) During the blastocyst stage,


A. Cells in the embryoblast differentiate into epiblast and hypoblast
B. Dorsal to ventral polarity is established in the embryo
C. Embryonic axis is established at the time of implantation
D. Anterior visceral endoderm is identified from epiblast
E. Blastocoel is developed

3) Events that occur during 11th and 12th days of gestation include,
A. Large lacunae appears in syncytium
B. Blastocyst is completely embedded in endometrial stroma
C. Uteroplacental circulation is established
D. Exocoelomic cyst forms ventrally
E. Primary villi appeared among the cytotrophoblasts

4) Derivatives of lateral plate mesoderm include,


A. Dermis of the skin in the body wall
B. Sternum
C. Body wall muscles
D. Vertebral body
E. Cranial facial skeleton

Presented by 15th Batch 296 FHCS | EUSL


5) Regarding the external appearance of the embryo,
A. Pharyngeal arches are visible after the 4th week
B. Limbs are visible after the 2nd month
C. Somite’s are visible after the 7th week
D. Pericardial swelling is visible during the 5th week
E. Larger head is prominent during the 2nd month

6) Regarding the prenatal diagnosis techniques,


A. By doing ultrasonography, fetal age can be calculated
B. By performing amniocentesis chromosome abnormalities can be detected
C. Chorionic villi sampling done at late stage of pregnancy
D. Maternal alpha-fetoprotein test done in mother's urine sample
E. Fetal blood sampling may occasionally lead to fetal death

7) Turner’s syndrome,
A. is a trisomy
B. shows gonadal dysgenesis
C. Survive after few months of birth
D. Can be detected by cytogenetic techniques
E. Both sexes are affected

8) Following disorders match the karyotyping,


A. Down's syndrome- 47XY, +21
B. Klinefelter syndrome- 45, X0
C. Patau’s syndrome- 47XY, +13
D. Prader willi syndrome- 47XY, +18
E. Cri du chat syndrome- 47XY, -5

9) Which of the following abnormalities correctly paired with the mutation that can lead to,
A. Frameshift mutation – Tay Sachs disease
B. Missense - hemolytic anemia
C. Disjunction - hemophilia
D. RNA splicing disorders - Myotonic dystrophy
E. Trinucleotide repeat expansion- Huntington disease

10) Regarding DNA polymerase in gene expression,


A. RNA polymerase needs a primer to begin synthesis of mRNA in transcription process
B. It initiates the transcription by binding to the promoter region of DNA double helix
C. RNA polymerase reads the information sequence on DNA template from 3' 5' direction
D. Guanine nucleotide regulate G protein
E. It is vital for the attachment of corresponding amino acid in t-RNA during translation

SBR
11) What is the relationship among DNA, a gene & a chromosome?
A. a chromosome contains genes, which are composed of DNA
B. a chromosome contains hundreds of genes, which are composed of protein
C. a gene contains hundreds of chromosomes, which are composed of DNA
D. a gene contains hundreds of chromosomes , which are composed of protein
E. a gene is composed of DNA but there is no relationship to chromosome

Presented by 15th Batch 297 FHCS | EUSL


12) Which of the following statement is more suitable regarding Y chromosome?
A. Male gametes can have Y chromosome only
B. Inhibit formation of female genitalia
C. Hemophilia Gene in the long arm of Y chromosome
D. It has less coding sequence than X chromosome
E. It is submetacentric type of chromosome

13) Which of the following causes aneuploidy?


A. Disjunction in Meiosis
B. Mosaicism
C. Non-disjunction
D. Translocation
E. Deletion

14) Extra embryonic mesoderm develops from,


A. Decidua basalis
B. Cytotrophoblast
C. Syncytiotrophoblast
D. Amnioblast
E. Yolk Sac

15) High level of alpha- fetoprotein in maternal serum after 16 weeks is indicative of which following condition?
A. Trisomy 21
B. Marfan syndrome
C. Anencephaly
D. Monosomy x
E. Klinefelter syndrome

15TH BATCH (EME) - SEQ

1.
1.1. Explain briefly the changes that takes in the mesoderm located closely to the central axis and state its
derivatives. (30 marks)
1.2. Write accounts on
1.2.1. Autosomal recessive inheritance. (20 marks)
1.2.2. Amniotic fluid. (20 marks)
1.3. Briefly describe the structure and functions of chromatin in eukaryotic cells. (30 marks)

Answers

MCQ

A True Matured secondary follicle: Graafian follicle - 25mm or more in diameter


B False LH induces preovulatory growth phase and ovulation
C True Theca interna & granulosa cells produce oestrogens
D True After ovulatin granulosa cells remaining in the wall of ruptured follicle with
cells of theca interna are vascularised. LH induce these cells to develop a

Presented by 15th Batch 298 FHCS | EUSL


yellowish pigment & change into lutean cells.
E True oocyte expelled together with its surrounding granulosa cells from the
cumulus oophorus.

2.

True Cells of the inner cell mass or embryoblast also differentiate into two layers:
A (1) a layer of small cuboidal cells adjacent to the blastocyst cavity, known as
the hypoblast layer (2) a layer of high columnar cells adjacent to the amniotic
cavity, the epiblast layer
B Since a layer of epiblast dorsally and hypoblast ventrally is formed Dorso
ventral polarity is established during this stage.
C the cranial—caudal embryonic axis is established near the time of
implantation
D False hypoblast cells are specified to form the anterior Visceral endoderm (AVE)
E

3.

True lacunar spaces produced in synciotrophoblasts


A • Concurrently cells of the syncytiotrophoblast penetrate deeper into the
stroma & erode the epithelium of maternal capillaries
• Capillaries get congested & dilated to form Sinusoids
• It results in the continuous flow thro trophoblast system produce :
Uteroplacental circulation.
B True Blastocyst is completely embedded.
C True
D False On day 13 hypoblast produce new cells along the exocoelomic cavity.
• They proliferate & form a new cavity inside the exocoelomic cavity.
•New cavity is the 2ndry yolk sac/ definitive yolk sac
•Due to its formation, exocelomic cavity pinched off & forms exocoelomic
cysts
E False Primary villi appear on day 13

4.

True The parietal layer of lateral píate mesoderm then forms


A  dermis of the skin in the body wall and limbs,
B True  the bones and connective tissue of the limbs,
C True  the sternum
D False  sclerotome and muscle precursor cells that migrate into the parietal
E False layer of lateral plate mesoderm form the costal cartilages, limb
muscles, and most of the body wall muscles
 The visceral layer of lateral plate mesoderm, together with embryonic
endoderm, forms the wall of the gut tube

5.

True At the end of the fourth week, when the embryo has approximately 28
A somites, the main external features are the somites and pharyngeal arches
During 2nd month external appearance of embryo is changed
Presented by 15th Batch 299 FHCS | EUSL
B True • Head size
C False • Limbs, face, ears, nose & eyes formed
D
E True

6.

A True
B True
C CVS is usually carried out between the 11th and 14th weeks of
pregnancy, although it's sometimes performed later than this if
necessary.
D False The AFP blood test is called MSAFP (maternal serum AFP). The AFP can
also be measured in the amniotic fluid, called AFAFP
E True

7.

False It is a monosomy
A
B True characterized by the absence of ovaries (gonadal dysgenesis) and short
stature.
C False 55% die within 2 months 5% survive for beyond 1 year
D True
E False Only females

8.

True
A
B false 47, XXY
C True
D False An example of microdeletion
E False Deletion of short arm at 5th chromosome

9.

true
A
B True Change of amino acid occurs in sickle cell anemia. Sickle cell anemia leads to
hemolytic anemia.
C False Mutation cause by pathogenic variants or deltion
D
E true Huntington's disease is caused when the trinucleotide repeats in the
HTT gene expand beyond the normal range

10.

False In contrast to DNA polymerase, RNA polymerase does not need a primer to
Presented by 15th Batch 300 FHCS | EUSL
A begin synthesis
B True the transcription is initiated by the binding of RNA polymerase to a specific
region of DNA double helix. This site is called promoter site or promoter
region.
C True RNA polymerase will read the information sequence on DNA template
from 3′ → 5′ direction
D
E False Amino acyl tRNA synthetase Required for attachment of aa to their
corresponding tRNAs

11. A

12.

A
B In XXY they have female external genitals but no gonads.
C Gene for hemophila is located on the x chromosome
D True
E Y chromosome is acrocentric while x chromosome is submetacentric.

13. C Nondisjunction is the most common mechanism resulting in aneuploidy.

14. E cells, derived from yolk sac cells, form a fine, loose connective tis- sue, the extraembryonic
mesoderm
15. C

SEQ

1.1


The mesoderm beside the midline thickens to form the paraxial mesoderm. This is formed by the cells
migrating from the lateral edges of the primitive node. Initially the mesoderm exisits as thin line of cells on
eother side of the midline. By the 17th day it is thickened to form the paraxial mesoderm.
 At the beginning of 3rd week the paraxial mesoderm is organized into segments known a somatomeres
these appear cephalocaudally. In the head region these get associated with segments of the neural plate to
turn into neuromeres. This contributes to the formation of mesoderm of the head.
 In the occipital region the first somite appears at day 20. They appear at a rate of 3 per day for 15 days in
the cranio caudal direction. By the end of 4th week a total of 42-44 pairs get formed.
 occipital: 4 - cervical: 8 - thoracic: 12 - lumbar: 5 - sacral: 5 - coccygeal : 10
 Out of this the first occipital and the last 5-7 coccygeal disappear later.
 The remaining somites form the axial skeleton. They are used to determine age of the embryo.
Development of somites
 The somites initially occur as a ball of cells which then undergo epithelialization to forma lumen, The
 smites now have a donut shape.
 Cell located in the ventral & medial walls of the somite become mesenchymal and shift to surround the
 neural tube & notochord and become the sclerotome
 Cell in the dorsomedial and ventrolateral edges become muscle precursor cells.
 Cells between these 2 group of cells become the dermatome.
 Cells from the muscle precursors cells migrate beneath the dermatome to form the dermomyotome.
 Sclerotome forms the vertebra and ribs.
 Dermatome forms the dermis of the back.
 And myotome forms the segmental muscular compartment.
Presented by 15th Batch 301 FHCS | EUSL
 Each somite and subsequently its derivatives, retain their segmental innervation, giving rise to specific
dermatome and myotomes which are exclusively supplied by a specific spinal segment.
 Each dermatome and myotome has its own segmental nerve component, and retains it no matter where
the cells ultimately migrate.
 In addition to axial structures, the paraxial mesoderm gives rise to the muscles of the limbs.

1.21
Two copies of the gene must be mutated for a person to be affected by an autosomal recessive disorder.
• An affected person usually has unaffected parents who each carry a single copy of the mutated gene and are
referred to as "carriers".
• Each parent with a defective gene normally do not have symptoms.
• Two unaffected people who each carry one copy of the mutated gene have a 25% risk with each pregnancy of
having a child affected by the disorder

Special features
1. Carrier states are usually normal but may show half normal enzyme levels such as in hexosaminidase A levels in
Tay Sach disease.
2. Consanguinity increases the chance of affected births
3. Screening is needed in certain racial groups carry higher frequency of recessive genes.
4. The sickle cell carriers develop sickling in high altitudes
Eg:
Homocystinuria
Cystic fibrosis
Sickle cell anaemia
Thalassaemias

1.22

 The amniotic cavity is filled with a clear, watery fluid that is produced in part by amniotic cells but is derived
primarily from maternal blood.
 The amount of fluid increases from approxi- mately 30 mL at 10 weeks of gestation to 450 mL at 20 weeks
to 800 to 1,000 mL at 37 weeks.
 During the early months of pregnancy, the embryo is suspended by its umbilical cord in this fluid, which
serves as a protective cushion.
 The fluid
 (1) absorbs jolts,
 (2) prevents adherence of the embryo to the amnion, and
 (3) allows for fetal movements.
 The volume of amniotic fluid is replaced every 3 hours.
 From the beginning of the flfth month, the fetus swallows its own amniotic fluid, and it is estimated that it
drinks about 400 mL a day, about half of the total amount. Fetal uriñe is added daily to the amniotic fluid in
the flfth month, but this uriñe is mostly water because the placenta is functioning as an exchange for
metabolic wastes

1.3

The DNA is extensively folded & packed into a protein. This DNA coplex is what is known as chromatin

• Chromatin is the complex combination of DNA that makes up chromosomes (mesh work of thread)

• The major proteins involved in chromatin are histone proteins

• The functions of chromatin are

Presented by 15th Batch 302 FHCS | EUSL


1) Package DNA into smaller volume to fit in the cell
2) Strengthen the DNA to allow mitosis & meiosis
3) Serve as a mechanism to control gene expression & DNA replication
The association of DNA with histones to form nucleosomes affects the ability of the transcription machinery to
access the DNA to be transcribed.
Chromatin is of 2 types. Euchromatin and heterochromatin.
Euchromatin:
 - Contributes to major portion of body of chromosome -
 loosely coiled, -
 Is active -
 Composed of high proportion of genes which transcribe -
 Lightly stained
Heterochromatin
Heterochromatin appears as
 small,
 darkly staining,
 irregular particles scattered throughout the nucleus or accumulated adjacent to the nuclear envelope

16TH BATCH (EME) - MCQ


01. Capacitation process,
A. Involve epithelial interaction between sperm and the mucosal surface of fallopian tube
B. It’s a process of conditioning sperm inside the female reproductive tract
C. Is induced by androgens
D. Lasts approximately for 2 hours
E. Leads to acrosomal reaction

02. Preimplantation event includes


A. Compaction of blastomeres
B. Differentiation of cytotrophoblast and syncytiotrophoblast
C. Differentiation of hypoblast and epiblast
D. Formation of blastocoel
E. Formation of embryonic axis

03. Primitive streak


A. Cells synthesize FGF8
B. Involves in the movement of the cells
C. Clearly visible at 15th day
D. Primitive streak is immediately caudal to the prechordal plate
E. Is needed for the formation of extra embryonic mesoderm

04. Mature placenta barrier is formed by


A. Capillary endothelium of fetal vessels
B. Capillary endothelium of maternal vessels
C. Connective tissue
D. Cytotrophoblast
E. Syncytium

05. Congenital anomalies that commonly occur during the first trimester include
A. Cleft lip
Presented by 15th Batch 303 FHCS | EUSL
B. Cryptorchidism
C. Finger webbing
D. Neural tube defect
E. Sirenomelia

06. Changes taking place during spermiogenesis includes


A. Condensation of the nucleus
B. Acrosome formation
C. Formation of haploid number of chromosomes
D. Formation of pro nucleus
E. Shedding of the cytoplasm

07. The disease that follows the multifactorial include


A. Achondroplasia
B. Diabetes mellitus
C. Fragile X syndrome
D. Marfan syndrome
E. Schizophrenia

08. Following disorder match the karyotype


A. Angelman syndrome- 47XY +18
B. Down’s syndrome- Trisomy, 21
C. Klinefelter’s syndrome- 45XO
D. Pataus syndrome- XY, +13
E. Turner’s syndrome- 47XXY

09. Regarding umbilical cord,


A. Intestinal loops are located distally
B. Distally it contains yolk sac stalk
C. Proximally it contains allantois
D. Proximally it contains amniotic fluid
E. Proximally it contains umbilical vessels

10. Gene mutation includes the following type/s,


A. Frame shift
B. Inversion
C. Missense
D. Silent
E. Non sense

SBR
11. Which of the following genetic disorders show reduce penetrance/ variable expressivity?
A. Autosomal dominant
B. Autosomal recessive
C. X linked recessive
D. X linked recessive
E. Y linked dominant

12. Turner’s syndrome


A. Can’t be detect before birth

Presented by 15th Batch 304 FHCS | EUSL


B. Do not survive after few months of birth
C. Is a trisomy
D. Present in both sexes
E. Shows gonadal dysgenesis

13. On ultrasound examination, it was found that a fetus had well-developed facial and thoracic region, but caudal
structures were abnormal. Lumbar and sacral vertebrae were missing, and the hind limbs were fused. What process
may have been distributed to cause such defects?
A. Formation of fate map
B. Formation of lateral folding
C. Gastrulation
D. Formation of neural tube
E. Notochord formation

14. Folding of embryo results in the production of


A. Connecting stalk
B. Definitive yolk sac
C. Neural tube
D. Hind gut
E. Stomodeum

15. Component that is not needed for translation procedure


A. Amino acids
B. Acyl synthetase
C. Anticodon
D. Ligase
E. mRNA

16TH BATCH (EME) - SEQ

1.
1.1. Write accounts on 1.1.1. Structural chromosomal disorders. (25 marks)
1.1.2. Ultrasound scanning techniques (15 marks)
1.1.3. Neurulation & fate of neural crest cells. (30 marks)
1.2. Draw and label the structure of eukaryotic gene (15 marks)
1.3. Briefly explain the principles of regulation of gene expression. (15 marks)

01.
A. True Capacitation involves epithelial interaction between sperm and mucosal surface of the uterine tube.
B. True
C. False Not by androgens. It’s an effect of epithelial interaction which leads to removal of a glycoprotein coat
and seminal plasma proteins from plasma membrane that overlies the acrosomal region of sperm.
D. False. This conditioning process lasts approximately 7hours.
E. True. Only capacitated sperm can undergo the acrosomal reaction.

02. Implantation occurs in 6th day in blastocyst stage of embryo.


A. True
B. False In 8th day
Presented by 15th Batch 305 FHCS | EUSL
C. True. In early blastocyst stage, embryoblast cells deferentiate as hypoblast and epiblast. Then, near the 6th
day, it is arranged to establish dorsal-ventral axis.
D. True
E. True
(C and E have to be confirmed)

03.
A. True
B. True Invagination process
C. True. Initially, streak is vaguely defined. But, in 15to 16 day, it is clearly visible as a narrow
groove.
D. False. There is notochord between primitive streak and prechordal plate
E. True. Extra embryonic mesoderm has 2 sources. 1. Primitive yolk sac (hypoblast). 2. Epibalst cells migrating
through the caudal most part of primitive streak.

04. Initially placental membrane/barrier has 4 layer-: endothelial lining of fetal vassels, connective tissue of
villous core, cytotrophoblastic layer, syncytium.
From 4th month on, placental barrier(mature) has 2layers-: endothelial lining of fetal vassels and syncytium.
A. T
B. F
C. F
D. F
E. T

05.
A. T. 2to 3rd month of pregnancy
B. F. Testes are present in scrotum before birth in 97% male new born. In most of the remainders, descent will
be completed during first 3rd month of postnatal period.
C. T. Finger webbing/syntactyly
D. T defect in neural tube formation
E. T sirenomelia/caudal dysgenesis os caused by problems with gastrulation.

06.
A. T
B. T
C. F Haploid chromosomes are formed in spermatids. Then, spermatids are changed into spermatozoa by
spermiogenesis process
D. F After fertilization (page no. 42)
E. T

07.
A. F Autosomal dominant
B. T
C. F x linked
D. F Autosomal dominant

Presented by 15th Batch 306 FHCS | EUSL


E. T

08.
A. F It is one of microdeletion syndrome. Not chromosome disorder.
B. T
C. F. Klinfelter-47,XXY
D. T
E. F Turners- 45,XO

09.
A. F proximally
B. T
C. T remnant of alantois.
D. F. Amniotic fluid is not a content of umbilical cord since it is out of umbilical ring. E. T

10.
A. T
B. F chromosomal mutation
C. T
D. T
E. T

SBR
11. A
They are special features of autosomal dominant inheritance.
Reduced penetrance- It is a clinical term which is an expression of the number of individuals who have the gene
and show the trait. It is an all or nothing phenomenon and if the frequency is less than 100%, reduced
penetrance exists.
Variable expressivity – It refers to the degree of expression of a trait. The individuals in a family may show mild
to moderate to severe forms of the disease and need to be examined carefully in order not to miss the
diagnosis. Multiple Neurofibromatosis and Marfan syndrome are good examples of reduced penetrance and
variable expressivity.

12. E
Can detect by prenatal diagnosis such as amniocentesis.

13. C
Page no-67

14. D

15. D

SEQ

Presented by 15th Batch 307 FHCS | EUSL


1)
1.1.1
Structural chromosomal disorder

These include deletions, ring chromosomes, duplications, isochromosomes, inversions, and translocations. The
acentric, centric fragments and dicentric chromosomes which may arise are unstable at cell division. These are
lost and result in cell death. However, stable alterations are also seen.
Deletion: A deletion leads to a loss of chromatin. In terminal deletions the break is at the end of an arm. Cri du
chat syndrome has the tip of the short arm of chromosome No. 5 deleted (5p-). They have a low birth weight
and have failure to thrive. These children have a round face, low set ears, with profound learning disability,
hypertelorism [widely spaced eyes), epicanthic folds and have malformed larynxes which cause them to cry
with a sound like a cat. Cri du chat (CAT LIKE CRY). In interstitial deletions two breaks occur within the
chromosome arms, the broken segment is lost and the ends unite to form a short arm. In ring chromosome
disorders, two breaks occur at the tips of both arms of the chromosome, acentric pieces are lost from the ends,
and the two broken arms curl towards each other and fuse to form a ring.
In microdeletion syndromes, very small deletions take place and are detected only by special high resolution
banding. An example is Prader-Willi syndrome involving the long arm of chromosome 15. When the deletion is
in the maternal chromosome 15, Angelman syndrome results. This is an example of imprinting involving genes.
Duplication: It results in a gain of chromosome material by doubling, in a particular region.
Isochromosome: It is formed when a chromosome with two chromatids splits at right angles to the normal
lengthwise separation seen at cell division. The resulting chromosomes will have either both short arms or both
long arms. The net loss will be an entire long arm or an entire short arm. Twenty percent of Turner syndrome
results from having one normal X and another X which is an isochromosome for the long arm, essentially lacking
a short arm (short at monosomy and long arm trisomy). Most isochromosomes cause a spontaneous abortion.
Inversion: In this category chromosomes break at two points and the intervening broken segment turns round
180 degrees to reverse the order of chromatin. If the break points are on the same arm, it is called a paracentric
inversion and if it is on either side of the centromere, including it in the broke segment, it is referred to as a
pericentric inversion. The medical significance is that it leads to chromosomally unbalanced gametes
Translocation: A translocation involves the exchange of chromosomal material between chromosomes. There
are three types of these exchanges.
1. Centric fusion or Robertsonian translocation: This results from a fusion of whole arms of acrocentric
chromosomes. The breakpoints are at or near the centromeres of both chromosomes. The fused long arm
chromosome survives while the fused short arm chromosome is lost. This loss produces no effect as the short
arms of acrocentrics contain genetically inert material or RNA genes. Basically the chromosome complement
here will be 45 and the carrier will be quite healthy as these are all acrocentric chromosomes that carry
multiple copies of RNA genes in their short arms whose loss produces no serious consequences. Eg: 4% of Down
syndrome.
2. Reciprocal Translocation: This results from breakage and exchange of segments between chromosomes.
The points of exchange may be anywhere along the chromosome. The reciprocal translocations show exchange
of segments with no loss of chromosome material. Eg: In 90% of patients with chronic myeloid leukemia (CML),
and in 10-15% of acute lymphocytic leukemia (ALL), the bone marrow shows the presence of the Philadelphia
chromosome (Ph).
3. Insertional translocation: This involves the insertion of a deleted segment of a chromosome interstitially or
inside another chromosome following a break at that point. This is very rare but if present may produce
chromosomally unbalanced offspring.

1.1.2
Ultrasound scanning
Presented by 15th Batch 308 FHCS | EUSL
Ultrasonography is a relatively non invasive technique that uses high-frequency sound waves reflected from
tissues to create images. The approach may be trans abdominal or trans vaginal, with the latter producing
images with higher resolution. In fact, the technique, which was first developed in the 1950s, has advanced to a
degree whereby detection of blood flow in major vessels, movement of heart valves, and flow of fluid in the
trachea and bronchi are possible. The technique is safe and commonly used.Important parameters revealed by
ultrasound include:
• Characteristics of fetal age and growth
• Presence or absence of congenital anomalies
• Status of the uterine environment, including the amount of amniotic fluid. Placental position and
umbilical blood flow Whether multiple gestations are present.

All of these factors are then used to determine proper approaches for management of the pregnancy.
Determination of fetal age and growth is crucial in planning pregnancy management, especially for low-birth-
weight infants. In fact, Studies show that ultrasoundscreened and managed pregnancies with low-birth-weight
babies. Reduced mortality rate by 60% compared with an unscreened group. Fetal age and growth are assessed
by crown-rump
length during the 5th to the 10th weeks of gestation. After that, a combination of measurements—including the
Biparietal diameter (BPD) of the skull, femur length, and abdominal circumference—are used. Multiple
measurements of these parameters over time improve the ability to determine the extent of fetal growth.
Congenital malformations that can be determined by ultrasound include:
• The neural tube defects( anencephaly and spina bifida)
• Abdominal wall defects (omphalocele and gastroschisis)
• Heart and facial defects ( cleft lip and palate)

1.1.3
Neuralation and fate of neural crest cells.
Neuralation
At the beginning of the third week of development, the ectodermal germ layer has the shape of a disc that is
broader in the cephalic than in the caudal region. Appearance of the notochord and prechordal mesoderm
induces the overlying ectoderm to thicken and form the neural plate. Cells of the plate make up the
neuroectoderm, and their induction represents the initial event in the process of neurulation. Neurulation is the
process whereby the neural plate forms the neural tube. One of the key events in this process is lengthening of
the neural plate and body axis by the phenomenon of convergent extension, whereby there is a lateral to
medial movement of cells in the plane of the ectoderm and mesoderm. The process is regulated by signaling
through the planar cell polarity pathway and is essential for neural tube development. As the neural plate
lengthens, its lateral edges elevate to form neural folds,and the depressed midregion forms the neural groove.
Gradually, the neural folds approach each other in the midline, where they fuse. Fusion begins in the cervical
region (fifth somite) and proceeds cranially and caudally. As a result, the neural tube is formed. Until fusion is
complete, the cephalic and caudal ends of the neural tube communicate with the amniotic cavity by way of the
anterior (cranial) and posterior (caudal) neuropores, respectively. Closure of the cranial neuropore occurs at
approximately day 25 (18- to 20somite stage), whereas the posterior neuropore closes at day 28 (25-somite
stage). Neurulation is then complete, and the central nervous system is represented by a closed tubular
structure with a narrow caudal portion, the spinal cord, and a much broader cephalic portion characterized by a
number of dilations, the brain vesicles.

Fate of Neural Crest Cells


As the neural folds elevate and fuse, cells at the lateral border or crest of the neuroectoderm begin to dissociate
from their neighbors. This cell dopulation, the neural crest cells undergoes an epithelial-to-mesenchymal

Presented by 15th Batch 309 FHCS | EUSL


transition as it leaves the neuroectoderm by active migration and displacement to enter the underlying
mesoderm.
• Crest cells from the trunk region leave the neuroectoderm after closure of the neural tube and migrate
along one of two pathways:
(1) A dorsal pathway through the dermis, where They will enter the ectoderm through holes in the basal
lamina to form melanocytes in the Skin and hair follicles.
(2) A ventral pathway through the anterior half of each somite to become sensory ganglia, sympathetic and
enteric neurons, Schwann cells, and cells of the adrenal medulla.
• NCC also form and migrate from cranial neural folds, leaving the neural tube before closure in this region.
These cells contribute to the craniofacial skeleton as well as neurons for cranial ganglia, glial cells,melanocytes,
and other cell types.

1.2

Presented by 15th Batch 310 FHCS | EUSL


Draw and label the eukayotic gene.

1.3
Briefly explain the principles of regulation of gene expression.
Gene expression is the information from a gene, is used in the synthesis of a functional gene product. These
products are often proteins, but in non- protein coding genes such as rRNA genes or tRNA genes, the product is
a functional RNA. The genes can be turned on and off as a result of interaction with regulatory proteins. The
expression of these genes can be controlled in many steps,
1) Transcriptional control
2) RNA processing control 3) RNA transport control 4) Translational control.

Feedback inhibition: When enough amount of product is made, the system will shuts down.
Gene expression: Genes are only expressed when needed. This is often regulated at transcription.

Presented by 15th Batch 311 FHCS | EUSL


Presented by 15th Batch 312 FHCS | EUSL
Phase 01 QUESTIONS

8TH BATCH
8TH PROPER MCQ

01) Regards to teratogens


A) Susceptibility to a teratogen is influenced by genetic factor
B) Act on the fetus by influencing metabolic process
C) Exposure to diagnostic radiological procedures produce permanent mutation in genes of gametes
D) Major psychiatric drugs taken during early months of pregnancy cause congenital malformation
E) Rh positive mother produce antibodies for action in hemolytic jaundice

02) Matured placental barriers


A) Syncitiotropoblast membrane
B) Connective tissues
C) Capillary endothelium of maternal blood vessels
D) Capillary end of fetal blood vessels
E) Cytotropoblast

SBR
03) Which embryonic tissue is correctly match to its ultimate derivative?
A) Epiblast- connective tissue
B) Endocyte- connective tissue
C) Hypoblast- Endoderm
D) Mesoderm- Epithelium of urinary tract
E) Ectoderm- Epithelium of nasal cavity

04) Which of the following is an autosomal dominant inheritant?


A) Cystic fibrosis
B) Sickle cell anemia
C) Marfan syndrome
D) Homocystine urea
E) Thalassemia

05) What is the structure remain active in umbilical cord?


A) Viteline duct
B) Allantois
C) Intestinal loops
D) Umbilical vessels
E) Amnion

06) Notochord
A) Is the early umbilical cord
B) Is precursor of vertebral column
C) lies dorsal to the spinal cord
D) Remains in the adults as nucleus pulposes
E) Formed by hypoblast

Presented by 15th Batch 313 FHCS | EUSL


8TH PROPER SEQ

3.
3.1. Define the embryonic period (05 marks)
3.2. List 2 major events that take place immediately after the implantation (05 marks)
3.3. Describe the process of one of the event stated for Q 3.2 (30 marks)
3.4. State the effects of teratogens by giving 5 examples (20 marks)
3.5. Describe the formation of placenta (40 marks) (4.0.)

4.
4.1. List 5 prenatal diagnostic techniques that are performed (15 marks)
4.2. Write an account of a non-invasive prenatal diagnostic technique which is used frequently (35 marks)
4.3. Give a brief account on
4.3.1. Karyotyping (25 marks)
4.3.2. Downs syndrome

Answers

MCQ

01)
A) F
Teratogen is an agent that can produce a permanent alteration of structure or function in an organism after
exposure during embryonic or fetal life.
B)
C) T A mutagen is an agent that can alter the structure of a chromosome or the DNA it carries. Unlike teratogens
they may affect at any stage of life.
D) T
E) F Rh- negative mothers.

02)
A) T
Mature placental barrier consists only of
1. Syncytium
2. Endothelium
B) F
C) T
D) F
E) F

SBR
03) ANSWER – E
Sensory epithelium of ear, nose and eye is derived from the ectoderm.

04) ANSWER – B
Cystic fibrosis – autosomal recessive
Sickle cell anemia – autosomal recessive
Marfan syndrome - autosomal dominant
Homocystein urea – autosomal recessive
Thalassemia –autosomal recessive

Presented by 15th Batch 314 FHCS | EUSL


05)ANSWER - D

06)ANSWER - D

SEQ-PROPER

2.0
.2.1.
2.1.1. Define the term “Monosomy” in chromosomal abnormality (05 marks)
Presence of only one member of a chromosome pair
.(ex-turners’ syndrome)

2.1.2. Write a brief account on Turners’ syndrome (30 marks)

Short stature Increased carrying angle


Absence of ovaries.
Hyper convex nails
Webbed neck.
Infertility
Oedema of hands and feet.
Low posteror hair line
Skeletal deformities.
Coarction of aorta
Widely space nipples.
Broad chest
Hypoplastic or inverted nipples.

2.2.
An affected son was born to a normal male & a heterozygous female. The affected trait was linked to the sex
chromosome.
2.2.1.
State the Mendelian pattern of the above condition. (10 marks)
X linked dominant inheritance

2.2.2.
Describe the features of the above inheritance pattern by giving 3 examples. (30 marks)

inactivation of one x and severely affected when both x chromosomes


carry it.

gene to all their daughters.

their daughters.
Presented by 15th Batch 315 FHCS | EUSL
rome and
incontinentia pigment.
Affected heterozygous female and normal male,

2.2.3. Write a short account on mitochondrial inheritance (20 marks)

can transmit the condition to both sexes but males do not


transmit to next generation.

systems like central nervous system are rich in mitochondria.


disorders are cardio myopathies, myopathies

4.0.
4.1. Outline the process of spermatogenesis
(30 marks)

erty.

Presented by 15th Batch 316 FHCS | EUSL


4.2. Explain how the oogenesis differs from spermatogenesis
(20 marks)

4.3. Write a brief account on


4.3.1. Neurulation (25 marks)
Neurulation is the process whereby the neural plate forms the neural tube.
One ofthe key events in this process is lengthening of the neural plate and body axis by the phenomenon of
convergent extension, whereby there is a lateral to medial movement of cells in the plane of the ectoderm and
mesoderm.
The process is regulated by signaling through the planar cell polarity pathway and is essential for neural tube
development. As the neural plate lengthens, its lateral edges elevate to form neural folds, and the depressed
Presented by 15th Batch 317 FHCS | EUSL
midregion forms the neural groove Gradually, the neural folds approach each other in the midline, where they fuse
Fusion begins in the cervical region (fifth somite) and proceeds cranially and caudally
As a result, the neural tube is formed.
Until fusion is complete, the cephalic and caudal ends of the neural tube communicate with the amniotic cavity by
way of the anterior (cranial) and posterior (caudal) neuropores, respectively .
Closure of the cranial neuropore occurs at approximately day 25 (18- to 20-somite stage), whereas the posterior
neuropore closes at day 28 (25-somite stage) (Fig. 6.4B).
Neurulation is then complete, and the central nervous system is represented by a closed tubular structure with a
narrow caudal portion, the spinal cord, and a much broader cephalic portion characterized by a number of dilations,
the brain vesicles

4.3.2. Teratogenesis and mutaenesis (25 marks)

• Both teratogens and mutagens cause alteration in the structure an functioning of the body, but the mechanisms
differ.
• Teratogens cause damage by altering the embryonic/ fetal development directly.
• Mutagens in contrast cause changes within the genetic material, restriction of growth or death of the embryo
/fetus.
➢It may also lead to inherited diseases if the germ cells are affected
➢Lead to cancer if somatic cells are involved.

SEQ-REPEAT

3.
3.1 Define the embryonic period (5)

3..2 list 2 major events that take place immediately after implantation
(5)
entiate

Blastodisk development

3.3. Describe the process of one of the event stated for Q 3.2

In the area over the embryoblast, the trophoblast has differentiated into two layers:
(1) an inner layer of mononucleated cells, the cytotrophoblast, and
(2) an outer multinucleated zone without distinct cell boundaries, the syncytiotrophoblast.
Mitotic figures are found in the cytotrophoblast but not in the syncytiotrophoblast.
Thus, cells in the cytotrophoblast divide and migrate into the syncytiotrophoblast, where they fuse and lose their
individual cell membranes.
The trophoblast shows considerable progress in development, particularly at the embryonic pole, where vacuoles
appear in the syncytium.
Presented by 15th Batch 318 FHCS | EUSL
When these vacuoles fuse, they form largelacunae, and this phase of trophoblast development is thus known as
the lacunar stage.

3.4. State the effects of teratogens by giving 5 examples

Presented by 15th Batch 319 FHCS | EUSL


3.5. Describe the formation of placenta
The fetal component of the placenta is derived from the trophoblast and extraembryonic mesoderm (the chorionic
plate); the maternal component is derived from the uterine endometrium.
By the beginning of the second month, the trophoblast is characterized by a great number of secondary and tertiary
villi, which give it a radial appearance.
Stem (anchoring) villi extend from the mesoderm ofthe chorionic plate to the cytotrophoblast shell.
The surface of the villi is formed by the syncytium, resting on a layer of cytotrophoblastic cells that in turn cover a
core of vascular mesoderm.
The capillary system developing in the core of the villous stems soon comes in contact with capillaries ofthe
chorionic plate and connecting stalk, thus giving rise to the extraembryonic vascular system.
Maternal blood is delivered to the placenta by spiral arteries in the uterus.
Erosion of these maternal vessels to release blood into intervillous spaces is accomplished by endovascular invasion
by cytotrophoblast cells.
These cells, released from the ends of anchoring villi, invade the terminal ends of spiral arteries, where they replace
maternal endothelial cells in the vessels’ walls, creating hybrid vessels containing both fetal and maternal cells.
To accomplish this process, cytotrophoblast cells undergo an epithelial-to-endothelial transition.
Invasion of the spiral arteries by cytotrophoblast cells transforms these vessels from small-diameter, high-
resistance vessels to larger diameter, low-resistance vessels that can provide increased quantities of maternal blood
to intervillous spaces.
During the following months, numerous small extensions grow out from existing stemvilli and extend as free villi
into the surrounding lacunar or intervillous spaces.
Initially, these newly formed free villi are primitive, but by the beginning of the fourth month, cytotrophoblastic
cells and some connective tissue cells disappear.
The syncytium and endothelial wall of the blood vessels are then the only layers that separate the maternal and
fetal circulations.
Frequently, the syncytium becomes very thin, and large pieces containing several nuclei may break off and drop
into the intervillous blood lakes.

Presented by 15th Batch 320 FHCS | EUSL


These pieces, known as syncytial knots, enter the maternal circulation and usually degenerate without causing any
symptoms.
Disappearance of cytotrophoblastic cells progresses from the smaller to larger villi, and although some always
persist in large villi, they do not participate in the exchange between the two circulations.
In the early weeks of development, villi cover the entire surface of the chorion.
As pregnancy advances, villi on the embryonic pole continue to grow and expand, giving rise to the chorion
frondosum (bushy chorion).
Villi on the abembryonic pole degenerate, and by the third month, this side of the chorion, now known as the
chorion laeve, is smooth.
The difference between the embryonic and abembryonic poles of the chorion is also reflected in the structure of
the decidua, the functional layer of the endometrium,
The decidua over the chorion frondosum, the decidua basalis, consists of a compact layer of large cells, decidual
cells, with abundant amounts of lipids and glycogen.
This layer, the decidual plate, is tightly connected to the chorion. The decidual layer over the abembryonic pole is
the decidua capsularis.
With growth of the chorionic vesicle, this layer becomes stretched
and degenerates. Subsequently, the chorion laeve comes into contact with the uterine wall (decidua parietalis) on
the opposite side of the uterus, and the two fuse, obliterating the uterine lumen.
Hence, the only portion of the chorion participating in the exchange process is the chorion frondosum, which,
together with the decidua basalis, makes up the placenta.

4.0.
4.1. List 5 prenatal diagnostic techniques that are performed
1. Amniocentesis
2. Chorionic villi sampling
3. Cordocentesis
4. Fetoscopy
5. Ultrasonography

4.2. Write an account of a non-invasive prenatal diagnostic technique which is used frequently

• Ultrasonography
Ultrasonography is a relatively noninvasive technique that uses high-frequency sound waves reflected from tissues
to create images.
The approach may be transabdominal or transvaginal, with the latter producing images with higher resolution .
In fact, the technique has advanced to a degree whereby detection of blood flow in major vessels, movement of
heart valves, and flow of fluid in the trachea and bronchi are possible. Important parameters revealed by
ultrasound include
characteristics of fetal age and growth;
presence or absence of congenital anomalies;
status of the uterine environment, including the amount of amniotic fluid ;
placental position and umbilical blood flow;
and whether multiple gestations are present .
All of these factors are then used to determine proper approaches for management of the pregnancy.
Fetal age and growth are assessed by crown-rump length during the 5th to the 10th weeks of gestation. After that,
combination of measurements—including the biparietal diameter (BPD) of the skull, femur length, and abdominal
circumference—are used .
Multiple measurements of these parameters over time improve the ability to determine the extent of fetal growth.
Congenital malformations that can be determined by ultrasound include the
neural tube defects anencephaly and spina bifida
abdominal wall defects, such as omphalocele and gastroschisis
heart and facial defects, including cleft lip and palate
Ultrasound can also be used to screen for Down syndrome and some other chromosome related abnormalities
through a test called nuchal translucency.
Presented by 15th Batch 321 FHCS | EUSL
The test is performed at 11 to 14 weeks of pregnancy.
Information from this test, combined with maternal serum screening test results and the mother’s age, can be
combined to provide a risk estimate for Down syndrome.

4.3. Give a brief account on


4.3.1. Karyotyping (25 marks)
Chromosome cultures are performed in rapidly dividing cells from the metaphase stage of mitosis.
• Eg. Lymphocytes, fibroblasts, cells from bone marrow biopsy.
• Fetal cells from amniocentosis, chorionic villi sampling, fetal blood cells from cordocentosis etc

• G banding appearance seen in Giemsa staining


• The good spreads are captured using photomicroscope.
• Individual chromosomes cut and arranged in pairs in
decreasing order of size and numbered from 1 to 22 and
sex chromosomes as separately
• The photographic representation of entire somatic
chromosome complement : KARYOTYPE
• Conventional notation is “46,XY” or “46,XX”
• G(iemsa)-banding, 500 bands per haploid recognizable
• Short (“p”-etit) arm = p, other (long) arm = q
Chromosome analysis/ Karyotyping

4.3.2. Downs syndrome

Down syndrome is caused by an extra copy of chromosome 21 [trisomy 21].


Features of children with Down syndrome include
growth retardation;
varying degrees of intellectual disability;
craniofacial abnormalities, including upward slanting eyes, epicanthal folds [extra skin folds at the medial corners
of the eyes], flat facies, and small ears;
cardiac defects;
Hypotonia

Presented by 15th Batch 322 FHCS | EUSL


These individuals also have an increased chance of developing leukemia, infections, thyroid dysfunction, and
premature aging
Furthermore, an increased frequency and earlier onset of Alzheimer disease is observed among persons with Down
syndrome.
In 95% of cases, the syndrome is caused by trisomy 21 resulting from meiotic nondisjunction, and in 75% of these
instances, nondisjunction occurs during oocyte formation.
In approximately 4% of cases of Down syndrome, there is an unbalanced translocation between chromosome 21
and chromosomes 13,14, 15, or 21
The final 1%is caused by mosaicism resulting from a trisomic conception followed by the loss of the extra
chromosome in some cells during mitosis.
These individuals have mosaicism, with some cells having a normal chromosome number and some having trisomy.
They may exhibit few or many of the characteristics of DownSyndrome.

9TH BATCH
9TH PROPER MCQ

01) Special features of autosomal dominant disease are


A) Clinical features appear at birth
B) Variation of age in onset
C) New mutation
D) Variable expressivity in a family
E) Variation of severity by sex

02) Day and appropriate events of the maturating embryo are correctly matched
A) Day 8 – differentiation of inner cell mass into 2 layers
B) Day 9 – formation of primitive yalk sac
C) Day 13 – formation of secondary villi
D) Day 11 – blastocyst completely embedded in the endometrial stroma
E) Day 16 – chorionic cavity

03) Oogenesis
A) Is started before birth
B) Primary oocyte may get mutation with age
C) Preovulatory phase is stimulated by LH
D) Granulosa cells contribute to produce zona pellucida
E) 2nd meiotic division finished immediately before ovulation

04) Derivatives of neural crest cells


A) Adrenal medulla
B) Craniofacial skeleton
C) Sympathetic nerves
D) Melanocytes
E) Spinal ganglia
05) Regarding mutations
A) Can alter the structure and functions of chromosomes
B) Rise due to exposure of radiations and chemicals
C) Make change in DNA that always harmful
D) Can result due to electron of base
E) Make insertions that are inheritable

Presented by 15th Batch 323 FHCS | EUSL


06) Primitive streak
A) Gastrulation begins with the formation of Primitive streak
B) Give rises neuroectoderm
C) Clearly visible during the 16 day development
D) Caudal end of the streak is the Primitive node
E) Secrete FGF 8

SBR
07) Fusion of male and female gemmates is called fertilization. What happened immediately after fertilization?
A) Formation of pronucleus
B) Restoration of diploid number of chromosomes
C) Second meiotic division of oocyte
D) Restriction reactions around zona pellucida
E) Degeneration of tail of spermatozoa

08) Turner syndrome


A) Is a trisomy
B) Gonad disgenesis
C) Survive few month after birth
D) Cannot be detected of birth
E) Present in both sex

09) The structure of umbilical cord that is remaining active in late pregnancy
A) Allantois
B) Vitelline vessels
C) Yolk sac
D) Umbilical vessels
E) Vitelline duct

9TH PROPER SEQ

4.
4.1. A young woman who has missed two menstrual periods complains of intense abdominal pain. She was
suspected of having ectopic pregnancy & thereafter admitted to the hospital for further investigations.
4.1.1. Explain the term “Ectopic Pregnancy” & list 3 common sites where it can occur (15 marks)
4.1.2. Describe the sequence of events during fertilization (35 marks)
4.2.
4.2.1. Describe the formation of placenta (35 marks)
4.2.2. Draw & label the structure of fully developed placenta (25 marks)

Answers

MCQ PROPER
01)
Special features of autosomal dominant
- new mutation - Variable expressively - variation in age on set - reduce penetrance - variation in severity
dependent on sex

Presented by 15th Batch 324 FHCS | EUSL


A) F
B) T
C) T
D) T
E) T

02)
A) T
B) T
C) F 3 rd week
D) T
E) F in 11th and 12th days

O3)
A) T
B) T
C) F By FSH
D) T
E) F Immediately after fertilization.

04)
A) T Langman’s pg. 69 – table 6.1
B) T
C) T
D) T
E) T

05)
A) T
B) T
C) F
D) T
E) F

06)
A) T
B) T By forward growth from primitive streak
C) T In 3rd week of development
D) F Not caudal its cranial end
E) T Fibroblast growth factor 8

07)
ANSWER- B
3 main results of fertilization;
1. Restoration of the diploid number of chromosomes.
2. Determination of the sex of the new individual.
3. Initiation of cleavage.

08)
ANSWER – B Biconcave shaped

09)
ANSWER-D

Presented by 15th Batch 325 FHCS | EUSL


SEQ-PROPER
04)
4.1)
4.1.1)

endometrium of corpus uteri. This is called as ectopregnancy.

1. Uterine tubes
2. Ovary
3. Abdominal cavity

4.1.2)
tes
fuse, occurs in the ampullary region of the uterine tube.

They go through the uterus & reach ampullary region of uterine


tube.
otile.

chemotactants produced by cumulus cells of egg.

1. Capacitation
-Period of conditioning
-Takes approximately 7 hours.
-Removal of glycoprotein & seminal plasma membrane that overlies acrosomal region of spermatozoa.
2. Acrosome reaction
-Occurs after binding to the zonapellucida.
-Release enzymes
which are needed to penetration of egg.

1. Penetration of the corona radiata.


2. Penetration of the zona pellucida.
3. Fusion of oocyte & sperm cell membranes.

1. Penetration of the corona radiata.

only one sperm can fertilize the ovum.

2. Penetration of the zona pellucida.

facilitate & maintains sperm binding & also induces


the acrosomal reactions.
Head of the sperm touch the zona pellucida.
Acrosome releases enzymes such as trypsin, acrosin
Enzymes allows sperms to penetrate the zona pellucida
Lysosomal enzymes are released by cortical granule
Lysosomal enzymes changes permeability of zona pellucida & become impermeable.

3. Fusion of the oocyte & sperm cell membrane,

eft behind the oocyte


surface.
Presented by 15th Batch 326 FHCS | EUSL
anged in a vesicular nucleus known as female pronucleus.

r nuclear envelops making a zygote.

tromere & the sister chromosomes move


to opposite poles providing each cell of the zygote with the normal diploid number of chromosomes.

,
1. Restoration of the diploid number of chromosomes.
2. Determination of sex of new individual
(“X” carrying sperms produce female baby & “Y” carrying sperms produce male baby)
3. Initiation of cleavage.

4.2)
4.2.1)
acilitates nutrient & gases exchange between the maternal & fetal compartments.

as a great number of secondary & tertiar villi which give a radial


appearance.
Primary villi appears at 13th day which are cellular columns with the syncytial coverings.

blast shell & they have syncytiotropnoblast covering.

uteroplacental circulation.
the endometrium.

ow resistance vessels.

thin & large pieces containing several nuclei breakdown & drop into intervillous blood
lake.

le continue to grow & expand giving rise to the chorionic frondosum.

um & desidua basalis which is over the chorionic frondosum together form the placenta

4.2.2

Presented by 15th Batch 327 FHCS | EUSL


9th SEQ-REPEAT
04.
4.1 Describe the important features of oogenesis.
(25 marks)

Process of the differentiation of oogonia into mature oocytes.


Once primordial germ cells arrived in gonads of a genetic female, they
differentiate into oogonia.

3 rd month, they are arrange in clusters surrounded by a layer of flat


epithelial cells known as follicular cells.

arrest their cell division in prophase in miosis I and form primary


oocytes.

rimary oocytes have entered prophase of meiosis i.

known as a primodial follicle.

meiosis 1. But instead of proceeding into metaphase , they enter the


diplotene stage.

first meiotic division, before puberty is reached.


uosly
maintained from the supply of primordial follicles.

immediately prior to ovulation, follicles are quite swollen and are called
graffian follicles.

Presented by 15th Batch 328 FHCS | EUSL


4.2 Explain the proess of fertilization & implantation
(45 marks)

4.2
The process by which the male and female gametes unite to give rise to
zygote which occurs in the ampullary region of the fallopian tube.
Ampullary region is the widest part of the tube & is close to the ovary.
Only 1% of sperm deposited in the vagina enter the cervix sperm become
less motile once they reach the isthmus .At ovulation sperm again

Presented by 15th Batch 329 FHCS | EUSL


become motile & swim to the ampulla due to chemoattractions produced
by cumulus cells.
Sperm under go capacitation and acrosomal reaction before fertilization
200-300 million sperms deposited in vagina
Only 300-500 sperms enter the fertilization site
Only one sperm fertilize the egg
Other aid the single sperm penetrating the barriers of the ovum.
There are 03 phases of fertilization.
Phase 1 – penetration of the corona radiata.
Capacitation process takes place.
It’s the conditioning in the uterine tube
Last for 7 hrs
Epithelial interactions occurs between sperm and mucosal surface of the
tube
Glycoprotein coat and the seminal plasma protein removed from the
plasma membrane that overlies the acrosomal cap.
Now the capacitated sperm can freely pass through the corona cells of
ovum.
Phase 02 – penetration of zona pellucida
Acrosome contain enzymes to old the sperm to penetrate the zona
pellucida
Release of acrosin enzyme allows the sperm to penetrate the zona by
digestion
Then it can contact with plasma membrane of oocyte.
Zona reaction – once the head of head of sperm touch the plasma
membrane. Of oocyte it result in release of lysosomal enzymes from the 115
cortical granules lining the plasma membrane .These enzymes alter the
properties of zona and prevent sperm penetration by inactirating the
species specific receptor site
Phase 03 – Fusion of the oocyte and sperm sell membrane
Plasma membranes of sperm and oocyte fuse
Both head and tail of spermatozoan enters the egg
Cortical and zona reactions prevent polyspermy
Oocyte finishes the 2nd meiotic division after entering of spermatozoa
and produce 2nd polar body and definite oocyte .
23 chromosomes arrange them selves to produce female pronucleus
Metabolic activation of the egg vegins for the early embryogenesis
Once the male nucleus reach and lies closer to the female
Pronucleus it swells to from male pronucleus
Their nuclear membrane disappear
Chromosomes become distract
Replication of DNA takes place in each 23 set
Each 46 chromosomes split into 2 gaps and more of each end of the
spindle

4.3
Write a brief account on Down,s syndrome.
(30 marks)
Down syndrome is caused by an extra copy of chromosome 21 [trisomy 21]. Features of children with Down
syndrome include growth retardation; varying degrees of intellectual disability;
craniofacial abnormalities, including upward slanting eyes, epicanthal folds [extra skin folds at the medial corners
of the eyes], flat facies, and small ears;
cardiac defects;
Hypotonia
These individuals also have an increased chance of developing leukemia, infections, thyroid dysfunction, and
premature aging
Furthermore, an increased frequency and earlier onset of Alzheimer disease is observed among persons with Down
syndrome.

Presented by 15th Batch 330 FHCS | EUSL


In 95% of cases, the syndrome is caused by trisomy 21 resulting from meiotic nondisjunction, and in 75% of these
instances, nondisjunction occurs during oocyte formation.
In approximately 4% of cases of Down syndrome, there is an unbalanced translocation between chromosome 21
and chromosomes 13,14, 15, or 21
The final 1%is caused by mosaicism resulting from a trisomic conception followed by the loss of the extra
chromosome in some cells during mitosis.
These individuals have mosaicism, with some cells having a normal chromosome number and some having trisomy.
They may exhibit few or many of the characteristics of Down syndrome.

10TH PROPER MCQ

26) Oogenesis
A) Corona radiata formed from granulosa cells
B) Diplotene stage is most suitable to protect oocytes
C) Oocyte complete second meiotic division at fertilization
D) Primordial follicle formed during prenatal life
E) Primary oocyte is vulnerable to damage as they advance in age

27) Abnormal karyotype


A) Cri du chat
B) Fragile x syndrome
C) Duchenne muscular syndrome
D) Turner syndrome
E) Jacob syndrome

28) Events during second week of development


A) Establishment of uteroplacental circulation
B) Formation of chorionic cavity
C) Formation of exocoelamic cavity
D) Formation of trilaminar germ disc
E) Formation of notochord

29) Regarding somites and somatomeres


A) Derived from neural crest cells
B) Facial skeleton develops from it
C) Somites only form the head region
D) The vertebrae originate from them
E) They form the segmental development of the body

30) Human placenta


A) Functions as an absolute barrier between maternal and fetal circulation
B) Is of a hemochonial type
C) Is separated after delivery
D) Undergoes changes during end of the pregnancy
E) Weights about 200g after maturation

31) Regarding early week of development


A) Implantation occurs by the 5th week
Presented by 15th Batch 331 FHCS | EUSL
B) Chorionic villi develop both from cytotrophoblast and syncytiotrophoblast
C) Trophoblast has haploid number of chromosomes
D) All three cell layers derived from hypoblast
E) Neural crest contributes to form dorsal root ganglia 7

32) X linked recessive diseases are


A) Hemophilia A
B) Duchenne muscular dystrophy
C) Cystic fibrosis
D) Cri-du-chat syndrome
E) Thalassemia

33) Klinefelter syndrome


A) Individual has cardiovascular deformities
B) Example for monosomy
C) Individual has large teeth
D) Seen in males
E) Osteoarthritis can seen

SBR
58) What is the most appropriate statement for notochord?
A) It consist of ectodermal cells
B) It develops during 2nd week
C) It is established out tail region
D) It is the precursor of spinal cord
E) It serve as the axial skeleton

59) Outcome of the flexion of the embryo


A) Formation of chorion
B) Formation of hindgut
C) Formation of extra embryonic mesoderm
D) Formation of secondary yolk sac
E) Formation of connecting stalk

60) Best statement about mitochondrial DNA


A) Contain over 100 coding sequence
B) Uses triplet TAG to code tryptophan
C) Double strand circular DNA
D) Inherited from parents
E) Has high percentage of introns

10TH PROPER SEQ

04.
4.1 Caudal dysgenesis (sirenomelia) is one of the major effect caused by teratogens during the 3rd week of
development.
4.1.1 List the sequence of events takes place during the 3rd week of development. (10 marks)
4.1.2 Describe the most characteristic event occurring at the beginning of the third week of gestation. (30
marks)
Presented by 15th Batch 332 FHCS | EUSL
4.2 Write an account on
4.2.1. Oogenesis (25 marks)
4.2.2. Turner’s syndrome (20 marks)
4.2.3. Sex determination (15 marks

Answers

MCQ-PROPER
26)
A) T Langman pg. 23 fig 2.19

- Granulosa cells surrounding the oocyte remain intact and from cumulus oophorus. -Corona radiate is the
innermost layer of cumulus Oophorus and is directly adjust to zona pellucida.
B) T
whether the diplotene stage is the most suitable phase to protect the oocyte against environmental influences is
unknown.
C) T
oocyte finishes its second meiotic division immediately after the entry of spermatozoon.
D) T
at birth – primary oocyte is formed.
E) T

27)
Abnormal karyotype are seen in chromosomal abnormalities.
Abnormal number or abnormal structure.
A) T deletion of short arm of chromosome 5 – structural abnormality
B) F X - linked recessive.
C) F X - linked recessive.
D) T 45, X monosomy
Numerical abnormalities
E) T 47, XYY trisomy

28)
A) T
Day 11 & 12
- Capillaries get congested and dilated to form sinusoids.
_ results in establishment of uteroplacental circulation.
B) T
Day 13
Extraemryonic coelom—chorionic cavity.
C) T
day 12
D) F
Presented by 15th Batch 333 FHCS | EUSL
third week.
E) F
third week.

29)
A) F
Somatomeres = organized segments of Paraxial Mesoderm.
B) F

Cells
C) F
Paraxial Mesodermal segments 02.
1. Somatomeres - Head region
2. Somites – Occipital region caudally.
D) T
Somites
Sclerotome cells
Vertebrae
E) T

30)
A) F
though it’s a barrier not an absolute barrier.it facilitate gas and
nutrient exchange between maternal and fetal compartments.
B) T
because the maternal blood in the intervillous space is separated
from fetal blood by a chorionic derivative, the placenta is called
hemochorial type.
C) T
approximately 30 min after delivery.
D) T
1 In the core of the villous.
2. Thickening of basement membranes in fetal capillaries.
3. Obliterative changes in small capillaries of the villi.
4. Deposition of fibrinoid on the surface of the villi in the
junctional zone.
- By these changes reduced exchange between two circulations.
E) F Full term placenta -
Weight - 500-600g
Diameter - 15-25cm
Thickness - 3cm

31)
A) F
7 th day
B) T
C) F
Diploid
D) F
All 3 layers from the epiblast.
E) T
Neural crest cells extend throughout the neural tube
Migrate laterally
Dorsal Root Ganglion.
Presented by 15th Batch 334 FHCS | EUSL
32)
A) T
X linked recessive -

-green color blindness

B) T
C) F autosomal recessive
D) F deletion of short arm of chromosome 5
E) F autosomal recessive

33)
Klinefelter syndrome – 47, XXY trisomy

genitalia.
and
varicose ulcers.

A) T
B) F
C) F
large teeth seen in Jacob’s disease (XYY)
D) T
E) T
58)
ANSWER – E
It underlies the neural tube and serve as basis for axial skeleton.27
Repeat campaign 2K18 presented by 10th wms

59)
ANSWER – D

60)
ANSWER – C
A) F
contain 37 coting sequence.
B) F
UGA
C) T
D) F
only from mother.
E) F
low percentage of introns.

Presented by 15th Batch 335 FHCS | EUSL


10th PROPER SEQ
4.
4.1.
4.1.1.

4.1.2.
ess that establishes all 3 germ layers (ectoderm,
mesoderm,endoderm) in the embryo.

epiblast.

elevated area surrounding the small primitive pit.

flask shaped,detach from the epiblast and slip beneath it.

some displace the hypoblast, creating


the embryonic endoderm and others come to lie between the epiblast
and newly created endoderm to form mesoderm.

4.2.
4.2.1.
Oogenesis
nia differentiate into mature oocytes.

differentiate into oogonia.

3 rd month, they are arrange in clusters surrounded by a layer of flat


epithelial cells known as follicular cells.

arrest their cell division in prophase in miosis I and form primary


oocytes.
f oogonia have degenerated except for afew.

known as a primodial follicle.


s have started prophase of
meiosis 1. But instead of proceeding into metaphase , they enter the
diplotene stage.

first meiotic division, before puberty is reached.


pool of growing follicles is established and continuosly
maintained from the supply of primordial follicles.

immediately prior to ovulation, follicles are quite swollen and are called
graffian follicles.

4.2.2.
Turner syndrome

– 45X
Presented by 15th Batch 336 FHCS | EUSL
 de chest






4.2.3.
Sex determination
ing a pair of sex
chromosomes.

chromosomes forming a pair of sex chromosomes.

chromosome included in the halved number of 23 and those carrying Y


chromosome.

includes a X chromosome.

fertilizes any of X ovum, an XX female zygote or an XY male zygote will


be formed.

10th REPEAT SEQ


6. Write an account on
6.1 Fertilization (35)
It is the process in which male and female gamete unite to form a zygote. It mainly
occur in the ampulla of oviduct. Sperms deposited in vagina, enter cervix, reach
isthmus and become less motile. Become motile again due to chemotactants
produced by cumulus cell at ovulation.
Before fertilization, the sperm must undergo capacitation and acrosomal reaction.
Capacitation is the functional maturation by conditioning of sperm within the oviduct.
Last for 7 hours. Releasing of enzymes like trypsin, acrosin to penetrate zona
pellucida is acrsomal reaction. Three phases of fertilization,
1. Penetration of corona radiata,- 300-500 sperms reach site of fertilization. Only
capacitated sperms can enter.
2. Penetration of zona pellucida,- acrosomal reaction
3. Fusion of oocyte and sperm cell membrane.- maintained partially by integrins.
Both head and tail enter
Events after entry of sperm are
1. Cortical and zona reactions of oocyte;-cotical granules release lysosomal
enzymes. Alter zona pellucida. Oocyte membrane becomes impenetrable to prevent
polyspermy.
2. Resumption of 2nd meiotic division of oocyte and
3. Metabolic activation of the egg.
Male and female pronuclei come into close contact and form zygote nucleus. The
main results of fertiilization are,
1. Restoration of diploid number of chromosomes,
2. Determination of sex of new individual and
Presented by 15th Batch 337 FHCS | EUSL
3. Initiation of cleavage.

6.2 Formation and fate of definitive yolk sac (30)


In the 2nd week of development of embryo, primitive yolk sac appears. Flat cells
from hypoblast form thin membrane- Heuser membrane. This lines inner surface of
cytotrophoblast, forming the lining of primitive yolk sac. This occurs about 9th day of
development. At the 13th day, hypoblast provides an additional cell layer. These cells
migrate along the inside of exocoelomic membrane. They proliferate and make new
cell cavity within exocoelomic cavity forming definitive yolk sac.
11th batch MedicineAt day 16, yolk sac send a diverticulum into the connecting stalk. As the embyo
folds, yolk sac becomes ventral. Gut tube formed by incorporating a large portion of
endoderm into embryos’ body. Definitive yolk sac remain outside the embyo. Mid gut
communicates with it by vitelline duct. In the 5th week, yolk sac duct, allantoise and
umbilical vessels are in umbilical cord. Vitellointestinal duct is fibrosed at the end of
6 th week. Yolk sac detached from mid gut. In 2% adults, its proximal intra-abdominal
part persists as Meckel diverticulum. In the 10th week, definitive yolk sac lies in the
chorionic cavity between amniotic and chorionic sacs. As pregnancy advances, it
atrophies.

6.3 Down’s syndrome (35)


A genetic disorder causing developmental and intellectual delays. Due to abnormal
cell division resulting extra chromosome 21, trisomy of 21 chromosome. Two types,
1. Trisomy 21 (96%)- Extra copy of 21 in every cell. Maybe due to either
nondisjunction of homologous chromosomes in meiosis 1 or nondisjunction of sister
chromatids in meiosis 11.
2. Translocation (4%) – Due to Robertsonian Translocation. Children have only extra
part of chromosome 21. 46 total chromosomes but 13/14/15/21/22 chromosomes
has an extra chromosome 21 attached.
Down’s syndrome causes distinct facial appearances, intellectual disability and
developmental delays. Maybe associated with heart and thyroids. Survival is usually
less than 50 years, but heart defects may cause even early death.
Facial defects are,
Small head,
flat occiput,
low set ears,
flat nasal bridge,
epicanthic fold,
upward slanting eyes and protruding tongue.
Cardiac defects like atrioventricular canal defects, tetralogy of Fallot may present.
Gastrointestinal atresia may also be seen.

11TH PROPER MCQ

22)oogenesis
A) is started before birth
B) primary oocyte may get mutation with age
C) preovulatory phase is stimulated by LH
D)granulosa cells contribute to produce Zona pellucida
Presented by 15th Batch 338 FHCS | EUSL
E) Second meiotic division finished immediately before ovulation

23. Regarding 3rd week development


A) Exposure to teratogen cause caudal dysgenesis
B) Neuroenteric canal temporarily connect amniotic cavity to chorionic cavity
C) Prechordal plate initiate notochord formation
D) Prenotochordal cells invaginate through the blastopore
E) Primitive streak can be seen in 16th day

24. Regarding somites and somatomeres


A) Derived from neural crest cells
B) facial skeleton develops from it
C) Somites only form the head region
D) the vertebra originates from them
E) they form the segmental development of the body

25. human placenta


A) function as an absolute barrier between maternal and fetal circulation
B) is of a hemochonial type
C) is separated after delivery
D)undergoes changes during end of the pregnancy
E) weight about 200g after maturation

26.Genetic diseases that occur due to deletion in the structure of chromosomes


A) acute lymphocytic leukaemia
B) angelman syndrome
C) di jeorge syndrome
D) Philadelphia syndrome
E) prader willi syndrome

28. regarding mutations


A) can alter the structure and functions of chromosomes
B) Rise due to exposure of radiation and chemicals
C) make change in DNA that always harmful
D) Can results due to metal ion
E) make insertion that are inheritable

29)The para meter that can be obtain by investigating ultrasonography


A) age of fetus
B) trisomy 18
C) position of placenta
D) length of femur
E) congenital anomalies

SBR
44. which of the following is not true regarding male infertility
A) undescendent testis
B) acrosomal defect
C) Sperm count is less than 20 million/ ml
D) Lesion of pelvic nerve

Presented by 15th Batch 339 FHCS | EUSL


E) Hypopituitarism

51. The structure of umbilical cord that is remaining active in late pregnancy
A) Allantois
B) Vitelline vessels
C) Yolk sac
D) Umbilical vessels
E) Vitelline duct

53. Most appropriate about y chromosome


A) Haemophilia in its long limb
B) Determine sex in fertilization
C) Has more number of genes than x chromosome.
D) Male gametes has only x chromosome
E) Decrease formation of genitalia

54. Which of thefollowing is an autosomal dominant inheritant?


A) Cystic fibrosis
B) sickle cell anemia
C) Marfan syndrome
D) Homocystine urea
E) Thalassemia

56. Failure of chromosome to separate during meiosis as known as


A) Duplication
B) Nondisjunction
C) point mutation
D) frame shift
E) translocation

11TH PROPER SEQ

3.4 Explain the differences in meiosis during gametogenesis in male and female gonads. (25)
4. 4.1 Describe the stages in the process of fertilization. (35)
4.2 Briefly describe the composition of fetal membranes in monozygotic twins. (25)
4.3 Observe the diagram given below.

Presented by 15th Batch 340 FHCS | EUSL


4.3.1What type of mode of transmission is most likely exhibited in the above pedigree. (10)
4.3.2 Explain the reason for your answer in 4.3.1

Answers

PROPER-MCQ

22 A True
B True
C true granulosa cells and the oocyte secretea layer of glycoproteins on the
surface of the oocyte, forming the zona pellucida
D false Pre ovulatory phase-follicular phase-stimulated by FSH
E False
23 A True
B faLSE Connects amniotic cavity and yolk sac cavity
C
D
E True
24 A
B Neural crest cells contribute to the craniofacial skeleton
C F
D T sclerotome that will differentiate into the vertebrae and ribs
E T Each myotome and dermatome retains its innervation from its
segment of origin, no matter where the cells migrate. Hence, each
somite forms its own sclerotome (the tendon cartilage and bone
component), its own myotome (providing the segmental muscle
component), and its own dermatome, which forms the dermis of the
back. Each myotome and dermatome also has its own segmental
nerve component.
25 A F
B T
C T
D
E F 500-600g
26 A False Reciprocal translocation of chromosome
B True Deletion in maternal chromosome 15
C True Deletion in small part of chromosome 22
D False Reciprocal translocation
E True Deletion in long arm of chromosome 15
28 A T
B T
C F
D T
E T
29 A T Important parameters revealed by ultrasound include
th
Presented by 15 Batch 341 FHCS | EUSL
characteristics of fetal age and growth;
presence or absence of congenital anomalies;
status of the uterine environment, including the amount of amniotic
fluid ;
placental position and umbilical blood flow;
and whether multiple gestations are present .
biparietal diameter (BPD) of the skull, femur length, and abdominal
circumference
B T Ultrasound can also be used to screen for Down syndrome and some
other chromosome related abnormalities through a test called nuchal
translucency.
C T
D T
E T
44 Answer
D
51

53 A False Carried by x chromosome


B True Y chromosome contains SRY gene which determine sex
C False Shorter than X and contains lower number of genes than X
D False Male gametes can be x or Y
E False
54 A False
B False Cells of the lateral boarder of the neurectoderm become dissociate to
form neural crest cells
C false endoderm
D False Syncytotrophoblast wont take over the function of neural crest cells
E True Neural crest cells give rise to
-craniofacial skeleton
-cranial ganglia/neurons
-glial cells
-melanocytes in face
56 A False Result in a gain of chromosome material by doubling in a region
B True Failure in separation of homologues chromosome
C False Also known as single gene mutation
D False Exchange of chromosomal material between chromosomes
E False Frameshift

PROPER-SEQ
3.4

4.

Presented by 15th Batch 342 FHCS | EUSL


4.1 Describe the stages in the process of fertilization. (35)
Fertilization is the process by which the male and female gamete unite to give rise to a
zygote. Occur in the Ampulla region of the fallopian tube. Before fertilization there are 3
phases,
1. Penetration of the Corona radiata
• Only capacitated sperms can cross corona radiata
• Capacitation is the conditioning of the sperm in the female reproductive
system.
• Lasts for 7 days
• Epithelial interactions occur between sperm and mucosal surface of of tube
• Glycoprotein coat and seminal plasma proteins removed from plasma
membrane that overlap acrosomal cap
2. Penetration of Zona pellucida
• Acrosomal reaction-release of enzymes like acrosin, trypsin, from acrosome
which allow sperm to penetrate the zona by digestion.
• Come in contact with plasma membrane of oocyte.
3. Fusion of the oocyte and the sperm cell membrane.
• Adhesion of sperm to oocyte mediated partially by integrins on oocyte.
• Their plasma membranes fuse- oocyte membrane and membrane covering
the posterior region of sperm head
• Both the head and the tail of spermatozoa enters the cytoplasm of oocyte
After entry of sperm, oocyte undergo
1. Cortical and zona reactions
• Zona pellucida alter its structure
• Oocyte membrane become impenetrable to other sperms
2. Resumption of 2nd meiotic division
• Release 2nd polar body• Chromosome arrange to form female pronucleus
3. Metabolic activation of egg
Male nucleus swell to form male pronucleus
Actual fertilization is when they fuse
Nuclear membrane disappear, chromosomes become distinct, replication of DNA resulting 2
daughter cells.

4.2 Briefly describe the composition of fetal membranes in monozygotic twins.


(25)
• Monozygotic twins developed from a single fertilized ovum.
• Identical to each other.
• Three possible relations of fetal membranes in monozygotic twins.
o Splitting occurs at the 2 cell stage.
▪ Each embryo has its own placenta, amniotic cavity and chorionic
cavity.
o Splitting of the inner cell mass into 2 completely separated groups.
▪ The 2 embryos have a common placenta and a common chorionic
cavity.
▪ But, separate amniotic cavities.
o Splitting of the inner cell mass at a late stage of development.
▪ The embryos have a common placenta, a common amniotic cavity
and a common chorionic cavity.

4.3 Observe the diagram given below.


4.3.1What type of mode of transmission is most likely exhibited in the above
pedigree. (10)
• Autosomal recessive.

Presented by 15th Batch 343 FHCS | EUSL


4.3.2 Explain the reason for your answer in 4.3.1
• In the last stage, one is affected and one is normal. It gives 4 normal children

If it is autosomal dominant, all will be affected or 2 affected and 2 normal

In the question male affected, female normal. If it is x linked 2 female children must be
carriers and 2 male children normal. Since they are normal, cannot be x-linked

12TH PROPER MCQ

22. Primitive streak


a. Gastrulation begins with formation of primitive streak
b. Give rises neuroectoderm
c. Clearly visible during 16th day of development
d. Caudal end of the streak is the primitive node
e. Secrete FGF8

24. Regarding gastrulation


a. Occur in 2nd week of gestation
b. Mesoderm is formed by migration of ectodermal cells
c. Followed by organogenesis
d. Established 3 germ layers
e. Mesoderm absent in precaudal plate

25. Regarding placenta


a. Is a hemochorial shape
b. Has 30-35 cotyledons
c. Separated 30min after delivery
d. Virus can go through placental barrier
e. Is a fetal organ

26. Regarding somites

Presented by 15th Batch 344 FHCS | EUSL


a. After 5th week,44 somites can be seen
b. Dermatomes give rise to muscles
c. Intermediate mesoderm give rise to back muscles
d. Para axial mesoderm give rise to axial skeletal
e. Sclerotome give rise to vertebras

27. Derivatives of neural crest cells


a. Adrenal medulla
b. Hyoid
c. Megakaryocytes
d. Neuroglia
e. Odontoblasts

28. Abnormalities of chromosomes


a. Acute lymphocytic syndrome
b. Downs
c. Plateau
d. Philadelphia
e. Marfan’s

29. Regarding maturation of oocytes,


a. Begins with puberty
b. Primordial germ cells differentiate into oogonia
c. Oogonia cells divided by mitosis
d. At 5th month of prenatal development 20 million germ cells reaches maximum number
e. Primary oocytes arrests in prophase of meiosis I

30. Spermiogenesis
a. Transformation of spermatids into spermatozoa
b. Include formation of acrosome
c. Include development of cytoplasm
d. 74 days required for produce mature spermatozoa from spermatogonia
e. 30 million sperms produced daily

33. Klinefelter syndrome


a. individual has cardiovascular deformities
b. example for monosomy
c. individual has large teeth
d. seen in males
e. osteoarthritis can seen

SBR
47. Which of the following genetic disorder show reduce penetrance/ variable expressivity?
a. X- linked recessive
b. Autosomal recessive
c. Autosomal dominant
d. Y-linked dominant
e. X-linked dominant

Presented by 15th Batch 345 FHCS | EUSL


12TH PROPER SEQ

1. 1.1 outline the events of meiosis that occur in ovary


1.2 explain the defects of meisosis that cause down syndrome
2. a fertilized ovum gradually becomes the blastocyst to get implanted in the wall of the uterus
2.1 describe the sequence of embryonic events that take place during 2nd week of gestation
2.2 explain structure and function of umbilical cord
2.3 a boy born to a normal male and a heterozygous carrier female, was affected by a genetic disease. the affected
trait was linked to the x chromosome
2.3.1 state mendalian pattern of the above conbition
2.3.2 describe the features of the above inheritance pattern by giving three examples
2.4 outline the karyotyping process and state its clinical importance.

13TH PROPER MCQ

22.Regarding spermatozoa
A. abnormal spermatozoa has lack normal motility
B. Capacitation occurs in female reproductive tract
C. acrosomal reaction helps to penetrate the zona pellucida
D. tail is left behind when it penetrate the ovum
E. spermatogenesis longer than oogenesis

24. external features of 4th week of embryo


A. ear bud
B. lung bud
C. pericardial swelling
D. pharyngeal swelling
E. somites

26. Mature placenta barrier


A. cytotropoblast
B. syncytio tropoblast
C. connective tissue
D. maternal blood endothelial layer
E. fetal blood endothelial layer

28. abnormal karyotype seen in


A. cri du chat syndrome
B. marfan syndrome
C. jacobs
D. turners
E. muscularis duchenne

48. How trisomy 21 can be happen?


A. Cross over of non sister cromatids
B. DNA replication
C. Dis junction of homologous chromosome
Presented by 15th Batch 346 FHCS | EUSL
D. Synapses of homologous chromosome
E. Splitting of telomere

51. Neural crest cells


A. Becomes smooth muscle cells within Wall of bowel
B. Gives rise to most of the cardio genic mesoderm
C. Induce the development of syncytiotrophoblast.
D. Derived from primitive streak.
E. Is the source of most of mescenchyme cells of brain

13TH PROPER SEQ

2.1. An ultrasound of a pregnant mother at 7 months gestation shows excessive fluid accumulation in the amniotic
cavity.
2.1.1.State the clinical condition of the above and mention 2 causes for it.
2.1.2.Write an account on amniotic fluid.
2.2. Write accounts on
2.2.1.Autosomal recessive inheritance
2.2.2.Gastrulation

14TH PROPER MCQ

06. Corpus luteum,


A) It is developed by the influence of the FSH
B) It supports the implantation of the embryo
C) Degeneration is prevented by hCG
D) It occupies the larger area in the ovary by the end of the 3rd month of pregnancy
E) Removal before the 3rd month of pregnancy lead to abortion

07. Regarding fertilization,


A) Movement of sperm from cervix to uterine tube occurs mainly on their own
B) Capacitation period is approximately 7 hours
C) Acrosome reaction is induced by zona proteins
D) Oocyte finishes 2nd meiotic division before entry of spermatozoan
E) Intracytoplasmic sperm injection can be used in sperm infertility

08. Primitive streak,


A) Visible on 16th day
B) Secrete FGF8
C) Need formation of EEM
D) Involve in formation of 3rd germ layer
E) Initiate neural tube formation

09. Glands that are derived from ectoderm include,


A) Mammary gland
Presented by 15th Batch 347 FHCS | EUSL
B) Pituitary gland
C) Salivary gland
D) Pancreatic gland
E) Thyroid gland

10. Chromosomal abnormalities are,


A) Prada willi syndrome
B) Edward syndrome
C) Cri du chat syndrome
D) Huntingdon’s syndrome
E) Sickle cell anemia

45. When primary oocyte finished its 1st meiotic cell division?
A) After fertilization
B) During ovulation
C) Just before ovulation
D) After ovulation
E) Just before fertilization

46. Neural tube develops under the inductive effect of


A) Notochord
B) Prechordal plate
C) Anterior visceral endoderm
D) Primitive node
E) Primitive streak

14TH PROPER SEQ

2. 2.1. Describe the formation of neural crest cells and list five (05) derivatives of those cells.
2.2. Write an account on monozygotic twins and the composition of their fetal membranes and placenta.
2.3. List four (04) clinical features of Turner syndrome

15TH PROPER MCQ

22) During spermiogenesis


A. Nuclear condensation occurs
B. Centrosome duplication
C. Formation of haploid number of chromosomes
D. formationofpronucleus
E. Shedding off cytoplasm

23) Regarding umbilical cord


A. Distallyitcontainsintestinalloop
B. distally it contains the yolk sac
C. proximally it contains allantois
D. proximallyitcontainsamnioticfluid
Presented by 15th Batch 348 FHCS | EUSL
E. proximally in contains umbilical vessels

24) Following disorders match the karyotype


A. Downsyndrome-45X,21
B. Patau's - 47XY,21
C. Prader Willi - 41XY,18
D. Klinefelter-45,X0
E. Cri- du chat

26) Regarding the Y chromosome


A. Itisinheritedbymalegametes
B. It inhibits genitalia formation in females
C. coding sequences of the y chromosome is decrease than x chromosome
D. Acrocentrictype
E. it carries SRY gene

50) On ultra sound examination it was determined that a fetus had fell developed facial and thoracic regions, but
caudal structures were abnormal. Kidneys were absent, lumbar and sacral vertebrae were missing, and the hind
limbs were fused. What process may have been disturbed to cause such defects?
A. Formed of fate map
B. Fusion of lateral folding
C. Gastrulation
D. Neural tube formation
E. Notochord formation

51) Following these answers which genetical disorder shows genetic heterogeneity in the affected populations
A. AutosomalDominance
B. Autosomal Recessive
C. X linked Dominance
D. X lined Recessive
E. Y linked Dominance

15TH PROPER SEQ

1.3 Turner's syndrome (50 marks)


1.4 Neural crest cells (50 marks)
2. 2.1 Briefly describe the events that takes place in the second week of embryonic development. (40 marks)
2.2 List five parameters that can be assessed using the ultrasound scanning technique, in a fetus (10 marks)

Presented by 15th Batch 349 FHCS | EUSL


HB 05

Presented by 15th Batch 350 FHCS | EUSL


BLOOD & IMMUNE SYSTEM
EME QUESTIONS
10TH BATCH (EME) - MCQ

10TH BATCH (EME) - SEQ


01.Regarding the body defence mechanism, describe the role of,
1.1 Skin (35)
1.2 Mucous membrane (35)
1.3 White blood cells (30)

Answers

SEQ

1.1

agents some of mechanisms protect body


from without knowing the indent of the biological agents non as initiate or nonspecific immunity.
or certbiological agents. Immune systems are well developed in organ systems that
are exposure to external environment.
entry of biological antigens.The bacterial flora
in the skin prevent the multifunction of harmful bacteria in body.Keratin layer skin takes pathogen with it.Has low
plt repels microorganism.
Sebum an sweat secreted by glands of skin have micro bacterial action.

It is act as First line mechanical barrier

1.2

branes in respiratory tract.Mucous trap pathogenCan be sneezed coughed away By ciliary action.
Macrophages in respiratory epithelium phagocyte
Pharynx there is waldayers ring collection of lymph follicles.

MUCUS PROTECTS. Mucus moistens and warms inhaled air and keeps the mucus membrane cells and the little hairs
called "cilia" lubricated

This is how mucous protect the body

It is act as First line mechanical barrier

1.3

Presented by 15th Batch 351 FHCS | EUSL


▪ innate leukocytes include: natural killer cells, mast cells, eosinophils, basophils; and the phagocytic cells include
macrophages, neutrophils, and dendritic cells
▪ Produced in bone marrow & lymph tissues

▪ Transported to areas of infection/inflammation ( except mast cells and macrophages )


▪ White blood cells are attracted to inflamed tissue areas by chemotaxis (chemotaxis factors)

▪ White blood cells enter the tissue spaces by diapedesis and move through tissue spaces by ameboid motion.

Phagocytes – Neutrophils ,Macrophages, dendritic cells


▪ Neutrophils - first cells to arrive at the site of an infection
▪ Neutrophils are the most abundant type of phagocyte

▪ Granules contain a variety of toxic substances that kill or inhibit growth of bacteria and fungi.

▪ Macrophages are the end-stage product of monocytes that enter the tissues from the blood.
▪ respiratory burst- causing the release of reactive oxygen species.
▪ lysosome contain digestive enzymes (proteolytic enzymes)
▪ macrophage produce chemokines- recruit other cells to the site of infection.[
▪ Neutrophils and macrophages – bactericidal agents/oxidizing agents
▪ basophils release histamine- important in the defence against parasites and play a role in allergic reactions, such
as asthma.
▪ Natural killer cells (NK cells) destroy compromised host cells, such as tumor cells or virus-infected cells
▪ Dendritic cells (DCs)-f antigen presentation, link between the innate and adaptive immune systems.

11TH BATCH (EME) - MCQ

01) Blood,
a) Express serum when it clots.
b) From a higher percentage of body weight in obese than in thin people.
c) Has low blood volume per kg body weight in neonate than in adults.
d) Make up about 7% body weight.
e) Volume rise after drink water.

02) Hemolytic disease of new born,


a) Affect mainly babies of positive mother.
b) Prevent by injecting anti – D antibodies just after the delivery to mother.
c) Treated by transfusion of blood to affected baby with Rh+ blood.
d) Causes jaundice with clears rapidly after birth.
e) Occurs mainly in babies who lack D agglutinogen.

03) Blood group agglutinogens,


Presented by 15th Batch 352 FHCS | EUSL
a) Attached to hemoglobin.
b) Before transfusion cross match with receptor’s one with donor’s agglutinin.
c) AB blood group have agglutinogen A and B.
d) Inherited as mendalian recessive.
e) Not present in new born.

04) Break down of erythrocyte in body,


a) Require for the synthesis of salt.
b) Due to Rh incompatibility.
c) Occur by reticular endothelium system.
d) Due to deficiency of G6P dehydrogenase.
e) Release Fe2+.

05) Person with B blood group,


a) Has anti n antibodies in plasma.
b) May have parent with O blood group.
c) May have children with B blood group.
d) May have the genotype AB.
e) Whose parent is A can have A child.

06) Blood platelets assist in arresting the bleeding by,


a) Adhering together to form plugs when exposed to collagen.
b) Von Willebrand factor which are releasing by platelets.
c) Prostacyclin which are in the plasma.
d) Platelets contain clotting factors.
e) Platelets are aggregated on the formed fibrin network.

07) White blood cells,


a) Has lobular nucleus.
b) Increase in allergic reactions.
c) Produced in lymph nodes.
d) Have lifespan of circulation of 3-4 weeks.
e) Contain proteolytic enzymes.

08) Regarding heme synthesis pathway,


a) All enzymes are located in the mitochondria.
b) Glycine and succinate are precursors of delta ALA.
c) ALA dehydrogenase catalyzes the synthesis of PBG from ALA.
d) Heme inhibits ALA synthesis activity by feed-back inhibition.
e) ALA synthase regulates heme biosynthesis both in liver and erythropoietic tissue.

09) Which of the following cells are involved with pathogenesis of acute immune decrease,
a) B-Lymphocyte
b) Mast cell
c) Neutrophils
d) Natural killer cell
e) T-Lymphocyte

10) γ-aminolemulin acid in heme biosynthesis pathway,


a) Is rate limit state in heme synthesis.

Presented by 15th Batch 353 FHCS | EUSL


b) Is decreasing by an individual use drug barbiturate phenol-barbiturate.
c) Inhibit by heavy metal ion.
d) Occur in the cytosol.
e) Hematin is an intermediate of heme synthesis.

SBR
11) Catabolism of Hb involves,
a) De-oxidative cleavage of the porphyrin ring.
b) Is the sole source of bilirubin.
c) Occur in RBC.
d) Synthesis bile salt.
e) Albumin is not considered for this.

12) Any of the following companions true, Except


a) Type 1 Hypersensitivity – Dengue shock syndrome
b) Type 2 Hypersensitivity – Autoimmune hemolytic anemia
c) Type 3 Hypersensitivity – Vasculitis
d) Type 4 Hypersensitivity – Contact dermatitis
e) Type 5 Hypersensitivity – Myasthenia gravis

13) Prothrombin time of the blood coagulation test,


a) Does not involve oxalation of thrombin in blood.
b) Indicate concentration of removal blood.
c)
d) Normally about 12 min.
e) Decreasing rise of prothrombin.

14) A 5 years old boy suffering from diarrhoea for about 2 week and recently becomes sever. From further
investigation it is observed that feces contain parasites. What’s the type of leukocyte that might increase?
a) Basophils
b) Monocyte
c) Neutrophil
d) Eosinophil
e) Lymphocyte

15) All the following cells are antigen presenting cell, Except
a) B – Lymphocytes
b) Dendritic cell
c) Cooper cells
d) Microglial cell
e) Neutrophil

11TH BATCH (EME) - SEQ

1. A mother of six-month-old boy cannot stop his bleeding after he hits his mouth on the kitchen floor. His mother
said that the boy’s cousin had a similar bleeding problem. He was admitted to the hospital and the investigation
gave the following results.
-Hb-9.6g/dl,
-Platelets-352000/ml
Presented by 15th Batch 354 FHCS | EUSL
-Bleeding time 7.5 min. (Normal 1-9Min)
-PT-12 sec (10-12 sec)
-APTT> 120 sec (28-35 sec)
1.1 What are the 3 processes involved to stop bleeding. (30 marks)
1.2 What coagulation disorder is likely to be present? (20 marks)
1.3 Explain briefly how you come to the above diagnosis. (35 marks)
1.4 How will conform your diagnosis (15 marks)
2
2.1
2.1.1. Briefly describe the function of albumin. (25 marks)
2.1.2. Explain the formation of Bilirubin from Heme. (25 marks)
2.2
2.2.1. List five characteristic difference between innate and acquired immunity. (20 marks)
2.2.2. Explain briefly the immunopathogenesis of allergy resulting from type 1 hyposensitivity. (30 marks)

Answers

MCQ

1. A) T

Serum= plasma - fibrinogen

B) F

C) F

D) T ( 6‰- 8%)

E) F

2. A) F

Occurs in negative mothers when having positive babies

B) T

C) F

D) T

E) F

3. A) F attached to RBC surface

B) F while transfusion of blood, antegen doner and antibody of the recipient are considered

C) T

D) F

E) F

4. A) F involve in synthesis of bile pigment not in bile salt


Presented by 15th Batch 355 FHCS | EUSL
B) T

C) T

D) T deficiency of NADPH and damage occur by free radicals

E) T

5. A) F

B) T

C) T

D) F ( Bi, BB)

E) T

6. A) T

B) T

C) F vasoconstriction by tromboxane A2

D) T factors 3,4,5,13 by platelet

E)

7. A) T

B) T eosinophil ex. Asthma

C) T

Protects your body against foreign invaders:


The lymphatic system is part of the immune
system. It produces and releases lymphocytes
(white blood cells) and other immune cells
that monitor and then destroy the foreign
invaders

D) F

Presented by 15th Batch 356 FHCS | EUSL


E) T

8. A) F cytosol also involve

B) T

C) T

D) T

E) F

9. A) T

B) F

C)

D)

E) T

10. A) T

B) F barbiturates increase the synthesis of ala synthase

C) F

D) F mitochondria and cytosol

E). Heam oxidised and form the hematin


Presented by 15th Batch 357 FHCS | EUSL
11. Answer B

A) Fault because it's occurred by oxidative cleavage

12. Answer A

13.

A, B, D, and E are false

14. D

15. E

11 th SEQ

1.

1.1 Hemostasis contain the following steps,


• Primary response
▪ Vasoconstriction
▪ Platelet response (Platelet plug formation)
• Secondary response
▪ Blood coagulation

Vasoconstriction

• After injury the blood vessel constricts and decreases the loss of blood from damaged Portion.
• Arterioles constrict.

• Obliteration of lumen of the damaged blood vessel Stop bleeding

➢ Nervous reflexes
▪ Pain fibers reach the CNS (spinal cord)

▪ Efferent signal reach the smooth muscles of the arterioles


➢ Local humoral factors – Serotonin, thromboxane A2
➢ Local myogenic contractions of blood vessel

Presented by 15th Batch 358 FHCS | EUSL


Platelet response (Platelet plug formation)

• Platelets to the collagen


is accelerated by von
Wille brand factor.
• This factor acts as a bridge
between a specific
Glycoprotein present on
the surface of platelet and
Collagen fibrils.
• Platelets get adhered to
the collagen of ruptured
blood vessel
• Secrete adenosine diphosphate (ADP) and thromboxane A
• attract more platelets and activate them
• All these platelets aggregate together and form a loose temporary platelet plug
• Vessel and prevents further blood loss.
Blood coagulation
Temporary platelet plug will be converted into definite blood clot Contains following steps,
• Formation of activated factor X (Xa) Intrinsic & Extrinsic pathways

• Conversion of Prothrombin to Thrombin


• Conversion of Fibrinogen to Fibrin

Presented by 15th Batch 359 FHCS | EUSL


1.2

May effected by
hereditary Hemophilia
• Hemophilia A
(low CF 8) or
Hemophilia B (low CF 9)

1.3

• The Platelet count


is normal so the person
platelet response is
normal • Bleeding
time is normal

• Which the Primary


response of hemostasis is
normal
• Normal vascular response and platelet response
Presented by 15th Batch 360 FHCS | EUSL
• PT is normal
• So the extrinsic pathway is tend to be normal

• APtt is high
• Which shows abnormal intrinsic pathway
• So the patient should have decreased production or action of Factors in intrinsic pathway
(Clotting factor 12, 11, 9 or 8)
• Which tend to be Hemophilia A (low CF 8) or Hemophilia B (low CF 9)

1.4. How will you confirm the diagnosis (15 marks)


• To check whether its hemophilia A or B,
• Both serum level of CF 8 and CF 9 is calculated
• Which lead to differentiate the diagnosis

2.1.1.
• Act as a transporter of the plasma
▪ Albumin carries insoluble substance in the plasma ( Steroid and thyroid hormones, Unconjugated bilirubin,
Drugs, Enzymes)
• Maintain the osmotic pressure of the blood
▪ Albumin is the highest found plasma protein in blood
▪ Cannot pass through the capillary membrane easily and remain in the blood.

▪ It creates an oncotic pressure plasma


▪ Maintain the blood volume

• Regulation of acid-base balance


▪ Regulate the pH of the blood by acting as an buffer
▪ 15% of buffering

• Provides viscosity to the blood


• Act as a reserve proteins
▪ During prolonged starvation albumin readily breakdown to provide energy
▪ Gives amino acid for the production of other proteins
• Provide Suspension stability of red blood cells

2.1.2.
During hemolysis in the reticular endothelial system (Spleen, Liver),
• Hemoglobin will be broken down
• Heme + globulin proteins result
• Heme breaks and iron + Porphyrin result
• Porphyrin will be broke down into Biliverdin
• Catalyst by Heme oxidase
• Biliverdin will be converted into Unconjugated bilirubin (UCB)

Presented by 15th Batch 361 FHCS | EUSL


• By biliverdin reductase
• UCB is
released to
the plasma
• Albumin
carries the
UCB to the
hepatocytes
of liver

Conjugation
of the UCB,

Unconjugated
Bilirubin will
be conjugated in the hepatocytes
• Glucuronidyl- transferase catalyze the reaction
• Conjugated bilirubin (CB) is formed
UPTO THIS IS MORE IMPORTANT

In biliary system,
• CB is released to the bile canaliculi
• Which lead to bile duct
• Open into the Duodenum

In intestine,
• CB is converted into urobilinogen
• By bacterial action
• Some urobilinogen reach the entero-hepatic circulation
• Remaining will be converted into stercobilin( Brown)

• Eliminate in faces
In Kidney,
• Urobilinogen in the blood reach the kidney

• Converted into urobilin


• Excreted through urine

2.2.2.
• Type 1 hypersensitivity is IgE mediated
✓ anaphylactic reactions

Presented by 15th Batch 362 FHCS | EUSL


✓ Urticaria
✓ Hay Fever
✓ Asthma

Anaphylaxis
• The allergen react with basophils and mast cells
• Basophils and mast cells have been sensitized by attachment of IgE antibody
• allergic reaction occurs throughout the vascular system and closely associated tissues
• This reaction is called anaphylaxis

• Histamines released into the circulation and causes


✓ Body-wide vasodilation
✓ Increased permeability of the capillaries with resultant marked loss of plasma from the circulation.

▪ Leads to circulatory shock


✓ Spasm of the smooth muscle of the bronchioles
▪ Bronchoconstriction
▪ Leads to asthma

Urticaria
• Antigen entering specific skin areas and causing localized anaphylactic reactions.
• Histamine released locally causes
✓ Vasodilation that induces an immediate red flare
✓ Increased local permeability of the capillaries that leads to swelling of the skin
• The swellings are commonly called hives.

.Hay Fever

• The allergen-reagin reaction occurs in the nose


• Histamine released in response to the reaction causes local intranasal vascular dilation
• Increased capillary permeability.
• Both these effects cause rapid fluid leakage into the nasal cavities
• Nasal linings become swollen and secretory.
Asthma
Asthma often occurs in the “allergic” type of person.
In such a person, the allergen-reagin reaction occurs in the bronchioles of the lungs
Causes spasm of the bronchiolar smooth muscle (Bronchoconstriction)

Difficulty in breathing

Presented by 15th Batch 363 FHCS | EUSL


12TH BATCH (EME) - MCQ

1)Breakdown in erythrocytes in body,


a. Occurs when there is incompatible of blood group
b. Done by reticuloendothelial system
c. Amino acids from globin are excreted with urine
d. If happens rapidly, sometimes can cause accumulation of bilirubin in brain
e. Yield by bilirubin transport to liver by plasma protein

2) Lead poisoning affects to enzymes in heam synthesis pathway,


a. ALA Synthase
b. ALA dehydratase
c. Ferrochelatase
d. PBG deaminase
e. Uroporphyrinogendecarboxylase

3) Albumin,
a. 40% in normal blood.
b. Initially synthesize as preproalbumin
c. It is a conjugated protein
d. Single polypeptide chain
e. It is served as source of amino acids to protein synthesizes

4) A person who has AB blood group,


a. A parent may have O blood group
b. Can have anti B antibodies
c. May have AB genotype
d. Siblings have A and B only
e. Mother can A and father is O

5) Eosinophil,
a. It's present in high concentration in pus
b. Secrete proteolytic enzymes
c. Increase in infections
d. Involve in allergy reactions
e. Lifespan is 4-5 weeks

6) Hemolytic disease in new born,


a. affected mainly babies of Rh - mothers
b. Occurs mainly in babies who lack D agglutinogen
c. Causes jaundice which clears rapidly after birth
d. Can be treated infant by transfusing Rh-blood
e. Prevent by injecting anti D soon after delivery to mother

7) Deficiency of factor IX,


a. Affect extrinsic pathway rather than intrinsic pathway
b. can diagnose with specific factor assay
c. cause small (pelechial) hemorrhages into skin to cause purpura
d. increase the APTT
e. Due to mutation of 21 chromosome

8) Primary lymphoid organs,


a. bone marrow
b. Liver
Presented by 15th Batch 364 FHCS | EUSL
c. Lymph nodes
d. spleen
e. thymus

9)What are compatible with each other,


a. CD20- B lymphocyte
b. CD3-Tlymphocyte
c. CD33-sterm cells
d. CD34-monocyte
e. CD8=ThI

10) Innate immunity is differ from acquired immunity. Innate immunity,


a.Demonstrates affinity maturation
b. Has no memory
c. is involved with auto immunity
d. More specificity
e. More efficiency

Select the most appropriate answer from each question & mark them inside the cages
11) What is the protein responsible for iron transportation?
a. Apo ferrin
b. Apo transferin
C. Ceruloplasmin
d. Ferritin
e. Hepatalmoglobulin

12) Reduction of prothombin level in blood is affected by,


a. Reduce platelets
b. Phospolipid
c. Sodium
d. Vit B12
e. Vit K

13) Kamala has classic hemophilia. What was the most probable one?
a. It was autosomal dominant
b. Father had hemophilia and mother was a carrier
c. Mother was a hemophilia carrier
d. One of new x chromosome lead for deficiency if factor viii
e. She is suffering from von willebrand

14) A person who don't like to eat vegetables because he thinks it is poisonous, clinically
diagnosed his MCV 122,WBC= 10000
a. Hemorrhage
b. Aplastic anemia
c. folic acid deficiency
d. hemolytic anemia
e. sickle cell anemia

15) All are antigen presenting cell except,


a. B lymphocyte
b. Dendrites cell
c. Kuffer cell
d. Microgial cell
e. T lymphocyte
Presented by 15th Batch 365 FHCS | EUSL
12TH BATCH (EME) - SEQ

1. 25 year old mother with a history of previous abortions delivered her first baby. Same Day, the baby developed
jaundice and admitted to baby care unit. The following Investigations were done ;
• Hb-6.4 g/di
• WBC-8500,
• platelet 290000
• Serum Bilirubin Total 23.5 mol/dl
• Direct Bilirubin total 3.5 mol/dl
• Mothers blood group B negative. Fathers blood group AB positive

1.1 What is the more likely condition of this baby? (15 marks)
1.2 Explain the pathophysiology of the above condition. (40 marks)
3. What are the possible phenotype/s and genotype/s of the baby's blood group? (20 marks)
4. What is/ are the blood groups that you will consider for the transfusion of baby's condition? (10 marks)
5. How will you prevent the above condition in the next pregnancy? (15 marks)

2.1
1. Explain the biochemical basis behind the "neonatal jaundice and phototherapy (35 marks)
2.Briefly explain "deficiency of alpha 1 antitrypsin causes emphysema' (15 marks)
2.2
3. List one clinical example in each type or hypersensitivity reaction.
4. List three clinical conditions causing secondary Immune deficiency.
5. State the difference between allograft and xenograft. (10 marks)
6. State the difference between primary and secondary immune response. (10 marks)
7. State the difference between active and passive immunity giving examples. (14 marks)

Answers

MCQ

1)
A-T Hemolysis will occur
B-T Liver spleen
C-F Globin will be reutilized
D- In children can accumulate bilirubin in brain because of immature blood brain barrier. In adults cannot
accumulate
E-T
2)
A-F Refer heam metabolism notes.
B-T
C-T
D-F
E-F
3)
A-T Refer plasma protein notes.
B-T
C-F
D-T

Presented by 15th Batch 366 FHCS | EUSL


E-T
4)
A-F To get AB blood group parents must have A&B groups
B-F AB blood group does not have antibodies
C-T IAIB
D-F
E-F
5)
A-F Neutrophil in pus
B-T
C-T
D-T
E-F
6)
A-T Erythroblastosis fetalis
B-F occur in rh+ babies
C-F physiological jaundice only clear rapidly after birth
D-F
E-T 24-72 hours after delivery
7)
A-F intrinsic pathway
B
C
D-T
E
8)
A-T
B-F
C-F
D-F
E-T
9)
A-
B
C
D
E
10)
A
B-T innate immunity has no memory. Adaptive immunity has memory
C-F in auto immunity T lymphocytes take part
D-F adaptive immunity
E-F adaptive immunitu
11)D
12)E
13)D classic hemophilia (factor viii deficiency)
14)C

Presented by 15th Batch 367 FHCS | EUSL


15)E Antigen presenting cells - B lymphocyte, Macrophages, Dendritic cells

SEQ
01)
1.1)
➢ Hemolytic disease of the new born (Erythroblastosis fetalis)
➢ Because of very low Hb & normal WBC & platelets
Normal new born Hb level 14-22g/dL
➢ Serum bilirubin is also in a high level

1.2)
➢ Fathers blood group is Rh positive & it is dominant than Rh negative.
➢ Therefore, every child of this parents will get Rh+ blood group.
➢ Rh incompatibility arises when an Rh-mother carries an Rh+
fetus.
➢ Small amount of fetal blood leak in to maternal circulation at the time of delivery & some
mothers develop significant titers of anti-Rh agglutinins during the postpartum period.
➢ During the next pregnancy the mother’s agglutinins cross the placenta to the fetus.
➢ In addition, there are some cases of fetal-maternal hemorrhage during pregnancy &
sensitization can occur during the pregnancy
➢ In any case, when anti-Rh agglutinins cross the placenta to Rh+ fetus, it can cause hemolysis
& various form of hemolytic disease of new born.
➢ If the hemolysis in the fetus is severe the infant may die in utero or may develop anemia,
severe jaundice & edema (Hydrops fetalis)
➢ Hemolysis release free bilirubin in to the blood.
➢ Large quantities of bilirubin in the ECF, either direct or indirect cause jaundice, which is
yellowish discoloration of skin, sclera, mucous membrane & deep tissue due to the
deposition of the of bilirubin.
➢ Here, elevation of indirect bilirubin is due to the increase production of unconjugated
bilirubin and bilirubin conjugating system not yet mature.
1.3)
Possible phenotypes of baby -AB /A/B
Possible genotypes of baby -AB+ /BB+/ AO+/BO+
1.4) O-

1.5)
Neutralize the mother’s blood by administrating anti-Rh antibodies in the form of Rh
immune globulin (Rhogam) during the postpartum period.
02)
2.1)
2.1.1)
➢ The majority of the new born infants (60% of full-term & 80% of pre-term) show a rise in
unconjugated bilirubin (UCB) in the 1st postnatal week
A transient physiologic jaundice because the activity of the hepatic bilirubin UDP-
glucuronosyltransferase (Bilirubin UGT) is low at birth.
(It reaches adult level in about 4 weeks)
Presented by 15th Batch 368 FHCS | EUSL
➢ Elevated UCB, in excess of the binding capacity of albumin(20-25mg/dL), Can diffuse into the
basal ganglia, cause toxic encephalopathy (Kernicterus) & a pathologic jaundice
➢ Therefore, newborns with significantly elevated bilirubin levels are treated with blue
fluorescent light (phototherapy)
➢ This phototherapy converts bilirubin into more polar & therefore water-soluble isomers.
➢ These photo isomers can be excreted in to the bile without conjugation to glucuronic acid.
(because of solubility differences, only UCB crosses the BBB & only CB appears in urine)

2.1.2 )
➢ Normally, α1-AT protects the lung form injurious effects by binding with the proteases.
Eg- active elastase
➢ Neutrophil elastase -Produced by WBC to break down harmful bacteria
-potentially damaging to the lungs if exposed
➢ A particular methionine (358 residue) is involved in binding with the protease
Active elastase + α1-AT->Inactive elastase: α1-AT complex->No proteolysis of lungs & No tissue damage
α1-AT deficiency
• α1-AT trapped in liver
• causing liver damage
• smoking oxidizes the methionine (358 residue) & inactivate the protein α1-AT
α1-AT cannot bind to the protease active elastase
Proteolysis of lungs & tissue damage occurs
accelerating the development of emphysema

2.2
2.2.1)
➢ type 1 – Utricaria
➢ type 2 – Rh incompatibility
➢ type 3 – Post streptococcal glomerulonephritis
➢ type 4 – post primary tuberculosis
➢ type 5 – Myasthenia gravis

2.2.2)
➢ Leukemia
➢ Post splenectomy
➢ AIDS

2.2.3 )
➢ Allograft – organ transplanted from a donor to a recipient of the same species who is not
genetically identical
➢ Xenograft – organ transplanted from a donor to a recipient of the different species

2.2.4 )
➢ Primary immune response – occurs when the antigen comes in contact to the immune
system for the first time
➢ Secondary immune system – Occurs when the 2nd (3rd/4

Presented by 15th Batch 369 FHCS | EUSL


th …) time the person is exposed to
the same antigen

2.2.5)

Active immunity Passive immunity

Result from an infection or an immunization Comes from naturally or artificially gaining


antibodies

Ex- Ex-
Measles-infection Natural- through the breast milk
Colostrum, Ig A
Artificial – injection of anti-rabies serum

13TH BATCH (EME) - MCQ

1) Macrocytic anemia is associated with


A. Gastrectomy
B. Iron deficency
C. Bile duct obstruction
D. Vitamin B12deficiency
E. Vitamin C deficiency

2) In healthy mature adults erythropoiesis occurs in


A. Epiphyseal plate of long bones
B. Flat bones
C. Liver
D. Shaft of long bone
E. spleen

3)thromboplastin is secreted by
A.
B. Blood vessels and surrounding tissues
C. Liver
D. Mass cells
E. Platelets

4)What are the phagocytic cells


A. Lymphocytes
B. Basophils
C. Eosinophils
D. Monocytes
E. Neutrophils

Presented by 15th Batch 370 FHCS | EUSL


5) Adaptive immunity provided by
A. Antigen presenting cells
B. B Lymphocytes
C. Eosinophils
D. Mucous membrane or respiratory tract
E. Reticulo endothelial system

6) hematocrit
A. Is increased in people live in high altitude
B. Is measuring using Westergren's tube
C. Is used in the calculation of MCH
D. Refers to total number of red cells in 100ml of blood
E. Refers to volume percentage or cells In 10Uml or blood

7)Hemolytic anemia
A. Is accompanied by dark colored urine
B. Accompanied with increased number of reticulocytes
C. Accompanied with jaundice
D. Is associated with hereditary spherocytosis
E. Is characterized by high MC

8) Platelets
A. Destroyed in spleen
B. Produced in megakaryocytes of red bone marrow
C. Life span is about 120 days
D. Have bilobed nucleus none
E. Deficiency results in bleeding disorders

9)Regarding red blood cells


A. Incapable of cell division
B. Hemoglobin occupies 70% of cell volume 95%
C. Outer surface of cell membrane contain blood group antibodies antigens
D. They become more spherical as they become older
E. They hemolyze when placed in isotonic NaCI solution

10) Found in innate immunity mechanism


A. B Lymphocytes
B. Kuffers cells
C. Monocytes
D. Neutrophils
E. T Lymphocytes

Single Best

11) Which of the following is not found in the peripheral blood


A. Lymphocytes
B. Megakaryocytes
C. Monocytes
D. Polymorphonuclear cells
E. Reticulocytes

Presented by 15th Batch 371 FHCS | EUSL


12) Most immediately response after minor injuries to small blood vessels
A. Clot formation
B. Clot retraction
C. Platelet plug formation
D. Fibrolysis
E. Vasoconstriction

13) Prolonged clotting time and excessive bleeding is associated with


A. Auto immune diseases
B. Heart diseases
C. Kidnev diseases
D. liver diseases
E. Lung diseases

14) What is the following plasma proteins produce outside the liver
A. Albumin
B. Gamma globulin plasma cells
C. Fibrinogen
D. B globulin
E. prothrombin

15) which of the following is not an acute phase protein


A. Alpha 1 antitrypsin
B. C relative proteins
C. Ceruloplasmin
D .Elastin
E. Haptoglobin

13TH BATCH (EME) - SEQ

1. when an Rh negative mother caries an Rh positive foetus


1.1. Describe the possible risk factors to the first baby and the subsequent Rh positive Babies (40 marks)
1.2 Explain the physiological basis of the risk factors (40 marks)
13 Briefly describe how the above risk factors may be prevented (20 marks)

2.1. name the components of the immune system (20 marks)


2.2. briefly describe the role of lymphocytes in immunity (30 marks)
2.3. mention the different types of hemoglobin (10 marks)
2.4. describe the structure of hemoglobin and how it is suited to its function (40 marks)

Answers

MCQ

1)

A-T Macrocytic anemia is associated with ,


Causes for macrocytic anemia

Presented by 15th Batch 372 FHCS | EUSL


Vit B12 deficiency= Pernicious anemia
Folate B9 deficiency= Megaloblastic anemia
Above 2 vitamins need to cell division-> Deficiency of above vitamins-> Mature RBC production is decreased->
Reticulocytes enter to the circulation->

Gastrectomy intrinsic factor secretion is Inhibited->Vit B12 absorption is Impaired / decreased.


B- F Iron deficiency RBC size is become small->Hb Concentration is reduced->Hypochromic microcytic anemia
C - F Bile duct obstruction.
D- T Vit B12 deficiency
E -F Vit C deficiency decrease absorption of iron->Hypochromic microcytic anemia

2)
A-T After 20 yrs (RBC are produced from membranous bones like vertebrae, sternum, ribs, scapula, iliac bone skull
bones & end of the long bones).
B - T Upto 20 years old red bone marrow of all bones [ long bones & all flat bones] (sembulingam pg 68).
C- F Site of erythropoiesis in fetal life.
D- F Shaft consists in yellow bone marrow
E - F Site of erythropoiesis in fetal life
1. Mesoblastic stage - Yolk sac(1st 2 months)
2. Hepatic stage - Liver (from 3 month)
3. Myeloid stage - Spleen

3)
A-
The thromboplastin is secreted by platelets that
catalyze the reaction which converts prothrombin
in to the thrombin. When tissue injury release thromboplastin [tissue
thromboplastin (factor iii)]

B- T
C - F
D - F
E-T

4)

A-F Phagocytic action - Neutrophil, Monocyte(Macrophages), Eosinophil (Weak Phagocytic action)

B-F

C-T

D-T

E-T

5)

A- T APC- B lymphocyte, Dendritic cells, Macrophages

Presented by 15th Batch 373 FHCS | EUSL


B- T
C- F Innate immunity
D - F Innate immunity
E- F

6)

A- T PCV= Hematocrit male 40-45% Female 38-42%


B- F Westregen tube only for ESR, Wintrobe tube(opened only one end) for ESR
& PCV

C- F [ MCHC, MCV]
MCV= PCV in 100ml * 10/RBC count in million /cumm
MCH= Hb in gram per 1000ml of blood/RBC count in million /cumm
MCHC= Hb in grams/ 100ml of blood *100/ PCV in 100ml of blood
D- T
E- F

7)

A- T Due to the presents of Hb


B- T
C- T Excessive hemolysis of RBC->Hb degradation is increased->Bilirubin concentration is increased->Increased
bilirubin 2mg/dl->jaundice
D-T

E- True MCV may be elevated in patients with high


reticulocyte count since these cells are larger than
mature RBCs

08)
A True Platelets are destroyed by tissue macrophages system in spleen
Increasing destruction of platelets-> Splenomegaly->Decrease number of platelets in circulation
B True
C True 10-12 days
D True No nucleus
E True

09)
A True
B False MCHC 30 -38%
C False Antigens present.
D True
E False Osmolarity RBC=NaCl

10)
A False
B True

Presented by 15th Batch 374 FHCS | EUSL


C True
D True
E False

11)
B
Platelets are derived from megakaryocytes
Megakaryocytes not in peripheral blood
Normally, human blood contains 4000-11000
WBC per microliter, of these, the granulocytes
(Polymorphonuclear leucocytes, PMNs) are
the numerous. [ganong 554pg]

12) E
Vascular spasm-> Platelet plug formation - > Blood coagulation

13) D
A lots of clotting factors are produced in liver
So some liver disease cause to decrease of
production of clotting factor
So clotting & bleeding time will be increased

14) B
B lymphocytes/ plasma cells

15) D
Acute phase protein
Non specific changes in certain plasma
proteins in response to acute inflammatory
stages or secondary to certain chronic tissue
damages.

They are synthesized in liver in response to


peptide mediators or cytokines.

Positive acute phase protein - C reactive protein, Ceruloplasmin, alpha 1 antitrypsin, alpha 2 macroglobulin

Negative acute phase protein - Albumin, prealbumin, retinol binding protein, transferrin

SEQ

01) 1.1)

Rh incompatibility occurs when the mother’s blood type is Rh negative and her fetus blood type is
Rh positive.
If some of the fetus blood passes into the mother’s blood stream, her body will produce antibodies
against Rh factor in response.
Presented by 15th Batch 375 FHCS | EUSL
This is known as iso-immunization. Mother’s immune system becomes sensitized for Rh positive
antigens.
If these antibodies enter to a fetus via placenta it can harm fetus red blood cells and cause
hemolysis. (Hemolytic disease of neonates/HDN)
Factors that determine HDN are
o Genetic makeup of the mother and fetus
o Trans placental hemorrhage
o Can occur at any stage of pregnancy
o Injury, abortion, c section, placental removal, ectopic pregnancy, amniocentesis,
chorionic villus sampling, umbilical blood sampling, external cephalic version may also
leads to iso-immunization of Rh negative mother.
o Fetomaternal hemorrhage
o Volume
o Degree of maternal immune response (antigenicity of fetal RBC and type of IgG)
o Concurrent ABO incompatibility.
Mother’s iso immunization is the main risk factor for HDN. Normally first Rh positive baby escapes if
the delivery is normal and there were no any above complications during pregnancy period. Because
normally mixing of fetomaternal blood occurs during delivery when the placenta separates. But, subsequent Rh
positive babies may affect if the mother is previously sensitized. Anti Rh
antibodies fro, 1st pregnancy remain in maternal blood and cross the placenta during delivery and
cause hemolysis in Rh positive fetus.

2.1)

The main components of the immune system are


• Bone marrow
o Cellular components
1. In innate immunity
Phagocytes (dendritic cells, macrophages and neutrophils)
Natural killer cells
Mast cells
Eosinophil
Basophils
2. In acquired immunity
Lymphocytes
B lymphocytes
T lymphocytes
Antigen presenting cells
• Molecular components
o Complement proteins
o Acute phase proteins
o Cytokines
o Antibodies
• Lymphatic system
• Lymphoid organs
o Spleen
o Thymus
o Tonsils
Presented by 15th Batch 376 FHCS | EUSL
2.2)

There are 2 main types of lymphocytes that play a major role in acquired immunity
T cells – in cellular immunity
B cells – in humoral immunity
• T cells
4 Types

a. Helper T cells
i. Helper-1
Secrete 2 substances
✓ Interleukin 1 – activates other T cells
✓ Gamma interferon – stimulates phagocytic activity of cytotoxic
cells, macrophages and natural killer cells
ii. Helper-2
Secrete
✓ IL-4 & IL-5 – Activates B cells
Proliferation of plasma cells
Production of antibodies by plasma cells
b. Cytotoxic T cells
✓ Receptors of cytotoxic T cells bind the antigen tightly.
Enlarge and release cytotoxic substance. (lysosomal
enzymes)
Destroy invading organisms.
✓ Also destroy cancer cells, transplanted cells, body’s own
tissues which are affected by foreign bodies (virus)
c. Suppressor T cells
✓ Suppress the activity of killer T cells.
✓ Prevent the t cells from destroying the body’s own tissue
along with invaded organisms.
✓ Suppress the activity of the helper T cells
d. Memory T cells
✓ When body exposed to some organism for 2nd time, memory
T cells identify them and immediately activate other T cells
and destroy them quickly.
• B cells
Proliferated B c ells transformed to
1. Plasma cells
✓ Produce the antibodies called immunoglobulin (IgG, IgA, IgM, IgD, IgE) and release in
to circulation. Produce antibodies until the end of the life span.
2. Memory B cells
✓ In the second exposure for an organism, memory cells are activated and produce
more antibodies.

2.3)

Presented by 15th Batch 377 FHCS | EUSL


1. Hb A1 / Adult Hb 1 – α2β2
2. Hb A2 / Adult Hb 2 – α2δ2
3. Hb F / Fetal Hb – α2γ2

4. Hb S / sickle cell Hb – abnormal globulin chain

2.4)

Hemoglobin found inside red blood cells.


Main hemoglobin (Hb) in human is adult Hb/HbA.
Hemoglobin is a globular hemeprotein.
Has a 3 dimensional quaternary structure.
Composed of 4 polypeptide chain subunits. (A tetrameric molecule)
HbA contain 2 α chains and 2 β chains.
Each subunit has α helical globulin chain and a heme group which is bind to heme binding pocket of
the chain.
Heme is a tightly bound prosthetic group.
Heme group serve to reversible bind of oxygen. Hb can carry 4 oxygen molecules.
• Structure of heme.
Fe2+ bind in center of heme molecule by 4 nitrogen of porphyrin ring.Fe2+ form 2 additional
bonds on each side of the planar porphyrin ring.one for a histidine residue of the globulin
chain and the other for binding of oxygen.
• Co-operative binding of oxygen
Binding of oxygen at 1 heme group increases the oxygen affinity for other remaining heme
groups in the same Hb molecule.
Globulin chains are held by non-covalent interactions.
Hb tetramer composed of identical dimers. (αβ)1 and (αβ)2
2 polypeptide chains within each dimer are held tightly together primarily by hydrophobic
interactions and form stable αβ dimer.
Ionic and hydrogen bonds are present between 2 dimers So 2 dimers are stabled primarily by polar
bonds.
2 dimers are able to move with respect to each other. According to the degree of mobility there are
2 forms.
T form and R form
• T form:-
Deoxy form of hemoglobin.
Taut/tense (T) form.
2 αβ dimers interact by a network of polar bonds that constrain the movement of
polypeptide chains.
T form is the low oxygen affinity form of Hb.
• R form:-
Oxyhemoglobin form.
Relaxed(R) form.
2 dimers held by weak interactions and have more freedom of movement.
High affinity for oxygen.
Main function of Hb is to transport oxygen from lungs to the capillaries of tissues.
Oxygen binding property of hemoglobin is regulated by interaction with allosteric effectors.
Ex:- H
+ , CO2 , 2,3 bisphospoglycerate.
Hb can transport H+ and CO2 from the tissues to the lungs.

Presented by 15th Batch 378 FHCS | EUSL


14TH BATCH (EME) - MCQ

01. Lymphatic capillaries,


A) Are blind ended tubules
B) Are absent in bone marrow
C) Are less permeable than that of blood capillaries
D) Formed by multiple layers of endothelial cells
E) Have anchoring filaments

02. Cells that mediate natural immunity include,


A) B-lymphocytes
B) Dendrites cells
C) Monocytes
D) Natural killer cells
E) Plasma cells

03. Tissue macrophages,


A) Are activated by T lymphocytes.
B) Are the monocytes that enter to the tissues from the circulation
C) Are the source of humeral antibodies
D) Include Kupffer cells of the liver
E) Their lifespan is about 3 days

04. Regarding blood volume in normal adult,


A) Form elements contribute 55% of the blood volume
B) About 50% of the body weight
C) It can be measured by diluted method using tritium oxide
D) It is maintained by hormonal mechanism
E) It is reducing in severe diarrhea

05. Serum protein include,


A) Albumin
B) Ferritin
C) Fibrinogen
D) Globulin
E) Prothrombin

06. Regarding plasma osmolality,


A) It has osmotic pressure of 32 mmHg.
B) It is about 290 mOsm/kg.
C) 0.9 NaCl is isotonic.
D) Reduced immediately after a hemorrhage.
E) Plasma proteins are mainly responsible for its total osmolarity.

07. Regarding clotting factors,


A) All clotting factors are vitamin K dependent
B) All clotting factors are proteins
Presented by 15th Batch 379 FHCS | EUSL
C) All are produced by liver
D) Heparin is a naturally occurring anticoagulant
E) Platelet factors initiate the intravascular clotting

08. Regarding ABO blood grouping,


A) AB negative is the rarest blood group.
B) About 85% of world's population are Rh+
C) Agglutinogens are find exclusively on the RBC membrane
D) Agglutinins are produced only in response to transfusion of red cells containing agglutinogens.
E) Rh negative is the safest blood for transfusion.

09. Delta ALA synthase in heme biosynthesis pathway,


A) Is the rate limiting step of the pathway.
B) Is decreased by iron in erythrocyte.
C) Is decreased in liver in individuals treated with certain drugs such as the barbiturate and phenobarbital.
D) Occurs in the cytosol.
E) Pyridoxal phosphate act as a coenzyme in this reaction.

10. Albumin transports


A) Bilirubin
B) Calcium
C) Fatty Acids
D) Retinol
E) Thyroxine

SBR
11. Human blood in normal condition,
A) Eosinophil is the most common type of WBC
B) Lymphocytes are greater than neutrophils
C) There are more platelets than RBC
D) There are more WBC than RBC
E) Iron mostly in hemoglobin

12. A hematocrit of 43% means that in the sample of blood analyzed


A) 43% of the hemoglobin is in the plasma
B) 43% of the hemoglobin is in the red blood cell
C) 43% of the total blood volume is made up of blood
D) 43% of the total blood volume is made up of WBC, RBC and platelets
E) 43% of the formed elements in the blood are red blood cells

13. Which of the following contributes most to colloid osmotic pressure?


A) Albumin
B) Globulin
C) Fibrinogen
D) Gamma globulin
E) Prothrombin

14. What is the primary abnormality in Wilson’s disease


A) Decreased excretion of copper in to bile
B) Increased intestinal absorption of copper

Presented by 15th Batch 380 FHCS | EUSL


C) Low serum total copper
D) Reduce synthesis of ceruloplasmin
E) Reduce urinary excretion of copper

15. Peyer's patches are aggregates of lymphatic nodules found in the area of the,
A) Small intestine
B) Appendix
C) Terminal ileum
D) Jejunum
E) Large intestine

14TH BATCH (EME) - SEQ

01.
1.1. Name the two most important hormones involved in the regulation of blood volume. (20 Marks)
1.2. Briefly explain the mechanisms by which the above hormones restore blood volume following severe diarrhea
and vomiting. (80 Marks)
02.
2.1. A healthy young male living in Colombo, and having an RBC count of 4.92 million/ cm3. He went to spend his
holiday in Horton Plains (altitude - 2,300 m, 7,546 ft) in Nuwara Eliya District. At the end of his one month vacation
his RBC count was found to be 6.11 million/ cm3.
Explain the physiologic mechanisms leading to a significant increase in RBC count after one month stay in Horton
Plains. (50 Marks)
2.2. Briefly describe about sickle cell anemia. (35 Marks)
2.3. Draw and label the microscopic features of a lymph node. (15 Marks)

Answers
EME

MCQ

01 A- T theory
B- T theory
C- F capable of returning leaked protein from plasma.
D- F single layer of endothelium.
E- T theory
02 A- F adaptive immunity
B- T innate immunity
C- T innate immunity
D- T innate immunity
E- F adaptive immunity
03 A- T
B- T
C- T functioning as antigen presenting cells.
D- T
E- F lifespan is uncertain check wheatear’s histology
04 A- F
B- F 8%
Presented by 15th Batch 381 FHCS | EUSL
C- F
D- T
E- T
05 A- T Serum contain all the components as same in plasma except clotting factors
B- F
C- T
D- T
E- F
06 A- F 25.4mmHg
B- T theory
C- T
D- F isotonic loss so not much change in plasma osmolarity.
E- F their contribution is normally less than 2mOsmoles.
07 A- F not all factors ii,vii,ix,x
B- F not all EX: calcium ion 4th factor.
C- F some are produced by tissues during damage and platelets.
D- T
E- T refer the clotting cascade pathway
08 A- T theory
B- F more than 85%.
C- F some are found in some specific organs as well. HLA antigen
D- F after birth produced in response to gut bacteria. Refer Ganong’s physiology
E- F no blood group is safe universal transfusion. proper matching transfusion is
always safety suggestive.
09 A- T theory
B- T theory
C- F it induces rather than decreases.
D- F occurs in mitochondria
E- T theory
10 A- T Substances transported by albumin - Hormones( steroids & thyroid), Fatty
acids, Bile salts, Bile acid, Bilirubin, Ca2+, Mg2+, Cu, Zn, Drugs, plasma
tryptophan
B- T
C- T
D- T
E- T
11 E could be the answer
12 E could be the answer.
13 A is the answer.
14 D is the answer. Reduced production of ceruloplasmin which is important for copper
transport. Its due to the deficiency of ATPase enzyme. So, this causes accumulation of
copper in kidney liver and brain due to the transporter deficiency.
15 C is the answer. theory

SEQ
01)
1.1) ADH & Aldosterone

Presented by 15th Batch 382 FHCS | EUSL


1.2)
Severe diarrhoea and vomiting loss the body water with electrolytes. Internal environment of the body is
in the homeostasis. When a huge loss of water happen it affects bodies function and even can cause
death due to severe dehydration. Severe diarrhoea causes huge loss of water and bicarbonate. loss of
water of the cells causes dehydration. So the cell absorb water from the circulatory vessel. So the blood
volume decreases. decreased blood volume result decrease blood pressure. decrease blood pressure
results in the stimulation of baroreceptors in carotid sinus and aorta. That stimulates posterior pituitary
and secrete A D H more ADH works on the renal tubules and increases sodium ion and water retention by
concentrating urine to decrease urine output increased tonicity of Plasma also increases ADH secretion by
stimulating osmoreceptors in supraoptic Nucleus. In severe vomiting also body water is lost with H +
mainly decreased blood pressure activate renin angiotensin Aldosterone system. hypotension is sense by
kidney to activate renin angiotensin Aldosterone system. Renin will be secreted. Renin converts
angiotensinogen into angiotensin. Angiotensin 1 is converted into angiotensin 2 by angiotensin converting
enzyme. Angiotensin 2 activates the secretion of aldosterone. Aldosterone increasing Sodium and water
reabsorption. Increased water retention in Renal tubules restore the blood volume. angiotensin 2 and
stimulation of a osmoreceptors in hypothalamus can activate the third mechanism. increased water loss
increases, the tonicity of blood plasma, and that stimulates osmoreceptors. when the thirst centres in the
hypothalamus activated it increases the desire for water intake. blood volume is restored by increasing
water intake. But the thirst mechanism activation is not practical in vomiting patients as increased vote
drinking also can trigger vomiting. although body activate those mechanism to reach the blood volume, it
is more important to monitor the patient closely and give rehydration Fluids to restore the ion loss.
02)
2.1)
When a healthy individual spend time in high altitude the body responds to their environmental
conditions there. in high altitude oxygen partial pressure in the atmosphere is less than sea level area
such as Colombo. so perfusion in the alveolar epithelium is decreased. hypoxia occurs. So tissues don't get
enough oxygen and oxygen partial pressure in the arterial blood is decreased. To increase the oxygen
partial pressure ventilation is increased as a short-term response as this person is staying there for such a
long time. Erythropoietin stimulates increase in Erythrocyte number. Erythropoietin is a hormone which is
important for erythropoiesis. So the RBC count is increased to increase the amount of hemoglobin present
in blood. It increases oxygen carrying capacity and hypoxia is relieved increasing of RBC count is called
polycythemia the process of red cell maturation take approximately two weeks and committed
themselves become totipotent stem cells and then committed stem cells until the generation to mature
RBC depends on Erythropoietin. they are proerythroblast early Normoblast intermediate Normoblast late
Normoblast and then reticulocytes which finally become mature cells. as all those maturation process are
the depend on erythropoietin, hypoxia-induced erythropoietin secretion increases the RBC count by
increased maturation rate.
2.2)
Sickle Cell anaemia is an blood disorder which is caused by point mutation in beta globulin protein
producing gene. Sickle Cell anaemia causes the production of abnormal shaped red cells normal red blood
cells as a lifespan about hundred to 120 days Sickle Cell has a shorter lifespan, which is about 10 to 20
days due to show the lifespan of RBC the RBC is break down easily and that causes condition that results
in shortness of breath fatigue delayed growth and like normal RBC is Sickle cells are rigid and sticky. They
Presented by 15th Batch 383 FHCS | EUSL
make lumped together and stick to the world of blood vessels causing obstruction of small vessels and
subsequent reduced oxygen supply to tissues. Autosomal recessive disorder. most of the time occurs as
non conservative missense when glutamate is replaced by valine in the Amino acid sequence. Sickle cells
carry oxygen, they changes shape when they are the oxygenated repeated sickling weakened the cell
membrane and causes premature destruction. Free hb release recycled by haptoglobin. Anaemia
increases production of reticulocytes and due to new bone formation expansion of medullary cavity of
skull occurs. extramedullary erythropoiesis can result in hepatomegaly.
2.3)

15TH BATCH (EME) - MCQ

1) Regarding blood volume in healthy adults,


A. It can be measured by dilution principal using titrated water
B. Composed of 5% of body weight
C. Increased in pregnancy
D. Increased in strenuous exercises
E. It is regulated by renal excretion of sodium and water

2) Hemolytic anemia is characterized by,


A. Increased serum conjugated bilirubin
B. Reduced hematocrits
C. Reduced MCV
D. Reticulocytosis
E. Yellow skin and mucous membrane

3) Regarding blood group,


A. Anti-A and anti-B antibodies can cross the placental barrier
B. Anti-A and anti-B are natural antibodies produced in fetal life
C. Blood group antigens are lipoproteins found in plasma
D. Father of blood group of A+ can have a child of blood A-
E. O+ is the most commonly found blood group in the world

4) Compared to adult Hemoglobin fetal hemoglobin,


A. About 3-7% hemoglobin in adults
B. Give rise to physiological jaundice in newborn babies
C. Affinity to 2,3 BPG
Presented by 15th Batch 384 FHCS | EUSL
D. Affinity to O2
E. Hb A1c transport O2 with ferrous ion

5) Components of innate immunity include,


A. Activated T cells
B. Bacteria commensals in the skin
C. Complement system
D. Gastric acids
E. Phagocytic action of neutrophils

6) Antigen presenting cells include,


A. B Lymphocyte
B. Dendritic cells
C. Macrophage
D. Neutrophils
E. T lymphocyte

7) Lymphatic capillaries,
A. Are blind ended
B. Are present in bone marrow
C. Have anchoring filaments
D. Have overlapped endothelial cells
E. Permeability less than blood vessel

8) Secondary lymphoid organs are,


A. Bone marrow
B. liver
C. lymph nodes
D. Spleen
E. Thymus

9) Which of the following Hb are normal except,


A. HbA
B. Hb A1c
C. HbF
D. HbS
E. HbM

10) Functions of albumin include,


A. Maintain oncotic pressure
B. Maintain pH of the body
C. Source of amino acids
D.
E. Transport cholesterol

SBR
11) Largest contribution to blood viscosity is by?
A. Electrolytes
B. Plasma protein

Presented by 15th Batch 385 FHCS | EUSL


C. Platelets
D. RBCs
E. WBCs

12) The vitamin closely associated with synthesis of clotting factors is?
A. Vitamin A
B. Vitamin B12
C. Vitamin C
D. Vitamin E
E. Vitamin K

13) There are differences and similarities between Primary and secondary immune response. Which of the
following is similar to Primary and secondary immune responses?
A. Amount of Antibody formation
B. Occurs after antigen exposed
C. Predominant IgG
D. Predominant IgM
E. Time taken to Antibody formation

14) Which of the following is not an acute phase protein?


A. Alpha 1 antitrypsin
B. C Reactive protein
C. Ceruloplasmin
D. Collagen
E. Haptoglobin

15) Which is the best description for lymph?


A. It contains large number of macrophages
B. Its composition is similar to plasma
C. Formed by diffusion and absorption of tissue fluid
D. Its hydrostatic pressure is greater than interstitial fluid
E. Its rate of flow is greater than blood

15TH BATCH (EME) - SEQ

01.
1.1 Briefly describe the important changes that are seen during the transformation of a haemopoietic stem cell into
a mature Red Blood Cell. (15marks)
1.2 Name the factors essential for maturation of the stem cell into a normal red cell and their specific role(25marks)
1.3 Briefly describe the role of platelets in hemostasis. (10 marks)
1.4 Explain why damage to one eye (exposure of lens protein to immune system) leads to development of
cataract in the other eye. (20 marks)
1.5 Write a brief note on Thalassemia

Answers

1)
A. T
Presented by 15th Batch 386 FHCS | EUSL
Total body water -> Radioactive H2O, Deutarium, Antipyrine
ECF -> Radioactive Na, Radioactive Cl, Iothalamate
Plasma -> Evans blue dye
B. F
8%
C. T
D. T
E. T
RAAS

2)
A. F
Unconjugated bilirubin will increase
B. T
RBC will decrease
PCV will increase
C. F
Normocytic
D. T
To compensate reduced RBC, Reticulocytes enter into the blood.
E. T

3)
A. F
D antibodies only can cross
B. T
C. F
Antigens present in RBC membrane
Antibodies present in plasma
D. F
+
Rh dominant
E. T
F.
4)
A. F
0.8 – 2 %
B. T
C. F
Low affinity to 2,3 BPG
D. T
Increase O2 affinity than Hb A1 / Hb A
E. T
Hb A,C -> Glycosylated Hb

5)
A. F
B. T

Presented by 15th Batch 387 FHCS | EUSL


C. T
D. T
E. F

6)
A. T
B. T
Antigen presenting cells - B Lymphocytes
- Macrophage
- Dendritic cells
C. T
D. F
E. F

7)
A. T
B. F
Absent in avascular structures, Brain, Spinal cord, Spleenic pulp, Bone marrow
C. T
D. T
E. F
More permeable

8)
A. F
B. F
C. T
D. T
E. F
Primary -> Stem cells divided and become immunocompetent
Secondary -> Show immune response

9)
A. F
B. F
C. F
D. T
E. T
Normal Hb -> Hb A, Hb A2, Hb F, Hb A1C
Abnormal Hb -> Hb S, Methemoglobin

10)
A. T
B. T
Maintain Acid Base balance.
C. T
D.

Presented by 15th Batch 388 FHCS | EUSL


E. T

11) D
12) E
13) B
14) D
Acute phase protein -> Positive -> C reactive protein, Ceruloplasmin, α1 Antitrypsin,
α2 Macroglobulin
-> Negative -> Albumin, Prealbumin, Transferrin, Retinol binding protein
15) B

SEQ
1)
1.1.
The process of transformation of RBC from proerythroblast to mature RBC is erythropoiesis. It takes 7 days.
Uncommited stem cell -> Committed stem cell -> Proerythroblast -> Early normoblast ->
Intermediate normoblast -> Late normoblast -> Mature RBC
Proerythroblast -> Synthesis of Hb starts.
Early normoblast-> Nucleoli disappears.
Intermediate -> Hb start to appears.
Late normoblast-> Nucleus disappears.
Reticulocyte -> Reticulum contain and cell enter to the capillaries from the site of production.
Mature RBC -> Reticular network disappears. And cell become biconcave.
Usually erythropoiesis occurs in bonemarrow in adults.

1.2.
Maturation factors – Vitamin B9, B12, Intrinsic factor
Vitamin B9 -> Syenthesis and maturation of nucleic acids.
-> Essential for maturation of RBC.
-> Absence of B9 - Decrease synthesis of DNA -> Decrease DNA replication -> Decrease cellular
replication -> Large cells and reduced no of cells -> Megaloblastic anemia.
Vitamin B12 -> Synthesis of DNA in RBC.
-> Deficiency – Decrease maturation of RBC -> Decrease DNA replication -> Decrease cellular
replication -> Macrocytic anemia
Intrinsic factors of castle -> Essential for absorption of Vitamin B12 .
-> Deficiency leads to poor absorption of Vitamin B12.

1.3.
Platelets produced in bonemarrow. Hemostasis means active sequence of responces that stop bleeding.
Hemostasis -> Vascular spasm
-> Platelet plug formation
-> Blood coagulation
Vascular spasm - Platelets release thromboxane A2. It helps to vasoconstriction.
Platelet plug formation - To form platelet plug, platelet adhesion, platelet activation & aggregation have to occur .
Platelet adhesion occurs when VWF connects exposed collagen to platelets. Then platelets release ADP,
Thromboxane A2. They help to activation of platelets by changing the shape of platelets and form pseudopodia.

Presented by 15th Batch 389 FHCS | EUSL


Platelet aggregationoccurs when fibrinogen receptors on activated platelets bind to fibrinogen, connective platelets
to one another. The accumulating mass of platelets form a platelet plug.
Blood coagulation - Temporary platelet plug changes into definitive clot. For blood coagulation clotting factors
are essential. Clotting factor 3( Thromboplastin), 4 (Calsium ions), 5(Labile factor) & 13 are syenthesised by
platelets. They help for blood clotting by facilitated extrinsic, intrinsic & common pathway for clotting.

1.5.
Thalassemia is a hereditary haemolytic disease. It’s an autosomal recessive disease caused by insufficient synthesis
of α / β chains in globulin protein. In thalassemia total absence of either chains α / β occur. It is a disorder formed
by mutation in the genes. They may include entire gene deletion/ substitution/ deletion of one to many nucleotides
in DNA.
2 types -> α Thalasemia – Insufficient production of α globulin chain.
-> β Thalasemia - Insufficient production of β globulin chain.

-> Thalasemia major -> Homozygous – Severe


-> Thalasemia minor -> Heterozygous – Minimaly asymptomatic

Lack of coordination in α & β chains -> Impaired Hb syenthesis -> Formation of insoluble aggregates of excess
chain -> Damages the RBC -> Reticuloendothelial cells detect those damaged RBC -> Destruction of damaged RBC ->
Hemolytic anemia

Presented by 15th Batch 390 FHCS | EUSL


PHASE 01 QUESTIONS
08TH BATCH
8TH BATCH (PROPER) - MCQ

35) Lymphocytes
A) Absolute count increase in pyogenic infection
B) Has a lobulated nucleus
C) Has longest lifespan out of white cells
D) is smaller in size than basophils
E) Transformed to plasma cells before making antibodies

36) IgG
A) Antibody formed against Rh factor when sensitized
B) Monomer
C) Neutralize the toxins
D) Response is greater in primary immune response than secondary immune response
E) Involves in type ІІІ hypersensitivity reactions

37) Complement system


A) Opsonization of bacteria
B) Chemo taxis of neutrophils when activated
C) Activates natural killer cells
D) Activated by antigen-antibody complex
E) Grouped under innate immunity

38) Platelets
A) Derived from megakaryocytes
B) Activate and release thromboxane A2
C) Prevent ischemic heart disease
D) Reduced in thrombosthenia
E) Are activated and attach to collagen

39) Vitamin B12


A) Deficiency cause macrocytic anemia
B) Deficiency in gastrectomy
C) Deficiency in terminal ileum resection
D) Cause to neurological symptoms
E) Formed by bacteria which cannot be absorbed

40) Erythropoietin concentration in blood is high


A) Iron deficiency anemia
B) Polycythemia
C) Chronic Kidney Disease
D) Lung fibrosis
E) High altitude

SBR
41) IgG
A) Present in mucosal cell barrier
B) Crosses the placenta
Presented by 15th Batch 391 FHCS | EUSL
C) Action is high in primary exposure than secondary
D) Produced by activated T lymphocytes
E) Contributes for Rh incompatibility

54) Edema is a feature in lymphatic obstruction. What is the main disturbance to starling forces in lymphatic
edema?
A) Decrease hydrostatic pressure interstitial space
B) Increase capillary hydrostatic pressure at arteriolar end
C) Increase capillary hydrostatic pressure at venular end
D) Decrease oncotic pressure in interstitial space
E) Decrease plasma oncotic pressure

55) Interstitial fluid is different to plasma though both are belongs to ECF. What is the most significant feature seen
in plasma?
A) Contain more Na+ than interstitial fluid
B) Higher buffering capacity than interstitial fluid
C) Has more protein than interstitial fluid
D) Electrically more negative than interstitial fluid
E) More viscous than interstitial fluid

57) 15-year-old boy came with fever and swelling on his leg after Trauma) His total WBC is 20 × 109/L, with
Neutrophil-18 × 109/L, with Lymphocytes- 2 × 109/What is the diagnosis?
A) Parasitic infection
B) Pyogenic bacterial infection
C) Tetanus
D) Tuberculosis
E) Viral infection

58) Mother is having O- while the blood group of the child is A+. What is the blood group of the father?
A) A-
B) AB-
C) A+
D) B+
E) O+

59) 35-year-old female came with having Hb% of 9g/dL & hypochromic microcytic blood picture. Her iron binding
capacity is higher than normal. What is the most likely diagnosis?
A)
B) Iron deficiency anemia
C)
D) Sideroblastic anemia
E)

60) A 40-year-old female presented with yellowish discoloration of sclera with anemia. It was suspected that she is
having hemolytic anemia. What is the most suitable test for conform the diagnosis of hemolytic anemia?
A) Hb%
B)
C)
D)
E) Urinary urobilinogen

Presented by 15th Batch 392 FHCS | EUSL


8TH BATCH (PROPER) - SEQ

01)
1.1.State 3 main groups of plasma proteins (10 marks)
1.2.Describe the role of plasma protein in,
1.2.2.Blood coagulation
02) Explain why,
2.1.PCV higher than normal in cyanotic heart diseases (30 marks)
2.2.Rhin compatibity is not seen in baby born to 𝑅ℎ+ mother (40 marks)
2.3.Macrocytic anaemia seen in folic acid deficiency (30 marks)

8TH BATCH (REPEAT) - SEQ

01)Explain why ,
1.1.Polycythemia is a feature of people living in high altitude. (30 marks)
1.2.Low dose aspirin is given to treat ischemic heart disease (40 marks)

8TH BATCH (PROPER) – MCQ ANSWERS

8th batch (proper)-MCQ

35) A) F in viral infections


B) F
C) T
D) T smallest WBC - Lymphocytes Largest - Monocytes
E) T B lymphocytes

36) A)T Ig G can cross the placenta. As a result, the Rh incompatibility may occur in Rh positive
fetus in sensitised Rh negative mothers.
B)T
C) T Tetanus and diphtheria
D) F Primary immune response- IgM
Secondary immune response – IgG
E) T In type III hypersensitivity reactions IgG, IgA and IgM

37) A) T Functions of complement system;


1. Act as a opsonin
2. Chemotaxis
3. Aid inflammation
4. Lysis of microbes
B)T
C)T
D) T Activation of complement system occurs via 3different pathways;
1. Activation by antigen-antibody
2. Activation by the antigen itself
Presented by 15th Batch 393 FHCS | EUSL
3. Mannose binding lectin pathway
E)T

38) A- T
B- T Thromboxane A2 – promotes platelet aggregation
C- F
D- F Thrombasthenia – defective function of platelets.
Thrombocytopenia – reduction in platelet count.
E- T Platelet plug formation

39) A-T Vit. B12 is essential to complete erythropoiesis


B-T Intrinsic factor secreted by gastric parietal cells is essential for absorption of B12
in the terminal ileum.
C- T
D- T Spina bifida and anencephaly
E- F Synthesized by bacterial flora and is absorbed.

40) A-T
B- T
C- F Erythropoietin is synthesized by the peritubular capillaries of the kidney
D- T Due to hypoxic stimulation
E- T Due to hypoxic stimulation

SBR
41) Answer- B/E
54) Answer- E
In lymphatic obstruction, the filtered proteins do not enter the blood reducing the
the plasma oncotic pressure.

55) Answer- C
57) Answer - B
WBC – 4.11 × 109
Neutrophil – 5.4 × 109
Lymphocytes – 2.7 × 109
Neutrophil count increases in bacterial infection
58) Answer- C
59) Answer- B
60) Answer- E

8TH BATCH (PROPER) - SEQ

01)
1.1• Albumin
• Globulin
• Fibrinogen
1.2

Presented by 15th Batch 394 FHCS | EUSL


1.2.2. Arrest of bleeding is called haemostasis which depends on vascular spasm,platelet plug formation and
blood clotting (cogulation).
Fibrinogen is the major protein which involves in blood coagulation. Conversion of Fibrinogen to insoluble
Fibrin is essential for blood clotting,.
Clotting response begins to appear within 15-20 seconds after the trauma.
Takes place via a complex cascade of plasma proteins called clotting factors.
Basic steps in clotting response are,
1- in response to ruptured of the vessel , cascade of plasma proteins get activated.
2- Prothrombin--------------------------------------------------->> thrombin
They all converge to activate factor X (Prothrombin activator)
3- Fibrinogen--‐------------------thrombin------------------------->> Fibrin
Fibrin fibres enmesh platelets, blood cells and plasma to form the definitive blood clot.
4- After a few minutes, clot begins to contract and release a fluid called serum.

02)
2.1cyanotic heart disease refers to a group of many different heart defects that are present at birth
(congenital). They result in a low blood oxygen level.
So Count of RBC increase to increase the oxygen capacity as a result PCV increase.
PCV is the proportion of blood occupied by RBCs expressed in percentage.

2.2 No one has Rh antibodies in their blood from birth.


Rh + individuals never develop Rh antibodies in their body because they have D antigen.
But Rh- individuals develop Rh antibodies if exposed to Rh+ blood.
Rh incompatibility occurs when a sensitised Rh negative mothers carries a Rh positive fetus. Antibodies for
Rh factor are IgG type which is a monomer. If Rh negative mother is sensitized, she carries antibodies for the Rh
factor. As IgG is a smaller molecule, it can cross the placenta and reach the foetal blood. If the foetus is Rh positive
fetus, the crossed Rh antibodies will react with the fetal red cells and causes hemolysis.
2.3.anemia is defined as reduction in haemoglobin concentration in blood lower than the normal level relative
to the age, sex and physiological status (pregnancy and altitude)
Folic acid, vitamin B12, intrinsic factor from stomach mucosa are required to synthesis of red blood cells. Loss
of any one of these can lead to slow reproduction of erythroblasts in the bone marrow. As a result the RBCs grow
too large , with odd shapes and are called megaloblasts. In megaloblastic anemia the RBCs are not matured .

Presented by 15th Batch 395 FHCS | EUSL


8TH BATCH( REPEAT) -SEQ
01)
1.1. Polycythemia is a feature of people living in high altitude
• With increasing altitude the barometric pressure is decrease.
• It cause to decrease the o2 partial pressure
•Due to that alveolar po2 is decrease
• Normal tissue level o2 concentration also decrease.
• Deficiency of tissue level o2
• Hypoxia occur
• Hypoxia stimulate secretion of erythropoietin in kidney (85%) and liver (15%)

•Erythropoietin act on heamopoietic stem cell and stimulate differentiate into
Proerythroblast
• Stimulate release of reticulocyte into capillary blood
• So formation of red blood cells more than normal amount in the body
• polycythemia occur

1.2. low dose aspirin is given to treat ischemic heart disease


• Ischemic heart disease is reduced blood supply to the heart due to obstruction of
Coronary arteries
•Obstruction occur due to formation of blood clots
•Thromboxane A2 is produced by the cyclooxygenase -1 in activated platelets
•It promote platelet adherence and aggregation circulating platelets and contracting
smooth muscles in vessels.
•It cause to prom
endothelial cells.
• It inhibit the platelet aggregation and stimulate vasodilation
• It inhibit the thrombogenesis
•Aspirin has anti – thrombogenic effect
•It inhibit the thromboxine synthesis and prostacyclin synthesis
•By irreversible acetylation of cyclooxygenase – 1 (cox 1) in platelets and cox -2 in
Vascular endothelial cells.
• Inhibition of cox – 1cannot overcome but cox -2 can be over come
• Increase cox -2 increase prostacycline synthesis, therefore reduce synthesis of
thrombi.
•So aspirin use to lower the risk of ischemic heart disease
9TH BATCH
9TH BATCH (PROPER) – MCQ

20) Erythrocytes
A) Erythrocytes are formed in bone marrow
B) Lifetime is 120 days
C) Production is stimulated by erythropoietin
D) Is dumbell in shape
E) is red in color

Presented by 15th Batch 396 FHCS | EUSL


21) Sweat
A) Contains less plasma
B) Secretion stimulate in adrenergic sympathetic stimulation
C) Has an antibacterial activity
D) Helps in the heat dissipation when temperature is below 37c
E) Has an antibacterial activity
F) Secretion is feature in hemorrhagic shock

29) Acute phase proteins


A) Are produced response to inflammation
B) Inhibit growth of microbes
C) Produced by granulocytes
D) Increase of ESR
E) Include interleukins, protein C and ferritin

30) Hemoglobin in adult


A) Synthesized in liver
B) Has O2 binding capacity of 19)5 ml/g
C) Is degraded in the reticuloendothelial system
D) Has 2 alpha chains and 2 gamma chains
E) Is completely deoxygenated in the tissues

31) Packed Cell Volume


A) Increase of high altitude
B) Decrease soon after hemorrhage
C) Increase after dehydration
D) Is measured using westergren tube
E) Is required for calculation of MCHC

33) T/F
A) Erythrocyte production is increased in high altitude
B) Reticulocytes are destructed by reticuloendothelial system
C) Haemopoisis in adult confined to red bone marrow
D) Average lifespan of erythrocyte in a normal human is 10 days
E) Conjugated bilirubin level is increased in hemolytic jaundice

34) IgG
A) Produced in primary hypersensitivity
B) Is a dimer
C) Cross placental barrier
D) Important in producing antibodies against tetanus virus
E) Most abundant

SBR
47) There is an increase in eosinophil count elevate a serum IgE. What is the most likely condition?
A) Insect bite
B) Bacterial infection
C) Fungi infection
D) Parasitic infection
E) Viral infection

Presented by 15th Batch 397 FHCS | EUSL


55) Which of the following isn’t true reference of hemoglobin?
A) Synthesize in endothelial system of bone marrow & spleen
B) Is included in proerythroblast
C) Degraded to form bile pigment
D) Its ability to carry O2 affected by PH
E) Buffering capacity is greater than that plasma protein

56) Increase in acute renal failure


A) Basophil
B) Eosinophil
C) Neutrophil
D) Monocyte
E) Lymphocyte

58) True about hemoglobin


A) Concentration Is Measured By Westergen Tube
B) Adult Hemoglobin Contains Alpha, Gama Chains
C) In Full Saturation an RBC Contains 19, 5 Mg/dl Hb
D) Is the Main Transporter of the CO2
E) Is More Darker When It Binds With CO

9TH BATCH (PROPER) – SEQ

01. A patient who came to the clinic complained of the following


a. passing dark urine (30 marks)
b. pale and bulk stool (35 marks)
c. prolonged bleeding (35 marks)

9TH BATCH (REPEAT) – SEQ

01.
1.1 Define “Natural immunity”. (20 marks)
1.2 Briefly describe three different mechanisms of natural immunity. (30 marks)
1.3 Explain the Physiological principle involved in vaccination and the benefits of vaccination. (50 marks)

9TH BATCH (PROPER) – MCQ ANSWERS

20)
A) T
B) T
C) T Erythropoietin is produced in the kidney
D) T In cross section
E) T

21)
Presented by 15th Batch 398 FHCS | EUSL
A) F
B) F Sympathetic cholinergic receptors
C) T
D) F Normal body temperature is 37C
E) F Cold pale skin

29)
A) T Acute phase proteins, A class of proteins whose plasma concentration increase (+ APPs) or decrease (-APPs) in
response to inflammation
B) T Reactive protein C by opsonin on microbes
C) F synthesized in liver
D) T APPs causes increased plasma viscosity, increased ESR
E) T C reactive protein, serum amyloid A, ferritin, Interleukins

30)
A) F In adult only red bone marrow, In embryo in liver, spleen, lymph nodes and bone marrow.
B) T Ganong pg. 640
C) T Liver and spleen
D) F 2 alpha chains and 2 beta chains.
E) F Oxygen saturation in venous blood = 70%, Oxygen saturation in arterial blood = 97%

31)
Packed cell volume = Hematocrit (Red cell volume fraction)
PCV= 45%/ hematocrit = 0.45
A) T High altitude ->polycythemia -> PCV increases
B) T PCV low in ->Cell destruction , Blood loss , All types of anemia
C) T PCV increase in Polycythemia, Dehydration
D) F PCV measure using winthrobe tube ESR can measure using Winthrobe tube / westergen tube
E) T MCHC= MCH/MCV= Hb%*100/pcv (Hemoglobin concentration in one RBC)

33)
A) T In high altitudes, hypoxic stimulation increases the erythropoietin Production in the kidneys.
B) T If the reticulocyte count is increased in blood they are destructed By the reticuloendothelial system.
C) T
D) F 120 days
E) F Unconjugated bilirubin level

34)
A) F Ig G
B) F Is a monomer
C) T
D) T Important in preventing tetanus and diphtheria as those diseases Are caused by toxins produced by bacteria.
E) T GAMED

47) D

54) ANSWER – D
Serum is Blood plasma without fibrinogen

55) ANSWER – B

Presented by 15th Batch 399 FHCS | EUSL


Proerythroblast is the 1st cell derived from CFU- E. Has no Hb

56) ANSWER-C In chronic renal failure monocytes increase and neutrophils Decrease.In acute renal failure due to
sudden tissue necrosis neutrophil Count increase.

58) ANSWER-C Arterial O2 is 97.5% saturated due to the presence of Physiological shunts.

Answers

MCQ
20)

A) T

B) T

C) T. Erythropoietin is produced in the kidney

D) T In cross section

E) T

21)

A) F

B) F Sympathetic cholinergic receptors

C) T

D) F Normal body temperature is 37C

E) F. Cold pale skin

29)

A) T Acute phase proteins

- A class of proteins whose plasma concentration increase

(+ APPs) or decrease (-APPs) in response to inflammation

B) T Reactive protein C by opsonin on microbes

C) F synthesized in liver

D) T APPs causes increasing in plasma viscosity

Increasing in ESR

E) T C reactive protein, serum amyloid A, ferritin, Interleukins

30)

A) F In adult only red bone marrow

In embryo in liver, spleen, lymph nodes and bone marrow.

Presented by 15th Batch 400 FHCS | EUSL


B) T Ganong pg. 640

C) T Liver and spleen

D) F 2 alpha chains and 2 beta chains.

E) F Oxygen saturation in venous blood = 70%

Oxygen saturation in arterial blood = 97%

31)

Packed cell volume = Haematocrit (Red cell volume fraction)

PCV= 45%/ haematocrit = 0.45

A) T High altitude - - > polycythaemia - - > PCV increases

B) T PCV low in - - > Cell destruction

Blood loss

All types of anaemia

C) T PCV high in - - > Polycythaemia

Dehydration

(In dehydration - - > plasma volume⬇️ fraction of RBC ⬆️so PCV⬆️)

D) F PCV measure using - - > winthrobe tube

ESR can measure using - - >Winthrobe tube / westergen tube

E) T 𝑀𝐶𝐻𝐶 = (𝑇𝑜𝑡𝑎𝑙 hemoglobin /𝑃𝐶𝑉) ×100%

(Hemoglobin concentration in one RBC)

33)

A) T In high altitudes, hypoxic stimulation increases the erythropoietin production in the kidneys.

B) T If the reticulocyte count is increased in blood they are destructed by the reticuloendothelial system.

C) T

D) F 120 days

E) F Unconjugated bilirubin level

34)

A) F Ig G

B) F Is a monomer

C) T

D) T Important in preventing tetanus and diphtheria as those diseases are caused by toxins produced by bacteria.

E) T

47)

ANSWER- D
Presented by 15th Batch 401 FHCS | EUSL
55)

ANSWER – B

Proerythroblast is the 1st cell derived from CFU- E. Has no Hb

56)

ANSWER-C

In chronic renal failure monocytes increase and neutrophils decrease.

In acute renal failure due to sudden tissue necrosis neutrophil count increase.

58)

ANSWER-C

Arterial O2 is 97.5% saturated due to the presence of physiological shunts.

SEQ (PROPER)
Question
ANSWERS

06)

6.1)

- Due to obstruction of bile duct bile cannot enter to the small intestine from Gall bladder

- Bile contains many substances including bile pigments & bile salts.

Process Of Hemolysis

Within the macrophages RBC breakdown into globin chain & heme ring

⬇️

Globin chain breakdown into amino acids

⬇️

Heme ring/ porphyrin ring oxidized by microsomal oxygenasel to biliverdin

⬇️

Biliverdin is reduced by biliverdin reductase to unconjugated bilirubin

⬇️

Unconjugated bilirubin released to plasma

⬇️

Bind with albumin

⬇️
Taken up by liver hepatocytes

Presented by 15th Batch 402 FHCS | EUSL


⬇️

Within liver unconjugated bilirubin conjugated by glucuronyl transferase forming conjugated bilirubin

⬇️

Conjugated bilirubin enter the small intestine with bile

⬇️

In small intestine bilirubin is degraded to urobilinogen by bacterial enzymes.

⬇️

10% of urobilinogen is reabsorbed by liver

⬇️

Remaining urobilinogen breakdown into urobilin & stercobilin. They excreted with feces.

⬇️

Reabsorbed urobilinogen (most) re-excreted into bile circulation.

⬇️

Little amount by pass the liver & excreted with urine.

- In this condition increase the plasma conjugated bilirubin due to obstruction of bile duct.

- Therefore excretion of bilr salt in urine

- There colour of urine become dark.

6.2)

- As mentioned above due to obstruction of bile duct conjugated bilirubin level in plasma increases.

- But entry of co-bilirubin to small intestine is blocked.

- Bile pigments give the unique colour of feces. Due to biliary obstruction bile pigments cannot enter to small
intestine.

- So feces become pale colour.

- Fat absorption is done by the terminal ileum.

- Fat absorption requires presence of bile salts.

- Absence of bile salts prevent fat absorption.

- This leads to bulky stool.

- This condition is called as steatorrhea.

6.3)

- As mentioned in 6.1 biliary obstruction leads to prevent bile acid entry to small intestine.

- Bile acids are required to absorption of fat by small intestine.

- Vitamin K is a fat soluble vitamin.

- Therefore Vitamin K absorption requires fat.

Presented by 15th Batch 403 FHCS | EUSL


- Due to biliary obstruction Vitamin K absorption is impaired.

- Vitamin K deficiency is the result.

- Vitamin K needs to activation of clotting factor II, VII, IX, X

- Due to Vitamin K deficiency these clotting mechanism is impaired.

- So bleeding may prolonged from skin injuries.

SEQ (REPEAT)
01)

1.1)

 Natural immunity is also called innate or , native immunity First line of defence system which blocks the
entry of microbes into the body.
 Once microbe enters body – phagocytosis, NK cells, some proteins, cytokines mechanisms will be activated.
This immune system founds in all multi-cellular organisms including plants.
 Always present; ready to recognize and eliminate microbes
 Do not react against non-microbial substances
 Less specific
 Powerful early defence mechanism
 Cross-talk with adaptive immunity
1.2)

Physical and chemical epithelial barriers are the very early defence system which prevent microbes from entering
into the body.

–Skin, GI, RS

–Produce peptides antibiotics that kill bacteria

–Intraepithelial lymphocytes (express antigen receptors of limited diversity

Once microbes enter into the body some other natural immune mechanisms are activated.
Phagocytes have a major role in natural immune system
–Neutrophils –Monocytes/macrophages- NK cells- dendritic cells.
 Complement system and cytokines are main mechanisms which are activated to response to pathogens.
1- Complement system
• Collection of circulating and membrane associated proteins

• Many are proteolytic enzymes

• Sequential activation occur –Alternative pathway, Classical pathway, Lectin pathway

• Promotes binding to phagocytes

• Chemoattraction

• Insert proteins that alter osmosis

 Activation of complement system


• Alternate pathway

–Activate C3 by binding to microbes


Presented by 15th Batch 404 FHCS | EUSL
• Classical pathway

–With antigen antibody complexes

• Lectin pathway

–Biding with mannose

2- Cytokines
• Dendritic cells, macrophages and other cells secrete

• TNF, IL-1 and chemokine are principal ones

• Communicate between immune cells –Autocrine –Paracrine –Endocrine also

1.3)

Vaccines work by stimulating a response from the immune system to a virus or bacterium. This creates a ‘memory’
in the immune system. This immune memory allows the body to ‘remember’ a specific virus or bacterium, so that it
can protect itself against this virus or bacterium and prevent disease that it causes.

Most vaccines contain a weakened or an inactivated (killed) form of a virus or bacterium, or a small part of the virus
or bacterium that cannot cause disease. This is called an antigen.

When a person gets a vaccine, their immune system recognises the antigen as foreign. This activates the immune
cells so that they kill the disease-causing virus or bacterium and make antibodies against it.

It also activates immune cells - called T-cells and B-cells - in the blood, in the bone marrow and throughout the
body.

Later, if the person comes into contact with the actual virus or bacterium, their immune system will remember it.

It can then produce the right antibodies and activate the right immune cells quickly, to kill the virus or bacterium.
This protects the person from the disease.

Different vaccines bring about different levels of protection. How long protection lasts also depends on the disease
it protects against. Some vaccines can only protect against a disease for a short period and may need booster doses;
for others, immunity can last a lifetime.

10TH BATCH
10TH BATCH (PROPER) – MCQ

19) When compare lymph with blood, the lymph


A) Is formed from interstitial fluid
B) Has no protein
C) Is intracellular fluid
D) Has no blood cells
E) Carries chylomicrons absorption from the small intestine

34) Serum protein include


A) Albumin
B) Fibrinogen
Presented by 15th Batch 405 FHCS | EUSL
C) Gamma globulin
D) Pre albumin
E) Prothrombin

36) Regarding iron deficiency anemia


A) It is accompanied by jaundice
B) It is accompanied by shortness of breath
C) Most common among females than males
D) RBC will have an MCV of above 150 cubic microns
E) RBC are normochromic

37) B12 deficiency results in


A) Hypochromic anemia
B) Megaloblastic anemia
C) Microcytic anemia
D) Pernicious anemia
E) Sickle cell anemia

38) T/F regarding hemoglobin


A) HbA is the major type in adult blood
B) 2, 3-DPG and H+ compete with oxygen to bind with deoxygenated Hemoglobin
C) Vitamin B12 and folic acid are not essential for hemoglobin synthesis
D) Glycosylated hemoglobin {HbAIC} level is a good indicator for monitoring control of diabetes mellitus
E) Abnormal polypeptide chains are produced in haemoglobinpathies

39) Cells involve in pathogenesis of autoimmune disease


A) T cells
B) B cells
C) Neutrophils
D) Mast cells
E) Monocyte

40) Regarding ABO system


A) A person group o is universal recipient
B) A person group B has anti A agglutinin in plasma
C) In bind transfusion incompatible reaction recipient cells lyse by donors agglutinin
D) Incompatible transfusion reaction followed by jaundice
E) A person has AB has anti A and anti B in plasma

55) Clot retraction mediated by


A) Lymphocyte
B) Neutrophil
C) Monocyte
D) Eosinophil
E) Platelets

56) Function of T lymphocytes?


A) Cell mediate immunity
B) Endocytosis
C) Humoral immunity

Presented by 15th Batch 406 FHCS | EUSL


D) Phagocytosis
E) Pinocytosis

57) False regarding spleen function


A) Detoxify alcohol
B) Blood reservoir in emergency
C) Lymphocyte activation
D) Destruction of aged RBCs
E) Remove large particles

10TH BATCH (PROPER) – SEQ

01. Explain the physiological basis of polycythaemia in the followings.


1.1 Chronic obstructive pulmonary disease (50 marks)
1.2 People living at high altitude (50 marks)

10TH BATCH (PROPER) – SEQ


4.
4.1 Describe the process of Erythropoiesis (60)
4.2 Name the factors essential for above process (40)

Answers

MCQ
19)

A) T lymph is formed as an ultra-filtrate of capillary micro circulation.

They are formed from interstitial fluid collected via lymph capillaries.

B) F composition of lymph is similar to ECF except the presence of high concentration of protein in lymph.

C) F is extracellular

D) F

E) T functions of lymph,

 Removal of filtered proteins from tissues


 Filtration of particles by lymph nodes
 Recirculation of lymphocytes
 Exposure of antigens to the immune system
 Formation of lymphocytes
 Absorption and transport of fatty acids and fats as chylomicrons from digestive system.
34)

A) T serum protein; albumin, ϒ globulin, fibrinogen.

B) T

Presented by 15th Batch 407 FHCS | EUSL


C) T

D) F pre pro albumin synthesized in liver,

pro albumin Golgi apparats albumin – secreted to bile

E) F

36)

A) F jaundice = yellowish discoloration of skin, sclera and mucus membrane due to increase in bilirubin
concentration in blood more than 2mg/dl. Normal plasma concentration less than 0.5mg/dl.

In Fe deficiency anemia - - > RBC are microcytic and hypochromic.

No hemolysis. So No jaundice.

B) T due to ⬇️ Hb concentration in blood there is an O2 deficiency at tissue level.

⬆️ Respiration - - >SOB

C) T due to blood loss during menstruation and pregnancy.

D) F Normocytic (MCV= 80-100) eg; - protein energy malnutrition.

Microcytic (MCV < 80) eg; - Fe deficiency anemia.

Macrocytic (MCV > 100) eg; - B12 and folic acid deficiency anemia.

E) F Hypochromic = ⬇️ hemoglobinisation. (Lighter colour)

37)

A) F B12 deficiency  macrocytic/ megaloblastic

⬆️ MCV and MCH

Normal MCHC

B) F

C) F

D) T pernicious anemia = autoimmune destruction of gastric parietalcells.

⬇️

Lack of intrinsic factor

E) F autosomal recessive disorder. Producing HbS instead of Hb A

38)

A) T

B) T 2, 3 BPG and H+ stabilize T form.

C) T they are essential for DNA synthesis/cell proliferation.

D) T Glucose binds to a β chain Valine residue. RBC have 120 days life-span. So it remains in blood longer.

E) T

39)

Presented by 15th Batch 408 FHCS | EUSL


Autoimmunity = formation of antibodies against self-antigens.

These can be B cell or T cell mediated.

A) T

B) T

C) F Neutrophil - - >phagocytosis, controlling pyogenic infection

D) F mast cells - - >inflammatory response.

E) F monocyte - - > phagocytosis (via become macrophages)

40)

A) F

B) T

C) F

D) T

E) F

55)

ANSWER – E

56)

ANSWER – A

T cells – cellular immunity

B cells—humoral immunity

57)

ANSWER – A

Liver detoxified alcohol

SEQ (PROPER)
6.

6.1.

Chronic obstructive pulmonary disease

 Obstructive disease occurs in air ways.


 Due to air way obstruction, respiration altered.
 Inspiration is not impaired ,but expiration is difficult.
 Obstructive lung disease is characterized by increased resistance to air
 flow and high lung volume.
 Mechanism of air way obstruction includes the following.
 Air way lumen may be partially obstructed by excessive secretions, oedema fluid or aspiration of food
or fluids.
Presented by 15th Batch 409 FHCS | EUSL
 The air way wall smooth muscle may be contracted or thickened because of inflammation and oedema
or mucus gland may be hypotropied.
 Outside the airway the obstruction of the lung parenchyma may decrease radial traction, causing the
airways to be narrowed.
 This condition causes the quantity of oxygen transported to the tissues to decrease, ordinarily increases the
rate of red cell production.
 Hypoxia situation in body tissues as well as renal tissues stimulate the production and secretion of
erythropoietin, 90% of erythropoietin is formed in kidney and 10% of liver.
 Erythropoietin is the principle stimulus for red blood cell production. It secreted in low oxygen levels.
 Hypoxia causes marked increase in erythropoietin production and in turn enhance red blood cell
production.
 Erythropoietin begins to be formed within minutes to hours and reaches maximum production in 24 hours.
 No new red cells appear in circulating blood until 5 days.
 It has been determined that the important effect of erythropoietin is to stimulate the production of
proerythroblasts from hematopoietic stem cells in bone marrow.
 Once erythroblasts are formed, the erythropoietin causes these cells to pass more rapidly through the
erythroblastic stades than they normally do.
 Further speeding up the production of ne red blood cells.
 Rapid production of cell continues as long as the person remains in allow oxygen state or until enough red
blood cells have been produced to carry adequate amount of oxygen to the tissue despite the low O2 at
this time rate of erythropoietin production decreases to the normal level.

 Polycythemia means the excess cells in blood. Due to this action more RBC produces than normal and
polycythemia occur (6 -7 million/ mm3)
6.2.

People living in high altitude

 Decrease in barometric pressure is the basic course of high altitude hypoxia.


 As altitude increases, barometric pressure decreases and Po2 decreases proportionally.
Presented by 15th Batch 410 FHCS | EUSL
 Alveolar Po2 also reduced.
 Quantity of O2 in air is greatly decreased. Insuffient oxygen is transported to tissues and red cell
production is greatly increased.
 Due to hypoxic condition erythropoietin production and secretion rate increases. It speed up the
production of red blood cells.
 Due to cells in blood increases with time polycythemia occurs.
 As number of RBC increases hypoxic condition also disappear. ( formation of polycythemia is in 6.1 answer
)

SEQ (REPEAT)
04)

4.1)

It is the process of formation of red blood cells. 2 million per second are produced in a healthy person. In adults it
occurs in the bone marrow. The pluripotent haemopoietin stem cells are divided to form committed stem cells.
Then into colony forming unit of erythrocytes. Erythropoietin normally acts on this level. CFUs form
proerythroblasts. They have no Hb but have nucleoli. Then convert to Early normoblasts( Basophil erythroblast)
Nucleolus disappear and Hb starts appearing. Then to Intermediate normoblasts.After multiplication, convert into
Late normoblast. Hb is increased. It takes2-7 days for pro erythroblasts to mature into late normoblast. There are 4
mitotic divisions in between forming 16 cells. Not all are good, so bad cells are destroyed. These form reticulocytes,
mature for 2-3 days in bone marrow and are released into blood stream by diapedesis. These form 1% of RBC.
There, after 2-3 days, convert into mature RBC.

4.2)

• Tissue oxygenation

• Erythropoietin

• Vitamins- B12, folic acid

• Fe

• Other Hormones –Sex hormones, Thyroxine, GH, Cortisol

11TH BATCH
11TH BATCH (PROPER) – MCQ

01) Blood,
a) Express serum when it clots.
b) From a higher percentage of body weight in obese than in thin people.
c) Has low blood volume per kg body weight in neonate than in adults.
d) Make up about 7% body weight.
e) Volume rise after drink water.

02) Hemolytic disease of new born,

Presented by 15th Batch 411 FHCS | EUSL


a) Affect mainly babies of positive mother.
b) Prevent by injecting anti – D antibodies just after the delivery to mother.
c) Treated by transfusion of blood to affected baby with Rh+ blood.
d) Causes jaundice with clears rapidly after birth.
e) Occurs mainly in babies who lack D agglutinogen.

03) Blood group agglutinogens,


a) Attached to hemoglobin.
b) Before transfusion cross match with receptor’s one with donor’s agglutinin.
c) AB blood group have agglutinogen A and B.
d) Inherited as mendalian recessive.
e) Not present in new born.

04) Break down of erythrocyte in body,


a) Require for the synthesis of salt.
b) Due to Rh incompatibility.
c) Occur by reticular endothelium system.
d) Due to deficiency of G6P dehydrogenase.
e) Release Fe2+.

05) Person with B blood group,


a) Has anti n antibodies in plasma.
b) May have parent with O blood group.
c) May have children with B blood group.
d) May have the genotype AB.
e) Whose parent is A can have A child.

06) Blood platelets assist in arresting the bleeding by,


a) Adhering together to form plugs when exposed to collagen.
b) Von Willebrand factor which are releasing by platelets.
c) Prostacyclin which are in the plasma.
d) Platelets contain clotting factors.
e) Platelets are aggregated on the formed fibrin network.

07) White blood cells,


a) Has lobular nucleus.
b) Increase in allergic reactions.
c) Produced in lymph nodes.
d) Have lifespan of circulation of 3-4 weeks.
e) Contain proteolytic enzymes.

08) Regarding heme synthesis pathway,


a) All enzymes are located in the mitochondria.
b) Glycine and succinate are precursors of delta ALA.
c) ALA dehydrogenase catalyzes the synthesis of PBG from ALA.
d) Heme inhibits ALA synthesis activity by feed-back inhibition.
e) ALA synthase regulates heme biosynthesis both in liver and erythropoietic tissue.

09) Which of the following cells are involved with pathogenesis of acute immune decrease,
a) B-Lymphocyte

Presented by 15th Batch 412 FHCS | EUSL


b) Mast cell
c) Neutrophils
d) Natural killer cell
e) T-Lymphocyte

10) γ-aminolemulin acid in heme biosynthesis pathway,


a) Is rate limit state in heme synthesis.
b) Is decreasing by an individual use drug barbiturate phenol-barbiturate.
c) Inhibit by heavy metal ion.
d) Occur in the cytosol.
e) Hematin is an intermediate of heme synthesis.

SBR
11) Catabolism of Hb involves,
a) De-oxidative cleavage of the porphyrin ring.
b) Is the sole source of bilirubin.
c) Occur in RBC.
d) Synthesis bile salt.
e) Albumin is not considered for this.

12) Any of the following companions true, Except


a) Type 1 Hypersensitivity – Dengue shock syndrome
b) Type 2 Hypersensitivity – Autoimmune hemolytic anemia
c) Type 3 Hypersensitivity – Vasculitis
d) Type 4 Hypersensitivity – Contact dermatitis
e) Type 5 Hypersensitivity – Myasthenia gravis

13) Prothrombin time of the blood coagulation test,


a) Does not involve oxalation of thrombin in blood.
b) Indicate concentration of removal blood.
C)
d) Normally about 12 min.
e) Decreasing rise of prothrombin.

14) A 5 years old boy suffering from diarrhoea for about 2 week and recently becomes sever. From further
investigation it is observed that feces contain parasites. What’s the type of leukocyte that might increase?
a) Basophils
b) Monocyte
c) Neutrophil
d) Eosinophil
e) Lymphocyte

15) All the following cells are antigen presenting cell, Except
a) B – Lymphocytes
b) Dendritic cell
c) Cooper cells
d) Microglial cell
e) Neutrophil

Presented by 15th Batch 413 FHCS | EUSL


11TH BATCH (PROPER) – SEQ

05.
5.1.
5.1.1. List two function of lymph. (10 Marks)
5.1.2. Describe the mechanisms that facilitate lymph flow in lymphatic vessels. (25 Marks)
5.1.3. Briefly explain why an infected wound in the leg may cause enlargement of lymph node in the groin.
(15 Marks)
5.2. Explain the physiological basis for numbness in the hand
5.2.1. after holding ice cube inside the palm for a few minutes. (25 Marks)
5.2.2. when BP cuff is kept inflated in the upper arm above systolic pressure for three minutes. (25 Marks)
06.
6.1. State two stages of haemostasis. (10 Marks)
6.2. Briefly describe each stage mentioned in 6.1. (40 Marks)
6.3. Explain why three is increased risk of intravascular thrombosis in
6.3.1. people who are immobilized for long periods. (25 Marks)
6.3.2. people with atherosclerotic disease. (25 Marks)

30.A-T Decreased protein intake lead to it

B-T Due to kidney damage protein loss in urine

C-T Because albumin synthesis in liver

D-F Because high protein intake increases albumin level

E-T Due to venous pooling albumin loss in interstitial fluid

31. -

32.A-F RBC not needed

B-T Fibrinogen converted to Fibrin by Thrombin

C- F Lymphocyte B become as plasma cell; it does not take part in coagulation

D -T Platelet activated, aggregated and blood clot

E-F not needed

33. A-F

B-T Type of macrocytic anemia, RBC enlarge, MCV>100

C-F RBC enlarge and reduce in number

D-T VB12 bound with intrinsic factor produce by stomach and absorb in intestine

E-F Caused by vitamin B12 deficiency

34.A-F Lymphocyte produce in bone marrow

B-F 20%-40% in leukocyte

Presented by 15th Batch 414 FHCS | EUSL


C-T Due to immune response, to produce antibodies against antigen

D- T

E- F Has a large nucleus

37.A-F They produce only after they expose to Rh+

B-T Ig G

C-T Ig A

D-

E-T When anti Rh agglutinin cross the placenta to an Rh + fetus, hemolysis occurs, in

severe stage lead to kernicterus

40.A-T Absent in avascular structures, brain, spinal cord, splenic pulp, bone marrow

B-F

C-

D-T

E-T

57. ANSWER- Positive acute phase protein

-c creative protein

-ceruloplasmin

-alpha 1 antitrypsin

-alpha 2 macroglobulin

-haptoglobin

-fibrinogen

Negative acute phase protein

-albumin

-transferrin

-transthyretin

-retinol binding protein

58. ANSWER- A Hemoglobin synthesis begins- basophil erythroblast

60.ANSWER- D Phagocytize bacteria

Presented by 15th Batch 415 FHCS | EUSL


11TH BATCH (PROPER)- SEQ
05.

5.1.

5.1.1. •Drains excess interstitial fluid

▪ Lymphatic vessels drain excess interstitial fluid from tissue spaces and

return it to the blood.

• Transports dietary lipids.

▪ Lymphatic vessels transport lipids and lipid-soluble vitamins (A, D, E, and K)

absorbed by the gastrointestinal tract.

• Carries out immune responses.

▪ Lymphatic tissue initiates highly specific responses directed against

particular microbes or abnormal cells

5.1.2.

Presented by 15th Batch 416 FHCS | EUSL


Formation of lymph

▪ Most of the filtered fluid out of blood capillaries returns to them by reabsorption.

▪ Some excess filtered fluid drains into lymphatic capillaries and lymph is formed

▪ Proteins that leave blood plasma cannot return to the blood

▪ Proteins can move through the lymphatic capillaries into lymph ( Lymphatic capillary more permeable for the
protein)

▪ Lymphatic capillaries has high oncotic pressure than Interstitial fluid

▪ Lymphatic vessels return the lost proteins and plasma to the bloodstream.

➢ lymphatic vessels contain valves

▪ ensure the one-way movement of lymph

➢ Two “pumps” maintain the flow of lymph.

▪ Skeletal muscle pump.

o Skeletal muscle contractions compresses lymphatic vessels and forces lymph

toward the junction of the internal jugular and subclavian veins.

▪ Respiratory pump.

o Lymph flow is also maintained by pressure changes that occur during inhalation

(breathing in).

o Lymph flows from the abdominal region toward the thoracic region

o When the pressures reverse during exhalation (breathing out)

➢ when a lymphatic vessel distends, the smooth muscle in its Wall contracts

➢ Which helps move lymph from one segment of the vessel to the next.

5.1.3 ➢ Infected wound in the leg

➢ Pathogens enter in to the lymphatic Capillaries on the infected wound of the leg

➢ Lymph capillaries lead to afferent lymphatic vessels

➢ They open into the lymph nodes

➢ Pathogens reaches Superficial and Deep inguinal nodes( lymph nodes in groin)

which are immediately near to the wound

➢ WBCs in the nodes produce immune response


Presented by 15th Batch 417 FHCS | EUSL
o Antibodies – Plasma cell

o Phagocytosis – Dendritic cells

➢ Destroying the pathogens take place

➢ Increased activity of the lymph nodes

➢ Swelling of the lymph nodes

5.2

5.2.1. • Cold receptors of the palm sense the change in temperature (Free nerve endings)

• Afferent fibers reach the thermoregulatory center of hypothalamus

• Efferent signals sent to the superficial arterioles

• Contraction of the smooth muscles in the wall of the Arterioles near the surface of

the palm

• Reduced blood flow to the surface of the palm

• Prolonged reduced blood flow lead to reduced O2 supply to the nerves of the

surface of the palm

• Reduced activity of the nerves • Lead to impaired cutaneous sensation

• Lead to numbness

5.2.2. Blood pressure is the force exerted by the blood on the walls of the vessels Systolic pressure is the
highest pressure exerted by the heart on the blood vessels

During inflation of the BP cuff above Systolic pressure,

• Inward force is exerted on the Artery ( Brachial Artery)

• Compress of the artery

• When the force is above the systolic pressure

• The artery is completely occluded

• Prevent the blood flow to the distal part of the upper limb

• Reduced o2 supply for the nerves of the distal part

• Leads to the reduced activity of the nerves

• Loss of the cutaneous sensation

• Leads to numbness

Presented by 15th Batch 418 FHCS | EUSL


06.

6.1. • Primary response

▪ Vasoconstriction

▪ Platelet response (Platelet plug formation)

• Secondary response

▪ Blood coagulation

6.2. Vasoconstriction

• After injury the blood vessel constricts and decreases the loss of blood from

damaged Portion.

• Arterioles constrict

. • Obliteration of lumen of the damaged blood vessel Stop bleeding

➢ Nervous reflexes

▪ Pain fibers reach the CNS (spinal cord)

▪ Efferent signal reach the smooth muscles of the arterioles

➢ Local humoral factors – Serotonin, thromboxane A2

➢ Local myogenic contractions of blood vessel

Platelet response (Platelet plug formation)

• Platelets to the collagen is accelerated by von Wille brand factor.

• This factor acts as a bridge between a specific Glycoprotein present on the surface of platelet and Collagen fibrils.

• Platelets get adhered to the collagen of ruptured blood vessel • Secrete adenosine diphosphate (ADP) and
thromboxane A

• attract more platelets and activate them

• All these platelets aggregate together and form a loose temporary platelet plug

• Vessel and prevents further blood loss.

Presented by 15th Batch 419 FHCS | EUSL


Blood coagulation

Temporary platelet plug will be converted into definite blood clot Contains following steps,

• Formation of activated factor X (Xa) Intrinsic & Extrinsic pathways

• Conversion of Prothrombin to Thrombin

• Conversion of Fibrinogen to Fibrin

Presented by 15th Batch 420 FHCS | EUSL


6.3

6.3.1. During Mobilization

, • Deep veins of the leg carries the blood to heart against the gravity

• Muscles of the lower limb act as a pump (muscular pump)

• Blood will be pumped towards the heart

• Valves of these veins prevent Back flow of the blood

If the person is immobilized,

• Muscular actions will be reduced

• Impaired muscular pump

• Blood can’t be pumped towards the heart

• Venous pooling occur


Presented by 15th Batch 421 FHCS | EUSL
• Blood begin to accumulate in the deep veins

• Because of the action of blood clotting factors

• Blood clots will be formed in the pooled blood

• Intravascular thrombus is formed

• Leads to deep vein thrombosis

6.3.2. • Due to the abnormal deposition of Cholesterol under the epithelium of the blood

vessels

• Plaque will be formed

• Narrows the lumen of the blood vessels

• The velocity of the blood through the narrow lumen is increased

• High velocity of blood may damage the wall of the blood vessels

• Due to the injury the collagen is exposed

• Which may activate the hemostasis

• Platelet will adhere and aggregate in damaged area

• Blood coagulation take place

• Blood clot will be formed and block the lumen of the blood vessels

• So the blood supply for the particular site is reduced

12TH BATCH

12TH BATCH (PROPER) – MCQ

6. T form of Hb
a. Deoxy form of Hb
b. Has high oxygen affinity form
c. Has low oxygen affinity form
d. H bonds and ionic bonds limit the movement of monomer
e. The binding of oxygen destabilizes some of the H bonds and ionic bonds particularly between alpha – beta dimers

Presented by 15th Batch 422 FHCS | EUSL


19. Histocytes
a. Involve in phagocytosis
b. Present antigens
c. Involve in innate immunity
d. From macrocytes
e. Lymphocytes

31. following cells are formed elements of blood


a. Erythrocytes
b. Fibroblasts
c. Megakaryocytes’
d. Thrombocytes
e. WBC

32. regarding production of blood cells


a. In adults blood cells are produced in marrow of all bones
b. In older adults blood cell production is confined to irregular and flat bones
c. In young children blood cells are produced in liver and spleen
d. Only red bone marrow produce blood cells
e. Part of marrow producing red blood cells greater than producing WBC

34. serum protein include


a. albumin
b. fibrinogen
c. gamma globulin
d. pre albumin
e. prothrombin

35. Regarding Rh blood grouping,


a. If baby born with erythroblastosis fetalis mothers’ blood can be transfused to the baby
b. If mother found to be Rh- & fetus Rh+, anti D(rhogam) should be provided to mother at 28th 34th week of
gustation
c. erythroblastosis fetalis is immature large blood cell in the peripheral blood
d. Rh incompatibility leads to severe anemia,jaundice,hypotonia
e. inheritance of Rh is Autosomal dominant

36. Regarding blood,


a. Leishman reagent only use for identification of WBC
b. Leishman is a mixture of eosin, methylene blue and some derivatives of methylene blue
c. both Rh system & ABO system has natural antibody
d. slide blood typing can be used for transfusion purpose
e. in mismatch transfusion occurs between recipient RBC & donors plasma

37. Regarding hemoglobin


a. When it is oxidized present in ferric state
b. It is responsible for color of blood
c. It form bile pigment
d. It can dilute with HCL
e. It is the main way of transporting of oxygen

Presented by 15th Batch 423 FHCS | EUSL


38. Regarding plasma
a. Plasma contain 8% of total body weight
b. Plasma protein contain 8g/ dl
c. It is isotonic with 5% of NaCl solution
d. hemoglobin is plasma protein
e. Plasma contain high amount of protein than intracellular fluid

43. responsible for special immunity


a. Lymphocyte
b. Neutrophil
c. Eosinophil
d. Basophil
e. Platelets

44. Which of the following is responsible for the seasonal allergic reactions such as runny nose, nasal congestion
and wheezing
a. Monocytes
b. Neutrophils
c. Mast cells
d. Lymphocytes
e. Basophils

49. Which condition increase the reticulocyte in blood?


a. Kidney disease
b. Vitamin B12 deficiency
c. Hemolytic anemia
d. Vitamin B9 deficiency
e. Bone marrow disease

53. Which of the following is not found in serum


a. Hormones
b. Glucose
c. Electrolytes
d. Fibrinogen
e. Antibodies

54. Function of T lymphocytes


a. Cell mediated immunity
b. Exocytosis
c. Humoral immunity
d. Production of histamine
e. Phagocytosis

59. 25 years old women complains she has tiredness immediately when she walk on a staircase. Physician found
that she has heavy menstrual bleeding , pale conjunctiva, palpitation like signs. What is the most possible cause?
a. Jaundice
b. Shortness of breathing
c. B12 deficiency
d. Microcytic anemia
e. Thyrotoxicosis

Presented by 15th Batch 424 FHCS | EUSL


60. Red cells of blood group O has
a. No antigen
b. H antigen
c. Anti A and anti B
d. D antigen
e. A antigen

12TH BATCH (PROPER) – SEQ

1.
1.1. Explain spherocytosis
1.2. Explain Physiological basis of
1.2.1 Anaemia in spherocytosis
1.2.2 Enlarged spleen in spherocytosis ▪ Shortness of breath and dizziness
1.3. Role of alpha 1 – antitrypsin activity in preventing emphysema
1.4. Regulation of heme synthesis

Answers

12TH BATCH (PROPER)-MCQ

6. A.T

B.F low oxygen affinity

C.T

D.T

E.T bind O2 move Fe , This move proximal his and its helix , moving helix alters alpha

beta interface

19. A.T phagocytic cell in connective tissue

B.T

C. T

D.

E.

31.A.T

B. F fibroblast involve in formation of extracelluar matrix and collagen

C. F megakaryocyte present in bone marrow

D. T platelets are also called thrombocytes

Presented by 15th Batch 425 FHCS | EUSL


E. T

32.A.F ganong page 554

In children blood cells are actively produce in the marrow cavity of all the bones.by age 20 marrow in
the cavities of the long bones except for upper humerus and femur has been inactive

B. F upper humerus and femur continuesly produce blood cells

C. F in the fetus blood cells produce in liver and spleen

D. F red blood cells and most wbc and platelets produce in bone marrow T and B lymphocytes also
produce in the lymph nodes and spleen

E. F red cell producing part-25% wbc producing part-75%

34.A.T

B.F

C.T

D.T

E. serum proteins are made up of albumin and globulin

35..A.T erythroblastosis fetalis means mother is sensitize for Rh factor at first Time rh+ baby delivery and in
second pregnancy preformed agglutinin cross placenta and leads to hemolysis

B.F first dosage receive 28weeks of gustation and again 72h after delivery

C.F

D.F. if hemolysis in the fetus is severe than infant may die in utero or may developed anemia jaundice
edema (hydrops fetalis) Ganong page 562

E. T

36.A.T

B.T

C.T ABO system antibodies developed shortly after birth but Rh Antibodies developed

only exposure to antigens

D.T slide typing is done before blood transfusion

E.F mismatch occur between donor blood cells and recipient plasma

37.A. T in hemoglobin iron atom present as fe2+ reduce formed when it get Oxidize to fe3+

B. T

C. T when hemoglobin is breakdown bilirubin and biliverdin are formed

Presented by 15th Batch 426 FHCS | EUSL


D.F HCL not use to dilute Hb it cause to produce acid hematin

E.T a little amount transport via dissolve form

38.A T normal plasma volume is about 5% of body weight gan0ng page 562

B.T

C.F 0.9 Nacl% isotonic to plasma

D F hemoglobin not release into plasma

E. F plasma protein protein not filter into interstitial space

43.A

Special/Specific/Acquired

Cells important for acquired immunity is Lymphocytes.

44.C

Mainly allergies are due to mast cells which releases allergic inducing substances.

49. C

The reticulocyte count rises when there is a lot of blood loss or in certain diseases in

which red blood cells are destroyed prematurely, such as hemolytic anemia.

53. D

Serum doesn't contain WBC,RBC PLATELET,and clotting factors.all electrolyte,

antibody,antigen,hormones,and exogenous substance are present in serum..

54.A

Function of T lymphocytes is cell mediated immunity..

Function of B lymphocytes is humoral immunity

59.D

Heavy menestural bleeding----->loss of more blood-----> iron defecieny anemia------>

microcytic anemia

60.A

Antigen present at the surface of RBC...but in O grp no antigen... present both antigen

12TH BATCH (PROPER)- SEQ

1.1. Hereditary spherocytosis is an inherited blood disorder. It happens because of a problem with the RBC.In
Hereditary spherocytosis, the RBCs are very small and spherical rather than being biconcave discs. These red cells
are more fragile than biconcave Shaped RBC.
Presented by 15th Batch 427 FHCS | EUSL
1.2

1.2.1 anemia- Reduced Hb concentration in blood lower than the normal level, relative to the age ,sex,race
and physiological status (ex:pregnancy, altitude)

Different abnormalities of the RBCs, many of which are acquired through hereditary, make the cells fragile,so
they rupture easily as they go through the capillaries, especially through spleen. Even though the number of RBCs
formed may be norm1, or evenmuch greater than normalin some hemolyticdiseases, the life span of the
fragileRBCs short that cells are destroyed faster than they can be formed and serious anemia results. This is called
hemolytic anemia ( in this anemia life span ofa RBC is shortened, increase breakdown of RBC occur.)

Because of increasing breakdown of Hb and deceased Hb concentration , anemia occurs in spherocytosis.

1.2.2 enlarged spleen (splenomegaly) - people developenlarged spleens in hereditaryspherocytosis when


their spleen fill up with spherocytes..In hereditary spherocytosis (congenital hemolytic icterus), the cells are
spherocytic in normal plasma and hemolyze more readily than normal cells in hypotonic sodium chloride solutions.
Abnormal spherocytes are also trapped and destroyed in the spleen, meaning that hereditary spherocytosis is one
of the most common causes of hereditary hemolytic anemia.

1.3. Alpha 1 - anti trypsin - it is synthesized by liver and it is the principal protease. Inhibitor of human plasma. It is
considered as acute phase reactant.

It increase in

• inflammatory conditons

• trauma

• burns

• infarction

• malignancy

decrease in

• emphysema

• nephrotic syndrome

•hypoproteinemia

Emphysema- is a lung condition that causes shortness of breath.in people with Emphysema the air sacs in the lungs
( alveoli) are damaged.

Normaly Alpha - 1 anti trypsin coats lungs, protecting them from Neutrophil elastase.neutrophil elastase produced
by white blood cells to break down harmful bacteria. Potentially damaging to lungs if exposed.

If alpha 1 anti trypsin deficiency occur, lungs lack alpha-1 anti trypsin coating, leaving them open to damage by
Neutrophil elastase. It trapped in liver, causing liver damage. Neutrophil elastase uninhibited causing lung damage

Presented by 15th Batch 428 FHCS | EUSL


1.4 Regulation of Heme synthesis

Presented by 15th Batch 429 FHCS | EUSL


Liver Erythroid cell

a.Heme or its oxidized product hemin (Fe3+) controls •controlled by


ALA synthase by Feedback inhibition b. Repression of
ALA synthase a. Uroporphyrinogen III synthase

c. Inhibition of transport of ALA synthase from the b. Ferrochelatase


cytosol to mitochondria (the site of action)
c. Erythropoietin

d. Availability of intra cellular iron.

drugs induce the activity of hepatic ALA synthase :


phenobarbital, griseofulvin or hydantoins,

Erythropoietin availability of intracellular iron regulate


ALA synthase in erythroid cell

13TH BATCH
13TH BATCH (PROPER) – MCQ

12. Serum is different from plasma. What is the main feature of serum?
A. it has higher osmolarity.
B. It is devoid of fibrinogen.
C. It has increased calcium concentration
D It has no antibodies.
E. It contains platelets.

13. Regarding lymphatic system


A. drain from interstitial fluid
B. no blood cells
C. destroy red blood cells
D. has proteins more than blood
E. transport Chyle from intestine to blood

18. involve in innate immunity of the skin


A. Melanocyte
B. dendrite cell
C. Langerhans cell
D. keratinocyte
E. Merkel cell

Presented by 15th Batch 430 FHCS | EUSL


30. In which conditions serum albumin level reduced.
A. Acute infection
B. Chronic infection
C. Liver cirrhosis
D. Malnutrition
E. Nephrotic syndrome

31. 02 binding to Hb
A. Co2 reduces oxygen affinity to Hb.
B. deoxy Hb increases affinity tor protons
C. Effect low affinity to 02 at high pH
D. Hb tetramer binds 4 moles of 2-3BPG
E. affinity for H increases with the increased percentage of saturation

32. True or false regarding erythropoiesis


A. duration for proerythroblast to become mature BC is 5-7days
B. In adults RBC are produced in marrow of all bone
C. stimulus to erythropoiesis is thrombopoietin
D. rate of production increase after hemorrhage
E. life span about 72days

33. antibodies are produced by


A. glial cells
B. Kupffer cells
C. Dendritic cell
D. plasma cells
E. reticulocytes

34. Component of adaptive immunity


A. B lymphocyte
B. Macrophage
C. Basophilic cell
D. Natural killer cell
E. T lymphocyte

35. Hemolytic anemia


A. dark urine
B. high serum bilirubin level
C. high reticulocyte count
D. Microcytic red cell
E. low MCH

37. hemolytic neonatal Jaundice due to


A. destruction of RBC with fetal HB
B. immature liver to remove bilirubin
C. increased plasma bilirubin
D. decreased hemopoiesis due to immature bone morrow
E. obstruction of bile duct

Presented by 15th Batch 431 FHCS | EUSL


38. Erythrocyte sedimentation rate is
A. Measure or concentration or RB
B. Direct correlation of fibrinogen levels
C. Higher in male than female
D. Increase in pregnancy
E. Measured by wintrobe tube

39. Red blood cells are normally destroyed by macrophages in the


A. Circulatory blood
B. Liver
C. Spleen
D. Thymus
E. Lymph nodes

40. regarding platelet


A. produce in end of the long bone.
B. life span 5-7 days
C. Help to clotting formation.
D. circulating average amount 4000-11000

Single best

55. Albumin contribute most of the plasma colloid osmotic pressure. what is the most
appropriate reason for that...?
A. Albumin is the smallest particle of the colloid particles
B. Albumin Is the most numerous in the colloid particles
C. Albumin Can't filtrate through the capillary
D. Albumin is rapid turnover in Liver
E. Albumin Found in high concentration in plasma

56. Which of the following tissues has the Highest oxygen consumption under resting condition
(ml/100mg/min)
A. Brain
B. Cardiac muscle
C. Kidney
D. Liver
E. Skeletal muscle

57. which of the following plasma protein is found in highest concentration?


A. Albumin
B. Globulin
C. Fibrinogen
D. Prothrombin
E. Transferrin

58. Serum is different from plasma. What is the main feature of serum?
A. it has higher osmolarity.
B. It is devoid of fibrinogen.
C. It has Increased calcium concentration
D. It has no antibodies

Presented by 15th Batch 432 FHCS | EUSL


E. It contains platelets.

59. largest white blood cell


A. Neutrophils
B. Eosinophils
C. Basophils
D. Monocytes
E. Lymphocytes

60. The most common method or analyzing separate serum plasma protein IS
A. Biuret Test
B. Chromatography
C. Electrophoresis
D. salt precipitation
E. Spectrophotometer

13TH BATCH (PROPER) – SEQ

3.1. Normal red blood cells placed in hypotonic saline do not hemolyze until the concentration is 0.45% (half
normal) whereas red blood cells in hereditary spherocytosis hemolyze when placed in 0.7% NaCl solution. Explain
the physiological
basis of
3.1.1 Normal BC not hemolyzing in solutions above 0.45% though hypotonic. (25 marks)
3.1.2 Increased fragility of spherocytes (25 marks)

4. Explain the Physiological basis,


Severe anemia is accompanied by
4.1. Shortness of breath after exertion
4.2. Heart murmurs
4.3. Dizziness
4.4. Pale skin

Answers

12)
A. False
B. True
C. False
D. False
E. False
13)
A. True Drain by lymphatic capillaries

Presented by 15th Batch 433 FHCS | EUSL


B. False Contain T and B lymphocytes

C. True By the red pulp of spleen


D. False
E. True Lipid & lipid soluble vitamins
18)
A. False Produce melanin pigment in the skin
B. True
C. True
D. False structural
E. false Sensory reception cells
30)
A. True
B. True
C. True
D. True
E. True
31)
A. True
B. True
C. False liberate o2 at low pH levels (low pH –high co2)
D. False
E. True Cooperative oxygen binding ,, sigmoid shape of the Hb ,O2 dissociation curve
32)
A. True
B. False

Presented by 15th Batch 434 FHCS | EUSL


C. False Erythropoietin
D. True Reticulocyte count of blood increased after a hemorrhage
E. False 120 days
33)
A. False
B. False
C. False
D. True Antibodies produced only B-lymphocytes . mature B lymphocytes called plasma cells
E. False
34)
A. True B-lymphocytes- Development of humeral immunity (antibody production)
B. True Antigen presenting cells
1.macrophages
2.Dendritic cells
3.B-lymphocytes
C. False
D. True NK cells- first line of defense in specific immunity , particularly against virus
E. True
35)
A. True
B. True Unconjugated form high
C. True
D. False Normocytic normochromic
E. False
37)
A. True
B. True
C. True Unconjugated form
D. False
E. False
38)
A. False Measures the rate of erythrocyte sedimentation.
B. True Globulin and fibrinogen increase the Rouleax formation
C. False Less number of RBC in female—high ESR in Female
D. True In pregnancy hemodilution occur- High ESR
E. True Two methods
1.westergren method –only to ESR
2.wintrobe tube –used to both ESR and PCV
39)
A. False
B. True
C. True Reticulo-endothelial system
D. False
E. False
40)
A. Produced in bone marrow—Megakaryocytes
B. 8-10 days
C. True
D.
E. False Normal platelet count ==150000 -450000

Presented by 15th Batch 435 FHCS | EUSL


55)
Ans-E
56)
Ans- B
57)
Ans-A
Albumin>globulin>fibrinogen
58)
Ans-B
59)
Ans-D
Largest – monocyte
Smallest – lymphocytes
60)
Ans- C

SEQ

Presented by 15th Batch 436 FHCS | EUSL


03)

3.1)
3.1.1)
• Water moves from lower tonicity to higher tonicity
• Rbc have higher tonicity than 0.9% Nacl
• So the water moves inside the cell accompanied by swelling of the cell
• Normal RBC is less fragile and it can tolerate the swelling until the surrounding fluid get 0.45% Nacl concentration
• After that the cell membrane will rupture
3.1.2)
• In spherocytosis RBC is already expanded, Surface/volume is decreased.
• When the RBC is placed in hypotonic solution, it will swell more.
• It will rupture in 0.7% Nacl concentration, because of increased fragility

04)
4.1)
After exercise --> Need more O2 to reduce lactic acid made during exercise (oxygen debt) --> In anaemia patients --
> O2 carrying ability of the blood is reduced-->Via chemoreceptors apneustic centre will be stimulated,
simultaneously pneumotaxic centre will be inhibited--> Causes increased inspiration and decreased expiration-->
Shortness of breath
4.2)
• RBC is the main component in maintaining viscosity of blood
• In anaemia --> RBC is reduced--> Viscosity of the blood will reduce--> Critical velocity of blood will decrease--> so
normal velocity of blood flow cause turbulent flow at the orifices of the heart--> Turbulent flow causes--> Murmurs
4.3)
In anaemia--> O2 transported by blood reduced-->O2 to organs reduce --> O2 for
brain also reduce ---> Metabolic rate in brain reduce--> Dizziness
4.4)
• Oxygenated haemoglobin gives the reddishness to the blood
• INJ anaemia O2 carrying capacity of blood reduced
• So the reddishness also reduced
• Veins in the superficial skin also contain reduced haemoglobin
• So the pale colour of the skin will prominent

Presented by 15th Batch 437 FHCS | EUSL


14th BATCH
14TH BATCH (PROPER) – MCQ

12. Thymus,
A) Primary lymphoid organ.
B) Supplied by internal thoracic artery.
C) Receive afferents
D) No venous drainage.
E) Secrete Thymic hormones

19. Pernicious anemia is associated with,


A) Reduced MCV
B) Fe deficiency
C) Gastrectomy
D) Increased MCH
E) Vitamin K deficiency

20. Creatinine,
A) Level points out urinary function
B) Converted to creatine phosphate by creatine kinase
C) High molecular compound
D) Found in plant food
E) Depends on muscle mass in the body

21. Immune cell capable of phagocytosis include,


A) Neutrophil
B) B lymphocyte
C) T lymphocyte
D) Kupffer cell
E) Monocytes

22. Bile pigments,


A) Secreted by hepatocytes
B) Transported binding to albumin
C) Conjugated in reticuloendothelial system of liver
D) Conjugated bilirubin can't be filtrated at renal glomerulus
E) Excreted as stercobilinogen and urobilinogen

23. Regarding agglutinins & agglutinogens of ABO system of blood group


A) Group A has agglutinin A in plasma
B) Group B has agglutinin B in plasma
C) Group AB has agglutinin A and B in plasma
D) Group O has agglutinin A and B in plasma
E) AB blood can transfer to any person safely

24. Following constitutes specific immune system


A) Neutrophil
B) B lymphocyte
C) Skin
D) Mucus membrane
Presented by 15th Batch 438 FHCS | EUSL
E) Thymus

25. Regarding reticulocytes,


A) Are nucleated cells
B) 2-4% are present in plasma
C) Number increases with the hemorrhage
D) From deficiency of iron the number increases
E) Are formed in the bone marrow

26. Regarding white blood cells,


A) T lymphocyte provide antibodies against virus.
B) Basophil secrete histamine during type I immunologic hypersensitivity.
C) Neutrophil count increases in allergic conditions.
D) Eosinophils are the most numerous of the white cells.
E) Monocyte are converted to macrophage.

27. Plasma osmolarity


A) Mainly due to electrolytes.
B) Is the same osmolarity as the interstitial fluid.
C) Protein concentration is 15g/dl
D) When separated from RBC do not clot
E) Can be increased by ingesting hyperosmotic IV

40. True/ False


A) CO2 decreases oxygen affinity of Hb.
B) In Oxyhemoglobin, Fe2+ change into Fe3+.
C) CO combines with NH2 in Hb and form carbamino Hb
D) H2S combines with oxy-Heam and form sulfhydrylheamoglobin
E) Heme remains as Fe2+ state in methemoglobin

SBR
47. Right lymphatic duct is formed by the union of,
A) Right Juglar, subclavian and bronchomediastinal
B) Right Juglar and subclavian
C) Right Juglar and bronchomediastinal
D) Right jugular left jugular
E) Right subclavian left subclavian

51. Blood cell that has longest lifespan,


A) Platelets
B) Neutrophil
C) Basophil
D) Monocyte
E) Erythrocyte

52. Which clotting factor deficiency will lead to the prolonged bleeding only in male?
A) Prothrombin
B) Tissue thromboplastin
C) Calcium
D) Antihemophilic factor

Presented by 15th Batch 439 FHCS | EUSL


E) Platelet factor

53. Follow the blood cells produce outside bone marrow in adults
A) RBC
B) Platelet
C) Lymphocyte
D) Eosinophil
E) Neutrophil

14TH BATCH (PROPER) – SEQ

4.3. Briefly describe on regulation heme biosynthesis. (30 marks)


4.4. State 5 conditions that will decrease the plasma albumin level (20 marks)

5.
5.1. Hypoalbuminemia leads to:
5.1.1. hyponatremia
5.1.2. hypovolemia
Explain the physiological basis of the abnormalities stated in 5.1.1 and 5.1.2. following hypoalbuminemia.
(2 x 25 marks)

6. For a Rh negative mother carrying Rh positive fetus the risk of Rh incompatibility is less than that for the second
Rh positive fetus.
6.1. Briefly describe the consequences of Rh incompatibility to the fetus and their physiological basis. (50 marks)
6.2. Explain why the second Rh positive fetus is at greater risk than the first Rh positive fetus. (50 marks)

Answers

12.A .T Also red bone marrow

B .T Also from the inferior thyroid artery

C. F No afferent lymphatics

D. F Has corresponding veins & thymic vein drains into left brachiocephalic vein

E.T Secrete thymosin which is necessary for T cell development & production

19.A .F

Pernicious anemia = vitamin B12 deficiency anemia

Macrocytic anemia

– Low PCV, Hb

– High MCV

– MCH and MCHC are normal or high


Presented by 15th Batch 440 FHCS | EUSL
– Low RBC count

B. F Fe deficiency → Hypochromic Microcytic anemia

C .T

Gastric atrophy ➢ Severe gastritis ➢ Gastric ulcers ➢ Gastrectomy

D .T MCH and MCHC are normal or high

E.F

20.A.T Creatinine is a breakdown product of creatine phosphate from muscle and protein

metabolism

B .F

C.F Both creatinine and creatine are small molecules

D.F Normally creatine found in animal tissues

E.T Creatine exists as creatine phosphate in skeletal muscles

21.A .T

B .F

C. F

D. T

E. T

22.A .T

B. T Bilirubin is slightly soluble in plasma so transported to the liver by binding non

covalently to the albumin

C. F Conjugation occurs in hepatocytes

D .F Conjugated bilirubin present in urine during

obstructive jaundice

E. F Excreted as stercobilin and urobilin

23A..F Group A → Agglutinogen A + Agglutinin B

B. F Group B → Agglutinogen B + Agglutinin A

Presented by 15th Batch 441 FHCS | EUSL


C .F Group AB → Agglutinogen A and B + No Agglutinin

D. T Group O → No Agglutinogen + Agglutinin A and B

E.F AB → universal recipient [ AB+ ] O → universal donor [ O- ]

24.A.F

B.T

C.F

D.F

E.T

25.A. F In erythropoiesis, nucleus disappear in late normoblast stage.

B. F Normal reticulocytes count in plasma = 0.5% to 2.5% [ adults ]

C. T Bleeding, hemolysis or decreased production of red cells → increase

reticulocytes Because the body is trying to compensate the red cell loss

D .F To make hemoglobin, cells require iron to build a component heme. Iron

deficiency anemia → reticulocyte count does not affect

E.T Reticulocytes develop and mature in the bone marrow

26. A.F Cellular immunity –T cells, Humoral immunity-B cells

Produce antibody → B lymphocytes

B.T Type 1- anaphylactic reactions (primary hypersensitivity) mediated by IgE Mast

cells and basophils release histamine

C.F Basophil involves in allergic reactions Neutrophil count increase in bacterial

infection

Presented by 15th Batch 442 FHCS | EUSL


D.F

E.T Monocyte is the precursors of the macrophages Macrophages are the

professional phagocytes

27.A .T Mainly by Na+

B. T Plasma and interstitial fluid are isotonic (same osmolarity)

C .F Plasma protein concentration is 60-80 mg/mL

D. -

E.T Hyperosmotic IV has more solutes than plasma. So increase plasma solute

concentration.in causes increase in osmolarity.

40.A .T Shifting O2 dissociation curve to right side.

B .F

C .F CO tightly binds to the hemoglobin iron forming carboxyhemoglobin

D.-

E.F Fe2+ oxidize to ferric in methemoglobin

SBR

47.A

51.E Platelets → 8 -12 days

Neutrophil → 2 - 5 days

Basophil → 12 - 15 days
Presented by 15th Batch 443 FHCS | EUSL
Monocyte → 2 - 5 days

Erythrocyte → 120 days

52.D Hemophilia A and B are fully expressed in males only. [ A is most common ]

Hemophilia C affects male and female in equal number. Hemophilia A is

characterized by a deficiency of factor VIII Clotting factor VIII = Antihemophilic

factor [ AHF ]

53.C Bone marrow produces all of the red blood cells, 60–70 percent of the white cells

( granulocytes), and all of the platelets. Lymphatic tissues ( thymus, the spleen, and the

lymph nodes ) produce the lymphocytes

14TH BATCH (PROPER)-SEQ

4.3.Briefly describe on regulation of heme biosynthesis.

The major protein in RBCs is hemoglobin.

Hemoglobin is made up of heme and globular protein known as globin.

Major sites in human eody for heme biosynthesis are liver and erythroid cells.

Heme synthesis rate in liver shows variability according to the demand of heme protein in body. But in case of this
rate in erythroid cells it show a constant and the rate is matched to the rate of globulin synthesis.

The ingredients of heme synthesis are glycine and succinyl co A.

When there is adequate amount is present the rate is increased.

When there is a defciency the rate becomes decreased.

Te inital step and fnal 3 steps occur in mitochondria matrix

wherease rest reactons occur in cytosol.

The rate of secreton increases when demand is high.

But when the demand is low which means there is adequate amount of heme in the body.

At such conditons the rate o heme synthesis is decreased and this is known as the end product Inhibition.

Presented by 15th Batch 444 FHCS | EUSL


The rate limiting enzyme of this process is ALA synthase which catalyoe the formaton of δ aminolevulinic acid ey
glycine and succinyl co A. In case of liver that is ALA synthase 1 and erythroid cells ALA synthase 2.

In liver when heme level increases in body these heme inhibits the action of ALA synthase 1 by repression of ALA
synthase 1 and inhibition of transport ALA synthase from cytosol to mitochondria matrix. Also in liver certain drugs
such as phenobarbital induce the actvity of hepatic ALA synthase. Heme synthesis in erythroid cells are controlled
by availability intracellular ions and following enzymes.

1. Urophophyrinogen.

2. Frrochelatase.

Also this is controlled ey erythropoietin.

4.4. State 5 conditons what will decrease plasma albumin level.

1. Dietary related proelems like malnutrition and malabsorption.

2. Decreased synthesis in advanced chronic liver diseases.

3. Abnormal distribution or diluton due to overhydration and increased capillary permeability.

4. Abnormal Excretion or degradation in clinical conditons like nephrotic syndrome, hemorrhage, burns

5. Congenital conditons.

5.

5.1.Hypoalbuminemia leads to

5.1.1 Hyponatremia

Hypoalbuminimia means reduce albumin level in blood. There are two types of starling forces. Hydrostatic
pressure Oncotic pressure. Hydrostatic pressure exerted by a fluid. Oncotic pressure is exerted ey proteins mainly
albumin. There are 4 forces act in both arterial and venous end of the capillary. They maintain the capillary
microcirculation. Hydrostatic pressure in the capillary Hydrostatic pressure in the interstitial space Oncotic pressure
in the capillary Oncotic pressure I the interstital space.There is a net fltration occurs in arterial end and net
reabsorption occurs in venous end. When albumin level is reduced in blood, plasma oncotic pressure is reduced in
both arterial and venous end. When oncotic pressure is reduced net fltration is increased in arterial end and net
reabsorption in reduced. So more fluid enters into interstital space from plasma. With this fluid sodium like ions
also enter into interstital space. It cause reduce sodium ion concentraton in blood. It is called hyponatremia.

Presented by 15th Batch 445 FHCS | EUSL


5.1.2 Hypovolemia

As mentioned above when oncotic pressure is decreased net fltration is increased and net reabsorption is
decreased. More fluid enters into interstitial space from plasma. Fluid accumulates in interstitial space. It cause
reduce blood volume in blood vessels. It is called hypovolemia.

6.

6.1.Briefly describe the consequences of Rh incompatbility to the fetus and their physiological
basis.(50marks)

•Rh factor is an antigen, present in RBC

• No one has Rh antibody in their blood from birth.

• Rh+ individuals never develop Rh antibodies in their body because they have D antigen.

•But Rh- individuals develop Rh antibodies if exposed to Rh+ blood.

• After this sensitization, if this Rh- individual is again exposed to Rh+ blood, transfusion reactions occur

•Rh incompatibility occurs when the mother’s blood type is Rh negative and her fetus blood type is Rh positive.

•In new born babies, Rh incompatbility is less commoner than ABO incompatbility

• But Rh incompatibility is severe.

•If Rh antibodies mix with Rh+ fetus, hemolytic diseases occur in fetal blood.

• Excessive intravascular hemolysis occur in fetus leads to erythroblastosis fetalis

- Disorder in fetus, characterized by the presence of erythroblasts in blood

- Severe hemolysis in the fetus causes jaundice

. - To compensate the hemolysis of a greater number of RBCs, there is rapid production of

RBCs.

- Now, many large and immature cells in proerythroblastic stage are released into

circulation.

- Because of this, the disease is called erythroelastosis fetalis.

•Ultimately due to excessive hemolysis severe complications develop,

1. Severe Anemia

Excessive hemolysis results in anemia and the infant dies when anemia becomes severe.

Presented by 15th Batch 446 FHCS | EUSL


2. Hydrops Fetalis

Characterized by edema. Severe hemolysis results in the development of edema,

enlargement of liver and spleen an cardiac failure.

When this conditon becomes more severe, it may lead to intrauterine death of fetus

3. Kernicterus

Form of brain damage in infants caused ey severe jaundice. Increase unconjugated bilirubin

in plasma. The blood-brain barrier is not well developed in infants as in the adults. Bilirubin

enters the brain and causes permanent brain damage.

6.2.Explain why the second Rh positve fetus is at greater risk than the frst Rh positve fetus.

• No one has Rh antibody in their blood from birth.

•Rh+ individuals never develop Rh antibodies in their body because they have D antigen.

•But Rh- individuals develop Rh antibodies if exposed to Rh+ blood.

• There is no risk when Rh+ mother carries Rh+ fetus.

•When Rh- mother carries a Rh+ fetus as her first baby, Antigen D is present in fetal blood but not antibody D
Antigen D is not present in mother’s blood. Antibody D also not present in maternal blood because she does not
expose to Rh+ blood yet.

• At the delivery, due to the separation of placenta, some amount of Rh+ fetal blood mixes with Rh- maternal
blood [ sensitization of the maternal blood ]

•Then Rh antibody develop in the mother’s circulation

• But this development takes some time.

• Within that time, the mixed fetal blood cells are dead.

• Therefore no agglutination takes place in maternal blood.

• Developed Rh antibodies remain in the maternal body forever.

•The first child escapes the complications of Rh incompatibility

•There is no chance to mix with antibody D which is developed in maternal blood.

•When the mother conceives for the second time and if the fetus happens to be Rh+ again, Rh antibody in the
maternal plasma crosses the placenta and goes to fetal circulation. Fetus is Rh+ and it has D antigens on its RBC
surface

. •Antibody D received from mother will react with D antigens and make clumps.

• This results in excessive intravascular hemolysis in the fetus.


Presented by 15th Batch 447 FHCS | EUSL
•Severe hemolysis in the fetus causes jaundice

• So the second Rh+ fetus is at greater risk than the first Rh+ fetus.

15TH BATCH
15TH BATCH (PROPER) – MCQ

12) Acute intermittent porphyria


A. Cause photosensitivity in skin
B. is a type of hepatic porphyria
C. ALA synthase activity is reduced
D. Skin rashes, blisters in early childhood
E. ALA and PBG concentration are increased in urine

27) Mucosa associated lymphoid tissue found in


A. Adenoid
B. Palatine tonsil
C. Payer’s patches
D. Lingula
E. Thymus

34) Features of hemolytic anemia


A. increase reticulocyte count
B. urinary urobilinogen
C. Increased amount of conjugated bilirubin in plasma
D. Increased production of erythropoietin
E. Yellow color discoloration of eye

35) Normal circulating blood contain


A. Fibrin
B. Heparin
C. Plasmin
D. Prothrombin
E. Thromboplastin

36) Regarding plasma cells


A. Found in plasma after centrifugation of whole blood
B. They are formed in bone marrow
C. They are small lymphocytes
D. Their life span is 4-6 days
E. T lymphocytes differentiate into plasma cells

Presented by 15th Batch 448 FHCS | EUSL


37) Jaundice with dark urine
A. Associated with pale stool in bile duct obstruction
B. Caused by hepatic failure
C. Due to increased concentration of conjugated bilirubin
D. Due to incompatible transfusion
E. Seen in erythroblastosis fetalis

38) what in left side test correctly evaluate right side situation
A. Anemia - MCV
B. Fragility of RBC - Hematocrit
C. Hypochromic - MCHC
D. Leukocytosis - WBC count
E. Microcytosis - RBC count

39) What are the events that take place of primary stage of blood clotting?
A. Initiate vasoconstriction
B. Involve fibrin clot formation
C. Initiate platelet plug formation
D. Increase bleeding time
E. Initiate by releasing tissue factors

40) Regarding primary Hemostasis


A. It is initiated with vasoconstriction
B. It involves with fibrin clot formation
C. It involves with platelet plug formation
D. Its deficiency would lead to increase bleeding time
E. It initiated by releasing tissue factor

SBR
41) Responsible for antibody production
A. Basophil
B. Neutrophil
C. Eosinophil
D. Plasma cells
E. T lymphocyte

42) Which stage of erythropoiesis in that hemoglobinization occurs?


A. Early normoblast
B. Intermediate normoblast
C. Late normoblast
D. Proerythroblast
E. Reticulocyte

55) which of the following involve to Heme synthesize?


A. Asparagine
B. Glutamate
C. Glycine
D. Lysine
E. Histidine

Presented by 15th Batch 449 FHCS | EUSL


15TH BATCH (PROPER) – SEQ

4.
4.1.
4.1.1. What is hyperbilirubinemia? (05 marks)
4.1.2. Differentiate between conjugated and unconjugated bilirubin. (10 marks)
4.1.3. Briefly describe how to determine direct and indirect bilirubin. (15 marks)

6. An elderly male underwent partial gastrectomy, later encountered with increasing fatigue and shortness of
breath on exertion. Full blood count test showed an RBC count of 2.8 x10/L, Hematocrit 42% and a hemoglobin
concentration of 8.4g /dl of blood.
6.1. Calculate his
6.1.1. Mean Corpuscular Volume (MCV) (15 marks)
6.1.2. Mean Corpuscular Hemoglobin (MCH) (15 marks)
6.2. Explain the physiological basis of the hematological findings of 6.1 (20 marks)
6.3. Explain the physiological basis of fatigue and shortness of breath (20 marks)
6.4. Briefly explain the immunological response against the primary and secondary exposure to pathogen (30marks)

Answers
MCQ
12) Acute intermittent porphyria
A. F
B. T
C. F
D. F
E. F
27) Mucosa associated lymphoid tissue found in
A. T
B. T
C. T
D. T
E. F
34) Features of hemolytic anemia
A. T
B. T
C. F
D. T
E. T
35) Normal circulating blood contain
A. F
B. T
C. T
D. T
E. F
36) Regarding plasma cells

Presented by 15th Batch 450 FHCS | EUSL


A. F
B. T
C. T
D. T
E. F
37) Jaundice with dark urine
A. T
B. F
C. T
D. F
E. F
38) what in left side test correctly evaluate right side situation
A. T
B. F
C. T
D. T
E. F
39) What are the events that take place of primary stage of blood clotting?
A. T
B. F
C. T
D. F
E. T
40) Regarding iron deficiency anemia
A. F - jaundice is yellowish discoloration of skin, sclera and mucus membrane due to increase in bilirubin
concentration in blood more than 2mg/dl. Normal plasma concentration less than 0.5mg/dl. In Fe deficiency
anemia RBC are microcytic and hypochromic. No hemolysis. Thus, No jaundice.
B. T - due to decrease Hb concentration in blood there is an O2 deficiency at tissue level. Increase Respiration
C. T - Due to blood loss during menstruation and pregnancy.
D. F
Normocytic (MCV= 80-100) Eg; - protein energy malnutrition.
Microcytic (MCV < 80) Eg; - Fe deficiency anemia.
Macrocytic (MCV > 100) Eg; - B12 and folic acid deficiency anemia.
E. F - Hypochromic = decrease hemoglobinization. (Lighter color)
41)
Answer D
Plasma cell is Final stage of B cell proliferation. It is a long-lived B cell that is unable to proliferate. It secretes the
highest number of antibodies.
42)
Answer B
55)
Answer C

SEQ
04)
4.1)
4.1.1)
Increased Bilirubin levels in the blood is called hyperbilirubenemia
4.1.2)

Presented by 15th Batch 451 FHCS | EUSL


Unconjugated bilirubin :Initially cyclic heme is degraded into linear biliverdin. Biliverdin, a green pigment, is
reduced, forming the red-orange Bilirubin. This occurs in the macrophages of reticulo-endothelial system. This
Bilirubin is called unconjugated and this complexed with albumin to transport to the liver through the blood.
Conjugated Bilirubin : In the hepatocytes, the solubility of Bilirubin is increased by the addition of two molecules of
glucuronic acid. This process is known as conjunction forming Bilirubin digulucuronide which is called conjugated
Bilirubin.
4.1.3)
 Bilirubin is commonly measured by van den Bergh reaction.
 In methanol both conjugated and unconjugated bilirubin are soluble and react with the reagent, providing
total Bilirubin value
 In aqueous solution, the water soluble conjugated Bilirubin rapidly reacts with the reagent, providing direct
Bilirubin value
 Indirect bilirubin value is obtained by subtracting the direct reacting Bilirubin from the total Bilirubin.
06)
6.1)
6.1.1)
MCV= Hematocrit % ×10/RBC count
= 42 ×10 / 2.8 × 10 12
=150 fl
6.1.2)
MCH= Hb (g/dL) × 10/RBC count
= 8.4 ×10/2.8 × 10 12
= 30 pg
6.2)
 Normal MCV= 80 to 100 fl
In this patient had increased MCV level.
 Normal MCH=27 to 31 pg
So patient has normal MCH level
 According to those numbers this person has Normochromic Macrocytic Anemia.
 This person undergoes partial gastrectomy surgery which refers to as the surgical removal of the part of the
stomach. Stomach is important for absorption of vit B12 in the ileum part because stomach secrets a
substance called intrinsic factors which are essential for vit B12 absorption. Due to the gastrectomy surgery
his surface of the stomach is reduced which leads to the decreased secretion of intrinsic factors. This causes
decreased absorption of vit B12 in the ileum. Therefore body’s B12 level is decreased has an effect on the
RBC’s size which causes macrocytic RBC production. Due to the normal MCH there is no colour changes in
RBC. So normochromic.
6.3)
 His RBC count is 2.8 million . It should be around 5 million for males. So his RBC count is low.
 His Hb concentration is 8.4g/dl . It should be around 15g/dl. So his Hb concentration is low.
 So he can’t carry the oxygen efficiently. He can’t fulfil the oxygen needed by the body. So oxygen level in
blood is low than normal. So during exertion he feels fatigue.
 Due to low oxygen level in blood body is trying to get high amount of oxygen via hyperventilation. Due to
this compensatory mechanism he feels shortness of breath
6.4)
Immunological response for Primary exposure to pathogen
 When pathogen is exposed it develops innate immunity and acquired immunity.
 Innate immunity is natural body response against pathogen.
 When body develops acquired immunity antigen presenting cells identify the invading organisms . antigen
presenting cells are macrophages , dendritic cells and B lymphocytes.
 Antigen presenting cells destroy the invading organism and release the antigens.
Presented by 15th Batch 452 FHCS | EUSL
 This released antigen will be presented to T lymphocytes and B lymphocytes.
 When the antigen is presented to T lymphocyte it develops cell –mediated immunity. It will activate helper
T cells . helper T cell will activate cytotoxic T cells . cytotoxic T cells will release some toxic substance and it
will destroy invading organisms.
 When the antigen is presented to B lymphocytes it develops humoral immunity. It will activate B cells and it
will form the plasma cells. Plasma cells will produce antibodies. Antibodies will destroy the invading
organisms.
Immunological response for secondary exposure to pathogen

 When the pathogen is exposed primarily it develops immunological responses. Those messages will be
saved in memory T cells and memory B cells.
 Messages for cell mediated immunity is saved in memory T cells and messages for humoral immunity saved
in memory B cell.
 So during secondary exposure this memory cells get activate and take fast immunological responses and
destroy pathogen quickly.

Presented by 15th Batch 453 FHCS | EUSL

You might also like